You are on page 1of 116

INSURANCE

ABOITIZ SHIPPING CORPORATION V INDIA ASSURANCE COMPANY, LTD. [CITATION] QUISUMBING; May 2, 2006 NATURE Petition for review on certiorari FACTS - Societe Francaise Des Colloides loaded a cargo of textiles and auxiliary chemicals from France on board a vessel owned by Franco-Belgian Services, Inc. - The cargo was consigned to General Textile, Inc., in Manila and insured by respondent New India Assurance Company, Ltd. - While in Hongkong, the cargo was transferred to M/V P. Aboitiz for transshipment to Manila. - Before departing, the vessel was advised that it was safe to travel to its destination, but while at sea, the vessel received a report of a typhoon moving within its path. - To avoid the typhoon, the vessel changed its course. However, it was still at the fringe of the typhoon when its hull leaked. - On October 31, 1980, the vessel sank, but the captain and his crew were saved. - On November 3, 1980, the captain of M/V P. Aboitiz filed his Marine Protest, stating that the wind force was at 10 to 15 knots at the time the ship foundered and described the weather as moderate breeze, small waves, becoming longer, fairly frequent white horse - Petitioner notified the consignee of the total loss of the vessel and all of its cargoes. - General Textile, lodged a claim with respondent for the amount of its loss. - Respondent paid General Textile and was subrogated to the rights of the latter. - Respondent hired a surveyor to investigate, and the same concluded that the cause was the flooding of the holds brought about by the vessels questionable seaworthiness. - Respondent filed a complaint for damages against petitioner Aboitiz, Franco-Belgian Services and the latters local agent, F.E. Zuellig, Inc. (Zuellig) - On November 20, 1989, the trial court ruled in favor of respondent and held petitioner Aboitiz liable for the total value of the lost cargo plus legal interest - The complaint with respect to Franco and Zuellig was dismissed NEW - Petitioner elevated the case to the Court of Appeals, which in turn, affirmed in toto the trial courts decision. - Petitioner moved for reconsideration but the same was denied. - Hence, this petition for review Petitioners Claim > Petitioner contends that respondents claim for damages should only be against the insurance proceeds and limited to its pro-rata share in view of the doctrine of limited liability Respondents Comments > Respondent counters that the doctrine of real and hypothecary nature of maritime law is not applicable in the present case because petitioner was found to have been negligent. Hence, according to respondent, petitioner should be held liable for the total value of the lost cargo ISSUE WON the limited liability doctrine applies in this case HELD NO Ratio Where the shipowner fails to overcome the presumption of negligence, the doctrine of limited liability cannot be applied. Reasoning - From the nature of their business and for reasons of public policy, common carriers are bound to observe extraordinary diligence over the goods they transport according to all the circumstances of each case. In the event of loss, destruction or deterioration of the insured goods, common carriers are responsible, unless they can prove that the loss, destruction or deterioration was brought about by the causes specified in Article 17341 of the Civil Code. In all other cases, common carriers are presumed to have been at fault or to have acted negligently, unless they prove that they observed extraordinary diligence. Moreover, where the vessel is found unseaworthy, the shipowner is also presumed to be negligent since it is tasked
1
Art. 1734. Common carriers are responsible for the loss, destruction, or deterioration of the goods, unless the same is due to any of the following causes only: (1) Flood, storm, earthquake, lightning, or other natural disaster or calamity; (2) Act of the public enemy in war, whether international or civil; (3) Act of omission of the shipper or owner of the goods; (4) The character of the goods or defects in the packing or in the containers; (5) Order or act of competent public authority.

Page 1
with the maintenance of its vessel. Though this duty can be delegated, still, the shipowner must exercise close supervision over its men. - In the present case, petitioner has the burden of showing that it exercised extraordinary diligence in the transport of the goods it had on board in order to invoke the limited liability doctrine. Differently put, to limit its liability to the amount of the insurance proceeds, petitioner has the burden of proving that the unseaworthiness of its vessel was not due to its fault or negligence. - Considering the evidence presented and the circumstances obtaining in this case, we find that petitioner failed to discharge this burden. Both the trial and the appellate courts, in this case, found that the sinking was not due to the typhoon but to its unseaworthiness. Evidence on record showed that the weather was moderate when the vessel sank. These factual findings of the Court of Appeals, affirming those of the trial court are not to be disturbed on appeal, but must be accorded great weight. These findings are conclusive not only on the parties but on this Court as well. Disposition Petition is denied for lack of merit. PHILAMCARE HEALTH SYSTEMS, (TRINOS) 379 SCRA 357 YNARES-SANTIAGO; March 18, 2002 NATURE Petition for review of CA decision FACTS - Ernani TRINOS, deceased husband of respondent Julita, applied for a health care coverage with Philamcare Health Systems, Inc. In the standard application form, he answered no to the question: Have you or any of your family members ever consulted or been treated for high blood pressure, heart trouble, diabetes, cancer, liver disease, asthma or peptic ulcer? (If Yes, give details). - The application was approved for period of one year; upon termination, it was extended for another 2 years. Amount of coverage was increased to a maximum sum of P75T per disability. - During this period, Ernani suffered a HEART ATTACK and was confined at the Manila Medical Center (MMC) for one month. While her husband was in the hospital, Julita tried to claim the hospitalization benefits. - Petitioner treated the Health Care Agreement (HCA) as void since there was a concealment INC. V CA

INSURANCE
regarding Ernanis medical history. Doctors at the MMC allegedly discovered at the time of his confinement, he was hypertensive, diabetic and asthmatic. Julita then paid the hospitalization expenses herself, amounting to about P76T. - After her husband died, Julita instituted action for damages against Philamcare and its Pres. After trial, the lower court ruled in her favor and ordered Philamcare to reimburse medical and hospital coverage amounting to P76T plus interest, until fully paid; pay moral damages of P10T; pay exemplary damages of P10T; attys fees of P20T. - CA affirmed the decision of the trial court but deleted all awards for damages and absolved petitioner Reverente. Petitioners Claims (1) Agreement grants living benefits such as medical check-ups and hospitalization which a member may immediately enjoy so long as he is alive upon effectivity of the agreement until its expiration. (2) Only medical and hospitalization benefits are given under the agreement without any indemnification, unlike in an insurance contract where the insured is indemnified for his loss. (3) HCAs are only for a period of one year; therefore, incontestability clause does not apply, as it requires effectivity period of at least 2 yrs. (4) It is not an insurance company, governed by Insurance Commission, but a Health Maintenance Organization under the authority of DOH. (5) Trinos concealed a material fact in his application. (6) Julita was not the legal wife since at the time of their marriage, the deceased was previously married to another woman who was still alive.* ISSUES 1. WON a health care agreement is an insurance contract (If so, incontestability clause under the Insurance Code is applicable) 2. WON the HCA can be invalidated on the basis of alleged concealment HELD YES Ratio Every person has an insurable interest in the life and health of himself2. The health care agreement was in the nature of non-life insurance, which is
2
Sec.10. Every person has an insurable interest in the life and health: (1) of himself, of his spouse and of his children;

Page 2
primarily a contract of indemnity. Once the member incurs hospital, medical or any other expense arising from sickness, injury or other stipulated contingent, the health care provider must pay for the same to the extent agreed upon under the contract. Reasoning - A contract of insurance3 is an agreement whereby one undertakes for a consideration to indemnify another against loss, damage or liability arising from an unknown or contingent event. - An insurance contract exists where the following elements concur: (a) The insured has an insurable interest; (b) The insured is subject to a risk of loss by the happening of the peril; (c) The insurer assumes the risk; (d) Such assumption of risk is part of a general scheme to distribute actual losses among a large group of persons bearing a similar risk; and (e) In consideration of the insurers promise, the insured pays a premium. 2. NO Ratio Where matters of opinion or judgment are called for, answers made in good faith and without intent to deceive will not avoid a policy even though they are untrue; since in such case the insurer is not justified in relying upon such statement, but is obligated to make further inquiry. Reasoning - The fraudulent intent on the part of the insured must be established to warrant rescission of the insurance contract. The right to rescind should be exercised previous to the commencement of an action on the contract. No rescission was made. Besides, the cancellation of health care agreements as in insurance policies requires: (a) Prior notice of cancellation to insured; (b) Notice must be based on the occurrence after effective date of the policy of one or more of the grounds mentioned; (c) Must be in writing, mailed or delivered to the insured at the address shown in the policy; (d) Must state the grounds relied upon provided in Section 64 of the Insurance Code and upon request of insured, to furnish facts on which cancellation is based. - These conditions have not been met. When the terms of insurance contract contain limitations on liability, courts should construe them in such a way as to preclude insurer from non-compliance of obligation. Being a contract of adhesion, terms of an insurance contract are to be construed strictly against the party which prepared it the insurer. - Also, Philamcare had 12 months from the date of issuance of the Agreement within which to contest the membership of the patient if he had previous ailment of asthma, and six months from the issuance of the agreement if the patient was sick of diabetes or hypertension. * The health care agreement is in the nature of a contract of indemnity. Hence, payment should be made to the party who incurred the expenses. It is clear that respondent paid all the hospital and medical bills; thus, she is entitled to reimbursement. Disposition Petition DENIED. PINEDA V CA (INSULAR COMPANY) 226 SCRA 755 DAVIDE; September 27, 1993 LIFE INSURANCE

NATURE Appeal by certiorari for review and set aside the Decision of the public respondent Court of Appeals and its Resolution denying the petitioners' motion for reconsideration FACTS - In 1983, Prime Marine Services, Inc. (PMSI) procured a group policy from Insular Life to provide life insurance coverage to its sea-based employees enrolled under the plan. During the effectivity of the policy, 6 covered employees perished at sea. They were survived by complainants-appellees, the beneficiaries under the policy. - complainants-appellees sought to claim death benefits due them and approached Capt. Roberto Nuval, President and GM of PMSI, then executed special powers of attorney authorizing Capt. Nuval to follow up, ask, demand, collect and receive for their benefit indemnities of sums of money due them - Unknown to the complainants, PMSI filed with Insular Life claims for and in behalf of them through Capt. Nuval, even using the 5 special powers of attorney that they executed as documents. Insular Life then released 6 checks, payable to the order of

(2) of any person on whom he depends wholly or in part for education or support, or in whom he has a pecuniary interest; (3) of any person under a legal obligation to him for the payment of money, respecting property or service, of which death or illness might delay or prevent the performance; and (4) of any person upon whose life any estate or interest vested in him depends.

Section 2 (1) of the Insurance Code

INSURANCE
the complainant-appellees, to the treasurer of PMSI (who happened to be Capt. Nuvals son-in-law). Capt. Nuval then endorsed and deposited these checks (which were for the complainants) in his bank account. - 3 years after, the complainants-appellees found out that they were entitled, as beneficiaries, to life insular benefits under a group policy with respondentappellant so they sought to recover these benefits from Insular Life. Insular Life denied the claim, saying that their liability to complainants was already extinguished upon delivery to and receipt by PMSI of the 6 checks issued in the complainants names. Complainants filed case with the Insurance Commission which decided in their favor. - Insurance Commission held that the special powers of attorney executed by complainants in favor of the complainants do not contain in unequivocal and clear terms authority to Capt. Nuval to obtain, receive, receipt from respondent company insurance proceeds arising from the death of the seaman-insured; also, that Insular Life did not convincingly refuted the claim of Mrs. Alarcon that neither she nor her husband executed a special power of authority in favor of Capt. Nuval (and therefore, the company should have not released the check to Capt. Nuval-PMSI); and that it did not observe Sec 180(3), as repealed by Art. 225 of the Family Code, when it released the benefits due to the minor children of Ayo and Lontok, when the said complainants did not post a bond as required - Insular Life appealed to the CA; CA modified the decision of the Insurance Commission, eliminating the award to the Lontoks and Ayo ISSUES 1. WON Insular Life should be liable to the complainants when they relied on the special powers of attorney, which Capt. Nuval presented as documents, when they released the checks to the latter 2. WON Insular Life should be liable to the complainants when they released the check in favor of Ayo and LOntok, even if no bond was posted as required HELD 1. YES Ratio Third persons deal with agents at their peril and are bound to inquire as to the extent of the power of the agent with whom they contract. The person dealing with an agent must also act with ordinary prudence and reasonable diligence. Obviously, if he knows or has good reason to believe that the agent is exceeding his authority, he cannot claim protection. So if the suggestions of probable limitations be of such a clear and reasonable quality, or if the character assumed by the agent is of such a suspicious or unreasonable nature, or if the authority which he seeks to exercise is of such an unusual or improbable character, as would suffice to put an ordinarily prudent man upon his guard, the party dealing with him may not shut his eyes to the real state of the case, but should either refuse to deal with the agent at all, or should ascertain from the principal the true condition of affairs. Reasoning - The execution by the principals of special powers of attorney, which clearly appeared to be in prepared forms and only had to be filled up with their names, residences, dates of execution, dates of acknowledgement and others, excludes any intent to grant a general power of attorney or to constitute a universal agency. Being special powers of attorney, they must be strictly construed. Insular Life knew that a power of attorney in favor of Capt. Nuval for the collection and receipt of such proceeds was a deviation from its practice with respect to group policies (that the employer-policyholder is the agent of the insurer). - The employer acts as a functionary in the collection and payment of premiums and in performing related duties. Likewise falling within the ambit of administration of a group policy is the disbursement of insurance payments by the employer to the employees. Most policies, such as the one in this case, require an employee to pay a portion of the premium, which the employer deducts from wages while the remainder is paid by the employer. This is known as a contributory plan as compared to a non-contributory plan where the premiums are solely paid by the employer. - the labor of the employees is the true source of the benefits, which are a form of additional compensation to them. - the employer is the agent of the insurer in performing the duties of administering group insurance policies. It cannot be said that the employer acts entirely for its own benefit or for the benefit of its employees in undertaking administrative functions. While a reduced premium may result if the employer relieves the insurer of these tasks, and this, of course, is advantageous to both the employer and the employees, the insurer also enjoys significant advantages from the arrangement. The reduction in

Page 3
the premium which results from employeradministration permits the insurer to realize a larger volume of sales, insurer to realize a larger volume of sales, and at the same time the insurer's own administrative costs are markedly reduced. - the employee has no knowledge of or control over the employer's actions in handling the policy or its administration. An agency relationship is based upon consent by one person that another shall act in his behalf and be subject to his control. It is clear from the evidence regarding procedural techniques here that the insurer-employer relationship meets this agency test with regard to the administration of the policy, whereas that between the employer and its employees fails to reflect true agency. The insurer directs the performance of the employer's administrative acts, and if these duties are not undertaken properly the insurer is in a position to exercise more constricted control over the employer's conduct. - ON GROUP INSURANCE: Group insurance is essentially a single insurance contract that provides coverage for many individuals. In its original and most common form, group insurance provides life or health insurance coverage for the employees of one employer. The coverage terms for group insurance are usually stated in a master agreement or policy that is issued by the insurer to a representative of the group or to an administrator of the insurance program, such as an employer. Although the employer may be the titular or named insured, the insurance is actually related to the life and health of the employee. Indeed, the employee is in the position of a real party to the master policy, and even in a non-contributory plan, the payment by the employer of the entire premium is a part of the total compensation paid for the services of the employee. 2. YES Ratio Regardless of the value of the unemancipated common child's property, the father and mother ipso jure become the legal guardian of the child's property. However, if the market value of the property or the annual income of the child exceeds P50,000,00, a bond has to be posted by the parents concerned to guarantee the performance of the obligations of a general guardian. Reasoning - Sec 180, Insurance Code: 'In the absence of a judicial guardian, the father, or in the latter's absence or incapacity, the mother of any minor, who is an insured or a beneficiary under a contract of life, health

INSURANCE
or accident insurance, may exercise, in behalf of said minor, any right under the policy, without necessity of court authority or the giving of a bond where the interest of the minor in the particular act involved does not exceed twenty thousand pesos " - repealed by Art. 225, Family Code: "ART. 225. The father and the mother shall jointly exercise legal guardianship over the property of their unemancipated common child without the necessity of a court appointment. In case of disagreement, the father's decision shall prevail, unless there is judicial order to the contrary. Where the market value of the property or the annual income of the child exceeds P50,000, the parent concerned shall be required to furnish a bond in such amount as the court may determine, but not less than ten per centum (10%) of the value of the property or annual income, to guarantee the performance of the obligations prescribed for general guardians." -"market value of the property or the annual income of the child": the aggregate of the child's property or annual income; if this exceeds P50,000.00, a bond is required - There is no evidence that the share of each of the minors in the proceeds of the group policy in question is the minor's only property. Without such evidence, it would not be safe to conclude that, indeed, that is his only property. Disposition the instant petition is GRANTED. The Decision of 10 October 1991 and the Resolution of 19 May 1992 of the public respondent in CA-G.R. SP No. 22950 are SET ASIDE and the Decision of the Insurance Commission in IC Case No. RD-058 is REINSTATED. Costs against the private respondent. SO ORDERED. CEBU SHIPYARD ENGINEERING WORKS, INC. V WILLIAM LINES, INC. and PRUDENTIAL GUARANTEE and ASSURANCE COMPANY, INC. [CITATION] PURISIMA; May 5, 1999 NATURE Petition for review on certiorari FACTS - Cebu Shipyard and Engineering Works, Inc. (CSEW) is engaged in the business of dry-docking and repairing of marine vessels while the Prudential Guarantee and Assurance, Inc. (Prudential) is in the non-life insurance business. - William Lines, Inc. is in the shipping business. It was the owner of M/V Manila City, a luxury passenger-cargo vessel, which caught fire and sank on Feb. 16, 1991. At the time of the unfortunate occurrence sued upon, subject vessel was insured with Prudential for P45M for hull and machinery. The Hull Policy included an Additional Perils (INCHMAREE) Clause covering loss of or damage to the vessel through the negligence of, among others, ship repairmen - Petitioner CSEW was also insured by Prudential for third party liability under a Shiprepairers Legal Liability Insurance Policy. The policy was for P10 million only, under the limited liability clause, to wit: - On Feb. 5, 1991, William Lines, Inc. brought its vessel, M/V Manila City, to the Cebu Shipyard in Lapulapu City for annual dry-docking and repair. - On Feb. 6, 1991, an arrival conference was held between representatives of William Lines, Inc. and CSEW to discuss the work to be undertaken on the M/V Manila City. The contracts, denominated as Work Orders, were signed thereafter., with the following stipulations: 10. The Contractor shall replace at its own work and at its own cost any work or material which can be shown to be defective and which is communicated in writing within one (1) month of redelivery of the vessel or if the vessel was not in the Contractors Possession, the withdrawal of the Contractors workmen, or at its option to pay a sum equal to the cost of such replacement at its own works. These conditions shall apply to any such replacements. 11. Save as provided in Clause 10, the Contractor shall not be under any liability to the Customer either in contract or for delict or quasi-delict or otherwise except for negligence and such liability shall itself be subject to the following overriding limitations and exceptions, namely: (a) The total liability of the Contractor to the Customer (over and above the liability to replace under Clause 10) or of any sub-contractor shall be limited in respect of any defect or event (and a series of accidents arising out of the same defect or event shall constitute one defect or event) to the sum of Pesos Philippine Currency One Million only. x x x 20. The insurance on the vessel should be maintained by the customer and/or owner of the vessel during the period the contract is in effect. - While the M/V Manila City was undergoing drydocking and repairs within the premises of CSEW, the master, officers and crew of M/V Manila City stayed in the vessel, using their cabins as living quarters. Other employees hired by William Lines to do repairs and

Page 4
maintenance work on the vessel were also present during the dry-docking. - On February 16, 1991, after subject vessel was transferred to the docking quay, it caught fire and sank, resulting to its eventual total loss. - On February 21, 1991, William Lines, Inc. filed a complaint for damages against CSEW, alleging that the fire which broke out in M/V Manila City was caused by CSEWs negligence and lack of care. - On July 15, 1991 was filed an Amended Complaint impleading Prudential as co-plaintiff, after the latter had paid William Lines, Inc. the value of the hull and machinery insurance on the M/V Manila City. As a result of such payment Prudential was subrogated to the claim of P45 million, representing the value of the said insurance it paid. On June 10, 1994, the trial court a quo came out with a judgment against CSEW: 1. To pay unto plaintiff Prudential Guarantee and Assurance, Inc., the subrogee, the amount of P45M, with interest at the legal rate until full payment is made; the amount of P56,715,000 representing loss of income of M/V MANILA CITY, with interest at the legal rate until full payment is made; 2. To pay unto plaintiff, William Lines, Inc. the amount of P11M as payment, in addition to what it received from the insurance company to fully cover the injury or loss, in order to replace the M/V MANILA CITY, with interest at the legal rate until full payment is made; the sum of P927,039 for the loss of fuel and lub oil on board the vessel when she was completely gutted by fire at defendant, Cebu Shipyards quay, with interest at the legal rate until full payment is made; the sum of P3,054,677.95 as payment for the spare parts and materials used in the M/V MANILA CITY during dry-docking with interest at the legal rate until full payment is made; P500,000 in moral damages;the amount of P10Min attorneys fees; and to pay the costs of this suit. On September 3, 1997, the Court of Appeals affirmed the appealed decision of the trial court, ordering CSEW to pay Prudential, the subrogee, the sum of P45 Million, with interest at the legal rate until full payment is made. CSEWs version: On Feb. 13, 1991, the CSEW completed the drydocking of M/V Manila City at its grave dock. It was then transferred to the docking quay of CSEW where the remaining repair to be done was the replating of the top of Water Ballast Tank No. 12 which was subcontracted by CSEW to JNB General Services. Tank Top No. 12 was at the rear section of

INSURANCE
the vessel, on level with the flooring of the crew cabins located on the vessels second deck. At around 7AM of Feb. 16, 1991, the JNB workers trimmed and cleaned the tank top framing which involved minor hotworks (welding/cutting works). The said work was completed at about 10AM. The JNB workers then proceeded to rig the steel plates, after which they had their lunch break. The rigging was resumed at 1PM While in the process of rigging the second steel plate, the JNB workers noticed smoke coming from the passageway along the crew cabins. When one of the workers, Mr. Casas, proceeded to the passageway to ascertain the origin of the smoke, he noticed that smoke was gathering on the ceiling of the passageway but did not see any fire as the crew cabins on either side of the passageway were locked. He immediately sought out the proprietor of JNB, Mr. Buenavista, and the Safety Officer of CSEW, Mr. Aves, who sounded the fire alarm. CSEWs fire brigade immediately responded as well as the other fire fighting units in Metro Cebu. However, there were no WLI representative, officer or crew to guide the firemen inside the vessel. - Despite the combined efforts of the firemen of the Lapulapu City Fire Dept., Mandaue Fire Dept., Cordova Fire Dept. Emergency Rescue Unit Foundation, and fire brigade of CSEW, the fire was not controlled until 2AM of the following day. - On the early morning of Feb. 17, 1991, gusty winds rekindled the flames on the vessel and fire again broke out. Then the huge amounts of water pumped into the vessel, coupled with the strong current, caused the vessel to tilt until it capsized and sank - When M/V Manila City capsized, steel and angle bars were noticed to have been newly welded along the port side of the hull of the vessel, at the level of the crew cabins. William Lines did not previously apply for a permit to do hotworks on the said portion of the ship as it should have done pursuant to its work order with CSEW. Prudentials version > At around 7AM of Feb. 16, 1991, the Chief Mate of M/V Manila City was inspecting the various works being done by CSEW on the vessel, when he saw that some workers of CSEW were cropping out steel plates on Tank Top No. 12 using acetylene, oxygen and welding torch. He also observed that the rubber insulation wire coming out of the air-conditioning unit was already burning, prompting him to scold the workers. > At 2:45 PM of the same day, witnesses saw smoke coming from Tank No. 12. The vessels reeferman reported such occurence to the Chief Mate who immediately assembled the crew members to put out the fire. When it was too hot for them to stay on board and seeing that the fire cannot be controlled, the vessels crew were forced to withdraw from CSEWs docking quay. - In the morning of Feb. 17, 1991, M/V Manila City sank. As the vessel was insured with Prudential Guarantee, William Lines filed a claim for constructive total loss, and after a thorough investigation of the surrounding circumstances of the tragedy, Prudential found the said insurance claim to be meritorious and issued a check in favor of William Lines in the amount of P45 million pesos representing the total value of M/V Manila Citys hull and machinery insurance. ISSUES 1. WON CSEW had management and supervisory control of the m/v manila city at the time the fire broke out 2. WON the doctrine of res ipsa loquitur applies against the crew 3. WON CSEWS expert evidence is admissible or of probative value 4. WON Prudential has the right of subrogation against its own insured THE CONTRACTUAL 5. 5. 5. 5. WON the provisions limiting csews liability for negligence to a maximum of p1 million are valid HELD 1. YES - The that factual findings by the CA are conclusive on the parties and are not reviewable by this Court. They are entitled to great weight and respect, even finality, especially when, as in this case, the CA affirmed the factual findings arrived at by the trial court. When supported by sufficient evidence, findings of fact by the CA affirming those of the trial court, are not to be disturbed on appeal. The rationale behind this doctrine is that review of the findings of fact of the CA is not a function that the Supreme Court normally undertakes. - The CA and the Cebu RTC are agreed that the fire which caused the total loss of subject M/V Manila City was due to the negligence of the employees and workers of CSEW. Both courts found that the M/V Manila City was under the custody and control of petitioner CSEW, when the ill-fated vessel caught fire. The decisions of both the lower court and the CA set forth clearly the evidence sustaining their finding of

Page 5
actionable negligence on the part of CSEW. This factual finding is accorded great weight and is conclusive on the parties. The court discerns no basis for disturbing such finding firmly anchored on enough evidence. - Furthermore, in petitions for review on certiorari, only questions of law may be put into issue. Questions of fact cannot be entertained. The finding of negligence by the CA is a question which this Court cannot look into as it would entail going into factual matters on which the finding of negligence was based. Such an approach cannot be allowed by this Court in the absence of clear showing that the case falls under any of the exceptions to the well-established principle. The finding by the trial court and the Court of Appeals that M/V Manila City caught fire and sank by reason of the negligence of the workers of CSEW, when the said vessel was under the exclusive custody and control of CSEW is accordingly upheld. 2. YES - For the doctrine of res ipsa loquitur to apply to a given situation, the following conditions must concur: (1) the accident was of a kind which does not ordinarily occur unless someone is negligent; and (2) that the instrumentality or agency which caused the injury was under the exclusive control of the person charged with negligence. The facts and evidence on record reveal the concurrence of said conditions in the case under scrutiny. First, the fire that occurred and consumed M/V Manila City would not have happened in the ordinary course of things if reasonable care and diligence had been exercised. In other words, some negligence must have occurred. Second, the agency charged with negligence, as found by the trial court and the CA and as shown by the records, is the herein petitioner, CSEW, which had control over subject vessel when it was docked for annual repairs. So also, as found by the RTC, other responsible causes, including the conduct of the plaintiff, and third persons, are sufficiently eliminated by the evidence. What is more, in the present case the trial court found direct evidence to prove that the workers and/or employees of CSEW were remiss in their duty of exercising due diligence in the care of subject vessel. The direct evidence substantiates the conclusion that CSEW was really negligent. Thus, even without applying the doctrine of res ipsa loquitur, in light of the direct evidence on record, the ineluctable conclusion is that CSEW was negligent and consequently liable for damages to the respondent, William Lines, Inc.

INSURANCE
3. NO - Petitioner maintains that the CA erred in disregarding the testimonies of the fire experts, Messrs. David Grey and Gregory Michael Southeard, who testified on the probable origin of the fire in M/V Manila City. Petitioner avers that since the said fire experts were one in their opinion that the fire did not originate in the area of Tank Top No. 12 where the JNB workers were doing hotworks but on the crew accommodation cabins on the portside No. 2 deck, the RTC and the CA should have given weight to such finding based on the testimonies of fire experts; petitioner argues. But courts are not bound by the testimonies of expert witnesses. Although they may have probative value, reception in evidence of expert testimonies is within the discretion of the court, under Section 49, Rule 130 of the Revised Rules of Court. It is never mandatory for judges to give substantial weight to expert testimonies. If from the facts and evidence on record, a conclusion is readily ascertainable, there is no need for the judge to resort to expert opinion evidence. In the case under consideration, the testimonies of the fire experts were not the only available evidence on the probable cause and origin of the fire. There were witnesses who were actually on board the vessel when the fire occurred. Between the testimonies of the fire experts who merely based their findings and opinions on interviews and the testimonies of those present during the fire, the latter are of more probative value. Prudential to William Lines, Inc., the former was subrogated to the right of the latter to indemnification from CSEW. As aptly ruled by the Court of Appeals, the law on the matter is succinct and clear, to wit: - Art. 2207. If the plaintiffs property has been insured, and he has received indemnity from the insurance company for the injury or loss arising out of the wrong or breach of contract complained of, the insurance company shall be subrogated to the rights of the insured against the wrongdoer or the person who has violated the contract. If the amount paid by the insurance company does not fully cover the injury or loss, the aggrieved party shall be entitled to recover the deficiency from the person causing the loss or injury. - Thus, when Prudential, after due verification of the merit and validity of the insurance claim of William Lines, Inc., paid the latter the total amount covered by its insurance policy, it was subrogated to the right of the latter to recover the insured loss from the liable party, CSEW. - Petitioner theorizes further that there can be no right of subrogation as it is deemed a co-assured under the subject insurance policy. To buttress its stance that it is a co-assured, petitioner placed reliance on Clause 20 of the Work Order which states: 20. The insurance on the vessel should be maintained by the customer and/or owner of the vessel during the period the contract is in effect. - According to petitioner, under the aforecited clause, William Lines, Inc., agreed to assume the risk of loss of the vessel while under drydock or repair and to such extent, it is benefited and effectively constituted as a co-assured under the policy. - This theory of petitioner is devoid of sustainable merit. Clause 20 of the Work Order in question is clear in the sense that it requires William Lines to maintain insurance on the vessel during the period of dry-docking or repair. Concededly, such a stipulation works to the benefit of CSEW as the shiprepairer. However, the fact that CSEW benefits from the said stipulation does not automatically make it as a coassured of William Lines. The intention of the parties to make each other a co-assured under an insurance policy is to be gleaned principally from the insurance contract or policy itself and not from any other contract or agreement because the insurance policy denominates the assured and the beneficiaries of the insurance. The hull and machinery insurance procured by William Lines, Inc. from Prudential named only William Lines, Inc. as the assured. There was

Page 6
no manifestation of any intention of William Lines, Inc. to constitute CSEW as a co-assured under subject policy. It is axiomatic that when the terms of a contract are clear its stipulations control.i] Thus, when the insurance policy involved named only William Lines, Inc. as the assured thereunder, the claim of CSEW that it is a co-assured is unfounded. - Then too, in the Additional Perils Clause of the same Marine Insurance Policy, it is provided that: Subject to the conditions of this Policy, this insurance also covers loss of or damage to vessel directly caused by the following: xxx Negligence of Charterers and/or Repairers, provided such Charterers and/or Repairers are not an Assured hereunder. - As correctly pointed out by respondent Prudential, if CSEW were deemed a co-assured under the policy, it would nullify any claim of William Lines, Inc. from Prudential for any loss or damage caused by the negligence of CSEW. Certainly, no shipowner would agree to make a shiprepairer a co-assured under such insurance policy; otherwise, any claim for loss or damage under the policy would be invalidated. Such result could not have been intended by William Lines, Inc. 5. NO - Although in this jurisdiction, contracts of adhesion have been consistently upheld as valid per se; as binding as an ordinary contract, the Court recognizes instances when reliance on such contracts cannot be favored especially where the facts and circumstances warrant that subject stipulations be disregarded. Thus, in ruling on the validity and applicability of the stipulation limiting the liability of CSEW for negligence to P1M only, the facts and circumstances vis-a-vis the nature of the provision sought to be enforced should be considered, bearing in mind the principles of equity and fair play. - It is worthy to note that M/V Manila City was insured with Prudential for P45M. To determine the validity and sustainability of the claim of William Lines, Inc., for a total loss, Prudential conducted its own inquiry. Upon thorough investigation by its hull surveyor, M/V Manila City was found to be beyond economical salvage and repair. The evaluation of the average adjuster also reported a constructive total loss. The said claim of William Lines, Inc., was then found to be valid and compensable such that Prudential paid the latter the total value of its insurance claim. Furthermore, it was ascertained that the replacement

4. YES - Petitioner contends that Prudential is not entitled to be subrogated to the rights of William Lines, Inc., theorizing that (1) the fire which gutted M/V Manila City was an excluded risk and (2) it is a co-assured under the Marine Hull Insurance Policy. - It is petitioners submission that the loss of M/V Manila City or damage thereto is expressly excluded from the coverage of the insurance because the same resulted from want of due diligence by the Assured, Owners or Managers which is not included in the risks insured against. Again, this theory of petitioner is bereft of any factual or legal basis. It proceeds from a wrong premise that the fire which gutted subject vessel was caused by the negligence of the employees of William Lines, Inc. To repeat, the issue of who between the parties was negligent has already been resolved against CSEW. Upon proof of payment by

INSURANCE
cost of the vessel (the price of a vessel similar to M/V Manila City), amounts to P55M. - Considering the aforestated circumstances, let alone the fact that negligence on the part of petitioner has been sufficiently proven, it would indeed be unfair and inequitable to limit the liability of petitioner to One Million Pesos only. As aptly held by the trial court, it is rather unconscionable if not overstrained. To allow CSEW to limit its liability to P1M notwithstanding the fact that the total loss suffered by the assured and paid for by Prudential amounted to P45M would sanction the exercise of a degree of diligence short of what is ordinarily required because, then, it would not be difficult for petitioner to escape liability by the simple expedient of paying an amount very much lower than the actual damage or loss suffered by William Lines, Inc. Disposition Petition is DENIED. Resolution of the CA is AFFIRMED. NEW LIFE ENTERPRISES V CA 207 SCRA 669 REGALADO; March 31, 1992 NATURE Appeal by certiorari FACTS - The antecedents of this case show that Julian Sy and Jose Sy Bang have formed a business partnership in the City of Lucena. Under the business name of New Life Enterprises, the partnership engaged in the sale of construction materials at its place of business, a two storey building situated at Iyam, Lucena City. The facts show that Julian Sy insured the stocks in trade of New Life Enterprises with Western Guaranty Corporation, Reliance Surety and Insurance Co. Inc., and Equitable Insurance Corporation. - On May 15, 1981, Western Guaranty Corporation issued Fire Insurance Policy No. 37201 in the amount of P350,000.00. This policy was renewed on May 13, 1982. - On July 30, 1981, Reliance Surety and Insurance Co., Inc. issued Fire Insurance Policy No. 69135 in the amount of P300,000.00 (Renewed under Renewal Certificate No. 41997). An additional insurance was issued by the same company on November 12, 1981 under Fire Insurance Policy No. 71547 in the amount of P700,000.00. On February 8, 1982, Equitable Insurance Corporation issued Fire Insurance Policy No. 39328 in the amount of P200,000.00. - Thus when the building occupied by the New Life Enterprises was gutted by fire at about 2:00 o'clock in the morning of October 19, 1982, the stocks in trade inside said building were insured against fire in the total amount of P1,550,000.00. According to the certification issued by the Headquarters, Philippine Constabulary/Integrated National Police, Camp Crame, the cause of fire was electrical in nature. According to the plaintiffs, the building and the stocks inside were burned. After the fire, Julian Sy went to the agent of Reliance Insurance whom he asked to accompany him to the office of the company so that he can file his claim. He averred that in support of his claim, he submitted the fire clearance, the insurance policies and inventory of stocks. He further testified that the three insurance companies are sister companies, and as a matter of fact when he was following-up his claim with Equitable Insurance, the Claims Manager told him to go first to Reliance Insurance and if said company agrees to pay, they would also pay. The same treatment was given him by the other insurance companies. Ultimately, the three insurance companies denied plaintiffs' claim for payment. Respondents comments > Western Guaranty Corporation through Claims Manager Bernard S. Razon told the plaintiff that his claim 'is denied for breach of policy conditions.' Reliance Insurance purveyed the same message as well as Equitable Insurance Corporation. - The said policy in question follows: "The insured shall give notice to the Company of any insurance or insurances already effected, or which may subsequently be effected, covering any of the property or properties consisting of stocks in trade, goods in process and/or inventories only hereby insured, and unless such notice be given and the particulars of such insurance or insurances be stated therein or endorsed on this policy pursuant to Section 50 of the Insurance Code, by or on behalf of the Company before the occurrence of any loss or damage, all benefits under this policy shall be deemed forfeited, provided however, that this condition shall not apply when the total insurance or insurances in force at the time of loss or damage is not more than P200,000.00." Petitioners comments > Petitioners contend that they are not to be blamed for the omissions, alleging that insurance agent Leon Alvarez (for Western) and Yap Kam Chuan (for Reliance and Equitable) knew about the existence of the additional insurance coverage and that they were

Page 7
not informed about the requirement that such other or additional insurance should be stated in the policy, as they have not even read said policies. ISSUE WON New Life Enterprises claim for payment be denied HELD YES Ratio Furthermore, when the words and language of documents are clear and plain or readily understandable by an ordinary reader thereof, there is absolutely no room for interpretation or construction anymore. Courts are not allowed to make contracts for the parties; rather, they will intervene only when the terms of the policy are ambiguous, equivocal, or uncertain. The parties must abide by the terms of the contract because such terms constitute the measure of the insurer's liability and compliance therewith is a condition precedent to the insured's right of recovery from the insurer. - While it is a cardinal principle of insurance law that a policy or contract of insurance is to be construed liberally in favor of the insured and strictly against the insurer company, yet contracts of insurance, like other contracts, are to be construed according to the sense and meaning of the terms which the parties themselves have used. If such terms are clear and unambiguous, they must be taken and understood in their plain, ordinary and popular sense. Moreover, obligations arising from contracts have the force of law between the contracting parties and should be complied with in good faith. Reasoning a. The terms of the contract are clear and unambiguous. The insured is specifically required to disclose to the insurer any other insurance and its particulars which he may have effected on the same subject matter. The knowledge of such insurance by the insurer's agents, even assuming the acquisition thereof by the former, is not the "notice" that would stop the insurers from denying the claim. Besides, the so-called theory of imputed knowledge, that is, knowledge of the agent is knowledge of the principal, aside from being of dubious applicability here has likewise been roundly refuted by respondent court whose factual findings we find acceptable. b. Petitioners should be aware of the fact that a party is not relieved of the duty to exercise the ordinary care and prudence that would be exacted in relation to other contracts. The conformity of the insured to

INSURANCE
the terms of the policy is implied from his failure to express any disagreement with what is provided for. FIRST QUEZON CITY INSURANCE CO. v. CA (DE DIOS MARIKINA TRANSPORT CO) 218 SCRA 526 GRINO-AQUINO; February 28, 1993 NATURE PETITION for review of the decision of the Court of Appeals. FQCIC seeks to limit to P12000, the amount specified in the insurance contract, its liability to indemnify the respomdemt DMTC, for the damages suffered by a passenger, who accidentally fell off the bug. FACTS - After sending off certain seamen at the departure area of MIA, Jose V. del Rosario proceeded to the public utility bus stop. While at the bus stop, the plaintiff saw a DMTC bus. While moving at a crawling pace, it was taking several passengers, all of whom managed to board the bus while it was already at the bus stop; plaintiff was the last one to board the bus. While the plaintiff was still on the bus with his hand on the bus door, the slowly moving bus sped forward at a high speed, as a result of which, the plaintiff lost balance and fell from the bus. As plaintiff clung instinctively to the handle bar, he was dragged by the bus along the asphalted road. The bus driver, Gil Agpalo, abruptly stopped the bus. Then fled from the scene, leaving the bus and the injured plaintiff behind. - The plaintiff was brought to the Manila Sanitarium and Hospital where the doctors performed 2 major surgical operations on plaintiffs right leg. - Plaintiff was confined at the hospital for (40) days, from June 10, 1984 to August 26, 1984. Medical expenses totaled the amount of P69,444.41. Plaintiffs medical expenses were advanced by his employer Maglines but he was required to reimburse Maglines on a staggered basis by way of salary deductions. After his release from the hospital, he returned to the hospital for further treatment and checkup. The injuries had left plaintiff with a huge scar on his right leg. Also, the plaintiff incurred lost earning by way of unearned salaries amounting to P7,500.00 due to said physical injuries and the consequent hospital confinement. - Plaintiff filed on June 26, 1985 the complaint against DMTC and its driver. Agpalo was later dropped as a party defendant because he could not be served with summons. Upon filing its answer, defendant DMTC filed a thirdparty complaint against First Quezon City Insurance Co., Inc. September 17, 1985, third-party defendant filed its answer to the third-party complaint. - TC held DMTC complaint dismissed for lack of merit and as regards the third-party complaint First Quezon City Insurance Co., Inc. was to indemnify third-party plaintiff DMTC in the sum of P12,000.00 with interest. There being no satisfactory warrant the court dismissed the rest of the claims in the complaint and third-party complaint. - The bus company appealed to the CA, which modified the dispositive as regards the third-party complaint, that the third-party defendant First Quezon City Insurance Co., Inc. be ordered to indemnify third-party plaintiff DMTC the SUM of P50,000.00 with legal interest. Insurance company filed a MFR which was denied. Hence, this petition for review, assailing the appellate courts' interpretation of the provision of the insurance contract on the limit of the insurer's liability. ISSUE WON the CA erred in the interpretation of the insurance contract on the limit of the insurers liability HELD YES - The insurance policy clearly placed the maximum limit of the petitioner's liability for damages arising from death or bodily injury at P12,000.00 per passenger and its maximum liability per accident at (P50,000.00. Since only one passenger was injured in the accident, the insurer's liability for the damages suffered by said passenger is pegged to the amount of P12,000.00 only. - The limit of P50,000.00 per accident means that the insurer's maximum liability for any single accident will not exceed P50,000.00 regardless of the number of passengers killed or injured therein. The bus company may not recover from the insurance company more than P12,000.00 per passenger killed or injured, or (P50,000.00) per accident even if under the judgment of the court, the erring bus operator will have to pay more than P12,000.00 to each injured passenger. The trial court's interpretation of the insurance contract was the correct interpretation. Disposition petition for review is GRANTED. The decision promulgated by the CA, ordering the third party defendent, First Quezon City Insurance Co., Inc. to indemnify theI private respondent, (DMTC), the

Page 8
sum of P50,000.00 for the damages of the passenger, Jose V. Del Rosario, is hereby modified by reducing the award to 12,000.00 only. Costs against the private respondent De Dios Marikina Transportation Co., Inc. TY V FIRST NATIONAL SURETY 1 SCRA 1324 LABRADOR; April 29, 1961 FACTS - At different times within a period of two months prior to 24 December 1953, Diosdado C. Ty, employed as operator mechanic foreman in the Broadway Cotton Factory insured himself in 18 local insurance companies, among which being the 8 above-named defendants, which issued to him personal accident policies. Plaintiffs beneficiary was his employer, Broadway Cotton Factory, which paid the insurance premiums. On 24 December 1953, a fire broke out which totally destroyed the Broadway Cotton Factory. Fighting his way out of the factory, plaintiff was injured on the left hand by a heavy object. He was brought to the Manila Central University hospital, and after receiving first-aid, he went to the National Orthopedic Hospital for treatment of his injuries (fractures in index, middle, fourth, and fifth fingers of left hand). From 26 December 1953 to 8 February 1954, he underwent medical treatment in the hospital. The abovedescribed physical injuries have caused temporary total disability of plaintiffs left hand. Plaintiff filed the corresponding notice of accident and notice of claim with all of the above-named defendants to recover indemnity. Defendants rejected plaintiffs claim for indemnity for the reason that there being no severance of amputation of the left hand, the disability suffered by him was not covered by his policy. - Plaintiff sued the defendants in the Municipality Court of this City, which dismissed his complaints. Thereafter, the plaintiff appealed to the Court of First Instance Manila, presided by Judge Gregorio S. Narvasa, which absolved the defendants from the complaints. Hence, the appeal. ISSUE WON Diosdado Ty is entitled to indemnity under the insurance policy for the disability of his left hand HELD

INSURANCE
- The agreement contained in the insurance policies is the law between the parties. As the terms of the policies are clear, express and specific that only amputation of the left hand should be considered as a loss thereof, an interpretation that would include the mere fracture or other temporary disability not covered by the policies would certainly be unwarranted. In the case at bar, due to the clarity of the stipulation, distinction between temporary disability and total disability need not be made in relation to ones occupation means that the condition of the insurance is such that common prudence requires him to desist from transacting his business or renders him incapable of working. While the Court sympathizes with the plaintiff or his employer, for whose benefit the policies were issued, it can not go beyond the clear and express conditions of the insurance policies, all of which define partial disability as loss of either hand by a amputation through the bones of the wrist. There was no such amputation in the case at bar. - The Supreme Court affirmed the appealed decision, with costs against the plaintiff-appellant. MISAMIS LUMBER V CAPITAL INSURANCE 17 SCRA 288 REYES; May 20, 1966 NATURE Direct appeal on a point of law from the judgment of the Court of First Instance of Manila FACTS - Misamis Lumber Corporation, under its former name, Lanao Timber Mills, Inc., insured its Ford Falcon motor car for the amount of P14,000 with Capital Insurance & Surety Company, Inc. The pertinent provisions of the policy provided, as follows: 1. The Company will subject to the Limits of Liability indemnify the Insured against loss or damage to the Motor Vehicle and its accessories and spare parts whilst thereon. 2. (a) by accidental collision or overturning or collision or overturning consequent when mechanical breakdown or consequent upon wear and tear. 3. At its option, the Company may pay in cash the amount of the loss or damage or may repair, reinstate or replace the Motor Vehicle or any part thereof or its accessories or spare parts. The liability of the Company shall not exceed the value of the parts lost or damaged and the reasonable cost of fitting such parts or the value of the Motor Vehicle at the time of the loss or damage whichever is the loss. The Insured's estimate of value stated in the schedule shall be the maximum amount payable by the Company in respect of any claim for loss or damage. 4. The Insured may authorize the repair of the Motor Vehicle necessitated by damage for which the Company may be liable under this policy provided that: (a) the estimated cost of such repair does not exceed the authorized Repair Limit. (b) a detailed estimate of the cost is forwarded to the Company without delay and providing also that the authorized repair limit is P150.00. - One night, the insured car, while traveling along in Aurora Boulevard, passed over a water hole which the driver did not see because an oncoming car did not dim its light. The crankcase and flywheel housing of the car broke when it hit a hollow block lying alongside the water hole. The car was towed and repaired by Morosi Motors at a total cost of P302.27. - When the repairs on the car had already been made, Misamis made a report of the accident to Capital Insurance. - Since Capital refused to pay for the total cost of to wage and repairs, suit was filed in the municipal court originally. - The defendant-appellant admits liability in the amount of P150, but not for any excess thereof. The lower court did not exonerate the said appellant for the excess because the company's absolution would render the insurance contract one-sided and that the said insurer had not shown that the cost of repairs in the sum of P302.27 is unreasonable, excessive or padded, nor had it shown that it could have undertaken the repairs itself at less expense. ISSUE WON Capital Insurance can be made to pay more than P150 HELD NO - The insurance policy stipulated in paragraph 4 that if the insured authorizes the repair the liability of the insurer, per its sub-paragraph (a), is limited to P150.00. The literal meaning of this stipulation must control, it being the actual contract, expressly and plainly provided for in the policy.

Page 9
- Recourse to legal hermeneutics is not called for because paragraph 4 of the policy is clear and specific and leaves no room for interpretation. - The option to undertake the repairs is accorded to the insurance company per paragraph 2. The said company was deprived of the option because the insured took it upon itself to have the repairs made, and only notified the insurer when the repairs were done. As a consequence, paragraph 4, which limits the company's liability to P150.00, applies. - The insurance contract may be rather onerous ("one-sided", as the lower court put it), but that in itself does not justify the abrogation of its express terms, terms which the insured accepted or adhered to and which is the law between the contracting parties. - To require the insurer to prove that the cost of the repairs ordered by the insured is unreasonable, when the insurer was not given an opportunity to inspect and assess the damage before the repairs were made, is contrary to elementary justice and equity. SUN INSURANCE OFFICE LTD. V CA (TAN) 195 SCRA 193 PARAS; March 13, 1991 NATURE Petition for certiorari to review the decision of the CA FACTS Private respondent Emilio Tan took from the petitioner a Peso 300,000 property insurance policy to cover his interest in the electrical insurance store of his brother housed in a building in Iloilo City on August 15, 1983. Four days after the issuance of the policy, the building including the insured store burned. - On August 20, 1983, Tan filed his claim for fire loss. Sun Insurance, on February 29, 1984, wrote the private respondent denying the claim. On April 3, 1984, private respondent wrote another letter to the insurance company requesting reconsideration of the denial. Tans lawyer wrote another letter to the insurance company inquiring about the April 3 letter which sought for a reconsideration of the denial. In its reply to the lawyers letter, Sun Insurance reiterated its denial of the claim and enclosed therein copies of the two previous denials dated February 29, 1984 and May 17, 1985. - On November 20, 1985, Tan filed a civil case with the RTC. Petition filed a motion to dismiss on the alleged ground that the action has already prescribed based on Condition 27 of the Insurance Policy which

INSURANCE 10
stated that the window to file the appropriate action with either the Insurance Commission or in any court of competent jurisdiction is twelve months from the rejection of the claim. RTC denied the motion and the subsequent motion for reconsideration. The CA likewise denied the petition of Sun Insurance. ISSUE 1. WON the court the filing of a motion for reconsideration interrupts the 12 months prescription period to contest the denial of the insurance claim 2. WON the rejection of the claim shall be deemed final only if it contains words to the effect that the denial is final HELD 1. NO - The SC held that Condition 27 of the Insurance policy is very clear and free from any doubt or ambiguity. It has to be taken in its plain, ordinary, and popular sense. The rejection letter of February 29, 1984 was clear and plain. The Court noted that the one year period is likewise in accord with Section 23 of the Insurance Code which states that any condition which limits the time for commencing an action to a period of less than one year when the cause of action accrues is void. The right of action, according to the SC, accrues at the time that the claim is rejected at the first instance. A request for reconsideration of the denial cannot suspend the running of the prescriptive period. The Court noted that the rationale for the one year period is to ensure that the evidence as to the origin and cause of the destruction have not yet disappeared. 2. NO - The Court clarified its ruling in Eagle Star Insurance Co. vs Chia Yu where it ruled that the cause of action in an insurance contract does not accrue until the Insureds claim is finally rejected by the Insurer by stating the use of the word finally cannot be construed to mean the rejection of a petition for reconsideration. What the court referred to in effect is the rejection in the first instance as claimed by Sun Insurance Disposition The decision of the CA is reversed and set aside. The case is dismissed FORTUNE INSURANCE AND SURETY CO. INC.V CA (PRODUCERS BANK OF THE PHILIPPINES) 244 SCRA 308 DAVIDE; May 23, 1995 NATURE Petition for Review on certiorari of CA decision FACTS - Producers Bank of the Philippines filed a complaint against Fortune Insurance and Surety Co., Inc. for recovery of P725,000.00 under the policy issued by Fortune. The sum was allegedly lost on June 29, 1987 during a robbery of Producer's armored vehicle while it was in transit to transfer the money from its Pasay City Branch to its head office in Makati under the custody of its teller, Maribeth Alampay. The armored car was driven by Benjamin Magalong Y de Vera, escorted by Security Guard Saturnino Atiga Y Rosete. Driver Magalong was assigned by PRC Management Systems. - After an investigation by the Pasay police, driver Magalong and guard Atiga were charged, together with Batigue , Aquino and John Doe, with violation of P.D. 532 (Anti-Highway Robbery Law) - Demands were made by the Producers upon the Fortune to pay the amount of the loss of P725,000.00 but the latter refused to pay as the loss is excluded from the coverage of the insurance policy specifically under "General Exceptions" > The company shall not be liable under this policy in respect of x x x (b) any loss caused by any dishonest, fraudulent or criminal act of the insured or any officer, employee, partner, director, trustee or authorized representative of the Insured whether acting alone or in conjunction with others. - Fortune opposes the contention of Producers that Atiga and Magalong are not its "officer, employee, x x x trustee or authorized representative x x x at the time of the robbery - Trial Court > On being EMPLOYEES Magalong and Atiga were not employees or representatives of Producers as their services as armored car driver and as security guard having been merely offered by PRC Management and by Unicorn Security and which latter firms assigned them to plaintiff. The wages and salaries of both Magalong and Atiga are presumably paid by their respective firms, which alone wields the power to dismiss them > On being AUTHORIZED REPRESENTATIVE They were merely an assigned armored car driver and security guard for the money transfer. It was teller

Page
Maribeth Alampay who had "custody" of the P725,000.00 cash being transferred along a specified money route - Court of Appeals > affirmed in toto > A policy or contract of insurance is to be construed liberally in favor of the insured and strictly against the insurance company (New Life Enterprises vs. Court of Appeals; Sun Insurance Office, Ltd. vs. Court of Appeals). Contracts of insurance, like other contracts, are to be construed according to the sense and meaning of the terms which the parties themselves have used. If such terms are clear and unambiguous, they must be taken and understood in their plain, ordinary and popular sense (New Life Enterprises Case; Sun Insurance Office). > The language used by Fortune in the policy is plain, ordinary and simple. No other interpretation is necessary. The word "employee" should be taken to mean in the ordinary sense. The Labor Code is a special law specifically dealing with/and specifically designed to protect labor and therefore its definition as to employer-employee relationships insofar as the application/enforcement of said Code is concerned must necessarily be inapplicable to an insurance contract. Had it intended to apply the Labor Code in defining what the word "employee" refers to, it must/ should have so stated expressly in the insurance policy. Said driver and security guard cannot be considered as employees of Producers bank because it has no power to hire or to dismiss said driver and security guard under the contracts except only to ask for their replacements from the contractors. - Fortunes Contention > when Producers commissioned a guard and a driver to transfer its funds from one branch to another, they effectively and necessarily became its authorized representatives in the care and custody of the money. Assuming that they could not be considered authorized representatives, they were, nevertheless, employees of Producers. It asserts that the existence of an employer-employee relationship "is determined by law and being such, it cannot be the subject of agreement." Thus, if there was in reality an employeremployee relationship between Producers, on the one hand, and Magalong and Atiga, on the other, the provisions in the contracts of Producers with PRC Management System for Magalong and with Unicorn Security Services for Atiga which state that Producers is not their employer and that it is absolved from any

INSURANCE 11
liability as an employer, would not obliterate the relationship. > an employer-employee relationship depends upon four standards: (1) the manner of selection and engagement of the putative employee (2) the mode of payment of wages (3) the presence or absence of a power to dismiss and (4) the presence and absence of a power to control the putative employee's conduct. > Of the four, the right-of-control test has been held to be the decisive factor. It asserts that the power of control over Magalong and Atiga was vested in and exercised by Producers. Fortune further insists that PRC Management System and Unicorn Security Services are but "labor-only" contractors under Article 106 of the Labor Code which provides: Art. 106. Contractor or subcontractor. - There is "labor-only" contracting where the person supplying workers to an employer does not have substantial capital or investment in the form of tools, equipment, machineries, work premises, among others, and the workers recruited and placed by such persons are performing activities which are directly related to the principal business of such employer. In such cases, the person or intermediary shall be considered merely as an agent of the employer who shall be responsible to the workers in the same manner and extent as if the latter were directly employed by him. > International Timber Corp. vs. NLRC - a "laboronly" contractor is equivalent to a finding that there is an employer-employee relationship between the owner of the project and the employee of the "laboronly" contractor - Producers Contention > Magalong and Atiga were not its employees since it had nothing to do with their selection and engagement, the payment of their wages, their dismissal, and the control of their conduct. > International Timber Corp. is not applicable to all cases but only when it becomes necessary to prevent any violation or circumvention of the Labor Code, a social legislation whose provisions may set aside contracts entered into by parties in order to give protection to the working man. > American President Lines vs. Clave should be applied which stated In determining the existence of employer-employee relationship, the following elements are generally considered, namely: (1) the selection and engagement of the employee; (2) the payment of wages; (3) the power of dismissal; and (4) the power to control the employee's conduct. - Since under Producers' contract with PRC Management Systems it is the latter which assigned Magalong as the driver of Producers' armored car and was responsible for his faithful discharge of his duties and responsibilities, and since Producers paid the monthly compensation of P1,400.00 per driver to PRC Management Systems and not to Magalong, it is clear that Magalong was not Producers' employee. As to Atiga, Producers relies on the provision of its contract with Unicorn Security Services which provides that the guards of the latter "are in no sense employees of the CLIENT." ISSUE WON Fortune Insurance and Surety Co. Inc. is liable under the Money, Security, and Payroll Robbery policy it issued to Producers Bank of the Philippines or WON recovery is precluded under the general exceptions clause of the policy HELD NO Ratio A contract of insurance is a contract of adhesion, thus any ambiguity therein should be resolved against the insurer, or it should be construed liberally in favor of the insured and strictly against the insurer. Limitations of liability should be regarded with extreme jealousy and must be construed in such a way as to preclude the insurer from non-compliance with its obligation. It goes without saying then that if the terms of the contract are clear and unambiguous, there is no room for construction and such terms cannot be enlarged or diminished by judicial construction. - An insurance contract is a contract of indemnity upon the terms and conditions specified therein. It is settled that the terms of the policy constitute the measure of the insurer's liability. In the absence of statutory prohibition to the contrary, insurance companies have the same rights as individuals to limit their liability and to impose whatever conditions they deem best upon their obligations not inconsistent with public policy. Reasoning - It should be noted that the insurance policy entered into by the parties is a theft or robbery insurance policy which is a form of casualty insurance. Section 174 of the Insurance Code provides:

Page
Sec. 174. Casualty insurance is insurance covering loss or liability arising from accident or mishap, excluding certain types of loss which by law or custom are considered as failing exclusively within the scope of insurance such as fire or marine. It includes, but is not limited to, employer's liability insurance, public liability insurance, motor vehicle liability insurance, plate glass insurance, burglary and theft insurance, personal accident and health insurance as written by non-life insurance companies, and other substantially similar kinds of insurance. (italics supplied) - Except with respect to compulsory motor vehicle liability insurance, the Insurance Code contains no other provisions applicable to casualty insurance or to robbery insurance in particular. These contracts are, therefore, governed by the general provisions applicable to all types of insurance. Outside of these, the rights and obligations of the parties must be determined by the terms of their contract, taking into consideration its purpose and always in accordance with the general principles of insurance law. - With the foregoing principles in mind, it may now be asked whether Magalong and Atiga qualify as employees or authorized representatives has been aptly observed that in burglary, robbery, and theft insurance, "the opportunity to defraud the insurer the moral hazard - is so great that insurers have found it necessary to fill up their policies with countless restrictions, many designed to reduce this hazard. Seldom does the insurer assume the risk of all losses due to the hazards insured against." Persons frequently excluded under such provisions are those in the insured's service and employment. The purpose of the exception is to guard against liability should the theft be committed by one having unrestricted access to the property. In such cases, the terms specifying the excluded classes are to be given their meaning as understood in common speech. The terms "service" and "employment" are generally associated with the idea of selection, control, and compensation. - There is marked disagreement between the parties on the correct meaning of the terms "employee" and "authorized representatives." It is clear to us that insofar as Fortune is concerned, it was its intention to exclude and exempt from protection and coverage losses arising from dishonest, fraudulent, or criminal acts of persons granted or having unrestricted access to Producers' money or payroll. When it used then the term "employee," it must have had in mind any person who qualifies as

INSURANCE 12
such as generally and universally understood, or jurisprudentially established in the light of the four standards in the determination of the employeremployee relationship or as statutorily declared even in a limited sense as in the case of Article 106 of the Labor Code which considers the employees under a "labor-only" contract as employees of the party employing them and not of the party who supplied them to the employer. - But even granting for the sake of argument that these contracts were not "labor-only" contracts, and PRC Management Systems and Unicorn Security Services were truly independent contractors, we are satisfied that Magalong and Atiga were, in respect of the transfer of Producer's money from its Pasay City branch to its head office in Makati, its "authorized representatives" who served as such with its teller Maribeth Alampay. Howsoever viewed, Producers entrusted the three with the specific duty to safely transfer the money to its head office, with Alampay to be responsible for its custody in transit; Magalong to drive the armored vehicle which would carry the money; and Atiga to provide the needed security for the money, the vehicle, and his two other companions. In short, for these particular tasks, the three acted as agents of Producers. A "representative" is defined as one who represents or stands in the place of another; one who represents others or another in a special capacity, as an agent, and is interchangeable with "agent." Disposition instant petition is hereby GRANTED. CA decision and RTC Makati decision are REVERSED and SET ASIDE. Civil Case is DISMISSED. VERENDIA V CA (FIDELITY & SURETY CO. OF THE PHILS) 217 SCRA 417 MELO; January 22, 1993 NATURE Petition to review decision of the CA FACTS - Fidelity Co. issued a Fire Insurance Policy covering Verendias residential building in the amount of P385k. Verendia also insured the same building with two other companies (Country Bankers Insurance for P56k, and Development Insurance for P400k). - While all 3 policies were in force, the insured property was completely destroyed by fire. Verendia filed a claim against Fidelity, but the latter refused payment, thus a complaint was filed in the RTC. Fidelitys reason for refusal: the policy was avoided by reason of over-insurance, and that Verendia maliciously represented that the building was under lease to a Roberto Garcia, when it was actually a Marcelo Garcia who was the lessee. - RTC: policy was violated by Verendia when it failed to inform Fidelity of his other insurance coverages, thus no need to pay. - CA: reversed decision ISSUE (There is a procedural issue involved here, but is irrelevant to our discussion. It concerns the filing of a motion for extension of time to file a motion for reconsideration, where the court said that although it now prohibits filing of such motion, the instant motion was filed before the effectivity of this rule, thus allowing the adjudication of the case) WON Fidelity was liable to pay Verendia considering the circumstances HELD 1. NO Ratio As the insurance contract is the law between the parties, Verendia is deemed to have forfeited his right to claim by the misrepresentation he made. Reasoning - the court reviewed the factual findings of the courts below, since it appears that there was a misapprehension of the facts by the CA. - Verendia is found to have concocted the lease contract to deflect responsibility for the fire towards an alleged lessee, even making it appear that the alleged lessee had disappeared, inflated the value of the property, and insured same property with two other companies. - An insurance contract is the law between the parties, its terms and conditions constitute the measure of the insurers liability and compliance therewith is a condition precedent to the insureds right to recovery from the insurer. - As it is also a contract of adhesion, an insurance contract should be liberally construed in favor of the insured and strictly against the insurer company which usually prepares it. - Considering, however, the fact that Verendia used a false lease contract to support his claim, the terms of the policy should be strictly construed against the insured. Verendia failed to live by the terms of the

Page
policy, specifically Section 13 thereof which is expressed in terms that are clear and unambiguous, that all benefits under the policy shall be forfeited If the claim be in any respect fraudulent, or if any false declaration be made or used in support thereof, or if any fraudulent means or devises are used by the Insured or anyone acting in his behalf to obtain any benefit under the policy. Verendia, having presented a false declaration to support his claim for benefits in the form of a fraudulent lease contract, he forfeited all benefits therein by virtue of Section 13 of the policy in the absence of proof that Fidelity waived such provision. Worse yet, by presenting a false lease contract, Verendia reprehensibly disregarded the principle that insurance contracts are uberrimae fidae and demand the most abundant good faith. Disposition Decision of CA reversed, and that of RTC is reinstated. FIELDMEN'S INSURANCE CO. INC V VDA. DE SONGCO 25 SCRA 20 FERNANDO; 1968 FACTS - An insurance firm, petitioner Fieldmen's Insurance Co., Inc., was not allowed to escape liability under a common carrier insurance policy on the pretext that what was insured, not once but twice, was a private vehicle and not a common carrier, the policy being issued upon the insistence of its agent who discounted fears of the insured that his privately owned vehicle might not fall within its terms, the insured moreover being "a man of scant education," finishing only the first grade. So it was held in a decision of the lower court thereafter affirmed by respondent Court of Appeals. Petitioner in seeking the review of the above decision of respondent Court of Appeals cannot be so sanguine as to entertain the belief that a different outcome could be expected. To be more explicit, we sustain the Court of Appeals. - The facts as found by respondent Court of Appeals, binding upon us, follow: "This is a peculiar case. Federico Songco of Floridablanca, Pampanga, a man of scant education being only a first grader ..., owned a private jeepney with Plate No. 41-289 for the year 1960. On September 15, 1960, as such private vehicle owner, he was induced by Fieldmen's Insurance Company Pampanga agent Benjamin Sambat to apply for a Common Carrier's Liability Insurance Policy covering his motor vehicle ... Upon paying an annual premium of P16.50, defendant

INSURANCE 13
Fieldmen's Insurance Company, Inc. issued on September 19, 1960, Common Carriers Accident Insurance Policy No. 45-HO- 4254 ... the duration of which will be for one (1) year, effective September 15, 1960 to September 15, 1961. On September 22, 1961, the defendant company, upon payment of the corresponding premium, renewed the policy by extending the coverage from October 15, 1961 to October 15, 1962. This time Federico Songco's private jeepney carried Plate No. J-68136-Pampanga1961. ... On October 29, 1961, during the effectivity of the renewed policy, the insured vehicle while being driven by Rodolfo Songco, a duly licensed driver and son of Federico (the vehicle owner) collided with a car in the municipality of Calumpit, province of Bulacan, as a result of which mishap Federico Songco (father) and Rodolfo Songco (son) died, Carlos Songco (another son), the latter's wife, Angelita Songco, and a family friend by the name of Jose Manuel sustained physical injuries of varying degree." 1 - It was further shown according to the decision of respondent Court of Appeals: "Amor Songco, 42-yearold son of deceased Federico Songco, testifying as witness, declared that when insurance agent Benjamin Sambat was inducing his father to insure his vehicle, he butted in saying: 'That cannot be, Mr. Sambat, because our vehicle is an "owner" private vehicle and not for passengers,' to which agent Sambat replied: 'whether our vehicle was an "owner" type or for passengers it could be insured because their company is not owned by the Government and the Government has nothing to do with their company. So they could do what they please whenever they believe a vehicle is insurable' ... In spite of the fact that the present case was filed and tried in the CFI of Pampanga, the defendant company did not even care to rebut Amor Songco's testimony by calling on the witness-stand agent Benjamin Sambat, its Pampanga Field Representative." 2 - The plaintiffs in the lower court, likewise respondents here, were the surviving widow and children of the deceased Federico Songco as well as the injured passenger Jose Manuel. On the above facts they prevailed, as had been mentioned, in the lower court and in the respondent Court of Appeals.1awphl.nt - The basis for the favorable judgment is the doctrine announced in Qua Chee Gan v. Law Union and Rock Insurance Co., Ltd., 3 with Justice J. B. L. Reyes speaking for the Court. It is now beyond question that where inequitable conduct is shown by an insurance firm, it is "estopped from enforcing forfeitures in its favor, in order to forestall fraud or imposition on the insured." 4 - As much, if not much more so than the Qua Chee Gan decision, this is a case where the doctrine of estoppel undeniably calls for application. After petitioner Fieldmen's Insurance Co., Inc. had led the insured Federico Songco to believe that he could qualify under the common carrier liability insurance policy, and to enter into contract of insurance paying the premiums due, it could not, thereafter, in any litigation arising out of such representation, be permitted to change its stand to the detriment of the heirs of the insured. As estoppel is primarily based on the doctrine of good faith and the avoidance of harm that will befall the innocent party due to its injurious reliance, the failure to apply it in this case would result in a gross travesty of justice. - That is all that needs be said insofar as the first alleged error of respondent Court of Appeals is concerned, petitioner being adamant in its far-fromreasonable plea that estoppel could not be invoked by the heirs of the insured as a bar to the alleged breach of warranty and condition in the policy. lt would now rely on the fact that the insured owned a private vehicle, not a common carrier, something which it knew all along when not once but twice its agent, no doubt without any objection in its part, exerted the utmost pressure on the insured, a man of scant education, to enter into such a contract. - Nor is there any merit to the second alleged error of respondent Court that no legal liability was incurred under the policy by petitioner. Why liability under the terms of the policy 5 was inescapable was set forth in the decision of respondent Court of Appeals. Thus: "Since some of the conditions contained in the policy issued by the defendant-appellant were impossible to comply with under the existing conditions at the time and 'inconsistent with the known facts,' the insurer 'is estopped from asserting breach of such conditions.' From this jurisprudence, we find no valid reason to deviate and consequently hold that the decision appealed from should be affirmed. The injured parties, to wit, Carlos Songco, Angelito Songco and Jose Manuel, for whose hospital and medical expenses the defendant company was being made liable, were passengers of the jeepney at the time of the occurrence, and Rodolfo Songco, for whose burial expenses the defendant company was also being made liable was the driver of the vehicle in question. Except for the fact, that they were not fare paying

Page
passengers, their status as beneficiaries under the policy is recognized therein." 6 - Even if it be assumed that there was an ambiguity, an excerpt from the Qua Chee Gan decision would reveal anew the weakness of petitioner's contention. Thus: "Moreover, taking into account the well known rule that ambiguities or obscurities must be strictly interpreted against the party that caused them, the 'memo of warranty' invoked by appellant bars the latter from questioning the existence of the appliances called for in the insured premises, since its initial expression, 'the undernoted appliances for the extinction of fire being kept on the premises insured hereby, ... it is hereby warranted ...,' admits of interpretation as an admission of the existence of such appliances which appellant cannot now contradict, should the parol evidence rule apply." 7 - To the same effect is the following citation from the same leading case: "This rigid application of the rule on ambiguities has become necessary in view of current business practices. The courts cannot ignore that nowadays monopolies, cartels and concentration of capital, endowed with overwhelming economic power, manage to impose upon parties dealing with them cunningly prepared 'agreements' that the weaker party may not change one whit, his participation in the 'agreement' being reduced to the alternative to 'take it or leave it' labelled since Raymond Saleilles 'contracts by adherence' (contrats d'adhesion), in contrast to those entered into by parties bargaining on an equal footing, such contracts (of which policies of insurance and international bills of lading are prime examples) obviously call for greater strictness and vigilance on the part of courts of justice with a view to protecting the weaker party from abuses and imposition, and prevent their becoming traps for the unwary (New Civil Code. Article 24; Sent. of Supreme Court of Spain, 13 Dec. 1934, 27 February 1942)." 8 - The last error assigned which would find fault with the decision of respondent Court of Appeals insofar as it affirmed the lower court award for exemplary damages as well as attorney's fees is, on its face, of no persuasive force at all. - The conclusion that inescapably emerges from the above is the correctness of the decision of respondent Court of Appeals sought to be reviewed. For, to borrow once again from the language of the Qua Chee Gan opinion: "The contract of insurance is one of perfect good faith (uberima fides) not for the insured alone,but equally so for the insurer; in fact, it is more

INSURANCE 14
so for the latter, since its dominant bargaining position carries with it stricter responsibility." 9 - This is merely to stress that while the morality of the business world is not the morality of institutions of rectitude like the pulpit and the academe, it cannot descend so low as to be another name for guile or deception. Moreover, should it happen thus, no court of justice should allow itself to lend its approval and support.1awphl.nt - We have no choice but to recognize the monetary responsibility of petitioner Fieldmen's Insurance Co., Inc. It did not succeed in its persistent effort to avoid complying with its obligation in the lower court and the Court of Appeals. Much less should it find any receptivity from us for its unwarranted and unjustified plea to escape from its liability. MALAYAN INSURANCE MARKETING CORP.) 270 SCRA 242 ROMERO; March 20, 1997 CORP. V CA (TKC However the cargo was sold in Durban, South Africa, for US$154.40 per metric ton or a total of P10,304,231.75 due to its perishable nature which could no longer stand a voyage of twenty days to Manila and another twenty days for the discharge thereof. It reduced its claim to US$448,806.09 (or its peso equivalent of P9,879,928.89 at the exchange rate of P22.0138 per $1.00) representing its loss after the proceeds of the sale were deducted from the original claim.Malayan maintained its position that the arrest of the vessel by civil authorities on a question of ownership was an excepted risk under the marine insurance policies. Petitioners Claim - an arrest by civil authority is not compensable since the term "arrest" refers to "political or executive acts" and does not include a loss caused by riot or by ordinary judicial process as in this case - the deletion of the Free from Capture or Seizure Clause would leave the assured covered solely for the perils specified by the wording of the policy itself - the rationale for the exclusion of an arrest pursuant to judicial authorities is to eliminate collusion between unscrupulous assured and civil authorities. - any loss which private respondent may have incurred was in the nature and form of unrecovered acquisition value brought about by a voluntary sacrifice sale and not by arrest, detention or seizure of the ship. - its act of rejecting the claim was a result of its honest belief that the arrest of the vessel was not a compensable risk under the policies issued Respondents Comments - petitioner, being the sole author of the policies, "arrests" should be strictly interpreted against it because the rule is that any ambiguity is to be taken contra proferentum. Risk policies should be construed reasonably and in a manner as to make effective the intentions and expectations of the parties. - the policies clearly stipulate that they cover the risks of non-delivery of an entire package and that it was petitioner itself that invited and granted the extensions and collected premiums thereon. ISSUES 1. WON the arrest of the vessel was a risk covered under the subject insurance policies 2. WON insurance policies should be strictly construed against the insurer HELD

Page
1.YES - With the incorporation of subsection 1.1 of Section 1 of the Institute War Clauses, "arrest" caused by ordinary judicial process is deemed included among the covered risks. This interpretation becomes inevitable when subsection 1.1 of Section 1 of the Institute War Clauses provided that "this insurance covers the risks excluded from the Standard Form of English Marine Policy by the clause 'Warranted free of capture, seizure, arrest, etc. x x x'" or the F.C. & S. Clause. Jurisprudentially, "arrests" caused by ordinary judicial process is also a risk excluded from the Standard Form of English Marine Policy by the F.C. & S. Clause. - Petitioner cannot adopt the argument that the "arrest" caused by ordinary judicial process is not included in the covered risk simply because the F.C. & S. Clause under the Institute War Clauses can only be operative in case of hostilities or warlike operations on account of its heading "Institute War Clauses." 2. YES Ratio Insurance Policies should be construed liberally in favor of the insured and strictly against the insurer. Reasoning - An insurance contract should be so interpreted as to carry out the purpose for which the parties entered into the contract which is, to insure against risks of loss or damage to the goods. Such interpretation should result from the natural and reasonable meaning of language in the policy. Where restrictive provisions are open to two interpretations, that which is most favorable to the insured is adopted. Indemnity and liability insurance policies are construed in accordance with the general rule of resolving any ambiguity therein in favor of the insured, where the contract or policy is prepared by the insurer. A contract of insurance, being a contract of adhesion, par excellence, any ambiguity therein should be resolved against the insurer.Limitations of liability should be regarded with extreme jealousy and must be construed in such a way as to preclude the insurer from noncompliance with its obligations - It must be borne in mind that such contracts are invariably prepared by the companies and must be accepted by the insured in the form in which they are written. Any construction of a marine policy rendering it void should be avoided. Such policies will, therefore, be construed strictly against the company in order to avoid a forfeiture, unless no other result is possible from the language used.

NATURE Petition for review on certiorari FACTS - TKC Marketing Corp. was the owner/consignee of some 3,189.171 metric tons of soya bean meal which was loaded on board the ship MV Al Kaziemah for carriage from the port of Rio del Grande, Brazil, to the port of Manila. Said cargo was insured against the risk of loss by petitioner Malayan Insurance Corporation for which it issued two (2) Marine Cargo Policies. - While the vessel was docked in Durban, South Africa the civil authorities arrested and detained it because of a lawsuit on a question of ownership and possession. TKC Marketing notified Malayan of the arrest of the vessel and made a formal claim for the dollar equivalent on the policies (US$916,886.66) for non-delivery of the cargo. It likewise sought the assistance of Malayan on what to do with the cargo. - Malayan replied that the arrest of the vessel by civil authority was not a peril covered by the policies. TKC advised Malayan that it might tranship the cargo and requested an extension of the insurance coverage until actual transhipment, which extension was approved upon payment of additional premium. The insurance coverage was extended under the same terms and conditions embodied in the original policies while in the process of making arrangements for the transhipment of the cargo from Durban to Manila.

INSURANCE 15
- If a marine insurance company desires to limit or restrict the operation of the general provisions of its contract by special proviso, exception, or exemption, it should express such limitation in clear and unmistakable language. Be that as it may, exceptions to the general coverage are construed most strongly against the company. Even an express exception in a policy is to be construed against the underwriters by whom the policy is framed, and for whose benefit the exception is introduced. WESTERN GUARANTY CORPORATION V CA (RODRIGUEZ, and DE DIOS TRANSPORTATION CO) 187 SCRA 652 FELICIANO; July 20, 1990 FACTS - At around 4:30 in the afternoon of 27 March 1982, while crossing Airport Road on a pedestrian lane on her way to work, respondent Priscilla E. Rodriguez was struck by a De Dios passenger bus owned by respondent De Dios Transportation Co., Inc., then driven by one Walter Saga y Aspero. The bus driver disregarded the stop signal given by a traffic policeman to allow pedestrians to cross the road. Priscilla was thrown to the ground, hitting her forehead. She was treated at the Protacio Emergency Hospital and later on hospitalized at the San Juan De Dios Hospital. Her face was permanently disfigured, causing her serious anxiety and moral distress. - Respondent bus company was insured with petitioner Western Guaranty Corporation ("Western") under its Master Policy which enumerated specific liabilities of the insurance company and ended with a clause to clarify the limitations of the amount which could be granted as indemnity. - Respondent Priscilla Rodriguez filed a complaint for damages before the Regional Trial Court of Makati against De Dios Transportation Co. and Walter A. Saga. Respondent De Dios Transportation Co., in turn, filed a third-party complaint against its insurance carrier, petitioner Western. - On 6 August 1985, the trial court rendered a decision in favor of respondent Priscilla E. Rodriguez, - On appeal, the Court of Appeals affirmed in toto the decision of the trial court. Petitioner moved for the reconsideration of the appellate court's decision. In a Resolution dated 10 January 1990, the Court of Appeals denied the motion for reconsideration for lack of merit. Petitioner Western is now before us on a Petition for Review alleging that the Court of Appeals erred in holding petitioner liable to pay beyond the limits set forth in the Schedule Indemnities and in finding Western liable for loss of earnings, moral damages and attorney's fees. Succinctly stated, it is petitioner Western's position that it cannot be held liable for loss of earnings, moral damages and attorney's fees because these items are not among those included in the Schedule Indemnities set forth in the insurance policy. - Petitioner Western in effect contends before this Court, as it did before the Court of Appeals, that because the Schedule of Indemnities limits the amount payable for certain kinds of expenses "hospital room", "surgical expenses", "an aesthesiologists' fee", "operating room" and "medical expenses" that Schedule should be read as excluding liability for any other type of expense or damage or loss even though actually sustained or incurred by the third party victim. We are not persuaded by Western's contention. ISSUE WON the Schedule of indemnities as stated in the insurance policy should be construed strictly to exclude all others not explicitly stated therein

Page
present. It appears to us self-evident that the Schedule of Indemnities was not intended to be an enumeration, much less a closed enumeration, of the specific kinds of damages which may be awarded under the Master Policy Western has issued. - Secondly, the reading urged by Western of the Schedule of Indemnities comes too close to working fraud upon both the insured and the third party beneficiary of Section 1, quoted above. For Western's reading would drastically and without warning limit the otherwise unlimited (save for the over-all quantitative limit of liability of P50,000.00 per person per accident) and comprehensive scope of liability assumed by the insurer Western under Section 1: "all sums necessary to discharge liability of the insured in respect of [bodily injury to a third party]". This result which is not essentially different from taking away with the left hand what had been given with the right hand we must avoid as obviously repugnant to public policy. If what Western now urges is what Western intended to achieve by its Schedule of Indemnities, it was incumbent upon Western to use language far more specific and precise than that used in fact by Western, so that the insured, and potential purchasers of its Master Policy, and the Office of the Insurance Commissioner, may be properly informed and act accordingly. - Petitioner Western would have us construe the Schedule of Indemnities as comprising contractual limitations of liability which, as already noted, is comprehensively defined in Section 1 "Liability to the Public" of the Master Policy. It is well-settled, however, that contractual limitations of liability found in insurance contracts should be regarded by courts with a jaundiced eye and extreme care and should be so construed as to preclude the insurer from evading compliance with its just obligations. - Finally, an insurance contract is a contract of adhesion. The rule is well entrenched in our jurisprudence that the terms of such contract are to be construed strictly against the party which prepared the contract, which in this case happens to be petitioner Western. QUA CHEE GAN V LAW UNION AND ROCK INSURANCE CO., LTD. 96 PHIL 85 REYES; December 17, 1955 NATURE

HELD NO Ratio An insurance policy being in the nature of an adhesion contract is to be strictly construed against the insurer and liberally in favor of the insured. Reasoning - Firstly, the Schedule of Indemnities does not purport to restrict the kinds of damages that may be awarded against Western once liability has arisen. Section 1, quoted above, does refer to certain "Limits of Liability" which in the case of the third party liability section of the Master Policy, is apparently P50,000.00 per person per accident. Within this over-all quantitative limit, all kinds of damages allowable by law "actual or compensatory damages"; "moral damages"; "nominal damages"; "temperate or moderate damages"; "liquidated damages"; and "exemplary damages" may be awarded by a competent court against the insurer once liability is shown to have arisen, and the essential requisites or conditions for grant of each species of damages are

INSURANCE 16
An appeal by defendant insurance company from the decision of CFI in favor of the plaintiff FACTS - before the last war, plaintiff-appellee owned 4 warehouses or bodegas in Tabaco, Albay, used for the storage of stocks of copra and of hemp, baled and loose, in which the appellee dealt extensively. They had been, with their contents, insured with the defendant Company since 1937, and the loose made payable to the Philippine National Bank as mortgage of the hemp and crops, to the extent of its interest. - Fire of undetermined origin that broke out in the early morning of July 21, 1940, and lasted almost one week, gutted and completely destroyed Bodegas Nos. 1, 2 and 4, with the merchandise stored therein. Plaintiff-appellee informed the insurer by telegram on the same date; and on the next day, the fire adjusters engaged by appellant insurance company arrived and proceeded to examine and photograph the premises, pored over the books of the insured and conducted an extensive investigation. The plaintiff having submitted the corresponding fire claims, totalling P398,562.81 (but reduced to the full amount of the insurance, P370,000), the Insurance Company resisted payment, claiming violation of warranties and conditions, filing of fraudulent claims, and that the fire had been deliberately caused by the insured or by other persons in connivance with him. - Que Chee Gan, with his brother, Qua Chee Pao, and some employees of his, were indicted and tried in 1940 for the crime of arson, it being claimed that they had set fire to the destroyed warehouses to collect the insurance. They were, however, acquitted by the trial court. - the civil suit to collect the insurance money proceeded to its trial with the CFI holding that: judgment is rendered for the plaintiff and against the defendant condemning the latter to pay the former (a) Under the first cause of action, the sum of P146,394.48; (b) Under the second cause of action, the sum of P150,000; (c) Under the third cause of action, the sum of P5,000; (d) Under the fourth cause of action, the sum of P15,000; and (e) Under the fifth cause of action, the sum of P40,000; all of which shall bear interest at the rate of 8% per annum in accordance with Section 91 (b) of the Insurance Act from September 26, 1940, until each is paid, with costs against the defendant. - In its first assignment of error, the insurance company alleges that the trial Court should have held that the policies were avoided for breach of warranty, specifically the one appearing on a rider pasted (with other similar riders) on the face of the policies.4 - It is argued that since the bodegas insured had an external wall perimeter of 500 meters or 1,640 feet, the appellee should have 11 fire hydrants in the compound, and that he actually had only 2, with a further pair nearby, belonging to the municipality of Tabaco. ISSUES 1. WON the defendant-appellant can claim the policies it had issued as void ab initio 2. WON the insured violated the "Hemp Warranty" provisions of Policy No. 2637165 against the storage of gasoline 3. WON the insured connived at the loss and fraudulently inflated the quantity of the insured stock in the burnt bodegas HELD 1. NO Ratio It is usually held that where the insurer, at the time of the issuance of a policy of insurance, has knowledge of existing facts which, if insisted on, would invalidate the contract from its very inception, such knowledge constitutes a waiver of conditions in the contract inconsistent with the facts, and the insurer is stopped thereafter from asserting the breach of such conditions. The law is charitable enough to assume, in the absence of any showing to the contrary, that an insurance company intends to executed a valid contract in return for the premium received; and when the policy contains a condition which renders it voidable at its inception, and this result is known to the insurer, it will be presumed to have intended to waive the conditions and to execute
4
Memo. of Warranty. The undernoted Appliances for the extinction of fire being kept on the premises insured hereby, and it being declared and understood that there is an ample and constant water supply with sufficient pressure available at all seasons for the same, it is hereby warranted that the said appliances shall be maintained in efficient working order during the currency of this policy, by reason whereof a discount of 2 1/2 per cent is allowed on the premium chargeable under this policy. Hydrants in the compound, not less in number than one for each 150 feet of external wall measurement of building, protected, with not less than 100 feet of hose piping and nozzles for every two hydrants kept under cover in convenient places, the hydrants being supplied with water pressure by a pumping engine, or from some other source, capable of discharging at the rate of not less than 200 gallons of water per minute into the upper story of the highest building protected, and a trained brigade of not less than 20 men to work the same.'

Page
a binding contract, rather than to have deceived the insured into thinking he is insured when in fact he is not, and to have taken his money without consideration. Reasoning - The appellant is barred estoppel to claim violation of the so-called fire hydrants warranty, for the reason that knowing fully all that the number of hydrants demanded therein never existed from the very beginning, the appellant neverthless issued the policies in question subject to such warranty, and received the corresponding premiums. The insurance company was aware, even before the policies were issued, that in the premises insured there were only two fire hydrants installed by Qua Chee Gan and two others nearby, owned by the municipality of Tabaco, contrary to the requirements of the warranty in question - The plain, human justice of this doctrine is perfectly apparent. To allow a company to accept one's money for a policy of insurance which it then knows to be void and of no effect, though it knows as it must, that the assured believes it to be valid and binding, is so contrary to the dictates of honesty and fair dealing, and so closely related to positive fraud, as to the abhorrent to fair-minded men. It would be to allow the company to treat the policy as valid long enough to get the premium on it, and leave it at liberty to repudiate it the next moment. This cannot be deemed to be the real intention of the parties. To hold that a literal construction of the policy expressed the true intention of the company would be to indict it, for fraudulent purposes and designs which we cannot believe it to be guilty of. - The appellant company so worded the policies that while exacting the greater number of fire hydrants and appliances, it kept the premium discount at the minimum of 2 1/2%, thereby giving the insurance company a double benefit. Such abnormal treatment of the insured strongly points at an abuse of the insurance company's selection of the words and terms of the contract, over which it had absolute control. - Receipt of Premiums or Assessments after Cause for Forfeiture Other than Nonpayment. It is a well settled rule of law that an insurer which with knowledge of facts entitling it to treat a policy as no longer in force, receives and accepts a premium on the policy, estopped to take advantage of the forfeiture. It cannot treat the policy as void for the

INSURANCE 17
purpose of defense to an action to recover for a loss thereafter occurring and at the same time treat it as valid for the purpose of earning and collecting further premiums. - Moreover, taking into account the well known rule that ambiguities or obscurities must be strictly interpreted against the party that caused them, the "memo of warranty" invoked by appellant bars the latter from questioning the existence of the appliances called for in the insured premises On the alleged violations of the plaintiff The alleged violation of the warranty of 100 feet of fire hose for every two hydrants, must be equally rejected, since the appellant's argument thereon is based on the assumption that the insured was bound to maintain no less than eleven hydrants, which requirement appellant is estopped from enforcing. - As to maintenance of a trained fire brigade of 20 men, the record is preponderant that the same was organized, and drilled, from time to give, although not maintained as a permanently separate unit, which the warranty did not require. 2. NO Ratio Here, again, by reason of the exclusive control of the insurance company over the terms and phraseology of the contract, the ambiguity must be held strictly against the insurer and liberally in favor of the insured, specially to avoid a forfeiture. Insurance is, in its nature, complex and difficult for the layman to understand. Policies are prepared by experts who know and can anticipate the hearing and possible complications of every contingency. So long as insurance companies insist upon the use of ambiguous, intricate and technical provisions, which conceal rather than frankly disclose, their own intentions, the courts must, in fairness to those who purchase insurance, construe every ambiguity in favor of the insured. An insurer should not be allowed, by the use of obscure phrases and exceptions, to defeat the very purpose for which the policy was procured. Reasoning - Appellee admitted that there were 36 cans of gasoline in the building designed. It However, gasoline is not specifically mentioned among the prohibited articles listed in the so-called "hemp warranty." The cause relied upon by the insurer speaks of "oils (animal and/or vegetable and/or mineral and/or their liquid products having a flash point below 300 Fahrenheit)", and is decidedly ambiguous and uncertain; for in ordinary parlance, "Oils" mean "lubricants" and not gasoline or kerosene. And how many insured, it may well be wondered, are in a position to understand or determine "flash point below 300 Fahrenheit. - If the company intended to rely upon a condition of that character, it ought to have been plainly expressed in the policy. - The contract of insurance is one of perfect good faith not for the insured alone, but equally so for the insurer; in fact, it is mere so for the latter, since its dominant bargaining position carries with it stricter responsibility. - Another point that is in favor of the insured is that the gasoline kept in Bodega No. 2 was only incidental to his business, being no more than a customary 2 day's supply for the five or six motor vehicles used for transporting of the stored merchandise. "It is well settled that the keeping of inflammable oils on the premises though prohibited by the policy does not void it if such keeping is incidental to the business." On the submission of books, voucbers, etc. The charge that the insured failed or refused to submit to the examiners of the insurer the books, vouchers, etc. demanded by them was found unsubstantiated by the trial Court, and no reason has been shown to alter this finding. The insured gave the insurance examiner all the date he asked for, and the examiner even kept and photographed some of the examined books in his possession. What does appear to have been rejected by the insured was the demand that he should submit "a list of all books, vouchers, receipts and other records", but the refusal of the insured in this instance was well justified, since the demand for a list of all the vouchers (which were not in use by the insured) and receipts was positively unreasonable, considering that such listing was superfluous because the insurer was not denied access to the records, that the volume of Qua Chee Gan's business ran into millions, and that the demand was made just after the fire when everything was in turmoil. That the representatives of the insurance company were able to secure all the date they needed is proved by the fact that the adjuster Alexander Stewart was able to prepare his own balance sheet that did not differ from that submitted by the insured except for the valuation of the merchandise, as expressly found by the Court in the criminal case for arson. 3. NO

Page
Ratio Both defenses are predicted on the assumption that the insured was in financial difficulties and set the fire to defraud the insurance company, presumably in order to pay off the Philippine National Bank, to which most of the insured hemp and copra was pledged. Both defenses are fatally undermined by the established fact that, notwithstanding the insurer's refusal to pay the value of the policies the extensive resources of the insured enabled him to pay off the National Bank in a short time; and if he was able to do so, no motive appears for attempt to defraud the insurer. While the acquittal of the insured in the arson case is not res judicata on the present civil action, the insurer's evidence, to judge from the decision in the criminal case, is practically identical in both cases and must lead to the same result, since the proof to establish the defense of connivance at the fire in order to defraud the insurer "cannot be materially less convincing than that required in order to convict the insured of the crime of arson." - As to the defense that the burned bodegas could not possibly have contained the quantities of copra and hemp stated in the fire claims, the insurer's case rests almost exclusively on the estimates, inferences and conclusions of its adjuster investigator who examined the premises during and after the fire. His testimony, however, was based on inferences from the photographs and traces found after the fire, and must yield to the contradictory testimony of those who actually saw the contents of the bodegas shortly before the fire, while inspecting them for the mortgagee Bank. Disposition We find no reversible error in the judgment appealed from, wherefore the same is hereby affirmed. DEL ROSARIO V EQUITABLE INSURANCE & CASUALTY CO., INC 8 SCRA 343 PAREDES; June 29, 1963 NATURE Appeal from judgment of CFI Rizal FACTS - Francisco del Rosario was insured by Equitable Insurance and Casualty Co. Inc under Personal Accident Policy no. 7136. The Company bound itself to pay P1000 to P3000 as indemnity for the death of the insured. - Under the policy:

INSURANCE 18
Part I. Indemnity for Death If the insured sustains any bodily injury which is effected solely through violent, external, visible and accidental means, and which shall result, independently of all other causes and within sixty days from the occurrence thereof, in the Death of the Insured, the Company shall pay the amount set opposite such injury: Section 1. Injury sustained other than those specified below unless excepted hereinafter P1000 Section 2. Injury sustained by the wrecking or disablement of a railroad passenger car or street railway car in or on which the Insured is traveling as a farepaying passenger P1500 Section 3. Injury sustained by the burning of a church, theatre, public library or municipal administration building while the Insured is therein at the commencement of the fire P2000 Section 4. Injury sustained by the wrecking or disablement of a regular passenger elevator car in which the Insured is being conveyed as a passenger (Elevator in mines exluded) P2500 Section 5. Injury sustained by a stroke of lightning or by a cyclone P3000 xxxx xxxx xxxx Part VI. Exceptions This policy shall not cover disappearance of the Insured nor shall it cover Death, Disability, Hospital fees, or Loss of time, caused to the insured: x x x (h) By drowning except as a consequence of the wrecking or disablement in the Philippine waters of a passenger steam or motor vessel in which the Insured is traveling as a farepaying passenger; x x x - A rider to the Policy contained the following; IV. DROWNING It is hereby declared and agreed that exemption clause Letter (h) in PART VI of the policy is hereby waived by the company, and to form a part of the provision covered by the policy. - Feb 24, 1957, Francisco del Rosario while on board the motor launch ISLAMA, with his beneficiary to the policy, Remedios Jayme, were forced to jump off said launch on account of fire which broke out on said vessel, resulting in the death by drowning of the insured and his beneficiary. - Simeon del Rosario, the insureds father, filed a claim for payment with the company. The company paid him P1000 pursuant to section 1 Part I of the policy. - On the same date, Atty. Francisco wrote to the company acknowledging receipt by his client of the P1000 but informing said company that said amount was not the correct one. He claimed that the amount payable should be P1500 under the provision of Section 2 Part I, based on the rule of pari materia. - The company referred the matter to the Insurance Coomissioner, who was of the opinion that the liability of the company was only P1000. thus the company refused to pay more that P1000. Atty. Francisco wrote a subsequent letter to company asking for p3000, which the company refused to pay. - A complaint for recovery of the balance of P2000 was instituted with the CFI Rizal, praying for a further sum of P10000 as attorneys fees, expenses of litigation and costs. - CFI ruled in favor of petitioner, ordering the company to pay P2000 to del Rosario. ISSUE How much should the indemnity be HELD - All the parties agree that indemnity has to be paid, but the conflict centers on how much it should be. - Where there is ambiguity with respect to the terms and conditions of the policy, the same will be resolved against the one responsible thereof. Generally, the insured has little, if any, participation in the preparation of the policy, together with the drafting of its terms and conditions. The interpretation of obscure stipulations in a contract should not favor the party who caused the obscurity. - SC agreed with the ruling of the lower court: x x x death by drowning is a ground for recovery apart from the bodily injury because death by bodily injury is covered by Part I of the policy while death by drowning is covered by Part VI thereof. But while the policy mentions specific amounts that may be recovered for death for bodily injury, yet, there is not specific amount mentioned in the policy for death thru drowning although the latter is, under Part VI of the policy,

Page
a ground for recovery thereunder. Since the defendant has bound itself to pay P1000 to P3000 as indemnity for the death of the insured but the policy does not positively state any definite amount that may be recovered in case of death by drowning, there is an ambiguity in this respect in the policy, which ambiguity must be interpreted in favor of the insured and strictly against the insurer so as to allow a greater indemnity. x x x plaintiff is therefore entitled to recover P3000. Disposition Judgment appealed from is affirmed. GEAGONIA v. CA INSURANCE) 8 SCRA 343 DAVIDE; February 6 1995 (COUNTRY BANKERS

FACTS -Geagonia is the owner of Norman's Mart located in the public market of San Francisco, Agusan del Sur. On 22 Dec 1989, he obtained from the private respondent fire insurance policy for P100,000.00. The period of the policy was from 22 Dec 1989 to 22 Dec 1990 and covered the ff: "Stock-in-trade consisting principally of dry goods such as RTW's for men and women wear and other usual to assured's business. -The policy contained the following condition: "3. The insured shall give notice to the Company of any insurance or insurances already effected, or which may subsequently be effected, covering any of the property or properties consisting of stocks in trade, goods in process and/or inventories only hereby insured, and unless notice be given and the particulars of such insurance or insurances be stated therein or endorsed in this policy pursuant to Section 50 of the Insurance Code, by or on behalf of the Company before the occurrence of any loss or damage, all benefits under this policy shall be deemed forfeited, provided however, that this condition shall not apply when the total insurance or insurances in force at the time of the loss or damage is not more than P200,000.00." -On 27 May 1990, fire of accidental origin broke out at around 7:30 p.m. at the public market of San Francisco, Agusan del Sur. The petitioner's insured stocks-in-trade were completely destroyed prompting him to file w/ the private respondent a claim under the policy. On 28 Dec 1990, the private respondent denied the claim because it found that at the time of the loss the petitioner's stocks-in-trade were likewise

INSURANCE 19
covered by two fire insurance policies for P100,000.00 each, issued by the Cebu Branch of the Philippines First Insurance Co., Inc. (PFIC). -The basis of the private respondent's denial was the petitioner's alleged violation of Condition 3 of the policy. - Geagonia then filed a complaint against the private respondent w/ the Insurance Commission for the recovery of P100,000.00 under fire insurance policy, for attorney's fees, and costs of litigation. He claims that the time he obtained the private respondent's fire insurance policy he knew that the two policies issued by the PFIC were already in existence; however, he had no knowledge of the provision in the private respondent's policy requiring him to inform it of the prior policies; this requirement was not mentioned to him by the private respondent's agent; and had it been so mentioned, he would not have withheld such information. He further asserted that the total of the amounts claimed under the three policies was below the actual value of his stocks at the time of loss, w/c was P1M. - The Insurance Commission found that the petitioner did not violate Condition 3 as he had no knowledge of the existence of the two fire insurance policies obtained from the PFIC; that it was Cebu Tesing Textiles w/c procured the PFIC policies w/o informing him or securing his consent; and that Cebu Tesing Textile, as his creditor, had insurable interest on the stocks. These findings were based on the petitioner's testimony that he came to know of the PFIC policies only when he filed his claim with the private respondent and that Cebu Tesing Textile obtained them and paid for their premiums w/o informing him. The Insurance Commission then ordered the respondent company to pay complainant the sum of P100,000.00 with legal interest from the time the complaint was filed until fully satisfied plus the amount of P10,000.00 as attorney's fees. -CA reversed the decision of the Insurance Commission because it found that the petitioner knew of the existence of the two other policies issued by the PFIC ISSUES 1. WON the petitioner had prior knowledge of the two insurance policies issued by the PFIC when he obtained the fire insurance policy from the private respondent, thereby, for not disclosing such fact, violating Condition 3 of the policy 2. if he had, WON he is precluded from recovering therefrom HELD 1. YES - We agree w/ the CA that the petitioner knew of the prior policies issued by the PFIC. His letter of 18 January 1991 to the private respondent conclusively proves this knowledge. His testimony to the contrary before the Insurance Commissioner and which the latter relied upon cannot prevail over a written admission made ante litem motam. It was, indeed, incredible that he did not know about the prior policies since these policies were not new or original. 2. NO - It must, however, be underscored that unlike the "other insurance" clauses involved in General Insurance and Surety Corp. vs. Ng Hua or in Pioneer Insurance & Surety Corp. vs. Yap, which read: "The insured shall give notice to the company of any insurance or insurances already effected, or which may subsequently be effected covering any of the property hereby insured, and unless such notice be given and the particulars of such insurance or insurances be stated in or endorsed on this Policy by or on behalf of the Company before the occurrence of any loss or damage, all benefits under this Policy shall be forfeited." or in the 1930 case of Santa Ana vs. Commercial Union Assurance Co. which provided "that any outstanding insurance upon the whole or a portion of the objects thereby assured must be declared by the insured in writing and he must cause the company to add or insert it in the policy, without which such policy shall be null and void, and the insured will not be entitled to indemnity in case of loss," Condition 3 in the private respondent's policy No. F-14622 does not absolutely declare void any violation thereof. It expressly provides that the condition "shall not apply when the total insurance or insurances in force at the time of the loss or damage is not more than P200,000.00." - Interpretation: It is a cardinal rule on insurance that a policy or insurance contract is to be interpreted liberally in favor of the insured and strictly against the company, the reason being, undoubtedly, to afford the greatest protection which the insured was endeavoring to secure when he applied for insurance. It is also a cardinal principle of law that forfeitures are not favored and that any construction which would result in the forfeiture of the policy benefits for the

Page
person claiming thereunder, will be avoided, if it is possible to construe the policy in a manner which would permit recovery, as, for example, by finding a waiver for such forfeiture. Stated differently, provisions, conditions or exceptions in policies which tend to work a forfeiture of insurance policies should be construed most strictly against those for whose benefits they are inserted, and most favorably toward those against whom they are intended to operate. The reason for this is that, except for riders which may later be inserted, the insured sees the contract already in its final form and has had no voice in the selection or arrangement of the words employed therein. On the other hand, the language of the contract was carefully chosen and deliberated upon by experts and legal advisers who had acted exclusively in the interest of the insurers and the technical language employed therein is rarely understood by ordinary laymen. - With these principles in mind, we are of the opinion that Condition 3 of the subject policy is not totally free from ambiguity and must be meticulously analyzed. Such analysis leads us to conclude that (a) the prohibition applies only to double insurance, and (b) the nullity of the policy shall only be to the extent exceeding P200,000.00 of the total policies obtained. - Furthermore, by stating within Condition 3 itself that such condition shall not apply if the total insurance in force at the time of loss does not exceed P200,000.00, the private respondent was amenable to assume a co-insurer's liability up to a loss not exceeding P200,000.00. What it had in mind was to discourage over-insurance. Indeed, the rationale behind the incorporation of "other insurance" clause in fire policies is to prevent over-insurance and thus avert the perpetration of fraud. When a property owner obtains insurance policies from two or more insurers in a total amount that exceeds the property's value, the insured may have an inducement to destroy the property for the purpose of collecting the insurance. The public as well as the insurer is interested in preventing a situation in which a fire would be profitable to the insured. Disposition Petition granted. The decision of the Court of Appeals in CA-G.R. SP No. 31916 is SET ASIDE and the decision of the Insurance Commission in Case No. 3340 is REINSTATED. SUN INSURANCE OFFICE, LTD. V CA (LIM)

INSURANCE 20
211 SCRA 554 CRUZ; July 17, 1992 NATURE Petition for review from the decision of the Court of Appeals FACTS - Felix Lim was issued a Personal Accident Policy insurance with petitioner company with a face value of P200,000. His beneficiary was his wife Nerissa. - October 6, 1982 Felix accidentally shot himself in the head with his own gun. - He was playing with the handgun after he had removed the guns magazine (kasi naman). - He pointed the gun at his secretary and only witness Pilar Nalagon as a joke and assured her that the gun was not loaded (are you sure). - He then put the gun to his temple and fired it (haaay, sabi ko na nga ba). - Both parties are in agreement that there was no suicide. - Nerissa claimed as Felixs beneficiary but Sun Insurance would not grant her claim, saying that her husbands death was not an accident. - Nerissa sued Sun Insurance and won the case. Sun Insurance was ordered to pay her P200,000 representing the face value of the claim along with moral, exemplary and compensatory damages and attorneys fees. The decision was affirmed by the CA. Petitioners Claim - Sun Insurance cites one of the four exceptions in the contract of insurance which includes bodily injury consequent upon the insured person attempting to commit suicide or willfully exposing himself to needless peril in an attempt to save a human life. - There mere act of pointing the gun to his temple showed that Felix willfully exposed himself to danger because a gun should always be handled with caution. Respondents Comments - Felix believed the gun to be safe because he had removed the magazine. - He repeatedly assured his secretary that the gun was not loaded. ISSUES 1. WON Felix Lims death was an accident, thus making his widow Nerissa liable to claim the accident insurance 2. WON the award of damages to Nerissa Lim was justified HELD 1. YES, Felix Lims death was an accident. Ratio There is no accident when a deliberate act is performed unless some additional, unexpected, independent and unforeseen happening occurs which produces or brings bout their injury or death. Reasoning - An accident has been defined to be that which happens by chance or fortuitously without intention or design and which is unexpected, unusual and unforeseen. It an event that takes pace without ones foresight or expectastion an event that proceeds from an unknown cause or is an unusual effect of a known case and therefore not expected. It happens without any human agency, an event which, under the circumstances, is unusual to and not expected by the person to whom it happens. - The firing of the gun was deemed to be the unexpected and independent and unforeseen occurrence that led to the insured persons death. - There was no willful exposure to needless peril for the part of Felix. Suicide and exposure to needless peril are similar in the sense that both signify disregard for ones life. Suicide imparts a positive act of ending ones life whereas the latter indicates recklessness that is almost suicidal in intent. - Accident insurance policies were never meant to reward the insured for his tendency to show off or for his miscalculations. They were intended to provide for contingencies. - Lim was unquestionably negligent but it should not prevent his widow from recovering from the insurance policy he obtained precisely against accident. - Insurance contracts are, as a rule, supposed to be interpreted liberally in favor of the assured. 2. NO, the claim for damages should not be granted for being unjust. Ratio A person may be made liable to the payment of moral damages if his act is wrongful. The adverse result of an action does not per se make the act wrongful and subject the act or to the payment of moral damages. Reasoning - Petitioner was acting in good faith when it resisted the private respondents claim on the ground that the death of the insured was covered by the exception. - The issue was debatable and was clearly not raised only for the purpose of evading a legitimate obligation.

Page
RIZAL SURETY & INSURANCE COMPANY V CA (TRANSWORLD KNITTING MILLS, INC.) 336 SCRA 12 PURISIMA; July 18, 2000 NATURE Petition for Review on Certiorari under Rule 45 of the Rules of Court FACTS - Rizal Surety & Insurance Company (Rizal Insurance) issued Fire Insurance Policy No. 45727 in favor of Transworld Knitting Mills, Inc. (Transworld). - Pertinent portions of subject policy on the buildings insured, and location thereof, read: "On stocks of finished and/or unfinished products, raw materials and supplies of every kind and description, the properties of the Insureds and/or held by them in trust, on commission or on joint account with others and/or for which they (sic) responsible in case of loss whilst contained and/or stored during the currency of this Policy in the premises occupied by them forming part of the buildings situate (sic) within own Compound at MAGDALO STREET, BARRIO UGONG, PASIG, METRO MANILA, PHILIPPINES, BLOCK NO. 601. x xx............ ...xxx....... ........xxx Said building of four-span lofty one storey in height with mezzanine portions is constructed of reinforced concrete and hollow blocks and/or concrete under galvanized iron roof and occupied as hosiery mills, garment and lingerie factory, transistor-stereo assembly plant, offices, warehouse and caretaker's quarters. 'Bounds in front partly by one-storey concrete building under galvanized iron roof occupied as canteen and guardhouse, partly by building of two and partly one storey constructed of concrete below, timber above undergalvanized iron roof occupied as garage and quarters and partly by open space and/or tracking/ packing, beyond which is the aforementioned Magdalo Street; on its right and left by driveway, thence open spaces, and at the rear by open spaces.'"

INSURANCE 21
- The same pieces of property insured with the petitioner were also insured with New India Assurance Company, Ltd., (New India). - Fire broke out in the compound of Transworld, razing the middle portion of its four-span building and partly gutting the left and right sections thereof. A two-storey building (behind said fourspan building) where fun and amusement machines and spare parts were stored, was also destroyed by the fire. - Transworld filed its insurance claims with Rizal Surety & Insurance Company and New India Assurance Company but to no avail. - Private respondent brought against the said insurance companies an action for collection of sum of money and damages. - Petitioner Rizal Insurance countered that its fire insurance policy sued upon covered only the contents of the four-span building, which was partly burned, and not the damage caused by the fire on the two-storey annex building. - The trial court dismissed the case as against The New India Assurance Co., Ltd. but ordered defendant Rizal Surety And Insurance Company to pay Transwrold (sic) Knitting Mills, Inc. - Both the petitioner, Rizal Insurance Company, and private respondent, Transworld Knitting Mills, Inc., went to the Court of Appeals, which required New India Assurance Company to pay plaintiff-appellant the amount of P1,818,604.19 while the Rizal Surety has to pay the plaintiff-appellant P470,328.67. - New India appealed to the Court theorizing inter alia that the private respondent could not be compensated for the loss of the fun and amusement machines and spare parts stored at the two-storey building because it (Transworld) had no insurable interest in said goods or items. - The Court denied the appeal with finality. - Petitioner Rizal Insurance and private respondent Transworld, interposed a Motion for Reconsideration before the Court of Appeals, which reconsidered its decision of July 15, 1993, as regards the imposition of interest. - Undaunted, petitioner Rizal Surety & Insurance Company found its way to the Court. ISSUE WON the fire insurance policy litigated upon protected only the contents of the main building (four-span), and did not include those stored in the two-storey annex building HELD NO - Resolution of the issue posited hinges on the proper interpretation of the stipulation in subject fire insurance policy regarding its coverage, which reads: "xxx contained and/or stored during the currency of this Policy in the premises occupied by them forming part of the buildings situate (sic) within own Compound xxx" - It can be gleaned unerringly that the fire insurance policy in question did not limit its coverage to what were stored in the four-span building. As opined by the trial court of origin, two requirements must concur in order that the said fun and amusement machines and spare parts would be deemed protected by the fire insurance policy under scrutiny, to wit: "First, said properties must be contained and/or stored in the areas occupied by Transworld and second, said areas must form part of the building described in the policy xxx" - Said building of four-span lofty one storey in height with mezzanine portions is constructed of reinforced concrete and hollow blocks and/or concrete under galvanized iron roof and occupied as hosiery mills, garment and lingerie factory, transistor-stereo assembly plant, offices, ware house and caretaker's quarter. - The Court is mindful of the well-entrenched doctrine that factual findings by the Court of Appeals are conclusive on the parties and not reviewable by this Court, and the same carry even more weight when the Court of Appeals has affirmed the findings of fact arrived at by the lower court. - In the case under consideration, both the trial court and the Court of Appeals found that the so called "annex " was not an annex building but an integral and inseparable part of the four-span building described in the policy and consequently, the machines and spare parts stored therein were covered by the fire insurance in dispute. - Verily, the two-storey building involved, a permanent structure which adjoins and intercommunicates with the "first right span of the lofty storey building", formed part thereof, and meets the requisites for compensability under the fire insurance policy sued upon. - So also, considering that the two-storey building aforementioned was already existing when subject fire insurance policy contract was entered into, petitioner should have specifically excluded the said two-storey

Page
building from the coverage of the fire insurance if minded to exclude the same but if did not, and instead, went on to provide that such fire insurance policy covers the products, raw materials and supplies stored within the premises of respondent Transworld which was an integral part of the four-span building occupied by Transworld, knowing fully well the existence of such building adjoining and intercommunicating with the right section of the fourspan building. - Indeed, the stipulation as to the coverage of the fire insurance policy under controversy has created a doubt regarding the portions of the building insured thereby. Article 1377 of the New Civil Code provides: "Art.1377. The interpretation of obscure words or stipulations in a contract shall not favor the party who caused the obscurity" - Conformably, it stands to reason that the doubt should be resolved against the petitioner, Rizal Surety Insurance Company, whose lawyer or managers drafted the fire insurance policy contract under scrutiny. Citing the aforecited provision of law in point, the Court in Landicho vs. Government Service Insurance System, ruled: "This is particularly true as regards insurance policies, in respect of which it is settled that the 'terms in an insurance policy, which are ambiguous, equivocal, or uncertain x x x are to be construed strictly and most strongly against the insurer, and liberally in favor of the insured so as to effect the dominant purpose of indemnity or payment to the insured, especially where forfeiture is involved' and the reason for this is that the 'insured usually has no voice in the selection or arrangement of the words employed and that the language of the contract is selected with great care and deliberation by experts and legal advisers employed by, and acting exclusively in the interest of, the insurance company.' " - Equally relevant is the following disquisition of the Court in Fieldmen's Insurance Company, Inc. vs. Vda. De Songco, to wit: "'This rigid application of the rule on ambiguities has become necessary in view of current business practices. The courts cannot ignore that nowadays monopolies, cartels and concentration of capital, endowed with overwhelming economic power, manage to impose upon parties dealing with them cunningly prepared 'agreements' that the weaker party may not change one whit, his

INSURANCE 22
participation in the 'agreement' being reduced to the alternative to 'take it or leave it' labelled since Raymond Saleilles 'contracts by adherence' (contrats [sic] d'adhesion), in contrast to these entered into by parties bargaining on an equal footing, such contracts (of which policies of insurance and international bills of lading are prime example) obviously call for greater strictness and vigilance on the part of courts of justice with a view to protecting the weaker party from abuses and imposition, and prevent their becoming traps for the unwary.'" - The issue of whether or not Transworld has an insurable interest in the fun and amusement machines and spare parts, which entitles it to be indemnified for the loss thereof, had been settled in G.R. No. L111118, entitled New India Assurance Company, Ltd., vs. Court of Appeals, where the appeal of New India from the decision of the Court of Appeals under review, was denied with finality by this Court on February 2, 1994. - The rule on conclusiveness of judgment, which obtains under the premises, precludes the relitigation of a particular fact or issue in another action between the same parties based on a different claim or cause of action. "xxx the judgment in the prior action operates as estoppel only as to those matters in issue or points controverted, upon the determination of which the finding or judgment was rendered. In fine, the previous judgment is conclusive in the second case, only as those matters actually and directly controverted and determined and not as to matters merely involved therein." Disposition Decision, and the Resolution of the CA WERE AFFIRMED in toto. No pronouncement as to costs. PAN MALAYAN INSURANCE CORPORATION vs. COURT OF APPEALS (ERLINDA FABIE & HER UNKNOWN DRIVER) 184 SCRA 55; G.R. No. 81026 CORTES; April 3, 1990 FACTS - December 10, 1985: PANMALAY filed a complaint for damages with the RTC of Makati against private respondents Erlinda Fabie and her driver. PANMALAY averred the following: that it insured a Mitsubishi Colt Lancer car registered in the name of Canlubang Automotive Resources Corporation [CANLUBANG]; that on May 26, 1985, due to the "carelessness, recklessness, and imprudence" of the unknown driver of a pick-up, the insured car was hit and suffered damages in the amount of P42,052.00; that PANMALAY defrayed the cost of repair of the insured car and, therefore, was subrogated to the rights of CANLUBANG against the driver of the pick-up and his employer, Erlinda Fabie; and that, despite repeated demands, defendants, failed and refused to pay the claim of PANMALAY. PANMALAY clarified that the damage caused to the insured car was settled under the "own damage", coverage of the insurance policy. - Private respondents filed a Motion to Dismiss alleging that PANMALAY had no cause of action against them. RTC dismissed PANMALAY's complaint for no cause of action and denied PANMALAY's motion for reconsideration. CA affirmed. Hence, this petition for review. ISSUE WON the insurer PANMALAY may institute an action to recover the amount it had paid its assured in settlement of an insurance claim against private respondents as the parties allegedly responsible for the damage caused to the insured vehicle HELD YES - Article 2207 of the Civil Code is founded on the wellsettled principle of subrogation. If the insured property is destroyed or damaged through the fault or negligence of a party other than the assured, then the insurer, upon payment to the assured, will be subrogated to the rights of the assured to recover from the wrongdoer to the extent that the insurer has been obligated to pay. - General Rule: Payment by the insurer to the assured operates as an equitable assignment to the former of all remedies which the latter may have against the third party whose negligence or wrongful act caused the loss. The right of subrogation is not dependent upon, nor does it grow out of, any privity of contract or upon written assignment of claim. It accrues simply upon payment of the insurance claim by the insurer. - Exceptions: a.) if the assured by his own act releases the wrongdoer or third party liable for the loss or damage, from liability b.) where the insurer pays the assured the value of the lost goods without notifying the carrier who has in good faith settled the assured's claim for loss

Page
c.) where the insurer pays the assured for a loss which is not a risk covered by the policy, thereby effecting "voluntary payment" - None of the exceptions are availing in the present case. - When PANMALAY utilized the phrase "own damage" a phrase which, incidentally, is not found in the insurance policy to define the basis for its settlement of CANLUBANG's claim under the policy, it simply meant that it had assumed to reimburse the costs for repairing the damage to the insured vehicle. - It is a basic rule in the interpretation of contracts that the terms of a contract are to be construed according to the sense and meaning of the terms which the parties thereto have used. In the case of property insurance policies, the evident intention of the contracting parties, i.e., the insurer and the assured, determine the import of the various terms and provisions embodied in the policy. It is only when the terms of the policy are ambiguous, equivocal or uncertain, such that the parties themselves disagree about the meaning of particular provisions, that the courts will intervene. In such an event, the policy will be construed by the courts liberally in favor of the assured and strictly against the insurer. - Considering that the very parties to the policy were not shown to be in disagreement regarding the meaning and coverage of Section III-1, specifically sub-paragraph (a) thereof, it was improper for the appellate court to indulge in contract construction, to apply the ejusdem generis rule, and to ascribe meaning contrary to the clear intention and understanding of these parties. - Although the terms "accident" or "accidental" as used in insurance contracts have not acquired a technical meaning, the Court has on several occasions defined these terms to mean that which takes place "without one's foresight or expectation, an event that proceeds from an unknown cause, or is an unusual effect of a known cause and, therefore, not expected." The concept "accident" is not necessarily synonymous with the concept of "no fault". It may be utilized simply to distinguish intentional or malicious acts from negligent or careless acts of man. - Obiter Dicta: Even if under the above circumstances PANMALAY could not be deemed subrogated to the rights of its assured under Article 2207 of the Civil Code, PANMALAY would still have a cause of action against private respondents. The insurer who may have no rights of subrogation due to "voluntary" payment may nevertheless recover from the third

INSURANCE 23
party responsible for the damage to the insured property under Article 1236 of the Civil Code. Disposition Petition is GRANTED. Petitioner's complaint for damages against private respondents is REINSTATED. Case remanded to the lower court for trial on the merits. AMERICAN HOME ASSURANCE TANTUCO ENTERPRISES 366 SCRA 740 PUNO; October 8, 2001 COMPANY V ISSUE WON new oil mill is insured by fire insurance policy HELD YES, new oil mill is insured. Ratio In construing the words used descriptive of a building insured, the greatest liberality is shown by the courts in giving effect to the insurance. In view of the custom of insurance agents to examine buildings before writing policies upon them, and since a mistake as to the identity and character of the building is extremely unlikely, the courts are inclined to consider that the policy of insurance covers any building which the parties manifestly intended to insure, however inaccurate the description may be. Reasoning - The parties manifestly intended to insure the new oil mill. On machineries and equipment with complete accessories usual to a coconut oil mill including stocks of copra, copra cake and copra mills whilst contained in the new oil mill building, situate (sic) at UNNO. ALONG NATIONAL HIGH WAY, BO. IYAM, LUCENA CITY UNBLOCKED. - If the parties really intended to protect the first oil mill, then there is no need to specify it as new. It would be absurd to assume that respondent would protect its first oil mill for different amounts and leave uncovered its second one. - As may be gleaned from the testimony of the petitioners employee, the source of the discrepancy happened during the preparation of the written contract. - Respondent is not estopped from claiming that the policy description is wrong. Evidence on record reveals that respondents operating manager, Mr. Edison Tantuco, notified the petitioners agent with whom respondent negotiated for the contract about the inaccurate description in the policy. However, Mr. Borja assured Mr. Tantuco that the use of the adjective new will distinguish the insured property. Regarding policy requirements that fire extinguishment appliances should be available and in good working condition, warranty did not require respondent to provide for all the fire extinguishing appliances enumerated therein. Neither did it require that the appliances are restricted to those mentioned in the warranty. (Within the vicinity of the new oil mill can be found the following devices: numerous portable fire extinguishers, two fire hoses, fire hydrant, and an emergency fire engine.)

Page
- The object of the court in construing a contract is to ascertain the intent of the parties to the contract and to enforce the agreement which the parties have entered into. In determining what the parties intended, the courts will read and construe the policy as a whole and if possible, give effect to all the parts of the contract. Disposition Petition is dismissed. PERLA COMPANIA DE SEGUROS, INC. v, CA (MILAGROS CAYAS) 185 SCRA 741 FERNAN; May 28, 1990 NATURE Petition for review on certiorari of a decision of the Court of Appeals FACTS - Private respondent Milagros Cayas was the registered owner of a Mazda bus, insured with Perla Compania de Seguros, Inc. (PCSI) under a policy issued on February 3, 1978. - On December 17, 1978, the bus figured in an accident in Naic, Cavite injuring several of its passengers. - One of them, 19-year old Edgardo Perea, sued Milagros Cayas for damages in the CFI of Cavite, while three others agreed to a settlement of P4,000.00 each. - After trial, the court rendered a decision in favor of Perea, ordering Cayas to compensate him, with an award of exemplary and moral damages, as well as attorneys fees. ( P32,000 total) - On November 11, 1981, Milagros Cayas filed a complaint for a sum of money and damages against PCSI in the Court of First Instance of Cavite. - In view of Milagros Cayas' failure to prosecute the case, the court motu propio ordered its dismissal without prejudice. - Alleging that she had not received a copy of the answer to the complaint, and that "out of sportsmanship", she did not file a motion to hold PCSI in default, Milagros Cayas moved for the reconsideration of the dismissal order. Said motion for reconsideration was acted upon favorably by the court. - About two months later, Milagros Cayas filed a motion to declare PCSI in default for its failure to file an answer.

NATURE Petition for Review on Certiorari assailing the Decision of the Court of Appeals. FACTS - Respondent Tantuco Enterprises, Inc. is engaged in the coconut oil milling and refining industry. It owns two oil mills. Both are located at its factory compound at Iyam, Lucena City. Respondent commenced its business operations with only one oil mill. In 1988, it started operating its second oil mill ( the new oil mill). - The two oil mills were separately covered by fire insurance policies issued by petitioner American Home Assurance Co. The first oil mill was insured Policy No. 306-7432324-3 for the period March 1, 1991 to 1992.The new oil mill was insured under Policy No. 306-7432321-9 for the same term. Official receipts indicating payment for the full amount of the premium were issued by the petitioner's agent. - Policy description: Front: by a driveway thence at 18 meters distance by Bldg. No. 2. Right: by an open space thence by Bldg. No. 4. Left: Adjoining thence an imperfect wall by Bldg. No. 4. Rear: by an open space thence at 8 meters distance. - A fire that broke out in the early morning of September 30,1991 gutted and consumed the new oil mill. Petitioner rejected respondents claim for the insurance proceeds on the ground that no policy was issued by it covering the burned oil mill. It stated that the description of the insured establishment referred to another building. Petitioners Claim The policies referred to the old mill, as stated in the description contained in the policy.

INSURANCE 24
- The motion was granted and plaintiff was allowed to adduce evidence ex-parte. - On July 13, 1982, the court rendered judgment by default ordering PCSI to pay Milagros Cayas P50,000 as compensation for the injured passengers, P5,000 as moral damages and P5,000 as attorney's fees. - Said decision was set aside after the PCSI filed a motion therefor. Trial of the case ensued. - In due course, the court promulgated a decision ordering defendant Perla Compania de Seguros, Inc. to pay plaintiff Milagros Cayas the sum of P50,000.00 under its maximum liability as provided for in the insurance policy; and the sum of P5,000.00 as reasonable attorney's fee - PCSI appealed to the Court of Appeals, which affirmed in toto the lower court's decision. - Its motion for reconsideration having been denied by said appellate court, PCSI filed this petition ISSUE WON, as maintained by petitioner, its liability is limited only to the payment made by private respondent to Perea and only up to the amount of P12,000.00 HELD YES - The insurance policy involved explicitly limits petitioner's liability to P12,000.00 per person and to P50,000.00 per accident. - In Stokes vs. Malayan Insurance Co., Inc., the Court held that the terms of the contract constitute the measure of the insurer's liability and compliance therewith is a condition precedent to the insured's right of recovery from the insurer. - In the case at bar, the insurance policy clearly and categorically placed petitioner's liability for all damages arising out of death or bodily injury sustained by one person as a result of any one accident at P12,000.00. - Said amount complied with the minimum fixed by the law then prevailing, Section 377 of Presidential Decree No. 612, which provided that the liability of land transportation vehicle operators for bodily injuries sustained by a passenger arising out of the use of their vehicles shall not be less than P12,000. - In other words, under the law, the minimum liability is P12,000 per passenger. Petitioner's liability under the insurance contract not being less than P12,000.00, and therefore not contrary to law, morals, good customs, public order or public policy, said stipulation must be upheld as effective, valid and binding as between the parties. - In like manner, we rule as valid and binding upon private respondent the condition requiring her to secure the written permission of petitioner before effecting any payment in settlement of any claim against her. - There is nothing unreasonable, arbitrary or objectionable in this stipulation as would warrant its nullification. The same was obviously designed to safeguard the insurer's interest against collusion between the insured and the claimants. - It being specifically required that petitioner's written consent be first secured before any payment in settlement of any claim could be made, private respondent is precluded from seeking reimbursement of the payments made to the three other passangers in view of her failure to comply with the condition contained in the insurance policy. - Clearly, the fundamental principle that contracts are respected as the law between the contracting parties finds application in the present case. - It was error on the part of the trial and appellate courts to have disregarded the stipulations of the parties and to have substituted their own interpretation of the insurance policy. - In Phil. American General Insurance Co., Inc vs. Mutuc, we ruled that contracts which are the private laws of the contracting parties should be fulfilled according to the literal sense of their stipulations, if their terms are clear and leave no room for doubt as to the intention of the contracting parties, for contracts are obligatory, no matter what form they may be, whenever the essential requisites for their validity are present. - In Pacific Oxygen & Acetylene Co. vs. Central Bank," it was stated that the first and fundamental duty of the courts is the application of the law according to its express terms, interpretation being called for only when such literal application is impossible. - We observe that although Milagros Cayas was able to prove a total loss of only P44,000.00, petitioner was made liable for the amount of P50,000.00, the maximum liability per accident stipulated in the policy. This is patent error. An insurance indemnity, being merely an assistance or restitution insofar as can be fairly ascertained, cannot be availed of by any accident victim or claimant as an instrument of enrichment by reason of an accident. Disposition Petition granted. The decision of the Court of Appeals is modified in that petitioner shall

Page
pay Milagros Cayas the amount of Twelve Thousand Pesos (P12,000. 00) plus legal interest from the promulgation of the decision of the lower court until it is fully paid and attorney's fees in the amount of P5,000.00. POLTAN v. BPI & JOHN DOE G.R. No. 164307 CHICO-NAZARIO; March 5, 2007 NATURE Petition for review of CA decision FACTS - Petitioners POLTAN obtained a loan evidenced by a promissory note from the MANTRADE Devt Corp. This was secured by a chattel mortgage over a 1-unit Nissan Sentra vehicle. - With notice to petitioners, MANTRADE assigned to BPI, by way of a Deed of Assignment, all its rights, title and interest to the promissory note and chattel mortgage. - Petitioners defaulted and so BPI demanded the whole balance of P286.5T including accrued interest, or to return to BPI the possession of the motor vehicle for foreclosure. It is specifically provided in the promissory note and chattel mortgage that failure to pay any installment when due shall make subsequent installments and the entire balance of the obligation due and payable. - After they refused to do so, BPI then filed complaint. - Petitioners claimed that BPI required them to obtain a motor vehicle insurance policy from FGU Insurance Corporation (FGU Insurance). This is a sister company of BPI. They had been paying the monthly installments on the vehicle until it figured in an accident where it became a total wreck. Under the terms of the insurance policy from FGU Insurance, the vehicle had to be replaced or its value paid to them. Due to the failure and refusal of FGU Insurance to replace the vehicle or pay its value, they stopped payment of the monthly installments. - RTC ordered POLTANS to pay BPI the said amount. - CA reversed and remanded case to RTC for trial on the merits. - RTC again ruled in favor of BPI. CA affirmed. ISSUES 1. WON contracts presented in evidence by BPI were unjust and unacceptable contracts of adhesion

INSURANCE 25
2. WON the terms and conditions of the comprehensive car insurance policy issued by FGU should be deemed as having automatically operated in favor of BPI as the assured mortgagee, and if so, it should be deemed as resulting in the extinguishment of petitioners obligation HELD 1. NO Ratio A contract of adhesion is one in which one of the parties imposes a ready-made form of contract, which the other party may accept or reject, but which the latter cannot modify. It is just as binding as ordinary contracts. Reasoning - Petitioners failed to show that they were under duress or forced to sign the loan documents. The natural presumption is that one does not sign a document without first informing himself of its contents and consequences. - Contracts of adhesion are not entirely prohibited even as the courts remain careful in scrutinizing the factual circumstances underlying each case to determine the respective claims of contending parties on their efficacy. 2. NO Reasoning - Petitioners failed to show any provision in the insurance policy or mortgage contract providing that the loss of the mortgaged vehicle extinguishes their principal obligation to BPI. - While it is true that the proceeds from the insurance policy over the mortgaged chattel is for the benefit of BPI, this will result in partial or full satisfaction of the obligation only if the insurer pays the mortgagee, BPI, or if the insurance proceeds were paid to BPI. In this case, upon the loss of the vehicle due to total wreck, the petitioners filed a claim under the insurance policy, collected and received the proceeds thereof, but did not settle their obligation with BPI which remained outstanding despite the loss of the vehicle. Disposition CA decision AFFIRMED with the modification that the interest rate be reduced to 12% per annum from 24 May 1994 until fully paid, and the award of attorneys fees be reduced to P50T. FILIPINO MERCHANTS INS. v. CA (CHOA TIEK SENG) 179 SCRA 638 REGALADO; November 28, 1989 NATURE Review of the decision of the CA FACTS - Plaintiff insured said shipment with defendant insurance company under said cargo for the goods described as 600 metric tons of fishmeal in new gunny bags of 90 kilos each from Bangkok, Thailand to Manila against all risks under warehouse to warehouse terms. - Some of the goods arrived in bad condition. Plaintiff made a claim against Filipino Merchants Insurance Company. The latter refused to pay. Plaintiff brought an action against them. The defendant insurance company presented a third party complaint against the vessel and the arrastre contractor. - Judgment was rendered against the insurance company. On the third party complaint, the third party defendants were ordered to pay the third party plaintiffs. The CA affirmed, but modified the same with regard to the adjudication of the third-party complaint ISSUES 1. WON some fortuity, casualty or accidental cause is needed to be proved despite the all risks policy (as asserted by the insurance company) 2. WON the respondent has an insurable interest HELD 1. NO - The very nature of the term "all risks" must be given a broad and comprehensive meaning as covering any loss other than a willful and fraudulent act of the insured. 7 This is pursuant to the very purpose of an "all risks" insurance to give protection to the insured in those cases where difficulties of logical explanation or some mystery surround the loss or damage to property. - Generally, the burden of proof is upon the insured to show that a loss arose from a covered peril, but under an "all risks" policy the burden is not on the insured to prove the precise cause of loss or damage for which it seeks compensation. The insured under an "all risks insurance policy" has the initial burden of proving that the cargo was in good condition when the policy attached and that the cargo was damaged when unloaded from the vessel; thereafter, the burden then shifts to the insurer to show the exception to the coverage. As we held in Paris-Manila Perfumery Co. vs. Phoenix Assurance Co., Ltd. the basic rule is that

Page
the insurance company has the burden of proving that the loss is caused by the risk excepted and for want of such proof, the company is liable. In the present case, there being no showing that the loss was caused by any of the excepted perils, the insurer is liable under the policy. 2. YES - Section 13 of the Insurance Code defines insurable interest in property as every interest in property, whether real or personal, or any relation thereto, or liability in respect thereof, of such nature that a contemplated peril might directly damnify the insured. In principle, anyone has an insurable interest in property who derives a benefit from its existence or would suffer loss from its destruction whether he has or has not any title in, or lien upon or possession of the property y. 16 Insurable interest in property may consist in (a) an existing interest; (b) an inchoate interest founded on an existing interest; or (c) an expectancy, coupled with an existing interest in that out of which the expectancy arises. - Respondents interest over the goods is based on the perfected contract of sale. The perfected contract of sale between him and the shipper of the goods operates to vest in him an equitable title even before delivery or before be performed the conditions of the sale. - Further, Article 1523 of the Civil Code provides that where, in pursuance of a contract of sale, the seller is authorized or required to send the goods to the buyer, delivery of the goods to a carrier, whether named by the buyer or not, for, the purpose of transmission to the buyer is deemed to be a delivery of the goods to the buyer, the exceptions to said rule not obtaining in the present case. The Court has heretofore ruled that the delivery of the goods on board the carrying vessels partake of the nature of actual delivery since, from that time, the foreign buyers assumed the risks of loss of the goods and paid the insurance premium covering them - Moreover, the issue of lack of insurable interest was not raised in petitioners answer. Disposition Petition denied GAISANO CAGAYAN v. INSURANCE NORTH AMERICA 490 SCRA 296 Austria-Martinez; June 8, 2006 NATURE Co. OF

INSURANCE 26
Petition for review on certiorari of the Decision of the Court of Appeals FACTS - Intercapitol Marketing Corporation (IMC) is the maker of Wrangler Blue Jeans. Levi Strauss (Phils.) Inc. (LSPI) is the local distributor of products bearing trademarks owned by Levi Strauss & Co.. IMC and LSPI separately obtained from respondent fire insurance policies with book debt endorsements. The insurance policies provide for coverage on book debts in connection with ready-made clothing materials which have been sold or delivered to various customers and dealers of the Insured anywhere in the Philippines. The policies defined book debts as the unpaid account still appearing in the Book of Account of the Insured 45 days after the time of the loss covered under this Policy. The policies also provide for the following conditions: 1. Warranted that the Company shall not be liable for any unpaid account in respect of the merchandise sold and delivered by the Insured which are outstanding at the date of loss for a period in excess of six (6) months from the date of the covering invoice or actual delivery of the merchandise whichever shall first occur. 2. Warranted that the Insured shall submit to the Company within twelve (12) days after the close of every calendar month all amount shown in their books of accounts as unpaid and thus become receivable item from their customers and dealers. x x x - Petitioner is a customer and dealer of the products of IMC and LSPI. On February 25, 1991, the Gaisano Superstore Complex in Cagayan de Oro City, owned by petitioner, was consumed by fire. Included in the items lost or destroyed in the fire were stocks of ready-made clothing materials sold and delivered by IMC and LSPI. On February 4, 1992, respondent filed a complaint for damages against petitioner. It alleges that IMC and LSPI filed with respondent their claims under their respective fire insurance policies with book debt endorsements; that as of February 25, 1991, the unpaid accounts of petitioner on the sale and delivery of ready-made clothing materials with IMC was P2,119,205.00 while with LSPI it was P535,613.00; that respondent paid the claims of IMC and LSPI and, by virtue thereof, respondent was subrogated to their rights against petitioner; that respondent made several demands for payment upon petitioner but these went unheeded. In its Answer with Counter Claim dated July 4, 1995, petitioner contends that it could not be held liable because the property covered by the insurance policies were destroyed due to fortuities event or force majeure; that respondents right of subrogation has no basis inasmuch as there was no breach of contract committed by it since the loss was due to fire which it could not prevent or foresee; that IMC and LSPI never communicated to it that they insured their properties; that it never consented to paying the claim of the insured. - At the pre-trial conference the parties failed to arrive at an amicable settlement. Thus, trial on the merits ensued. On August 31, 1998, the RTC rendered its decision dismissing respondents complaint. It held that the fire was purely accidental; that the cause of the fire was not attributable to the negligence of the petitioner; that it has not been established that petitioner is the debtor of IMC and LSPI; that since the sales invoices state that it is further agreed that merely for purpose of securing the payment of purchase price, the above-described merchandise remains the property of the vendor until the purchase price is fully paid, IMC and LSPI retained ownership of the delivered goods and must bear the loss. Dissatisfied, petitioner appealed to the CA. On October 11, 2000, the CA rendered its decision setting aside the decision of the RTC. The CA held that the sales invoices are proofs of sale, being detailed statements of the nature, quantity and cost of the thing sold; that loss of the goods in the fire must be borne by petitioner since the proviso contained in the sales invoices is an exception under Article 1504 (1) of the Civil Code, to the general rule that if the thing is lost by a fortuitous event, the risk is borne by the owner of the thing at the time the loss under the principle of res perit domino; that petitioners obligation to IMC and LSPI is not the delivery of the lost goods but the payment of its unpaid account and as such the obligation to pay is not extinguished, even if the fire is considered a fortuitous event; that by subrogation, the insurer has the right to go against petitioner; that, being a fire insurance with book debt endorsements, what was insured was the vendors interest as a creditor. Petitioner filed a motion for reconsideration but it was denied by the CA in its Resolution dated April 11, 2001. ISSUES

Page
1. WON the CA erred in construing a fire insurance policy on book debts as one covering the unpaid accounts of IMC and LSPI since such insurance applies to loss of the ready-made clothing materials sold and delivered to petitioner. 2. WON IMC bears the risk of loss because it expressly reserved ownership of the goods by stipulating in the sales invoices that [i]t is further agreed that merely for purpose of securing the payment of the purchase price the above described merchandise remains the property of the vendor until the purchase price thereof is fully paid. 3. WON the petitioner liable for the unpaid accounts HELD 1. NO - It is well-settled that when the words of a contract are plain and readily understood, there is no room for construction. In this case, the questioned insurance policies provide coverage for book debts in connection with ready-made clothing materials which have been sold or delivered to various customers and dealers of the Insured anywhere in the Philippines.; and defined book debts as the unpaid account still appearing in the Book of Account of the Insured 45 days after the time of the loss covered under this Policy. Nowhere is it provided in the questioned insurance policies that the subject of the insurance is the goods sold and delivered to the customers and dealers of the insured. - Indeed, when the terms of the agreement are clear and explicit that they do not justify an attempt to read into it any alleged intention of the parties, the terms are to be understood literally just as they appear on the face of the contract. Thus, what were insured against were the accounts of IMC and LSPI with petitioner which remained unpaid 45 days after the loss through fire, and not the loss or destruction of the goods delivered. 2. NO - The present case clearly falls under paragraph (1), Article 1504 of the Civil Code: ART. 1504. Unless otherwise agreed, the goods remain at the sellers risk until the ownership therein is transferred to the buyer, but when the ownership therein is transferred to the buyer the goods are at the buyers risk whether actual delivery has been made or not, except that: (1) Where delivery of the goods has been made to the buyer or to a bailee for the buyer, in pursuance of the contract and the ownership in the goods

INSURANCE 27
has been retained by the seller merely to secure performance by the buyer of his obligations under the contract, the goods are at the buyers risk from the time of such delivery; (Emphasis supplied) - Thus, when the seller retains ownership only to insure that the buyer will pay its debt, the risk of loss is borne by the buyer. Accordingly, petitioner bears the risk of loss of the goods delivered. - IMC and LSPI did not lose complete interest over the goods. They have an insurable interest until full payment of the value of the delivered goods. Unlike the civil law concept of res perit domino, where ownership is the basis for consideration of who bears the risk of loss, in property insurance, ones interest is not determined by concept of title, but whether insured has substantial economic interest in the property. - Section 13 of our Insurance Code defines insurable interest as every interest in property, whether real or personal, or any relation thereto, or liability in respect thereof, of such nature that a contemplated peril might directly damnify the insured. Parenthetically, under Section 14 of the same Code, an insurable interest in property may consist in: (a) an existing interest; (b) an inchoate interest founded on existing interest; or (c) an expectancy, coupled with an existing interest in that out of which the expectancy arises. - Therefore, an insurable interest in property does not necessarily imply a property interest in, or a lien upon, or possession of, the subject matter of the insurance, and neither the title nor a beneficial interest is requisite to the existence of such an interest, it is sufficient that the insured is so situated with reference to the property that he would be liable to loss should it be injured or destroyed by the peril against which it is insured. Anyone has an insurable interest in property who derives a benefit from its existence or would suffer loss from its destruction. Indeed, a vendor or seller retains an insurable interest in the property sold so long as he has any interest therein, in other words, so long as he would suffer by its destruction, as where he has a vendors lien. In this case, the insurable interest of IMC and LSPI pertain to the unpaid accounts appearing in their Books of Account 45 days after the time of the loss covered by the policies. 3. YES - Petitioners argument that it is not liable because the fire is a fortuitous event under Article 1174 of the Civil Code is misplaced. As held earlier, petitioner bears the loss under Article 1504 (1) of the Civil Code. - Moreover, it must be stressed that the insurance in this case is not for loss of goods by fire but for petitioners accounts with IMC and LSPI that remained unpaid 45 days after the fire. Accordingly, petitioners obligation is for the payment of money. Where the obligation consists in the payment of money, the failure of the debtor to make the payment even by reason of a fortuitous event shall not relieve him of his liability. The rationale for this is that the rule that an obligor should be held exempt from liability when the loss occurs thru a fortuitous event only holds true when the obligation consists in the delivery of a determinate thing and there is no stipulation holding him liable even in case of fortuitous event. It does not apply when the obligation is pecuniary in nature. - Under Article 1263 of the Civil Code, [i]n an obligation to deliver a generic thing, the loss or destruction of anything of the same kind does not extinguish the obligation. If the obligation is generic in the sense that the object thereof is designated merely by its class or genus without any particular designation or physical segregation from all others of the same class, the loss or destruction of anything of the same kind even without the debtors fault and before he has incurred in delay will not have the effect of extinguishing the obligation. This rule is based on the principle that the genus of a thing can never perish. Genus nunquan perit. An obligation to pay money is generic; therefore, it is not excused by fortuitous loss of any specific property of the debtor. - Thus, whether fire is a fortuitous event or petitioner was negligent are matters immaterial to this case. What is relevant here is whether it has been established that petitioner has outstanding accounts with IMC and LSPI. - With respect to IMC, the respondent has adequately established its claim. Petitioner has an outstanding account with IMC in the amount of P2,119,205.00, check voucher evidencing payment to IMC, subrogation receipt executed by IMC in favor of respondent upon receipt of the insurance proceeds. All these documents have been properly identified, presented and marked as exhibits in court. The subrogation receipt, by itself, is sufficient to establish not only the relationship of respondent as insurer and IMC as the insured, but also the amount paid to settle

Page
the insurance claim. The right of subrogation accrues simply upon payment by the insurance company of the insurance claim. Respondents action against petitioner is squarely sanctioned by Article 2207 of the Civil Code which provides: Art. 2207. If the plaintiffs property has been insured, and he has received indemnity from the insurance company for the injury or loss arising out of the wrong or breach of contract complained of, the insurance company shall be subrogated to the rights of the insured against the wrongdoer or the person who has violated the contract. x x x - Petitioner failed to refute respondents evidence. - As to LSPI, respondent failed to present sufficient evidence to prove its cause of action. No evidentiary weight can be given to Exhibit F Levi Strauss, a letter dated April 23, 1991 from petitioners General Manager, Stephen S. Gaisano, Jr., since it is not an admission of petitioners unpaid account with LSPI. It only confirms the loss of Levis products in the amount of P535,613.00 in the fire that razed petitioners building on February 25, 1991. - Moreover, there is no proof of full settlement of the insurance claim of LSPI; no subrogation receipt was offered in evidence. Thus, there is no evidence that respondent has been subrogated to any right which LSPI may have against petitioner. Failure to substantiate the claim of subrogation is fatal to petitioners case for recovery of the amount of P535,613.00. Disposition Petition is partly GRANTED. The assailed Decision dated October 11, 2000 and Resolution dated April 11, 2001 of the Court of Appeals in CA-G.R. CV No. 61848 are AFFIRMED with the MODIFICATION that the order to pay the amount of P535,613.00 to respondent is DELETED for lack of factual basis. TAI TONG CHUACHE & CO v. INSURANCE COMMISSION and TRAVELLERS MULTIINDEMNITY CORPORATION 158 SCRA 366 GANCAYCO; February 29, 1988 NATURE Petition for review on certiorari of the decision of the Insurance Commission FACTS - Complainants Palomo acquired a parcel of land and a building located in Davao City. They assumed the

INSURANCE 28
mortgage of the building in favor of SSS, which building was insured with respondent SSS Accredited Group of Insurers for P25K. - On April 19, 1975, Azucena Palomo obtained a P100K loan from Tai Tong Chuache Inc. (TTCC) and executed a mortgage over the land and the building in favor of Tai Tong Chuache & Co. as security of payment .On April 25, 1975, Arsenio Chua, representative of TTCC insured the latter's interest with Travellers Multi-Indemnity Corporation (Travellers) for P100K (P70K for bldg and P30K for the contents thereof) - On June 11, 1975, Pedro Palomo secured a Fire Insurance Policy, covering the building for P50K with respondent Zenith Insurance Corporation (ZIC). Another Fire Insurance Policy was later procured from respondent Philippine British Assurance Company (PBAC), covering the same building for P50K and contents thereof for P70K. On July 31, 1975, the building and the contents were totally razed by fire. - Based on the computation of the loss, including the Travellers, respondents, ZIC, PBAC, and SSS paid their corresponding shares of the loss. Complainants were paid the following: P41,546.79 by PBAC, P11,877.14 by ZIC, and P5,936.57 by SSS. Demand was made from respondent Travellers for its share in the loss but was refused. Hence, complainants demanded from the other 3 respondents the balance of each share in the loss based on the computation excluding Travellers Multi-Indemnity in the amount of P30,894.31 (P5,732.79-ZIC: P22,294.62, PBAC: and P2,866.90, SSS) but was refused, hence, this action. ISSUE WON petitioner Tai Tong has insurable interest in the said policy HELD YES - First, respondent insurance commission based its findings on mere inference. Respondent Insurance Commission absolved respondent insurance company from liability on the basis of the certification issued by the then CFI, that in a certain civil action against the Palomos, Arsenio Lopez Chua stands as the complainant and not Tai Tong Chuache. From said evidence respondent commission inferred that the credit extended by herein petitioner to the Palomos secured by the insured property must have been paid. Such is a glaring error which this Court cannot sanction. - Second, it has been held in a long line of cases that when the creditor is in possession of the document of credit, he need not prove non-payment for it is presumed. The validity of the insurance policy taken b petitioner was not assailed by private respondent. Moreover, petitioner's claim that the loan extended to the Palomos has not yet been paid was corroborated by Azucena Palomo who testified that they are still indebted to herein petitioner. So at the time of the fire, petitioner as mortgagee still had insurable interest therein. - And third, petitioner's declaration that Arsenio Lopez Chua acts as the managing partner of the partnership was corroborated by respondent insurance company. Thus Chua as the managing partner of the partnership may execute all acts of administration including the right to sue debtors of the partnership in case of their failure to pay their obligations when it became due and demandable. Or at the least, Chua being a partner of petitioner Tai Tong Chuache & Company is an agent of the partnership. Being an agent, it is understood that he acted for and in behalf of the firm. Disposition Appealed decision SET ASIDE and ANOTHER judgment is rendered order private respondent Travellers to pay petitioner the face value of Fire Insurance Policy in the amount of P100K. Costs against said private respondent. PEREZ v. CA (BF LIFEMAN INSURANCE CORP.) 323 SCRA 613 YNARES-SANTIAGO; January 28, 2000 NATURE Petition for review on certiorari FACTS - Primitivo Perez has been insured with the BF Lifeman Insurance Corporation (BF hereafter) since 1980 for Php20,000. Sometime in 1987, Rodolfo Lalog (agent of BF) convinced him to apply for additional insurance coverage of Php50,000. Perez accomplished the application form and passed the required medical examination. He also paid Php2,075 premium) to Lalog. - On November 25, 1987, Perez died while riding a banca which capsized during a storm. During this time his application papers for the additional insurance coverage was still with the Gumaca, Quezon office of BF.

Page
- Without knowing that Perez died on November 25, 1987, BF approved Perez's application and issued the corresponding policy for the Php50,000 on December 2, 1987. - Virginia Perez (wife of the deceased) claimed the benefits under the insurance policies of the deceased, but she was only able to receive Php40,000 under the first insurance policy. BF refused to pay the proceeds amounting to Php150,000 under the additional policy coverage of Php50,000 because they maintain that such policy had not been perfected. - On September 21, 1990, BF filed a complaint against Mrs. Perez seeking recission and declaration of nullity of the insurance contract in question. Mrs. Perez filed a conterclaim for the collection of Php150,000 plus damages. ISSUE WON there was a consummated contract of insurance between Perez and BF HELD NO - An essential requisite of a valid contract is consent. Consent must be manifested by the meeting of the offer and the acceptance upon the thing and the cause which are to constitute the contract. - The offer must be certain and the acceptance absolute. When Perez filed the application, it was subject to the acceptance of BF. The perfection was also further conditioned upon (1) the issuance of the policy, (2) the payment of the premium, and (3) the delivery to and acceptance by the applicant in good health. - The delivery and acceptance by the applicant was a suspensive condition which was not fulfilled inasmuch as the applicant was already dead at the time the policy was issued. The non-fulfillment of the condition resulted in the non-perfection of the contract. - An application for insurance is merely an offer which requires the overt act of the insurer for it to ripen to a contract. Delay in acting on the application does not constitute acceptance even though the insured has forwarded his first premium with his application. Delay, in this case, does not constitute gross negligence because the application was granted within the normal processing time.

INSURANCE 29
Disposition Decision of CA affirmed in so far as it declared the insurance policy for Php50,000 issued by BF null and void (no recission because it presupposes a valid contract) VDA. DE SINDAYEN v. INSULAR 62 Phil 51 BUTTE; September 4, 1935 FACTS - Arturo Sindayen, up to the time of his death on January 19, 1933, was employed as a linotype operator in the Bureau of Printing at Manila and had been such for eleven years prior thereto. While there he made a written application on December 26, 1932, to the defendant Insular Life Assurance Co., Ltd., through its agent, Cristobal Mendoza, for a policy of insurance on his life in the sum of P1,000 and he paid to the agent P15 cash as part of the first premium. It was agreed with the agent that the policy, when and if issued, should be delivered to his aunt. Felicidad Estrada, with whom Sindayen left the sum of P26.06 to complete the payment of the first annual premium of P40.06. - On January 1, 1933, Sindayen, who was then twenty-nine years of age, was examined by the company's doctor who made a favorable report, to the company. On January 11, 1933, The company accepted the risk and issued policy No. 47710 dated back to December 1, 1932, and mailed the same to its agent, Cristobal Mendoza, in Camiling, Tarlac, for delivery to the insured. -On January 11, 1933, Sindayen was at work in the Bureau of Printing. On January 12, he complained of a severe headache and remained at home. On January 15, he called a physician who found that he was suffering from acute nephritis and uremia and on January 19, 1933, he died. - On January 18, 1933, the agent, in accordance with his agreement with the insured, delivered the policy to Felicidad Estrada upon her payment of the balance of the first year's annual premium. The agent asked Felicidad Estrada if her nephew was in good health and she replied that she believed so because she had no information that he was sick and he thereupon delivered to her the policy. - On January 20, 1933, the agent learned of the death of Arturo Sindayen and called on Felicidad Estrada and asked her to return the policy. But he did not return or offer to return the premium paid. Felicidad Estrada on his aforesaid statement gave him the policy. - On February 4, 1933 Insular Life obtained from the beneficiary, Sindayens wife, her signature to a legal document entitled "ACCORD, SATISFACTION AND RELEASE" whereby in consideration of the sum of P40.06 paid to her by a check of the company, she "assigns, releases and forever discharges said Isular Life Assurance Co., Ltd., its successors and assigns, of all claims, obligation in or indebtedness. The said check for P40.06 was never cashed but returned to the company and appears in the record of this case as Exhibit D. Thereupon this action was brought to enforce payment of the policy. By the terms of the policy, an annual premium of P40.06 is due on the first day of December of each year, the first premium already paid by the insured covering the period from December 1, 1932. It is to December 1, 1933. It is to be noted that the policy was not issued and the company assumed no actual risk prior to January 11, 1933.The application which the insured signed in Camiling, Tarlac, on December 26, 1932, contained among others the following provisions: 3 That the said policy shall not take effect until the first premium has been paid and the policy has been delivered to and accepted by me, while I am in good health. -Main defense of the company in this case, namely, that the said policy never took effect because of paragraph 3 of the application above quoted, for at the time of its delivery by the agent as aforesaid the insured was not in good health ISSUE WON the insurance policy is valid HELD YES - There is one line of cases which holds that the stipulation contained in paragraph 3 is in the nature of a condition precedent, that is to say, that there can be no valid delivery to the insured unless he is in good health at the time; that this condition precedent goes to the very essence of the contract and cannot be waived by the agent making delivery of the policy - On the other hand, a number of American decisions hold that an agent to whom a life insurance policy similar to the one here involved was sent with instructions to deliver it to the insured has authority to bind the company by making such delivery,

Page
although the insured was not in good health at the time of delivery, on the theory that the delivery of the policy being the final act to the consummation of the contract, the condition as to the insurer's good health was waived by the company. - we are inclined to the view that it is more consonant with the well known practice of life insurance companies and the evidence in the present case to rest our decision on the proposition that Mendoza was authorized by the company to make the delivery of the policy when he received the payment of the first premium and he was satisfied that the insured was in good health. As was well said in the case of MeLaurin vs. Mutual Life Insurance Co. It is plain, therefore, that upon the facts it is not necessarily a case of waiver or of estoppel, but a case where the local agents, in the exercise of the powers lodged in them, accepted the premium and delivered the policy. That act binds their principal, the defendant. - Mendoza was duly licensed by the Insurance Commissioner to act as the agent of the defendant insurance company. The well known custom of the insurance business and the evidence in this case prove that Mendoza was not regarded by the company as a mere conduit or automaton for the performance of the physical act of placing the policy in the hands of the insured - Granted that Mendoza's decision that the condition had been met by the insured and that it was proper to make a delivery of the policy to him is just as binding on the company as if the decision had been made by its board of directors. Granted that Mendoza made a mistake of judgement because he acted on insufficient evidence as to the state of health of the insured. But it is not charged that the mistake was induced by any misconduct or omission of duty of the insured. - It is the interest not only the applicant but of all insurance companies as well that there should be some act which gives the applicant the definite assurance that the contract has been consummated. This sense of security and of peace of mind that one's defendants are provided for without risk either of loss or of litigation is the bedrock of life insurance. When the policy is issued and delivered, in the absence of fraud or other grounds for rescission, it is plainly not within the intention of the parties that there should be any questions held in abeyance or reserved for future determination that leave the very existence of the contract in suspense and doubt. - It is therefore in the public interest, for the public is profoundly and generally interested in life insurance,

INSURANCE 30
as well as in the interest of the insurance companies themselves by giving certainly and security to their policies, that we are constrained to hold, as we, do, that the delivery of the policy to the insured by an agent of the company who is authorized to make delivery or without delivery is the final act which binds the company (and the insured as well) in the absence of fraud or other legal ground for rescission - The company therefore having decided that all the conditions precedent to the taking effect of the policy had been complied with and having accepted the premium and delivered the policy thereafter to the insured, the company is now estopped to assert that it never intended that the policy should take effect. SEPARATE OPINION IMPERIAL [dissent] - "A local agent of an insurance company, whose only power is to solicit applications for insurance, and forward them to the company for approval, when, if approved to the insured, has no power to waive any of the provision of the policy so delivered." - It is clear, therefore, that the delivery of the policy by Mendoza does not bind the defendant, nor is the defendant estopped from alleging its defense, for the simple reason that Mendoza was not an agent with authority to issue policies or to accept risks in the name of his principle. -There is another ground upon which the majority opinion is based, namely, that the defendant waived the defense it now invokes, by reason of the delivery of the policy by its invokes, by reason of the delivery of the policy by its agent. It is admitted that if the delivery of the policy was due to fraud, legally there could have been no waiver. In view of the facts established and admitted, there is no doubt, as to the existence of the fraud. -Estrada, as a representative of the insured was not only bound to give a truthful information on the state of health of the insured, but it was her duty to find out it his true state of health in order to give true and correct information. When she gave Mendoza an incorrect information tending to create the impression that the insured was well when in fact he was seriously ill, there is no doubt that she committed fraud and imparted a deceitful information to the defendant agent ENRIQUEZ v. SUN LIFE OF CANADA 41 PHIL 269 MALCOLM; November 29, 1920 NATURE Appeal from judgment of trial court denying plaintiffs (administrator of the estate of the late Joaquin Ma. Herrer) action to recover from the defendant life insurance company the sum of pesos 6,000 paid by the deceased for a life annuity. FACTS - On September 24, 1917, Joaquin Herrer made application to the Sun Life Assurance Company of Canada through its office in Manila for a life annuity. Two days later he paid the sum of P6,000 to the manager of the company's Manila office and was given a receipt. - The application was immediately forwarded to the head office of the company at Montreal, Canada. On November 26, 1917, the head office gave notice of acceptance by cable to Manila. (Whether on the same day the cable was received, notice was sent by the Manila office of Herrera that the application had been accepted, is a disputed point, which will be discussed later.) On December 4, 1917, the policy was issued at Montreal. On December 18, 1917, attorney Aurelio A. Torres wrote to the Manila office of the company stating that Herrer desired to withdraw his application. The following day the local office replied to Mr. Torres, stating that the policy had been issued, and called attention to the notification of November 26, 1917. This letter was received by Mr. Torres on the morning of December 21, 1917. Mr. Herrer died on December 20, 1917. - The chief clerk of the Manila office of Sun Life testified that he prepared the letter and handed it to the local manager, Mr. E. E. White, for signature. The local manager, Mr. White, testified to having received the cablegram accepting the application of Mr. Herrer from the home office on November 26, 1917. He said that on the same day he signed a letter notifying Mr. Herrer of this acceptance. The witness further said that letters, after being signed, were sent to the chief clerk and placed on the mailing desk for transmission. Mr. Tuason, who was the chief clerk on November 26, 1917, was not called as a witness. - For the defense, attorney Manuel Torres testified to having prepared the will of Joaquin Ma. Herrer. That on this occasion, Mr. Herrer mentioned his application for a life annuity, and that he said that the only document relating to the transaction in his possession was the provisional receipt. Rafael Enriquez, the administrator of the estate, testified that he had gone

Page
through the effects of the deceased and had found no letter of notification from the insurance company to Mr. Herrer. ISSUE WON there exists a contract for life annuity between Herrer and defendant HELD NO Ratio The law applicable to the case is found to be the second paragraph of article 1262 of the Civil Code providing that an acceptance made by letter shall not bind the person making the offer except from the time it came to his knowledge. Reasoning - Until quite recently, all of the provisions concerning life insurance in the Philippines were found in the Code of Commerce and the Civil Code. After July 1, 1915, there was, however, in force the Insurance Act. No. 2427. Chapter IV of this Act concerns life and health insurance. The Act expressly repealed Title VIII of Book II and Section III of Title III of Book III of the code of Commerce. The law of insurance is consequently now found in the Insurance Act and the Civil Code. - While, as just noticed, the Insurance Act deals with life insurance, it is silent as to the methods to be followed in order that there may be a contract of insurance. On the other hand, the Civil Code, in article 1802, not only describes a contact of life annuity markedly similar to the one we are considering, but in two other articles, gives strong clues as to the proper disposition of the case. For instance, article 16 of the Civil Code provides that "In matters which are governed by special laws, any deficiency of the latter shall be supplied by the provisions of this Code." On the supposition, therefore, which is incontestable, that the special law on the subject of insurance is deficient in enunciating the principles governing acceptance, the subject-matter of the Civil code, if there be any, would be controlling. In the Civil Code is found article 1262 providing that "Consent is shown by the concurrence of offer and acceptance with respect to the thing and the consideration which are to constitute the contract. An acceptance made by letter shall not bind the person making the offer except from the time it came to his knowledge. - According to the provisional receipt, three things had to be accomplished by the insurance company before there was a contract: (1) There had to be a

INSURANCE 31
medical examination of the applicant; (2) there had to be approval of the application by the head office of the company; and (3) this approval had in some way to be communicated by the company to the applicant. The further admitted facts are that the head office in Montreal did accept the application, did cable the Manila office to that effect, did actually issue the policy and did, through its agent in Manila, actually write the letter of notification and place it in the usual channels for transmission to the addressee. - The contract for a life annuity in the case at bar was not perfected because it has not been proved satisfactorily that the acceptance of the application ever came to the knowledge of the applicant. Disposition Judgment is reversed, and the plaintiff shall have and recover from the defendant the sum of P6,000 with legal interest from November 20, 1918, until paid, without special finding as to costs in either instance. defendants liable for repair of the car, medical expenses, etc. BUT Maharlike was exonerated on the gnd that the policy was not in force. - Petitioners elevated this case to this court, faulting the respondent judge for considering the defense of late payment of premium when the same was waived at the pre-trial, hence the evidence of late payment should be disregarded supposedly because the private respondent had admitted that such fact was not in issue. - (More pertinent to this class: ) petitioners assert that the private respondent had agreed to grant the then prospective insured a credit extension of the premium due. - This controversy arose under the old insurance law, Act No. 2427. - The accident occurred in 1973. The complaint was filed on July 20, 1974. both before the effectivity of Presidential Decree no. 612, the subsequent insurance law which repealed its predecessor - The former insurance law, which applies to the case here, provided: An insurer is entitled to the payment of premium as soon as the thing insured is exposed to the peril insured against, unless there is clear agreement to grant the insured credit extension of the premium due. No policy issued by an insurance company is valid and binding unless and until the premium thereof has been paid. ISSUE WON the insurance policy would be valid and binding notwithstanding the non-payment of the premium HELD NO Ratio Act No. 2427: an insurance policy would be valid and binding notwithstanding the non-payment of the premium if there was a clear agreement to grant to the insured credit extension. Such agreement may be express or implied. Reasoning - Petitioners maintain that in spite of their late payment, the policy is binding because there was an implied agreement to grant a credit extension so as to make the policy effective. To them, the subsequent acceptance of the premium and delivery of the policy estops the respondent company from asserting that the policy is ineffective. The court however sees no proof of any such implied agreement. The purported nexus between the delivery

Page
of the policy and the grant of credit extension is too tenuous to support the conclusion for which petitioners contend. Parenthetically mention: in the present law, Section 77 of the Insurance Code of 1978 has deleted the clause "unless there is clear agreement to grant the insured credit extension of the premium due" which was then involved in this controversy. Disposition Fnding no reversible error, the judgment appealed from is hereby AFFIRMED.

TIBAY v. CA (FORTUNE INSURANCE) 257 SCRA 126 BELLOSILLO; May 24, 1996

LIFE

&

GENERAL

VELASCO and ACOSTA v. APOSTOL MAHARLIKA INSURANCE CO., INC. 173 SCRA 228 REGALADO.; May 9, 1989 NATURE Petition for review on certiorari

and

FACTS - Petitioners were plaintiffs in a civil case of which public respondent Hon. Apostol was the judge. - The case was an offshoot of an incident: plaintiffs were riding in their car, when a taxicab crossed a center island in the road and collided with their car. Private respondent Maharlika was eventually impleaded as a defendant in this case, with an allegation that the taxicab involved was insured against third party liability for P20,000.00 with private respondent at the time of the accident - Maharlika claimed there was no cause of action against it because at the time of the accident, the alleged insurance policy was not in force due to the non-payment of the premium thereon. Also, even if the cab had been insured, the complaint would be premature since the policy provides that the insurer would be liable only when the insured becomes legally liable. - Trial court ruled in favor of the plaintiff, holding the

FACTS - On 22 January 1987 Fortune Life and General Insurance Co., Inc. (FORTUNE) issued Fire Insurance Policy No. 136171 in favor of Violeta R. Tibay and/or Nicolas Roraldo on their two-storey residential building located at 5855 Zobel Street, Makati City, together with all their personal effects therein. The insurance was for P600,000 covering the period from 23 January 1987 to 23 January 1988. On 23 January 1987, of the total premium of P2,983.50, Violeta Tibay only paid P600 thus leaving a considerable balance unpaid. - On 8 March 1987 the insured building was completely destroyed by fire. Two days later, Violeta Tibay paid the balance of the premium. On the same day, she filed with FORTUNE a claim on the fire insurance policy. Her claim was accordingly referred to its adjuster, Goodwill Adjustment Services, Inc. (GASI), which immediately wrote Violeta requesting her to furnish it with the necessary documents for the investigation and processing of her claim. Petitioner forthwith complied. On 28 March 1987 she signed a nonwaiver agreement with GASI to the effect that any action taken by the companies shall not be, or be claimed to be, an admission of liability. - FORTUNE denied the claim of Violeta for violation of Policy Condition No. 2 and of Sec. 77 of the Insurance Code. Efforts to settle the case before the

This policy including any renewal thereof and/or any endorsement thereon is not in force until the premium has been fully paid to and duly receipted by the Company in the manner provided herein.

INSURANCE 32
Insurance Commission proved futile. On 3 March 1988 Violeta and the other petitioners sued FORTUNE for damages in the amount of P600,000 representing the total coverage of the fire insurance policy plus 12% interest per annum, P100,000 moral damages, and attorney's fees equivalent to 20% of the total claim. The trial court ruled for petitioners. CA reversed. ISSUE WON a fire insurance policy is valid, binding and enforceable upon mere partial payment of premium HELD NO Ratio Where the insurer and the insured expressly stipulated that the policy is not in force until the premium has been fully paid the payment of partial premium by the assured in this particular instance should not be considered the payment required by the law and the stipulation of the parties. Rather, it must be taken in the concept of a deposit to be held in trust by the insurer until such time that the full amount has been tendered and duly receipted for. Reasoning - As expressly agreed upon in the contract, full payment must be made before the risk occurs for the policy to be considered effective and in force. Thus, no vinculum juris whereby the insurer bound itself to indemnify the assured according to law ever resulted from the fractional payment of premium. The insurance contract itself expressly provided that the policy would be effective only when the premium was paid in full. It would have been altogether different were it not so stipulated. Ergo, petitioners had absolute freedom of choice whether or not to be insured by FORTUNE under the terms of its policy and they freely opted to adhere thereto. - Indeed, and far more importantly, the cardinal polestar in the construction of an insurance contract is the intention of the parties as expressed in the policy. Courts have no other function but to enforce the same. The rule that contracts of insurance will be construed in favor of the insured and most strongly against the insurer should not be permitted to have the effect of making a plain agreement ambiguous and then construe it in favor of the insured. Verily, it is elemental law that the payment of premium is requisite to keep the policy of insurance in force. If the premium is not paid in the manner prescribed in the policy as intended by the parties the policy is ineffective. Partial payment even when accepted as a partial payment will not keep the policy alive even for such fractional part of the year as the part payment bears to the whole payment. Disposition Petition is DENIED. Decision of the CA is AFFIRMED.

Page
- A second policy was issued to renew the first one, this time covering the period 1 March 1983 to 1 March 1984. This was also pain in installment basis. - A third policy was again issued for the period 1 March 1984 to 1 March 1985. For this, petitioner made two installment payments, both accepted by AHAC. Thereafter, petitioner refused to pay the balance of the premium. AHAC filed an action to recover the unpaid balance of P314,103.05. - Petitioner explained that it discontinued the payment of premiums because the policy did not contain a credit clause in its favor and the receipts for the installment payments covering the policy for 1984-85, as well as the two (2) previous policies, stated the following reservations: 2. Acceptance of this payment shall not waive any of the company rights to deny liability on any claim under the policy arising before such payments or after the expiration of the credit clause of the policy; and 3. Subject to no loss prior to premium payment. If there be any loss such is not covered. - Petitioner further claimed that the policy was never binding and valid, and no risk attached to the policy. It then pleaded a counterclaim for P152k for the premiums already paid for 1984-85, and in its answer with amended counterclaim, sought the refund of P924,206.10 representing the premium payments for 1982-85. - Trial court dismissed the complaint and the counterclaim upon the following findings: (1) payment of the premiums of the three policies were made during the term of said policies, hence, it could not be said, inspite of the reservations, that no risk attached under the policies; (2) as regards the unpaid premiums, in view of the reservation in the receipts ordinarily issued by AHAC on premium payments the only plausible conclusion is that AHAC has no right to demand their payment after the lapse of the term of said policy on March 1, 1985. Therefore, Tuscany was justified in refusing to pay the same. - CA modified the decision by ordering Tuscany to pay the balance of the premiums due on the third policy plus legal interest until fully paid, and affirming the denial of the counterclaim. Petitioners Claims Petitioner argues that where the premiums is not actually paid in full, the policy would only be effective if there is an acknowledgment in the policy of the receipt of premium pursuant to Sec. 78 of the Insurance Code. The absence of an express

SEPARATE OPINION VITUG [dissent] - The law neither requires, nor measures the strength of the vinculum juris by, any specific amount of premium payment. It should thus be enough that payment on the premium, partly or in full, is made by the insured which the insurer accepts. In fine, it is either that a juridical tie exists (by such payment) or that it is not extant at all (by an absence thereof). Once the juridical relation comes into being, the full efficacy, not merely pro tanto, of the insurance contract naturally follows. Verily, not only is there an insurance perfected but also a partially performed contract. In case of loss, recovery on the basis of the full contract value, less the unpaid premium can accordingly be had; conversely, if no loss occurs, the insurer can demand the payment of the unpaid balance of the premium. The insured, on the one hand, cannot avoid the obligation of paying the balance of the premium while the insurer, upon the other hand, cannot treat the contract as valid only for the purpose of collecting premiums and as invalid for the purpose of indemnity. MAKATI TUSCANY v. CA ( AMERICAN HOME ASSURANCE CO.) 215 SCRA 462 BELLOSILLO; November 6, 1992 NATURE Appeal from decision of the CA FACTS - American Home Assurance Co. (AHAC), represented by American International Underwriters (Phils.), Inc., issued in favor of petitioner Makati Tuscany Condominium Corporation an insurance policy on the latter's building and premises, for the period 1 March 1982 to1 March 1983. The premium was paid on installments all of which were accepted by AHAC.

INSURANCE 33
acknowledgment in the policies of such receipt of the corresponding premium payments, and petitioner's failure to pay said premiums on or before the effective dates of said policies rendered them invalid. Petitioner thus concludes that there cannot be a perfected contract of insurance upon mere partial payment of the premiums because under Sec. 77 of the Insurance Code, no contract of insurance is valid and binding unless the premium thereof has been paid, notwithstanding any agreement to the contrary. ISSUE WON payment by installment of the premiums due on an insurance policy invalidates the contract of insurance HELD Ratio Where the risk is entire and the contract is indivisible, the insured is not entitled to a refund of the premiums paid if the insurer was exposed to the risk insured for any period, however brief or momentary. Reasoning - The obligation to pay premiums when due is ordinarily as indivisible obligation to pay the entire premium. Here, the parties herein agreed to make the premiums payable in installments, and there is no pretense that the parties never envisioned to make the insurance contract binding between them. And the insured never informed the insurer that it was terminating the policy because the terms were unacceptable. - There is nothing in Section 77 which suggests that the parties may not agree to allow payment of the premiums in installment, or to consider the contract as valid and binding upon payment of the first premium. - The records clearly show that petitioner and private respondent intended subject insurance policies to be binding and effective notwithstanding the staggered payment of the premiums. Acceptance of payments speaks loudly of the insurer's intention to honor the policies it issued to petitioner. - Section 78 of the Insurance Code in effect allows waiver by the insurer of the condition of prepayment by making an acknowledgment in the insurance policy of receipt of premium as conclusive evidence of payment so far as to make the policy binding despite the fact that premium is actually unpaid. Section 77 merely precludes the parties from stipulating that the policy is valid even if premiums are not paid, but does not expressly prohibit an agreement granting credit extension, and such an agreement is not contrary to morals, good customs, public order or public policy. - At the very least, both parties should be deemed in estoppel to question the arrangement they have voluntarily accepted. Disposition Judgment affirmed. Costs against petitioner. SOUTH SEA SURETY AND INSURANCE v. CA (VALENZUELA HARDWOOD) 244 SCRA 744 VITUG; June 2, 1995 NATURE Petition for review on certiorari FACTS - Hardwood entered into agreement with Seven Bros Shipping, where latter undertook to load the formers logs on vessel. Hardwood insured the logs with South Sea Surety which issued Marine Cargo Insurance Policy. The vessel sank Jan 25, 1984. - Hardwood filed claim with South Sea and Seven Bros. Trial Court favored Hardwood. CA decided against South Sea, but absolved Seven Bros. South Sea filed this instant petition. ISSUES WON the insurance contract was already in effect when the vessel sank HELD YES - It is already in effect because Hardwood has already paid the insurance premium. It delivered the check to Victorio Chua before the vessel sank, but Victorio Chua was only to deliver the check to South Sea five days after the vessel sank. Appellant argues that Chua was not its broker, but it was found that Chua was authorized by South Sea to receive the premium on its behalf. AREOLA v. CA (PRUDENTIAL GUARANTEE AND ASSURANCE, INC.) 236 SCRA 643 ROMERO; September 22, 1994 NATURE CERTIORARI

Page
FACTS - June 29, 1985- 7 months after the issuance of Santos Areola's Personal Accident Insurance Policy No. PA-20015 (covering a period of one year), Prudential unilaterally cancelled the same since company records revealed that Areola failed to pay his premiums. o Under the terms of the statement of account issued by Prudential, Areola was supposed to pay the total amount of P1,609.65 which included the premium of P1,470.00, documentary stamp of P110.25 and 2% premium tax of P29.40. o The statement of account stated that it must not be considered a receipt as an official receipt will be issued upon payment of the account. And if payment was made to a representative, the client must demand for a Provisional Receipt and if Official Receipts arent received within 7 days, Prudential should be notified. If payment is made to their office, clients should demand for an OR. - August 3, 1985- Prudential offered to reinstate same policy it had previously cancelled and even proposed to extend its lifetime to December 17, 1985, upon a finding that the cancellation was erroneous and that the premiums were paid in full by Areola but were not remitted by Teofilo M. Malapit, Prudential's branch manager. Petitioners Claims - The fraudulent act of in misappropriating Areolas premium payments is the proximate cause of the cancellation of the insurance policy. - Areola theorized that Malapit's act of signing and even sending the notice of cancellation himself, notwithstanding his personal knowledge of petitionerinsured's full payment of premiums, further reinforces the allegation of bad faith. - Such fraudulent act committed by Malapit is attributable to Prudential. - Malapit's actuations are therefore not separate and distinct from that of Prudentials. It must, therefore, bear the consequences of the erroneous cancellation of subject insurance policy caused by the nonremittance by its own employee of the premiums paid. - Subsequent reinstatement could not possibly absolve respondent insurance company from liability, there being an obvious breach of contract. After all damage had already been inflicted on him and no amount of rectification could remedy the same. Respondents Argument

INSURANCE 34
- Prudential argues that where reinstatement, the equitable relief sought by Areola was granted at an opportune moment, i.e. prior to the filing of the complaint, Areola is left without a cause of action on which to predicate his claim for damages. - Reinstatement effectively restored Areola to all his rights under the policy. ISSUES 1. WON the erroneous act of canceling subject insurance policy entitle petitioner-insured to payment of damages 2. WON the subsequent act of reinstating the wrongfully cancelled insurance policy obliterate whatever liability for damages Prudential has HELD 1. YES 2. NO Reasoning - Malapit's fraudulent act of misappropriating the premiums paid by petitioner-insured is beyond doubt directly imputable to Prudential. - A corporation, such as respondent insurance company, acts solely thru its employees. The latters acts are considered as its own for which it can be held to account. - The facts are clear as to the relationship between private respondent insurance company and Malapit. His act of receiving the premiums collected is well within the province of his authority as manager. Thus, his receipt of said premiums is receipt by private respondent insurance company who, by provision of law, particularly under Article 1910 of the Civil Code, is bound by the acts of its agent. - Article 1910 thus reads: Art. 1910. The principal must comply with all the obligations which the agent may have contracted within the scope of his authority. As for any obligation wherein the agent has exceeded his power, the principal is not bound except when he ratifies it expressly or tacitly. - Malapit's failure to remit the premiums he received cannot constitute a defense for private respondent insurance company; no exoneration from liability could result therefrom. - Prudentials earlier act of reinstating the insurance policy can not obliterate the injury inflicted on petitioner-insured. - Respondent company should be reminded that a contract of insurance creates reciprocal obligations for both insurer and insured. - Reciprocal obligations are those which arise from the same cause and in which each party is both a debtor and a creditor of the other, such that the obligation of one is dependent upon the obligation of the other. - Under the circumstances of instant case, the relationship as creditor and debtor between the parties arose from a common cause: i.e., by reason of their agreement to enter into a contract of insurance under whose terms, Prudential promised to extend protection to Areola against the risk insured for a consideration in the form of premiums to be paid by the latter. - Under the law governing reciprocal obligations, particularly the second paragraph of Article 1191, the injured party, Areola in this case, is given a choice between fulfillment or rescission of the obligation in case one of the obligors, such as respondent insurance company, fails to comply with what is incumbent upon him. - However, said article entitles the injured party to payment of damages, regardless of whether he demands fulfillment or rescission of the obligation. - Untenable then is reinstatement insurance company's argument, namely, that reinstatement being equivalent to fulfillment of its obligation, divests petitioner-insured of a rightful claim for payment of damages. Such a claim finds no support in our laws on obligations and contracts. DAMAGES: - The nature of damages to be awarded, however, would be in the form of nominal damages - Although the erroneous cancellation of the insurance policy constituted a breach of contract, Prudential within a reasonable time took steps to rectify the wrong committed by reinstating the insurance policy of petitioner. - Moreover, no actual or substantial damage or injury was inflicted on petitioner Areola at the time the insurance policy was cancelled. - Nominal damages are "recoverable where a legal right is technically violated and must be vindicated against an invasion that has produced no actual present loss of any kind, or where there has been a breach of contract and no substantial injury or actual damages whatsoever have been or can be shown. Disposition Petition for review on certiorari is hereby GRANTED. RTC s DECISION is REINSTATED. UCPB GENERAL INSURANCE MASAGANA TELAMART, INC. 308 SCRA 259 PARDO; June 15, 1999 CO.,

Page
INC. v.

NATURE Petition for review on certiorari of a decision of the Court of Appeals. FACTS - On April 15, 1991, petitioner issued five (5) insurance policies covering respondent's various property described therein against fire, for the period from May 22, 1991 to May 22, 1992. - In March 1992, petitioner evaluated the policies and decided not to renew them upon expiration of their terms on May 22, 1992. Petitioner advised respondent's broker, Zuellig Insurance Brokers, Inc. of its intention not to renew the policies. - On April 6, 1992, petitioner gave written notice to respondent of the non-renewal of the policies at the address stated in the policies. - On June 13, 1992, fire razed respondent's property covered by three of the insurance policies petitioner issued. - On July 13, 1992, respondent presented to petitioner's cashier at its head office five (5) manager's checks in the total amount of P225,753.95, representing premium for the renewal of the policies from May 22, 1992 to May 22, 1993. No notice of loss was filed by respondent under the policies prior to July 14, 1992. - On July 14, 1992, respondent filed with petitioner its formal claim for indemnification of the insured property razed by fire. On the same day, petitioner returned to respondent the five manager's checks that it tendered, and at the same time rejected respondent's claim for the reasons (a) that the policies had expired and were not renewed, and (b) that the fire occurred on June 13, 1992, before respondent's tender of premium payment. - On July, 21, 1992, respondent filed with the Regional Trial Court, Branch 58, Makati City, a civil complaint against petitioner for recovery, of P18.645,000.00, representing the face value of the policies covering respondent's insured property razed by fire, and for attorney's fees. - On October 23, 1992, after its motion to dismiss had been denied, petitioner filed an answer to the complaint. It alleged that the complaint "fails to state a cause of action"; that petitioner was not liable to

INSURANCE 35
-respondent for insurance proceeds under the policies because at the time of the loss of respondent's property due to fire, the policies had long expired and were not renewed. After due trial, on March 10, 1993, the Regional Trial Court, Branch 58, Makati, rendered decision, the dispositive portion of which reads: "WHEREFORE, premises considered, judgment is hereby rendered in favor of the plaintiff and against the defendant, as follows. "(1) Authorizing and allowing the plaintiff to consign/deposit with this Court the sum of P225,753.95 (refused by the defendant) as full payment of the corresponding premiums for the replacement-renewal policies for Exhibits A, B, C, D and E; "(2) Declaring plaintiff to have fully complied with its obligation to pay the premium thereby rendering the replacement-renewal policy of Exhibits A, B, C, D and E effective and binding for the duration May 22, 1992 until May 22, 1993; and, ordering defendant to deliver forthwith to plaintiff the said replacement-renewal policies; "(3) Declaring Exhibits A & B, in force from August 22, 1991 up to August 23, 1992 and August 9, 1991 to August 9, 1992, respectively; and "(4) Ordering the defendant to pay plaintiff the sums of. (a) P18,645,000.00 representing the latter's claim for indemnity under Exhibits A, B & C and/or its replacement-renewal policies; (b) 25% of the total amount due as and for attorney's fees; (c) P25,000.00 as necessary litigation expenses; and, (d) the costs of suit. xxx - In due time, petitioner appealed to the Court of Appeals (CA). The CA promulgated its decision affirming that of the Regional Trial Court with the modification that item No. 3 of the dispositive portion was deleted, and the award of attorney's fees was reduced to 10% of the total amount due. It held that following previous practice, respondent was allowed a 60- to 90-day credit term for the renewal of its policies, and that the acceptance of the late premium payment suggested an understanding that payment could be made later. Hence, this appeal. ISSUE WON the fire insurance policies issued by petitioner to the respondent covering the period May 22, 1991 to May 22, 1992, had expired on the latter date or had been extended or renewed by an implied credit arrangement though actual payment of premium was tendered on a later date after the occurrence of the risk (fire) insured against HELD NO - An insurance policy, other than life, issued originally or on renewal, is not valid and binding until actual payment of the premium. Any agreement to the contrary is void. The parties may not agree expressly or impliedly on the extension of credit or time to pay the premium and consider the policy binding before actual payment. Disposition Judgment reversed and set aside

Page
courts below correctly found that no notice of nonrenewal was made within 45 days before 22 May 1992, or before the expiration date of the fire insurance policies. Thus, the policies in question were renewed by operation of law and were effective and valid on 30 June 1992 when the fire occurred, since the premiums were paid within the 60- to 90-day credit term. - Respondent likewise disagrees with its ruling that parties may neither agree expressly or impliedly on the extension of credit or time to pay the premium nor consider a policy binding before actual payment. It urges the Court to take judicial notice of the fact that despite the express provision of Section 77 of the Insurance Code, extension of credit terms in premium payment has been the prevalent practice in the insurance industry. Most insurance companies, including Petitioner, extend credit terms because Section 77 of the Insurance Code is not a prohibitive injunction but is merely designed for the protection of the parties to an insurance contract. The Code itself, in Section 78, authorizes the validity of a policy notwithstanding non-payment of premiums. - Respondent also asserts that the principle of estoppel applies to Petitioner. Despite its awareness of Section 77 Petitioner persuaded and induced Respondent to believe that payment of premium on the 60- to 90-day credit term was perfectly alright; in fact it accepted payments within 60 to 90 days after the due dates. By extending credit and habitually accepting payments 60 to 90 days from the effective dates of the policies, it has implicitly agreed to modify the tenor of the insurance policy and in effect waived the provision therein that it would pay only for the loss or damage in case the same occurred after payment of the premium. - Petitioner filed an opposition to the Respondents motion for reconsideration. It argues that both the trial court and the Court of Appeals overlooked the fact that on 6 April 1992 Petitioner sent by ordinary mail to Respondent a notice of non-renewal and sent by personal delivery a copy thereof to Respondents broker, Zuellig. Both courts likewise ignored the fact that Respondent was fully aware of the notice of nonrenewal. A reading of Section 66 of the Insurance Code readily shows that in order for an insured to be entitled to a renewal of a non-life policy, payment of the premium due on the effective date of renewal should first be made. Respondents argument that Section 77 is not a prohibitive provision finds no authoritative support.

UCPB GENERAL INSURANCE CO., INC. MASAGANA TELAMART, INC. (EN BANC) 356 SCRA 307 DAVIDE; April 4, 2001

v.

NATURE Motion for reconsideration of the decision of the Supreme Court. FACTS - In its decision of 15 June 1999, the SC defined the main issue to be whether the fire insurance policies issued by petitioner to the respondent covering the period from May 22, 1991 to May 22, 1992 had been extended or renewed by an implied credit arrangement though actual payment of premium was tendered on a later date and after the occurrence of the (fire) risk insured against. The Court resolved this issue in the negative in view of Section 77 of the Insurance Code and its decisions in Valenzuela v. Court of Appeals; South Sea Surety and Insurance Co., Inc. v. Court of Appeals; and Tibay v. Court of Appeals. Accordingly, it reversed and set aside the decision of the Court of Appeals. - Respondent seasonably filed a motion for the reconsideration of the adverse verdict. It alleges in the motion that the SC had made in the decision its own findings of facts, which are not in accord with those of the trial court and the Court of Appeals. The

INSURANCE 36
- The following facts, as found by the trial court and the Court of Appeals, are indeed duly established: 1. For years, Petitioner had been issuing fire policies to the Respondent, and these policies were annually renewed. 2. Petitioner had been granting Respondent a 60- to 90-day credit term within which to pay the premiums on the renewed policies. 3. There was no valid notice of non-renewal of the policies in question, as there is no proof at all that the notice sent by ordinary mail was received by Respondent, and the copy thereof allegedly sent to Zuellig was ever transmitted to Respondent. 4. The premiums for the policies in question in the aggregate amount of P225,753.95 were paid by Respondent within the 60- to 90-day credit term and were duly accepted and received by Petitioners cashier. ISSUE WON Sec. 77 of the Insurance Code of 1978 must be strictly applied to Petitioners advantage despite its practice of granting a 60- to 90-day credit term for the payment of premiums HELD NO - Section 77 of the Insurance Code of 1978 provides: SEC. 77. An insurer is entitled to payment of the premium as soon as the thing insured is exposed to the peril insured against. Notwithstanding any agreement to the contrary, no policy or contract of insurance issued by an insurance company is valid and binding unless and until the premium thereof has been paid, except in the case of a life or an industrial life policy whenever the grace period provision applies. - This Section is a reproduction of Section 77 of P.D. No. 612 (The Insurance Code) promulgated on 18 December 1974. In turn, this Section has its source in Section 72 of Act No. 2427 otherwise known as the Insurance Act as amended by R.A. No. 3540, approved on 21 June 1963, which read: SEC. 72. An insurer is entitled to payment of premium as soon as the thing insured is exposed to the peril insured against, unless there is clear agreement to grant the insured credit extension of the premium due. No policy issued by an insurance company is valid and binding unless and until the premium thereof has been paid. (Underscoring supplied) - It can be seen at once that Section 77 does not restate the portion of Section 72 expressly permitting an agreement to extend the period to pay the premium. But there are exceptions to Section 77. The first exception is provided by Section 77 itself, and that is, in case of a life or industrial life policy whenever the grace period provision applies. The second is that covered by Section 78 of the Insurance Code, which provides: SEC. 78. Any acknowledgment in a policy or contract of insurance of the receipt of premium is conclusive evidence of its payment, so far as to make the policy binding, notwithstanding any stipulation therein that it shall not be binding until premium is actually paid. - A third exception was laid down in Makati Tuscany Condominium Corporation vs. Court of Appeals, wherein we ruled that Section 77 may not apply if the parties have agreed to the payment in installments of the premium and partial payment has been made at the time of loss. Tuscany has provided a fourth exception to Section 77, namely, that the insurer may grant credit extension for the payment of the premium. This simply means that if the insurer has granted the insured a credit term for the payment of the premium and loss occurs before the expiration of the term, recovery on the policy should be allowed even though the premium is paid after the loss but within the credit term. Moreover, there is nothing in Section 77 which prohibits the parties in an insurance contract to provide a credit term within which to pay the premiums. That agreement is not against the law, morals, good customs, public order or public policy. The agreement binds the parties. Article 1306 of the Civil Code provides: ART. 1306. The contracting parties may establish such stipulations clauses, terms and conditions as they may deem convenient, provided they are not contrary to law, morals, good customs, public order, or public policy. - Finally, it would be unjust and inequitable if recovery on the policy would not be permitted against Petitioner, which had consistently granted a 60- to 90-day credit term for the payment of premiums despite its full awareness of Section 77. Estoppel bars it from taking refuge under said Section since Respondent relied in good faith on such practice. Estoppel then is the fifth exception to Section 77. Disposition Judgment reconsidered and set aside, that of the Court of Appeals affirmed in toto. SEPARATE OPINION

Page

VITUG - An essential characteristic of an insurance is its being synallagmatic, a highly reciprocal contract where the rights and obligations of the parties correlate and mutually correspond. - By weight of authority, estoppel cannot create a contract of insurance, neither can it be successfully invoked to create a primary liability, nor can it give validity to what the law so procribes as a matter of public policy. PARDO [dissent] - An assureds failure to give notice of the fire immediately upon its occurrence blatantly showed the fraudulent character of its claims. Respondent is required by law and by express terms of the policy to give immediate written notice of loss. This must be complied with in the utmost good faith. - Assuming arguendo that the 60- to 90-day credit has been agreed between the parties, respondent could not still invoke estoppel to back up its claim. Estoppel cannot give validity to an act that is prohibited by law or against public policy. The actual payment of premiums is a condition precedent to the validity of an insurance contract other than life insurance policy. Any agreement to the contrary is void as against law and public policy. ACME SHOE RUBBER & PLASTIC CORP. v. CA (DOMESTIC INSURANCE COMPANY OF THE PHILS.) 134 SCRA 155 MELENCIO-HERRERA; January 17, 1985. NATURE Petition for Review on Certiorari of the Decision of the then Court of Appeals (CA-G. R. No. 58917-R), denying recovery on an insurance policy, thereby reversing the judgment of the Court of First Instance of Rizal, Branch XII, at Caloocan City, which had allowed such recovery. FACTS - ACME Shoe Rubber and Plastic Corporation (ACME) had been insuring yearly against fire its building, machines and general merchandise with Domestic Insurance Company (INSURER) since 1946. On May 14, 1962, ACME continued to insure its properties

INSURANCE 37
with INSURER in the amount of P200,000 for the period May 15, 1962 up to May 15, 1963. - On May 14, 1963, INSURER issued Renewal Receipt to cover the period May 15, 1963 to May 15, 1964. - On January 8, 1964, ACME paid P3,331.26 as premium. The INSURER applied the payment as renewal premium for the period of May 15, 1963 to May 15, 1964. - On May 15, 1964, INSURER issued a Renewal Receipt for the period of May 15, 1964 to May 15, 1965 (for renewal premium of P3,331.26 yet to be paid) with a stamped note that says that the insurance will be deemed valid and binding only when the premium and documentary stamps have actually been paid in full and duly acknowledged in an official receipt. ACME was given 90 days to pay otherwise the policy would automatically become void and ineffective. (ACME should pay short period premium for 90 days before the period expires. If they are able to pay the whole amount before the 90-day period, the automatic termination wont apply anymore). - On May 26, 1964, ACME, through its President, signed a promissory note saying that they promise to pay the premium and documentary stamps and agreed to the automatic cancellation penalty for not complying. - On October 13, 1964, ACMEs properties were completely destroyed by fire. ACME filed insurance claim but the INSURER disclaimed liability on the ground that as of the date of loss, the properties burned were not covered by insurance. - ACME claims that the January 8, 1964 payment was for the period 1964-1965 and that INSURER had no right to apply it to the period 1963-1964 because under RA 3540, the policy was void and INSURER could have validly disclaimed liability for loss had one occurred then. - TC found INSURER liable for P200k and opined that there was a clear intention on the INSURER's part to grant ACME a credit extension for the payment of the premium due; and that to allow the INSURER to apply the premium ACME paid on January 8, 1964. CA reversed TC and dismissed the suit on the ground that, as of the moment of loss, ACME's properties were not insured and the INSURER could not be held liable for any indemnity as a result of the loss. ISSUE WON the premium payment for 1964-1965 was paid HELD NO - Not having paid the 1964-1965 premium within the extension granted, and pursuant to R.A. No. 3540, the policy was automatically cancelled and there was no insurance coverage to speak of as of the date of the fire on October 13, 1964. - The pertinent provision of Republic Act No. 3540 reads: "Sec. 72. An insurer is entitled to payment of the premium as soon as the thing insured is exposed to the peril insured against, unless there is clear agreement to grant the insured credit extension of the premium due. No policy issued by an insurance company is valid and binding unless and until the premium thereof has been paid." - RA 3540 was approved on June 20, 2963 and was put into effect on Oct 1, 1963. It could not be applied retroactively to the renewal of the policy for the 19631964 period because said policy was renewed on May 14, 1963. (Laws have no retroactive effect unless the contrary is provided.) Therefore, the Jan 8, 1964 payment was properly applied to the 1963-1964 premium. The Trial Court's opinion that there was a clear agreement to grant ACME credit extension for 1964-1965 is negated by ACME's Promissory Note binding itself to pay within ninety days from the effective date of this policy, 15th May, 1964. The credit extension was granted for 90 days only. (So wala na by August 16, 1964.) - If ACME was granted credit extensions in the past, the promissory note it signed did away with such credit arrangement. Also, before RA 3540, the Renewal Receipts issued by INSURER did not contain the auto-cancellation after 90 days note. By 1964, however, the situation had changed by the passage of the RA: no policy could be valid and binding unless and until the premium thereof had been paid. - What became automatically cancelled by R.A. No. 3540 was the 1964-1965 policy for ACME's failure to pay the premium within the 90-day extension granted, and in accordance with the express terms of the Promissory Note that it had signed. Disposition The judgment under review is hereby affirmed. Without pronouncement as to costs. PEDRO ARCE v. THE CAPITAL INSURANCE & SURETY CO., INC. 11 SCRA 63 ABAD SANTOS; September 30, 1982.

Page
NATURE Appeal from CFI decision on question of law. FACTS - Arce (INSURED) owned a residential house which was insured with the appellant COMPANY since 1961. In November 1965, the COMPANY sent to the INSURED a Renewal Certificate to cover the period from December 5, 1965 to December 5,1966, and requested payment of the corresponding premium. Anticipating that the premium could not be paid on time, the INSURED asked for an extension which was granted by the COMPANY. After the lapse of the requested extension, INSURED still failed to pay the premium. Thereafter, the house of the INSURED was totally destroyed by fire. Upon INSURED's presentation of claim for indemnity, he was told that no indemnity was due because the premium was not paid. The INSURED sued the COMPANY for indemnity. - The trial court held the COMPANY liable to indemnify the INSURED on the ground that since the COMPANY could have demanded payment of the premium, mutuality of obligation required that it should be liable on the policy. ISSUE WON the COMPANY can be held liable on its policy HELD NO. - The Court commiserates with the INSURED. They are well aware that many insurance companies have fallen into the condemnable practice of collecting premiums promptly but resort to all kinds of excuses to deny or delay payment of just claims. Unhappily the instant case is one where the insurer has the law on its side. - Sec. 72 of the Insurance Act, as amended by R.A. No. 3540 reads: "SEC. 72. An insurer is entitled to payment of premium as soon as the thing insured is exposed to the perils insured against, unless there is clear agreement to grant credit extension for the premium due. No policy issued by an insurance company is valid and binding unless and until the premium thereof has been paid." - It is obvious from both the Insurance Act, as amended, and the stipulation of the parties that time is of the essence in respect of the payment of the insurance premium so that if it is not paid the contract does not take effect unless there is still

INSURANCE 38
another stipulation to the contrary. In the instant case, the INSURED was given a grace period to pay the premium but the period having expired with no payment made, he cannot insist that the COMPANY is nonetheless obligated to him. - Prior to the amendment (italicized portion above), an insurance contract was effective even if the premium had not been paid so that an insurer was obligated to pay indemnity in case of loss and correlatively he had also the right to sue for payment of the premium. But the amendment to Sec. 72 has radically changed the legal regime in that unless the premium is paid there is no insurance. Disposition The decision of the court a quo is reversed; the appellee's complaint is dismissed. No special pronouncement as to costs. - Irrelevant facts: The premium costs P38.10. After the fire, the COMPANY issued a check for P300 to Arce as donation. Arce accepted the check, but still sued the company. CAPITAL INC. v. PLASTIC ERA CO. 65 SCRA 134 MARTIN; July 18, 1975 NATURE Petition for review of a decision of the CA affirming the decision of the CFI of Manila FACTS - On December 17, 1960, petitioner Capital Insurance & Surety Co., Inc. delivered to the respondent Plastic Era Manufacturing Co., Inc., its open Fire Policy No. 22760 wherein the former undertook to insure the latter's building, equipments, raw materials, products and accessories located at Sheridan Street, Mandaluyong, Rizal. The policy expressly provides that if the property insured would be destroyed or damaged by fire after the payment of the premiums, at anytime between the 15th day of December 1960 and one o'clock in the afternoon of the 15th day of December 1961, the insurance company shall make good all such loss or damage in an amount not exceeding P100,000.00. When the policy was delivered, Plastic Era failed to pay the corresponding insurance premium. On January 8, 1961, in partial payment of the insurance premium, Plastic Era delivered to Capital Insurance, a check for the amount of P1,000.00 postdated January 16, 1961. However, Capital Insurance tried to deposit the check only on February 20, 1961 and the same was dishonored by the bank for lack of funds. - Two days after the insurance premium became due, at about 4:00 to 5:00 o'clock in the morning, the property insured by Plastic Era was destroyed by fire. In less than a month Plastic Era demanded from Capital Insurance the payment of the sum of P100,000.00 as indemnity for the loss of the insured property under Policy No. 22760 but the latter refused for the reason that, among others, Plastic Era failed to pay the insurance premium. ISSUES 1. WON a contract of insurance has been duly perfected between petitioner and respondent 2. WON the dishonored check constituted payment HELD 1. YES - Tender of draft or check in order to effect payment that would extinguish the debtor's liability should be actually cashed. If the delivery of the check of Plastic Era to Capital Insurance were to be viewed in the light of the foregoing, no payment of the premium had been effected. Significantly, Capital Insurance accepted the promise of Plastic Era to pay the insurance premium within 30 days from the effective date of policy. By so doing, it has implicitly agreed to modify the tenor of the insurance policy and in effect, waived the provision therein that it would only pay for the loss or damage in case the same occurs after the payment of the premium. Considering that the insurance policy is silent as to the mode of payment, Capital Insurance is deemed to have accepted the promissory note in payment of the premium. This rendered the policy immediately operative on the date it was delivered. 2. YES - Although the check was due for payment on January 16, 1961 and Plastic Era had sufficient funds to cover it as of January 19, 1961, Capital Insurance decided to hold the same for thirty-five (35) days before presenting it for payment. Having held the check for such an unreasonable period of time, Capital Insurance was estopped from claiming a forfeiture of its policy for non-payment even if the check had been dishonored later. Where the check is held for an unreasonable time before presenting it for payment, the insurer may be held estopped from claiming a forfeiture if the check is dishonored. Disposition The decision of the CA is AFFIRMED in toto.

Page
MALAYAN INSURANCE CO., INC. v. ARNALDO and PINCA 154 SCRA 672 CRUZ; October 12, 1987 FACTS - On June 7, 1981, Malayan Insurance Co. (MICO), issued fire insurance for the amount of P14,000 on the property of private respondent, Pinca, effective July 1981-1982. MICO later allegedly cancelled the policy for non-payment of the premium and sent a notice to Pinca. On Dec. 24 Adora, an agent of MICO, received Pincas payment, which was remitted to MICO. On Jan. 18, 1982, Pincas property was completely burned. On Feb. 5, MICO returned Pincas payment to Adora on the ground that her policy had been cancelled; the latter refused to accept it. Her demand for payment having been rejected by MICO, Pinca went to the Insurance Commission. Public respondent Arnaldo, the Insurance Commissioner, sustained Pinca, hence this petition from MICO. Records show MICO received Arnaldos decision on April 10; MICO filed a MFR on April 25 which was denied on June 4; MICO received notice of this denial on June 14; instant petition was filed on July 2. ISSUES Procedural 1. WON the petition should be dismissed for late filing Substantive 2. WON there was a valid insurance contract at the time of the loss 3. WON Adora was authorized to receive such payment 4. WON an adjuster is indispensable in the valuation of the loss HELD Procedural 1. YES - Petitioner invokes Sec 416 of the Insurance Code which grants it 30 days from notice of the Insurance Commission within which to appeal by certiorari with the Court. MICO filed its MFR on April 25, 15 days after the notice; the reglementary period began to run again after June 13. Since the petition was filed only on July 2, it was tardy by 4 days. Alternatively it invokes Rule 45 of the Rules of Court for certiorari but the petition still exceeds the 15 day limit from the June 13 notice.

INSURANCE 39
-Respondents, on the other hand, invoke Sec. 39 of B.P. 129 which pegs the period for appeal from decisions of any court in all cases at 15 days from the notice of the decision appealed from. Since the MFR was filed only 15 days after receiving notice of the decision, it was already 18 days late by July 2. So whichever is applied, the petition is still late. Substantive 2. YES - A valid cancellation requires the following conditions based on Sections 64-65 of the Code: prior notice which must be based on the occurrence of one or more of the grounds mentioned in Sec 64 (in this case, non-payment of premium), after the effective date of the policy; the notice must be written and mailed to the address on the policy; it must state the ground(s) for cancellation and the insurer must furnish details upon the request of the insured. - It is undisputed that payment of premium was made. Petitioner relies heavily on Sec 77 of the Insurance Code to contest this, the said provision requiring payment of premium as soon as the thing is exposed to the peril insured against and that the policy is invalid without it. However, this is not applicable in the instant case as payment was eventually made. It is to be noted that the premium invoice was stamped Payment Received, indicating an understanding between the parties that payment could be made later. This is furthered by the fact that Adora had earlier told her to call him anytime she was ready with her payment. The Court also finds it strange that MICO only sought to return Pincas Jan. 15 payment only on Feb. 5, long after her house had burned downthis makes petitioners motives highly suspect. - MICO claims to have sent a notice to Pinca, who flatly denied receiving one. Pinca did not have to prove this since the strict language of Sec 64 requires that MICO ensure the cancellation was actually sent to and received by the insured. - MICO also suggests that Pinca knew the policy had been cancelled and was paying the premium in order to renew the policy. A close study of the transcripts show, however, that Pinca only meant to renew the policy had it been cancelled but not if it was still in effectit was conditional. Payment was thus legally made on the original transaction and validly received by Adora, who was not informed of the alleged cancellation and thus saw no reason to reject the payment. 3. YES - Sec. 306 of the Insurance Code provides that any insurance company that delivers a policy to its agent is deemed to have authorized such agent to receive payment of premium on its behalf. It is a well-known principle under the law of agency that payment to an authorized agent is equivalent to payment to the principal himself. MICOs acknowledgement of Adora as its agent thus defeats its contention that he was not authorized to receive payments on its behalf. 4. NO - In absence of fraud, the amount of the loss may be determined on the basis of such proof offered by the insured. Here. The certification of the Integrated National Police as the extent of the loss should suffice. Disposition petition is DENIED MANUFACTURERS LIFE INSURANCE CO. v. MEER 89 PHIL 351 BENGZON, June 29, 1951 NATURE APPEAL from a judgment of the Court of First Instance of Manila FACTS (this is a tax case. Whats really important here is the definition of CASH SURRENDER VALUE). - Manufacturers Life Insurance Company is a duly organized corporation which has its head office at Toronto. It is duly registered and licensed to engage in life insurance business in the Philippines, and, maintains a branch office in Manila. It was engaged in such business in the Philippines for more than five years before and including the year 1941. But due to the exigencies of the war It closed the branch office at Manila during 1942 up to September 1945. - Plaintiff issued a number of life insurance policies in the Philippines containing stipulations referred to as NONFORFEITURE CLAUSES5
5"'8. Automatic Premium Loan.-This Policy shall not lapse for non-payment of any premium after it has been three full
years in force, it, at the due date of such premium, the Cash Value of this Policy and of any bonus additions and dividends left on accumulation (after deducting any indebtedness to the company and the interest accrued thereon) shall exceed the amount of said premium. In which event the company will, without further request, treat the premium then due as paid, and the amount of such premium, with interest from its actual due date at six per cent per annum, compounded yearly, and one per cent, compounded yearly, for expenses, shall be a first lien on this Policy in the Company's favour in priority to the claim of any assignee or any other person. The accumulated lien may at any time, while the Policy is in force, be paid in whole or in part. 'When the premium falls due and is not paid in cash within the month's grace, if the Cash Value of this policy and of any bonus additions and dividends left on accumulation (after deducting any accumulated indebtedness) be less than the premium then due, the Company will, without further requests, continue this insurance in force for a period * * *. '10. Cash and Paid-Up Insurance Values.-At the end of the third policy year or thereafter, upon the legal surrender of this Policy to the Company while there is no default in premium payments or within two months after the due date of the premium in default, the Company will (1) grant a cash value as specified in Column (A) increased by the cash value of any bonus additions and dividends left on accumulation, which have been alloted to this Policy, less all indebtedness to the

Page
- From January 1, 1942 to December 31, 1946, Plaintiff head office at Toronto applied the provisions of the automatic premium loan clauses upon the nonpayment of the corresponding premiums by the people who subscribed to the insurance. The net amount of premiums advanced (by the company) or loaned (to the insured) as payment for the premium due totaled P1,069,254.98. - Meer, the Collector of the National Internal Revenue assessed the net amount of premium at P17,917.12 pursuant to SEC.255, National Internal Revenue Code6 - Company protested the assessment, but paid the taxes anyway. Then they filed a complaint to recover money paid under protest for taxes - CFI: Dissmiss complaint - PLAINTIFFs MAIN CONTENTION: when it made premium loans or premium advances by virtue of the non-forfeiture clauses, it did not collect premiums within the meaning of the above sections of the law, and therefore it is not amenable to the tax therein provided. ISSUES 1. WON premium advances made by plaintiffappellant under the automatic premium loan clause of its policies are premiums collected' by the Company subject to tax 2. WON, in the application of the automatic premium loan clause of plaintiff-appellant's policies, there is 'payment in money, notes, credits, or any substitutes for money 3. WON the collection of the alleged deficiency premium taxes constitutes double taxation 4. WON the making of premium advances, granting for the sake of argument that it amounted to collection of premiums, were done in Toronto, Canada 5. WON the fact that plaintiff-appellant was not doing business in the Philippines during the period from January 1, 1942 to September 30, 1945, inclusive, exempts it from payment of premium taxes corresponding to said period
Company on this Polley an the date of ouch surrender, or (2) endorse this Policy as a Non-Participating Paid-up Polley for the amount as specified In Column (B) of the Table of Guaranteed Values * * *. '11. Extended Insurance-After the premiums for three or more full years have been paid hereunder in cash, if any subsequent premium is not paid when due, and there is no indebtedness to the Company on the written request of the insured * * *."

6"SEC. 255. Taxes on insurance premiums.-There shall be collected from every person, company, or corporation (except
purely cooperative companies or associations) doing insurance business of any sort in the Philippines a tax of one per centum of the total premiums collected * * * whether such permiums are paid in money, notes, credits, or any substitute for money but premiums refunded within six months after payment on account of rejection of risk or returned for other reason to person insured shall not be included in the taxable receipts * * *."

INSURANCE 40
premiums paid." (Cyclopedia Law Dictionary 3d. ed. 1077.) The cash value or cash surrender value is therefore an amount which the insurance company holds In trust for the insured to be delivered to him upon demand. It is therefore a liability of the company to the insured. Now then, when the company's credit for advances is paid out of the cash value or cash surrender value, that value and the company's liability is thereby diminished pro tanto. 2. YES - the insurer agreed to consider the premium paid on the strength of the automatic loan. The premium was therefore paid by means of a "note" or "credit" or "other substitute for money" and the tax is due because section 255 above quoted levies taxes according to the total premiums collected by the insurer "whether such premiums are paid in money, notes, credits or any substitute for money. 3. NO - No constitutional prohibition against double taxation. 4. NO - The loans are made to policyholders in the Philippines, who in turn pay therewith the premium to the insurer thru the Manila branch. Approval of appellant's position will enable foreign insurers to evade the tax by contriving to require that premium payments shall be made at their head offices. What is important, the law does not contemplate premiums collected in the Philippines. It is enough that the insurer is doing insurance business in the Philippines, irrespective of the place of its organization or establishment. 5. NO - Although during those years the appellant was not open for new business because its branch office was closed, still it was practically and legally, operating in this country by collecting premiums on its outstanding policies, incurring the risks and/or enjoying the benefits consequent thereto, without having previously taken any steps indicating withdrawal in good faith from this field of economic activity. Disposition finding no prejudicial error in the appealed decision, we hereby affirm it with costs. ANDRES v. CROWN LIFE INSURANCE 102 Phil. 919 REYES, J.B.L., Jan.28, 1958 NATURE Appeal from judgment of CFI

Page

HELD NOTE (example given by the plaintiff): "Suppose that 'A', 30 years of age, secures a 20-year endowment policy for P5,000 from plaintiff-appellant Company and pays an annual premium of P250. 'A' pays the first ten yearly premiums amounting to P2,500 and on this amount plaintiff-appellant pays the corresponding taxes under section 255 of the National Internal Revenue Code. Suppose also that the cash value of said policy after the payment of the 10th annual premium amounts to P1,000." When on the eleventh year the annual premium fell due and the insured remitted no money within the mouth grace, the insurer treated the premium then over due as paid from the cash value, the amount being a loan to the policyholder1 who could discharge it at any time with interest at 6 per cent. The insurance contract, therefore, continued in force for the eleventh year. 1. YES - Based on the example given by the plaintiff, the insurer collected the amount of P250 as the annual premium for the eleventh year on the said policy when it loaned to A the sum of P250. The insurer became a creditor of the loan, but not of the premium that had already been paid (advanced by the insurer). The insurer is entitled to collect interest on the loan, not on the premium. "A" paid the premium for the eleventh year; but in turn he became a debtor of the company for the sum of P250. This debt he could repay either by later remitting the money to the insurer or by letting the cash value compensate for it. The debt may also be deducted from the amount of the policy should "A" die thereafter during the continuance of the policy. - ON ARGUMENT THAT THE ASSETS OF THE INSURER REMAINED THE SAME AFTER THE APPLICATION OF THE AUTOMATIC PREMIUM LOAN CLAUSE: there was an increase in assets in the form of CREDIT for the advances made (in the example, the P250 for the 11th year). - ON ARGUMENT THAT IF THE CREDIT IS PAID OUT OF THE CASH SURRENDER VALUE, THERE WERE NO NEW FUNDS ADDED TO THE COMPANY'S ASSETS: Cash surrender value "as applied to a life insurance policy, is the amount of money the company agrees to pay to the holder of the policy if he surrenders it and releases his claims upon it. The more premiums the insured has paid the greater will be the surrender value; but the surrender value is always a lesser sum than the total amount of

FACTS - Feb. 13, 1950: For the sum of P5,000, defendantappellee Crown Life issued an insurance policy in the name of plaintiff-appellant Rufino and his wife, with the stipulation that the premiums are to be paid semiannually. - The premiums for the 1st and 2nd semester of the 1st year, in the amount of P165.15 were paid by Rufino but the premium for the third semester, in the same amount, was not paid. - Jan. 6, 1951, Crown Life, through its branch secretary, wrote to Mr. and Mrs. Andres advising them that their insurance policy lapsed on Dec. 26, 1950 and the amount of P165.15 was overdue, giving them 60 days from the date of lapse to file an application for reinstatement. Crown Life later sent another letter telling the spouses Andres that their insurance policy was no longer in force. - Feb. 1951: Plaintiff and his wife executed a Statement of Health and application for reinstatement of the aforesaid policy. - Feb. 20, 1951: Plaintiff wrote a letter to the defendant, enclosed with a money order for P100. Upon acceptance, defendant advised Rufino that its main office had approved the application and that the reinstatement of the lapsed policy was subject to the payment of the remaining premium balance of P65.15. - May 3, 1951: Severa Andres died of dystocia, contracted pelvis. - May 5, 1951: Plaintiff sent a letter enclosed with a money order in the amount of P65, for the remaining balance due. - May 15, 1951: Defendant sent a letter with official receipt of the P165.15 paid by Rufino as well as a Certificate of Reinstatement. - June 7, 1951: Rufino presented a death claim as survivor-beneficiary of his deceased wife. Payment was denied by the defendant. - April 1952: Rufino filed a complaint in CFI against Crown Life for the recovery of the amount of P5,000 as the face value of a joint 20-year endowment insurance policy issued by defendant in favor of plaintiff and his wife, on Feb. 13, 1950. In its answer,

INSURANCE 41
Crown Life disclaimed liability and set forth the special defense that the aforementioned policy had already lapsed. - Aug. 5, 1954: CFI rendered a decision absolving the defendant company from any liability on the ground that the policy had lapsed and it was not reinstated at the time of the plaintiffs wifes death. Plaintiff later appealed to the CA but the same was certified by the CA to the SC for having no question of fact. ISSUE WON the insurance policy, which has been in a state of lapse before May 3, 1951, has been validly and completely reinstated after said date (Was there a perfected contract of reinstatement after the policy lapsed due to non-payment of premiums?) HELD NO Ratio The stipulation in a life insurance policy giving the insured the privilege to reinstate it upon written application does not give the insured absolute right to such reinstatement by the mere filing of an application. The Company has the right to deny the reinstatement if it is not satisfied as to the insurability of the insured and if the latter does not pay all overdue premiums and all other indebtedness to the Company. After the death of the insured the insurance Company cannot be compelled to entertain an application for reinstatement of the policy because the conditions precedent to reinstatement can no longer be determined and satisfied. Reasoning - The stipulations of facts render it undisputable that the original policy lapsed for non-payment of premiums on Dec. 26, 1950, upon expiration of the 31-day grace period. - As found by the lower court, the conditions set forth in the policy for reinstatement as provided in the contract itself are the following: (A) application shall be made within 3 years from the date of lapse; (B) there should be a production of evidence of the good health of the insured; (C) if the rate of premium depends upon the age of the Beneficiary, there should likewise be a production of evidence of his or her good health; (D) there should be presented such other evidence of insurability at the date of application for reinstatement; (E) there should be no change which has taken place in such good health and insurability subsequent to the date of such application and before the policy is reinstated; and (F) all overdue premiums and other indebtedness in respect of the policy, together with interest at 6%, compounded annually, should first be paid. - The plaintiff did not comply with the last condition; for he only paid P100 before his wifes death; and despite the Companys reminders, he only remitted the balance of P65.15 two days after his wife died. On the face of such facts, the Company had the right to treat the contract as lapsed and refuse payment of the policy. - Rufino contends that the condition regarding payment of the premium was waived by the insurance Company through its letters, wherein it made statements such as: If you are unable to pay the full amount immediately, send as large amount as possible and advise us how soon you expect to be able to pay the balance; we will work out an adjustment most beneficial to you. The Court found the statements to be too vague and indefinite to indicate an intention on the insurers part to waive the full payment as prerequisite to the reinstatement of the lapsed policy. The Court reiterated the rule that a waiver must be clear and positive, the intent to waive shown clearly and convincingly. On the other hand, It found subsequent letters sent by defendant indicating that they insisted on full payment of the premium before the policy was reinstated and that defendant did not consider partial payment as sufficient consideration for the reinstatement. PlaintiffAppellants failure to remit the balance before the death of his wife operated to deprive him of any right to waive the policy and recover the face value thereof. Disposition Judgment appealed from is affirmed. VALENZUELA v. CA (PHILIPPINE AMERICAN GENERAL INSURANCE COMPANY, INC.) 191 SCRA 1 GUTIERREZ; October 19, 1990 NATURE Petition for review of the decision of theca. FACTS - Petitioner Arturo P. Valenzuela is a General Agent of private respondent Philippine American General Insurance Company, Inc. (Philamgen for short) since 1965. As such, he was authorized to solicit and sell in behalf of Philamgen all kinds of non-life insurance, and in consideration of services rendered was entitled to receive the full agent's commission of 32.5% from Philamgen under the scheduled commission rates.

Page
- From 1973 to 1975, Valenzuela solicited marine insurance from one of his clients, the Delta Motors, Inc. (Division of Electronics Airconditioning and Refrigeration) in the amount of P4.4 Million from which he was entitled to a commission of 32%. However, Valenzuela did not receive his full commission which amounted to P1.6 Million from the P4.4 Million insurance coverage of the Delta Motors. During the period 1976 to 1978, premium payments amounting to P1,946,886.00 were paid directly to Philamgen and Valenzuela's commission to which he is entitled amounted to P632,737.00. - In 1977, Philamgen started to become interested in and expressed its intent to share in the commission due Valenzuela on a fifty-fifty basis. Valenzuela refused. - Because of the refusal of Valenzuela, Philamgen and its officers took drastic action against Valenzuela. They: (a) reversed the commission due him by not crediting in his account the commission earned from the Delta Motors, Inc. insurance ; (b) placed agency transactions on a cash-and-carry basis; (c) threatened the cancellation of policies issued by his agency; and (d) started to leak out news that Valenzuela has a substantial account with Philamgen. All of these acts resulted in the decline of his business as insurance agent. - Then on December 27, 1978, Philamgen terminated the General Agency Agreement of Valenzuela. - Lower court: the termination of Valenzuela as General Agent was improper because the record will show the principal cause of the termination of the plaintiff as General Agent of defendant Philamgen was his refusal to share his Delta commission. - CA: In any event the principal's power to revoke an agency at will is so pervasive, that the Supreme Court has consistently held that termination may be effected even if the principal acts in bad faith, subject only to the principal's liability for damages. (CA ordered Valenzuela to pay Philamgen the amount of One Million Nine Hundred Thirty-Two Thousand Five Hundred Thirty-Two and 17/100 Pesos (P1,932,532.17) with legal interest) ISSUES 1. WON whether or not Philamgen and/or its officers can be held liable for damages due to the termination of the General Agency Agreement it entered into with the petitioners 2. WON petitioners are liable to Philamgen for the unpaid and uncollected premiums

INSURANCE 42
HELD 1. YES - If a principal acts in bad faith and with abuse of right in terminating the agency, then he is liable in damages. - There is an exception to the principle that an agency is revocable at will and that is when the agency has been given not only for the interest of the principal but for the interest of third persons or for the mutual interest of the principal and the agent. In these cases, it is evident that the agency ceases to be freely revocable by the sole will of the principal (PROCEDURAL: Where the findings of the Court of Appeals and the trial court are contrary to each other, this Court may scrutinize the evidence on record - After a painstaking review of the entire records of the case and the findings of facts of both the court a quo and respondent appellate court, the Court affirmed the trial courts findings.) - The principal cause of the termination of Valenzuela as General Agent of Philamgen arose from his refusal to share his Delta commission. The records sustain the conclusions of the trial court on the apparent bad faith of the private respondents in terminating the General Agency Agreement of petitioners. - It is also evident from the records that the agency involving petitioner and private respondent is one "coupled with an interest," and, therefore, should not be freely revocable at the unilateral will of the latter. - The private respondents by the simple expedient of terminating the General Agency Agreement appropriated the entire insurance business of Valenzuela. With the termination of the General Agency Agreement, Valenzuela would no longer be entitled to commission on the renewal of insurance policies of clients sourced from his agency. Worse, despite the termination of the agency, Philamgen continued to hold Valenzuela jointly and severally liable with the insured for unpaid premiums. Under these circumstances, it is clear that Valenzuela had an interest in the continuation of the agency when it was unceremoniously terminated not only because of the commissions he should continue to receive from the insurance business he has solicited and procured but also for the fact that by the very acts of the respondents, he was made liable to Philamgen in the event the insured fail to pay the premiums due. They are estopped by their own positive averments and claims for damages. - "The principal may not defeat the agent's right to indemnification by a termination of the contract of agency (Erskine v. Chevrolet Motors Co. 185 NC 479, 117 SE 706, 32 ALR 196). - For the pivotal factor rendering Philamgen and the other private respondents liable in damages is that the termination by them of the General Agency Agreement was tainted with bad faith. This is in accordance with the precepts in Human Relations enshrined in our Civil Code. 2. NO. The respondent court erred in holding Valenzuela liable. There was no factual and legal basis for the award. Under Section 77 of the Insurance Code, the remedy for the non-payment of premiums is to put an end to and render the insurance policy not binding - "Sec. 77 . . . [N]otwithstanding any agreement to the contrary, no policy or contract of insurance is valid and binding unless and until the premiums thereof have been paid except in the case of a life or industrial life policy whenever the grace period provision applies (P.D. 612, as amended otherwise known as the Insurance Code of 1974) - This is buttressed by Section 776 of the Insurance Code (Presidential Decree No. 612, promulgated on December 18, 1974), which now provides that no contract of Insurance by an insurance company is valid and binding unless and until the premium thereof has been paid, notwithstanding any agreement to the contrary." Disposition Petition is GRANTED. CA decision SET ASIDE. The decision of the TC REINSTATED with the MODIFICATIONS. And that the contractual relationship between Arturo P. Valenzuela and Philippine American General Insurance Company shall be deemed terminated upon the satisfaction of the judgment as modified. CHAPTER V THE POLICY, PARTIES THERETO, & RIGHTS THEREON DE LIM v. SUN LIFE ASSURANCE COMPANY OF CANADA 41 PHIL 263 MALCOLM; November 29, 1920 NATURE Appeal from an order of the CFI of Zamboanga sustaining a demurrer to plaintiff's complaint upon the ground that it fails to state a cause of action. FACTS

Page
- On July 6, 1917, Luis Lim of Zamboanga made application to the Sun Life Assurance Company of Canada for a policy of insurance on his life in the sum of P5,000. In his application Lim designated his wife, Pilar de Lim, the plaintiff herein, as the beneficiary. The first premium of P433 was paid by Lim, and upon such payment the company issued what was called a ''provisional policy." Luis Lim died on August 23, 1917, after the issuance of the provisional policy but before approval of the application by the home office of the insurance company. Pilar de Lim brought an action to recover from the Sun Life sum of P5,000, the amount named in the provisional policy. - The "provisional policy" reads: "Received (subject to the following stipulations and agreements) the sum of P433, being the amount of the first year's premium for a Life Assurance Policy on the life of Mr. Luis D. Lim of Zamboanga for P5,000, for which an application dated the 6th day of July, 1917, has been made to the Sun Life Assurance Company of Canada. - The above-mentioned life is to be assured in accordance with the terms and conditions contained or inserted by the Company in the policy which may be granted by it in this particular case for four months only from the date of the application, provided that the Company shall confirm this agreement by issuing a policy on said application when the same shall be submitted to the Head Office in Montreal. Should the Company not issue such a policy, then this agreement shall be null and void ab initio, and the Company shall be held not to have been on the risk at all, but in such case the amount herein acknowledged shall be returned. ISSUE WON the contract of insurance between Luis Lim and Sun Life Assurance Company of Canada was perfected HELD NO. - The document it is to be a provisional policy "for four months only from the date of this application." Immediately following the words fixing the four months period comes the word "provided" which has the meaning of "if." Otherwise stated, the policy for four months is expressly made subject to the affirmative condition that the company shall confirm this agreement by issuing a policy on said application when the same shall be submitted to the head office in Montreal. To re-enforce the same there follows the negative condition - "Should the company not issue

INSURANCE 43
such a policy, then this agreement shall be null and void ab initio, and the company shall be held not to have been on the risks." Certainly language could hardly be used which would more clearly stipulate that the agreement should not go into effect until the home office of the company should confirm it by issuing a policy. As we read and understand the socalled provisional policy, it amounts to nothing but an acknowledgment on behalf of the company, that it has received from the person named therein the sum of money agreed upon as the first year's premium upon a policy to be issued upon the application, if the application is accepted by the company. - It is of course a primary rule that a contract of insurance, like other contracts, must be assented to by both parties either in person or by their agents. So long as an application for insurance has not been either accepted or rejected, it is merely an offer or proposal to make a contract. The contract, to be binding from the date of the application must have been a completed contract, one that leaves nothing to be done, nothing to be completed, nothing to be passed upon, or determined, before it shall take effect. There can be no contract of insurance unless the minds of the parties have met in agreement. Our view is, that a contract of insurance was not here consummated by the parties. - The trial court committed no error in sustaining the demurrer and dismissing the case. It is to be noted, however that counsel for appellee admits the liability of the company for the return of the first premium to the estate of the deceased. GREAT PACIFIC LIFE v. CA (LEUTERIO) 316 SCRA 677 QUISUMBING; October 13, 1999 NATURE Petition for Review of CA decision FACTS - A contract of group life insurance was executed between petitioner Great Pacific Life Assurance Corporation (hereinafter Grepalife) and Development Bank of the Philippines (hereinafter DBP). Grepalife agreed to insure the lives of eligible housing loan mortgagors of DBP. - In Nov. 1983, Dr. Wilfredo Leuterio, a physician and a housing debtor of DBP applied for membership in the group life insurance plan. In an application form, Dr. Leuterio answered Qs concerning his health condition as follows: Q: Have you ever had, or consulted, a physician for a heart condition, high blood pressure, cancer, diabetes, lung, kidney or stomach disorder or any other physical impairment? No. Q: Are you now, to the best of your knowledge, in good health? Yes. - Grepalife issued an insurance coverage of Dr. Leuterio, to the extent of his DBP mortgage indebtedness of P86,200.00. In Aug. 1984, Dr. Leuterio died due to "massive cerebral hemorrhage." DBP submitted a death claim to Grepalife. Grepalife denied the claim because Dr. Leuterio was not physically healthy when he applied for an insurance. Grepalife insisted that Dr. Leuterio did not disclose he had been suffering from hypertension, which caused his death. Allegedly, such non-disclosure constituted concealment that justified the denial of the claim. - Herein respondent Medarda Leuterio, widow, filed a complaint with RTC against Grepalife for "Specific Performance with Damages." Dr. Mejia, who issued the death certificate, testified that Dr. Leuterio complained of headaches presumably due to high blood pressure. The inference was not conclusive because Dr. Leuterio was not autopsied, hence, other causes were not ruled out. - RTC ruled in favor of respondent widow and against Grepalife. CA sustained the RTC decision. Hence, the present petition.

Page
for the benefit of another person named or unnamed, and although it is expressly made payable to another as his interest may appear or otherwise. Although a policy issued to a mortgagor is taken out for the benefit of the mortgagee and is made payable to him, yet the mortgagor may sue thereon in his own name, especially where the mortgagee's interest is less than the full amount recoverable under the policy. (See Sec. 8, Insurance Code) Reasoning [a] The insured private respondent did not cede to the mortgagee all his rights or interests in the insurance, the policy stating that: In the event of the debtor's death before his indebtedness with the Creditor (DBP) shall have been fully paid, an amount to pay the outstanding indebtedness shall first be paid to the creditor and the balance of sum assured, if there is any, shall then be paid to the beneficiary/ies designated by the debtor. When DBP submitted the insurance claim against Grepalife, the latter denied payment thereof, interposing the defense of concealment committed by the insured. Thereafter, DBP collected the debt from the mortgagor and took the necessary action of foreclosure on the residential lot of private respondent. [b] Since a policy of insurance upon life or health may pass by transfer, will or succession to any person, whether he has an insurable interest or not, and such person may recover it whatever the insured might have recovered, the widow of the decedent Dr. Leuterio may file the suit against the insurer, Grepalife. 2. NO Ratio The fraudulent intent on the part of the insured must be established to entitle the insurer to rescind the contract. Misrepresentation as a defense of the insurer to avoid liability is an affirmative defense and the duty to establish such defense by satisfactory and convincing evidence rests upon the insurer. In the case at bar, the petitioner failed to clearly and satisfactorily establish its defense, and is therefore liable to pay the proceeds of the insurance. Reasoning [a] The insured, Dr. Leuterio, had answered in his insurance application that he was in good health and that he had not consulted a doctor or any of the enumerated ailments, including hypertension; when he died the attending physician had certified in the death certificate that the former died of cerebral hemorrhage, probably secondary to hypertension. From this report, petitioner Grepalife refused to pay

ISSUES 1. WON CA erred in holding petitioner liable to DBP as beneficiary in a group life insurance contract from a complaint filed by the widow of the decedent/mortgagor 2. WON CA erred in not finding that Dr. Leuterio concealed that he had hypertension, which would vitiate the insurance contract 3. WON CA erred in holding Grepalife liable for P86,200.00 without proof of the actual outstanding mortgage payable by the mortgagor to DBP HELD 1. NO Ratio Insured, being the person with whom the contract was made, is primarily the proper person to bring suit. Subject to some exceptions, insured may thus sue, although the policy is taken wholly or in part

INSURANCE 44
the insurance claim. It alleged that the insured had concealed the fact that he had hypertension. [b] Contrary to Grepalifes allegations, there was no sufficient proof that the insured had suffered from hypertension. Aside from the statement of the insured's widow who was not even sure if the medicines taken by Dr. Leuterio were for hypertension, the appellant had not proven nor produced any witness who could attest to Dr. Leuterio's medical history. [c] Grepalife had failed to establish that there was concealment made by the insured, hence, it cannot refuse payment of the claim. 3. NO - Considering the supervening event that DBP foreclosed in 1995 their residential lot, in satisfaction of mortgagor's outstanding loan, the insurance proceeds shall inure to the benefit of the heirs of the deceased person or his beneficiaries. Equity dictates that DBP should not unjustly enrich itself at the expense of another. Hence, it cannot collect the insurance proceeds, after it already foreclosed on the mortgage. The proceeds now rightly belong to Dr. Leuterio's heirs represented by his widow, herein private respondent. - The Court ruled this issue based on the clear provisions of the policy. The mortgagor paid the premium according to the coverage of his insurance, which states that: "The policy states that upon receipt of due proof of the Debtor's death during the terms of this insurance, a death benefit in the amount of P86,200.00 shall be paid In the event of the debtor's death before his indebtedness with the creditor shall have been fully paid, an amount to pay the outstanding indebtedness shall first be paid to the Creditor and the balance of the Sum Assured, if there is any shall then be paid to the beneficiary/ies designated by the debtor." From this, it is clear that Grepalife is liable and that Dr. Leuterios heirs must get the proceeds. Disposition Petition DENIED. CA Decision AFFIRMED with modification. PACIFIC TIMBER EXPORT CORPORATION v. CA (WORKMENS INSURANCE CO) 112 SCRA 199 DE CASTRO; February 25, 1982 FACTS - March 19, 1963 - the plaintiff secured temporary insurance from the defendant for its exportation of 1,250,000 board feet of Philippine Lauan and Apitong logs to be shipped from the Diapitan Bay, Quezon to Okinawa and Tokyo, Japan. The defendant issued on said date Cover Note No. 1010, insuring the said cargo of the plaintiff "Subject to the Terms and Conditions of the WORKMEN'S INSURANCE COMPANY, INC. printed Marine Policy form as filed with and approved by the Office of the Insurance Commissioner. - April 2, 1963 - The two (2) regular marine cargo policies were issued by the defendant in favor of the plaintiff. The total cargo insured under the two marine policies accordingly consisted of 1,395 logs, or the equivalent of 1,195,498 bd. ft. - After the issuance of cover note but before the issuance of the two marine policies some of the logs intended to be exported were lost during loading operations in the Diapitan Bay due to bad weather. - April 4, 1963 - The plaintiff informed the defendant about the loss of 'approximately 32 pieces of logs' during loading through a letter. The plaintiff subsequently submitted a 'Claim Statement' demanding payment of the loss under the second marine cargo policy. - July 17, 1963 - the defendant requested the First Philippine Adjustment Corporation to inspect the loss and assess the damage. - August 23, 1963 - the adjuster reported that 'the loss of 30 pieces of logs is not covered by the two policies inasmuch as said policies covered the actual number of logs loaded on board. But it is covered by Cover Note. - On January 13, 1964 - the defendant wrote the plaintiff denying the latter's claim, on the ground that defendant's investigation revealed that the entire shipment of logs covered by the two marines policies were received in good order at their point of destination. It was further stated that the said loss may not be considered as covered under cover note because the said note had become 'null and void by virtue of the issuance of two marine policies. - The CFI of Manila ruled in favour of the petitioner. - The Court of Appeals reversed the decision of the CFI. ISSUES 1. WON the cover note is null and void for lack of valuable consideration because no separate premiums are collected by private respondent on all its cover notes

Page
2. WON the court of appeals erred in holding that private respondent was released from liability under the cover note due to unreasonable delay in giving notice of loss because the court disregarded the proven fact that private respondent did not promptly and specifically object to the claim on the ground of delay in giving notice of loss and, consequently, objections on that ground are waived under section 84 of the insurance act HELD 1. NO Ratio Cover note is issued with a consideration when, by express stipulation, the cover note is made subject to the terms and conditions of the marine policies, and the payment of premiums is one of the terms of the policies. Reasoning a. the cover note in question is subject to the terms and conditions of the marine policies b. Nature of the Cover Note: The fact that no separate premium was paid on the Cover Note before the loss insured against occurred, does not militate against the validity of petitioner's contention, for no such premium could have been paid, since by the nature of the Cover Note, it did not contain, as all Cover Notes do not contain particulars of the shipment that would serve as basis for the computation of the premiums. As a logical consequence, no separate premiums are intended or required to be paid on a Cover Note. c. The petitioner paid in full all the premiums as called for by the statement issued by private respondent after the issuance of the two regular marine insurance policies, thereby leaving no account unpaid by petitioner due on the insurance coverage, which must be deemed to include the Cover Note. If the Note is to be treated as a separate policy instead of integrating it to the regular policies subsequently issued, the purpose and function of the Cover Note would be set at naught or rendered meaningless, for it is in a real sense a contract, not a mere application for insurance which is a mere offer. Had all the logs been lost during the loading operations, but after the issuance of the Cover Note, liability on the note would have already arisen even before payment of premium. This is how the cover note as a "binder" should legally operate; otherwise, it would serve no practical purpose in the realm of commerce, and is supported by the doctrine that where a policy is delivered without requiring payment of the premium, the

INSURANCE 45
presumption is that a credit was intended and policy is valid. 2. NO - The private respondent company never raised this ground in the proceedings. It must be because it did not find any delay, as this Court fails to find a real and substantial sign thereof. But even on the assumption that there was delay, this Court is satisfied and convinced that as expressly provided by law, waiver can successfully be raised against private respondent. Thus Section 84 of the Insurance Act provides: "Section 84. - Delay in the presentation to an insurer of notice or proof of loss is waived if caused by any act of his or if he omits to take objection promptly and specifically upon that ground." - From what has been said, We find duly substantiated petitioner's assignments of error. Disposition The appealed decision is set aside and the decision of the Court of First Instance is reinstated in toto with the affirmance of this Court. DEVELOPMENT INSURANCE v. IAC (PHIL UNION REALTY DEVELOPMENT CORP) 143 SCRA 62 CRUZ; July 16, 1986 FACTS - A fire occurred in the building of the private respondent and it sued for recovery of damages from the petitioner on the basis of an insurance contract between them. The petitioner allegedly failed to answer on time and was declared in default by TC. A judgment of default was rendered on the strength of the evidence submitted ex parte by the private respondent, which was allowed full recovery of its claimed damages. - On learning of this decision, the petitioner moved to lift the order of default, invoking excusable neglect, and to vacate the judgment by default. Its motion was denied. - On appeal, IAC affirmed the TC decision in toto. ISSUE 1. WON default of petitioner is based on excusable neglect 2. What is the amount of indemnity due to the private respondent under its insurance contract? WON CFI was correct in interpreting the contract HELD 1. NO - Summons was served through its vice-president. There were even several extensions to the original period to answer. As a consequence, the TC, on motion of the private respondent filed declared the petitioner in default. This was done almost one month later. Even so, the petitioner made no move at all for two months thereafter. It was only more than one month after the judgment of default was rendered by the TC that it filed a motion to lift the order of default and vacate the judgment by default. - There is a pattern of inexcusable neglect. 2. The policy is an open policy which means that the actual loss, as determined, will represent the total indemnity due the insured from the insurer except only that the total indemnity shall not exceed the face value of the policy. - The petitioner argues that since at the time of the fire the building insured was worth P5,800,000.00, the private respondent should be considered its own insurer for the difference between that amount and the face value of the policy and should share pro rata in the loss sustained. Accordingly, the private respondent is entitled to an indemnity of only P67,629.31, the rest of the loss to be shouldered by it alone. The petitioner cites Condition 17 of the policy, which provides: "If the property hereby insured shall, at the breaking out of any fire, be collectively of greater value than the sum insured thereon then the insured shall be considered as being his own insurer for the difference, and shall bear a ratable proportion of the loss accordingly. Every item, if more than one, of the policy shall be separately subject to this condition." - However, there is no evidence on record that the building was worth P5,800,000.00 at the time of the loss. On the contrary, the building was insured at P2,500,000.00, and this must be considered, by agreement, the actual value of the property insured on the day the fire occurred. This valuation becomes even more believable if it is remembered that at the time the building was burned it was still under construction and not yet completed. - The Court notes that the policy in this case is an open policy and is subject to the express condition that: "Open Policy.

Page
This is an open policy as defined in Sec57 of the Insurance Act. In the event of loss, whether total or partial, it is understood that the amount of the loss shall be subject to appraisal and the liability of the company, if established, shall be limited to the actual loss, subject to the applicable terms, conditions, warranties and clauses of this Policy, and in no case shall exceed the amount of the policy." - As defined in the aforestated provision, which is now Sec60 of the Insurance Code, "an open policy is one in which the value of the thing insured is not agreed upon but is left to be ascertained in case of loss.". - The actual loss has been ascertained in this case and the Court will respect such factual determination in the absence of proof that it was arrived at arbitrarily. There is no such showing. Hence, applying the open policy clause as expressly agreed upon by the parties in their contract, we hold that the private respondent is entitled to the payment of indemnity under the said contract in the total amount of P508,867.00. - The refusal of its vice-president to receive the private respondent's complaint, as reported in the sheriff's return, was the first indication of the petitioner's intention to prolong this case and postpone the discharge of its obligation to the private respondent under this agreement. That intention was revealed further in its subsequent acts ---- or inaction ---- which indeed enabled it to avoid payment for more than five years from the filing of the claim against it in 1980. Disposition The appealed decision is affirmed in full, with costs against the petitioner. HARDING v. COMMERCIAL UNION ASSURANCE 38 PHIL 464 FISHER; August 10, 1918 FACTS - Mrs. Harding was the owner of a Studebaker automobile; in consideration of the payment to the defendant of the premium of P150, by said plaintiff, Mrs. Henry E. Harding, with the consent of her husband, the defendant by its duly authorized agent, Smith, Bell & Company (limited), made its policy of insurance in writing upon said automobile was set forth in said policy to be P3,000 that the value of said automobile was set forth in said policy to be P3,000; that on March 24, 1916, said automobile was totally

INSURANCE 46
destroyed by fire; that the loss thereby to plaintiffs was the sum of P3,000. - The defendants version is that by request of Mrs. Harding, it issued the policy of insurance on an automobile alleged by the said plaintiff to be her property. It was made by means of a proposal in writing signed and delivered by said plaintiff to the defendant, guaranteeing the truth of the statements contained therein which said proposal is referred to in the said policy of insurance made a part thereof; that certain of the statements and representations contained in said proposal and warranted by said plaintiff to be true, to wit: (a) the price paid by the proposer for the said automobile; (b) the value of said automobile at the time of the execution and delivery of the said proposal and (c) the ownership of said automobile, were false and known to be false by the said plaintiff at the time of signing and delivering the said proposal and were made for the purpose of misleading and deceiving the defendant, and inducing the defendant, relying upon the warranties, statements, and representations contained in the said proposal and believing the same to be true, issued the said policy of insurance. - The evidence shows that Hermanos, the Manila agents for the Studebaker automobile, sold the automobile to Canson for P3,200 (testimony of Mr. Diehl); who sold the said automobile to Henry Harding for the sum of P1,500. Harding sold the said automobile to J. Brannigan for the sum of P2,000 who sold the said automobile Henry Harding for the sum of P2,800; Henry Harding gave the said automobile to his wife as a present; that said automobile was repaired and repainted at the Luneta Garage at a cost of some P900; that while the said automobile was at the Luneta Garage; the latter solicited of Mrs. Harding the insurance of said automobile by the Company; that a proposal was filled out by the said agent and signed by the plaintiff Mrs. Henry E. Harding, and in said proposal under the heading "Price paid by proposer," is the amount of "3,500" and under another heading "Present value" is the amount of "3,000". - After the said proposal was made a representative of the Manila agent of defendant went to the Luneta Garage and examined said automobile and Mr. Server, the General Manager of the Luneta Garage, an experienced automobile mechanic, testified that at the time this automobile was insured it was worth about P3,000, and the defendant, by and through its said agent Smith, Bell & Company (limited), thereafter issued a policy of insurance upon proposal in which policy the said automobile was described as of the "present value" of P3,000 and the said defendant charged the said plaintiff Mrs. Henry E. Harding as premium on said policy the sum of P150, or 5 per cent of the then estimated value of P3,000. - The "Schedule" in said policy of insurance describes the automobile here in question, and provides in part of follows: "That during the period above set forth and during any period for which the company may agree to renew this policy the company will subject to the exception and conditions contained herein or endorsed hereon indemnify the insured against loss of or damage to any motor car described in the schedule hereto (including accessories) by whatever cause such loss or damage may be occasioned and will further indemnify the insured up to the value of the car or P3,000 whichever is the greater against any claim at common law made by any person (not being a person in the said motor car nor in the insured's service) for loss of life or for accidental bodily injury or damage to property caused by the said motor car including law costs payable in connection with such claim when incurred with the consent of the company." - On March 24, 1916, the said automobile was totally destroyed by fire, and that the iron and steel portions of said automobile which did not burn were taken into the possession of the defendant by and through its agent Smith, Bell & Company (limited), and sold by it for a small sum, which had never been tendered to the plaintiff prior to the trial of this case, but in open court during the trial the sum of P10 as the proceeds of such sale was tendered to plaintiff and refused. - Trial judge decided that there was no proof of fraud on the part of plaintiff in her statement of the value of the automobile, or with respect to its ownership; that she had an insurable interest therein; and that defendant, having agreed to the estimated value, P3,000, and having insured the automobile for that amount, upon the basis of which the premium was paid, is bound by it and must pay the loss in accordance with the stipulated insured value. ISSUE 1. WON Mrs. Harding was not the owner of the automobile at the time of the issuance of the policy, and, therefore, had no insurable interest in it 2. WON the statement regarding the cost of the automobile was a warranty, that the statement was

Page
false, and that, therefore, the policy never attached to the risk HELD 1. NO - Article 1334 of the Civil Code which provides that "All gifts between spouses during the marriage shall be void. Moderate gifts which the spouses bestow on each other on festive days of the family are not included in this rule." - Even assuming that defendant might have invoked article 1334 as a defense, the burden would be upon it to show that the gift in question does not fall within the exception therein established. We cannot say, as a matter of law, that the gift of an automobile by a husband to his wife is not a moderate one. Whether it is or is not would depend upon the circumstances of the parties, as to which nothing is disclosed by the record. - We are of the opinion that it would be unfair to hold the policy void simply because the outlay represented by the automobile was made by the plaintiff's husband and not by his wife, to whom he had given the automobile. It cannot be assumed that defendant should not have issued the policy unless it were strictly true that the price representing the cost of the machine had been paid by the insured and by no other person ? that it would no event insure an automobile acquired by gift, inheritance, exchange, or any other title not requiring the owner to make a specific cash outlay for its acquisition. 2. NO - It has not been shown by the evidence that the statement was false; on the contrary we believe that it shows that the automobile had in fact cost more than the amount mentioned. The court below found, and the evidence shows, that the automobile was bought by plaintiff's husband a few weeks before the issuance of the policy in question for the sum of P2,800, and that between that time and the issuance of the policy some P900 was spent upon it in repairs and repainting. - The witness Server, an expert automobile mechanic, testified that the automobile was practically as good as new at the time the insurance was effected. The form of proposal upon which the policy was issued does not call for a statement regarding the value of the automobile at the time of its acquisition by the applicant for the insurance, but merely a statement of its cost. The amount stated was less than the actual outlay which the automobile represented to Mr.

INSURANCE 47
Harding, including repairs, when the insurance policy was issued. - The court below found and the evidence shows, without dispute, that the proposal upon which the policy in question was issued was made out by defendant's agent by whom the insurance was solicited, and that appellee simply signed the same. It also appears that an examiner employed by the defendant made an inspection of the automobile before the acceptance of the risk, and that the sum after this examination. The trial court found that Mrs. Harding, in fixing the value of the automobile at P3,000, acted upon information given her by her husband and by Mr. Server, the manager of the Luneta Garage. She merely repeated the information which had been given her by her husband, and at the same time disclosed to defendant's agent the source of her information. There is no evidence to sustain the contention that this communication was made in bad faith. We do not think that the facts stated in the proposal can be held as a warranty of the insured, even if it should have been shown that they were incorrect in the absence of proof of willful misstatement. Under such circumstance, the proposal is to be regarded as the act of the insurer and not of the insured. Disposition Plaintiff was the owner of the automobile in question and had an insurable interest therein; that there was no fraud on her part in procuring the insurance; that the valuation of the automobile, for the purposes of the insurance, is binding upon the defendant corporation, and that the judgment of the court below is, therefore, correct and must be affirmed, with interest, the costs of this appeal to be paid by the appellant. WHITE GOLD MARINE SERVICES v. PIONEER INSURANCE 464 SCRA 448 QUISUMBING; July 28, 2005 NATURE This petition for review assails the Decision of the Court of Appeals, affirming the Decision of the Insurance Commission. Both decisions held that there was no violation of the Insurance Code and the respondents do not need license as insurer and insurance agent/broker. FACTS - White Gold procured a protection and indemnity coverage for its vessels from Steamship Mutual through Pioneer Insurance. Subsequently, White Gold was issued a Certificate of Entry and Acceptance. Pioneer also issued receipts evidencing payments for the coverage. When White Gold failed to fully pay its accounts, Steamship Mutual refused to renew the coverage. - Steamship Mutual thereafter filed a case against White Gold for collection of sum of money to recover the latters unpaid balance. White Gold on the other hand, filed a complaint before the Insurance Commission claiming that Steamship Mutual violated Sections 186 and 187 of the Insurance Code, while Pioneer violated Sections 299, 300 and 301 in relation to Sections 302 and 303, thereof. - The Insurance Commission dismissed the complaint. It said that there was no need for Steamship Mutual to secure a license because it was not engaged in the insurance business. It explained that Steamship Mutual was a Protection and Indemnity Club (P & I Club). Likewise, Pioneer need not obtain another license as insurance agent and/or a broker for Steamship Mutual because Steamship Mutual was not engaged in the insurance business. Moreover, Pioneer was already licensed, hence, a separate license solely as agent/broker of Steamship Mutual was already superfluous. - The Court of Appeals affirmed the decision of the Insurance Commissioner. In its decision, the appellate court distinguished between P & I Clubs vis-vis conventional insurance. The appellate court also held that Pioneer merely acted as a collection agent of Steamship Mutual. ISSUES 1. WON Steamship Mutual, a P & I Club, is engaged in the insurance business in the Philippines 2. WON Pioneer needs a license as an insurance agent/broker for Steamship Mutual HELD 1. YES - The test to determine if a contract is an insurance contract or not, depends on the nature of the promise, the act required to be performed, and the exact nature of the agreement in the light of the occurrence, contingency, or circumstances under which the performance becomes requisite. It is not by what it is called. Basically, an insurance contract is a contract of indemnity. In it, one undertakes for a

Page
consideration to indemnify another against loss, damage or liability arising from an unknown or contingent event. - In particular, a marine insurance undertakes to indemnify the assured against marine losses, such as the losses incident to a marine adventure. Section 99 of the Insurance Code enumerates the coverage of marine insurance. - Relatedly, a mutual insurance company is a cooperative enterprise where the members are both the insurer and insured. In it, the members all contribute, by a system of premiums or assessments, to the creation of a fund from which all losses and liabilities are paid, and where the profits are divided among themselves, in proportion to their interest. Additionally, mutual insurance associations, or clubs, provide three types of coverage, namely, protection and indemnity, war risks, and defense costs. - A P & I Club is a form of insurance against third party liability, where the third party is anyone other than the P & I Club and the members. By definition then, Steamship Mutual as a P & I Club is a mutual insurance association engaged in the marine insurance business. - The records reveal Steamship Mutual is doing business in the country albeit without the requisite certificate of authority mandated by Section 187 of the Insurance Code. It maintains a resident agent in the Philippines to solicit insurance and to collect payments in its behalf. We note that Steamship Mutual even renewed its P & I Club cover until it was cancelled due to non-payment of the calls. Thus, to continue doing business here, Steamship Mutual or through its agent Pioneer, must secure a license from the Insurance Commission. - Since a contract of insurance involves public interest, regulation by the State is necessary. Thus, no insurer or insurance company is allowed to engage in the insurance business without a license or a certificate of authority from the Insurance Commission. 2. YES - SEC. 299 . . . - No person shall act as an insurance agent or as an insurance broker in the solicitation or procurement of applications for insurance, or receive for services in obtaining insurance, any commission or other compensation from any insurance company doing business in the Philippines or any agent thereof, without first procuring a license so to act from the Commissioner, which must be renewed annually on

INSURANCE 48
the first day of January, or within six months thereafter. Disposition The petition is PARTIALLY GRANTED. The Decision dated July 30, 2002 of the Court of Appeals affirming the Decision dated May 3, 2000 of the Insurance Commission is hereby REVERSED AND SET ASIDE. The Steamship Mutual Underwriting Association (Bermuda) Ltd., and Pioneer Insurance and Surety Corporation are ORDERED to obtain licenses and to secure proper authorizations to do business as insurer and insurance agent, respectively. The petitioners prayer for the revocation of Pioneers Certificate of Authority and removal of its directors and officers, is DENIED. PANDIMAN v. MARINE MANNING MNGT CORP. 460 SCRA 418 GARCIA; June 21, 2005 NATURE Petition for certiorari to review CA decision FACTS - Benito Singhid was hired as chief cook on board the vessel MV Sun Richie Five for a term of one year by Fullwin Maritime Limited through its Philippine agent, Marine Manning and Management Corporation. While the said vessel was on its way to Shanghai from Ho Chih Minh City, Benito suffered a heart attack and subsequently died on June 24, 1997. - Apparently, the vessel and the crew were insured with Ocean Marine Mutual Insurance Association Limited (OMMIAL), a Protective and Indemnity Club of which Sun Richie Five Bulkers S.A. is a member. Pandiman Philippines, the petitioner, is the local correspondent of OMMIAL. - Benitos widow, Rosita, filed a claim for death benefits with Marine which referred her to Pandiman. After her submission of the required documentation, Pandiman recommended payment of the death benefits amounting to $79,000. However, payment has not been made. - Rosita filed a complaint with the Labor Arbiter naming Marine, Pandiman, OMMIAL, and Fullwin as respondents. The Arbiter ordered all the respondents, except Pandiman, to jointly and severally pay the widow the death benefits plus legal fees. The NLRC, on appeal by Marine, limited the liable parties to Pandiman and OMMIAL but maintained the money award. The CA sustained the decision of the NLRC. Hence this appeal.

Page
FACTS - October 1, 1941 - Christern Huenefeld, & Co., Inc., after payment of corresponding premium, obtained from the Filipinas Cia. de Seguros a fire policy in the sum of P1000,000, covering merchandise contained in No. 711 Roman Street, Binondo Manila. - February 27, 1942 or during the Japanese military occupation - building and insured merchandise were burned. In due time the Huenefeld Co submitted to the Filipinas Cia its claim under the policy. The salvage goods were sold at public auction and, after deducting their value, the total loss suffered by the respondent was fixed at P92,650. - Filipinas Cia refused to pay the claim on the ground that the policy in favor of the respondent had ceased to be in force on the date the United States declared war against Germany, the respondent Corporation (though organized under and by virtue of the laws of the Philippines) being controlled by the German subjects and the Filipinas Cia being a company under American jurisdiction when said policy was issued on October 1, 1941. Filipinas Cia, however, in pursuance of the order of the Director of Bureau of Financing, Philippine Executive Commission, dated April 9, 1943, paid to the Huenefeld Co the sum of P92,650 on April 19, 1943. - August 6, 1946 action filed in CFI Manila to recover from the Huenefeld Co the sum of P92,650 above mentioned. The theory of the Filipinas Cia is that the insured merchandise were burned up after the policy issued in 1941 in favor of Huenefeld Co has ceased to be effective because of the outbreak of the war between the United States and Germany on December 10, 1941, and that the payment made by the Filipinas Cia to Huenefeld Co during the Japanese military occupation was under pressure. - CFI: dismissed the action without pronouncement as to costs. - CA: CFI judgment affirmed, with costs. The case is now before us on appeal by certiorari from the decision of the Court of Appeals. ISSUE WON the policy in question became null and void upon the declaration of war between United States and Germany HELD YES Ratio The Philippine Insurance Law (Act No. 2427, as amended,) in section 8, provides that "anyone

ISSUE 1. WON Pandiman may be held liable for the death benefits 2. WON Marine and its foreign principal, Fullwin, should be absolved from the death claim liabilities HELD 1. NO - Pandiman is not an insurance agent as defined by Section 3007 of the Insurance Code. In this case, there was no showing that Pndiman in fact negotiated the insurance contract between Sun Richie Five and the insurer OMMIAL. Even, if Pandiman were an agent, payment for claims arising from peril insured against, to which the insurer is liable, is definitely not one of the liabilities of an insurance agent. Thus, there is no legal basis whatsoever for holding petitioner solidarily liable with insurer OMMIAL for the widows claim for death benefits. Also, Pandiman is not a party to the insurance contract and hence under Article 1311 of the Civil Code, it is not liable for the obligation arising out of the insurance contract. 2. NO - Fullwin, as Benitos principal employer is liable under the employment contract. Marine is also bound by its undertaking pursuant to the Rules and Regulations Governing Overseas Employment that it shall assume joint and solidary liability with the employer for all the claims and liabilities which may arise in connection with the implementation of the contract, including but not limited to the payment of wages, heath and disability compensation and repatriation. In other words, both Fullwin and Marine should be held liable for whatever death benefits the widow of Benito may be entitled to. Disposition The petition is granted and the CA decision is reversed and set aside. FILIPINAS COMPANIA DE SEGUROS CHRISTERN, HUENEFELD AND CO INC 89 PHIL 54 PARAS; May 25, 1951
7

Section 300. Any person who for compensation solicits or obtains on behalf of any insurance company transmits for a person other than himself an application for a policy or contract of insurance to or from such company or offers or assumes to act in the negotiating of such insurance shall be an insurance agent within the intent of this section and shall thereby become liable to all the duties, requirements, liabilities, and penalties to which an insurance agent is subject.

INSURANCE 49
except a public enemy may be insured." It stands to reason that an insurance policy ceases to be allowable as soon as an insured becomes a public enemy. > Effect of war, generally. All intercourse between citizens of belligerent powers which is inconsistent with a state of war is prohibited by the law of nations. Such prohibition includes all negotiations, commerce, or trading with the enemy; all acts which will increase, or tend to increase, its income or resources; all acts of voluntary submission to it; or receiving its protection; also all acts concerning the transmission of money or goods; and all contracts relating thereto are thereby nullified. It further prohibits insurance upon trade with or by the enemy, upon the life or lives of aliens engaged in service with the enemy; this for the reason that the subjects of one country cannot be permitted to lend their assistance to protect by insurance the commerce or property of belligerent, alien subjects, or to do anything detrimental too their country's interest. The purpose of war is to cripple the power and exhaust the resources of the enemy, and it is inconsistent that one country should destroy its enemy's property and repay in insurance the value of what has been so destroyed, or that it should in such manner increase the resources of the enemy, or render it aid, and the commencement of war determines, for like reasons, all trading intercourse with the enemy, which prior thereto may have been lawful. All individuals therefore, who compose the belligerent powers, exist, as to each other, in a state of utter exclusion, and are public enemies. (6 Couch, Cyc. of Ins. Law, pp. 5352-5353.) > In the case of an ordinary fire policy, which grants insurance only from year, or for some other specified term it is plain that when the parties become alien enemies, the contractual tie is broken and the contractual rights of the parties, so far as not vested. lost. (Vance, the Law on Insurance, Sec. 44, p. 112.) Reasoning - The Court of Appeals overruled the contention of the petitioner that the respondent corporation became an enemy when the United States declared war against Germany, relying on English and American cases which held that a corporation is a citizen of the country or state by and under the laws of which it was created or organized. It rejected the theory that nationality of private corporation is determined by the character or citizenship of its controlling stockholders. - There is no question that majority of the stockholders of the respondent corporation were German subjects. Therefore, Huenefeld Co became an enemy corporation upon the outbreak of the war between the United States and Germany. The English and American cases relied upon by the Court of Appeals have lost their force in view of the latest decision of the Supreme Court of the United States in Clark vs. Uebersee Finanz Korporation, decided on December 8, 1947, in which the controls test has been adopted. In "Enemy Corporation" by Martin Domke, a paper presented to the Second International Conference of the Legal Profession held at the Hague (Netherlands) in August. 1948 also discussed this dilemma > In Clark vs. Uebersee Finanz Korporation, A. G., dealing with a Swiss corporation allegedly controlled by German interest, the Court: "The property of all foreign interest was placed within the reach of the vesting power (of the Alien Property Custodian) not to appropriate friendly or neutral assets but to reach enemy interest which masqueraded under those innocent fronts. . . . The power of seizure and vesting was extended to all property of any foreign country or national so that no innocent appearing device could become a Trojan horse." - The respondent having become an enemy corporation on December 10, 1941, the insurance policy issued in its favor on October 1, 1941, by the petitioner (a Philippine corporation) had ceased to be valid and enforcible, and since the insured goods were burned after December 10, 1941, and during the war, the respondent was not entitled to any indemnity under said policy from the petitioner. However, elementary rules of justice (in the absence of specific provision in the Insurance Law) require that the premium paid by the respondent for the period covered by its policy from December 11, 1941, should be returned by the petitioner. Disposition the appealed decision is hereby reversed and the respondent corporation is ordered to pay to the petitioner the sum of P77,208.33, Philippine currency, less the amount of the premium, in Philippine currency, that should be returned by the petitioner for the unexpired term of the policy in question, beginning December 11, 1941.

Page

INSULAR LIFE ASSURANCE CO. v. EBRADO 80 SCRA 181 MARTIN; October 28, 1977 NATURE Appeal from judgment of RTC. FACTS - Buenaventura Ebrado obtained a whole-life insurance policy from Insular, for P5,882.00 with a rider for accidental death benefits for the same amount. He designated Carponia Ebrado as the revocable beneficiary, referring to her as the wife. - Afterwards, he died as a result of an accident when he was hit by a falling branch of a tree. Carponia filed a claim for the proceeds as the designated beneficiary in the policy, although she admits that she and Buenaventura were merely living as husband and wife without the benefit of marriage. The legal wife, Pascuala Vda De Ebrado, also filed her claim as the widow of the deceased. - Insular then filed an interpleader in court (CFI Rizal) to determine to whom the proceeds should be paid. CFI declared that Carponia was disqualified from becoming beneficiary of the insured and directing the Insular to pay the proceeds to the estate of Buenaventura. ISSUE 1. WON a common-law wife named as beneficiary in the insurance policy of a legally married man claim the proceeds of the same HELD 1. NO Ratio The prohibition that husband and wife cannot donate to each other applies to common-law relationships. As the appointment of a beneficiary in insurance may be considered a donation, one cannot name as beneficiary his common-law wife. Reasoning - It is quite unfortunate that the Insurance Code does not contain any specific provision grossly resolutory of the prime question at hand. - Rather, general rules of civil law should be applied to resolve the issue. Art.2011, CC states: The contract of insurance is governed by special laws. Matters not expressly provided for in such special

INSURANCE 50
laws shall be regulated by this Code. Thus, when not otherwise specifically provided for by the Insurance Law, the contract of life insurance is governed by the general rules of the civil law regulating contracts. - Also, Art.2012 any person who is forbidden from receiving any donation under Article 739 cannot be named beneficiary of a life insurance policy by the person who cannot make a donation to him. Common-law spouses are, definitely, barred from receiving donations from each other. - Art.739, CC: The following donations shall be void: 1. Those made between persons who were guilty of adultery or concubinage at the time of donation; - In essence, a life insurance policy is no different from a civil donation insofar as the beneficiary is concerned. Both are founded upon the same consideration: liberality. A beneficiary is like a donee, because from the premiums of the policy which the insured pays out of liberality, the beneficiary will receive the proceeds or profits of said insurance. As a consequence, the proscription in Art.739 CC should equally operate in life insurance contracts. The mandate of Art.2012 cannot be laid aside: any person who cannot receive a donation cannot be named as beneficiary in the life insurance policy of the person who cannot make the donation. - Policy considerations and dictates of morality rightly justify the institution of a barrier between commonlaw spouses in regard to property relations since such relationship ultimately encroaches upon the nuptial and filial rights of the legitimate family. There is every reason to hold that the bar in donations between legitimate spouses and those between illegitimate ones should be enforced in life insurance policies since the same are based on similar consideration. - So long as marriage remains the threshold of family laws, reason and morality dictate that the impediments imposed upon married couple should likewise be imposed upon extra-marital relationship. If legitimate relationship is circumscribed by these legal disabilities, with more reason should an illicit relationship be restricted by these disabilities. Disposition Decision AFFIRMED. CONSUEGRA v. GSIS 37 SCRA 315 ZALDIVAR; January 30, 1971 NATURE Appeal from the decision of the Court of First Instance of Surigao del Norte awarding the 8/16 part of the proceeds of the deceased Consuegras retirement benefits to Rosario Diaz. FACTS - The late Jose Consuegra, at the time of his death, was employed as a shop foreman of the office of the District Engineer in the province of Surigao del Norte. In his lifetime, Consuegra contracted two marriages, the first with herein respondent Rosario Diaz, solemnized in the parish church of San Nicolas de Tolentino, Surigao, Surigao, on July 15, 1937, out of which marriage were born two children, namely, Jose Consuegra, Jr. and Pedro Consuegra, but both predeceased their father; and the second, which was contracted in good faith while the first marriage was subsisting, with herein petitioner Basilia Berdin, on May 1, 1957 in the same parish and municipality, out of which marriage were born seven children, namely, Juliana, Pacita, Maria Lourdes, Jose, Rodrigo, Lenida and Luz, all surnamed Consuegra. - Being a member of the Government Service Insurance System (GSIS, for short) when Consuegra died on September 26, 1965, the proceeds of his life insurance under policy No. 601801 were paid by the GSIS to petitioner Basilia Berdin and her children who were the beneficiaries named in the policy. - However, Consuegra did not designate any beneficiary who would receive the retirement insurance benefits due to him. Respondent Rosario Diaz, the widow by the first marriage, filed a claim with the GSIS asking that the retirement insurance benefits be paid to her as the only legal heir of Consuegra, considering that the deceased did not designate any beneficiary with respect to his retirement insurance benefits. Petitioner Basilia Berdin and her children, likewise, filed a similar claim with the GSIS, asserting that being the beneficiaries named in the life insurance policy of Consuegra, they are the only ones entitled to receive the retirement insurance benefits due the deceased Consuegra. Resolving the conflicting claims, the GSIS ruled that the legal heirs of the late Jose Consuegra were Rosario Diaz, his widow by his first marriage who is entitled to one-half, or 8/16, of the retirement insurance benefits, on the one hand; and Basilia Berdin, his widow by the second marriage and their seven children, on the other hand, who are entitled to the remaining one-half, or 8/16, each of them to receive an equal share of 1/16. - Dissatisfied with the foregoing ruling and apportionment made by the GSIS, Basilia Berdin and

Page
her children filed on October 10, 1966 a petition for mandamus with preliminary injunction in the Court of First Instance of Surigao. - The CFI of Surigao ruled in favor of respondent Rosario Diaz and upheld the ruling of GSIS in all aspect. Thus, Basilia Berdin and her children appealed said decision to the Supreme Court. ISSUE WON GSIS was correct in awarding half of the retirement benefit of the deceased to Rosario Diaz, the first wife, notwithstanding the fact that the petitioners were named as beneficiaries of the life insurance HELD YES - The GSIS offers two separate and distinct systems of benefits to its members, one is the life insurance and the other is the retirement insurance. These two distinct systems of benefits are paid out from two distinct and separate funds that are maintained by the GSIS. Thus, it doesnt necessarily mean that the beneficiaries in the life insurance are also the beneficiaries in the retirement insurance. - Consuegra started in the government service sometime during the early part of 1943, or before 1943. In 1943 Com. Act 186 was not yet amended, and the only benefits then provided for in said Com. Act 186 were those that proceed from a life insurance. Upon entering the government service Consuegra became a compulsory member of the GSIS, being automatically insured on his life, pursuant to the provisions of Com. Act 186 which was in force at the time. During 1943 the operation of the Government Service Insurance System was suspended because of the war, and the operation was resumed sometime in 1946. When Consuegra designated his beneficiaries in his life insurance he could not have intended those beneficiaries of his life insurance as also the beneficiaries of his retirement insurance because the provisions on retirement insurance under the GSIS came about only when Com. Act 186 was amended by Rep. Act 660 on June 16, 1951. Hence, it cannot be said that because herein appellants were designated beneficiaries in Consuegra's life insurance they automatically became the beneficiaries also of his retirement insurance. - The provisions of subsection (b) of Section 11 of Commonwealth Act 186, as amended by Rep. Act 660, clearly indicate that there is need for the

INSURANCE 51
employee to file an application for retirement insurance benefits when he becomes a member of the GSIS, and he should state in his application the beneficiary of his retirement insurance. Hence, the beneficiary named in the life insurance does not automatically become the beneficiary in the retirement insurance unless the same beneficiary in the life insurance is so designated in the application for retirement insurance. - In the case of the proceeds of a life insurance, the same are paid to whoever is named the beneficiary in the life insurance policy. As in the case of a life insurance provided for in the Insurance Act, the beneficiary in a life insurance under the GSIS may not necessarily be an heir of the insured. The insured in a life insurance may designate any person as beneficiary unless disqualified to be so under the provisions of the Civil Code. And in the absence of any beneficiary named in the life insurance policy, the proceeds of the insurance will go to the estate of the insured. - On the other hand, the beneficiary of the retirement insurance can only claim the proceeds of the retirement insurance if the employee dies before retirement. If the employee failed or overlooked to state the beneficiary of his retirement insurance, the retirement benefits will accrue to his estate and will be given to his legal heirs in accordance with law, as in the case of a life insurance if no beneficiary is named in the insurance policy. Disposition Petition Denied. It is Our view, therefore, that the respondent GSIS had correctly acted when it ruled that the proceeds of the retirement insurance of the late Jose Consuegra should be divided equally between his first living wife Rosario Diaz, on the one hand, and his second wife Basilia Berdin and his children by her. SSS v. DAVAC 17 SCRA 863 BARRERA: July 30, 1966 NATURE APPEAL from a resolution Of the Social Security Commission. FACTS - Petronilo Davac, became a member of the Social Security System (SSS for short) on September 1, 1957. In the Member's Record he designated respondent, Candelaria Davac as his beneficiary and indicated his relationship to her as that of "wife". - He died on April 5, 1959. It appears that the deceased contracted two marriages, the first, with Lourdes Tuplano on August 29, 1946, who bore him a child, Romeo Davac, and the second, with Candelaria Davac on January 18, 1949, with whom he had a minor daughter, Elizabeth Davac. Both filed their claims for death benefit with the SSS. - Social Security Commission issued the resolution declaring respondent Candelaria Davac as the person entitled to receive the death benefits payable for the death of Petronilo Davac. ISSUES 1. WON the Social Security Commission Candelaria Davac is entitled to receive the death benefits 2. WON a beneficiary under the Social Security System partakes of the nature of a beneficiary in a life insurance policy and, therefore the designation made in the person DAVAC as bigamous wife is null and void, because it contravenes the provisions of the Civil Code 3. WON the benefits accruing from membership with SSS forms part of the conjugal property thus the resolution deprives the lawful wife of her share in the conjugal property as well as of her own and her child's legitime in the inheritance HELD 1. YES - Section 13, RA1161 provides that the beneficiary "as recorded" by the employee's employer is the one entitled to the death benefits. - Section 13, Republic Act No. 1161, as amended by Republic Act No. 1792, in force at the time of Petronilo Davac's death provides: Upon the covered employee's death or total and permanent disability under such conditions as the Commission may define, before becoming eligible for retirement and if either such death or disability is not compensable under the Workmen's Compensation Act, he or. in case of his death, his beneficiaries, as recorded by his employer shall be entitled to the following benefit: - In Tecson vs. Social Security System. Section 13 was construed:"it may be true that the purpose of the coverage under the Social Security System is protection of the employee as well as of his family, but this purpose or intention of the law cannot be enforced to the extent of contradicting the very

Page
provisions of said law contained in Section 13, thereof - When the provisions of a law are clear and explicit, the courts can do nothing but apply its clear and explicit provisions (Velasco vs. Lopez) 2. NO - The disqualification mentioned in Article 739 is not applicable to herein appellee Candelaria Davac because she was not guilty of concubinage, there being no proof that she had knowledge of the previous marriage of her husband Petronilo. ART. 2012. Any person who is forbidden from receiving any donation under Article 739 cannot be named beneficiary of a life insurance policy by the person who cannot make any donation to him according to said article. ART. 739. The following donations shall be void: (1) Those made between persons who were guilty of adultery or concubinage at the time of the donation; (the court did not decide whether this partakes the nature of a life insurance policy) 3. NO - The benefit receivable under the Act is in the nature of a special privilege or an arrangement secured by the law pursuant to the policy of the State to provide social security to the workingmen. The amounts that may thus be received cannot be considered as property earned by the member during his lifetime. His contribution to the fund constitutes only an insignificant portion thereof. Then, the benefits are specifically declared not transferable, and exempted from tax, legal processes, and lien. Furthermore, in the settlement of claims thereunder, the procedure to be observed is governed not by the general provisions of law, but by rules and regulations promulgated by the Commission. Thus, if the money is payable to the estate of a deceased member, it is the Commission, not the probate or regular court that determines the person or persons to whom it is payable. - They are disbursed from a public special fund created by Congress.The sources of this special fund are the covered employee's contribution (equal to 21/2 per cent of the employee's monthly compensation) ; the employer's 'Contribution (equivalent to 3-1/2 per cent of the monthly compensation of the covered employee) ;and the Government contribution which consists in yearly appropriation of public funds to assure the maintenance of an adequate working balance of the funds of the System. Additionally, Section 21 of the Social Security Actprovides that the benefits

INSURANCE 52
prescribed in this Act shall not be diminished and the Government of the Republic of the Philippines accepts general responsibility for the solvency of the System. - The benefits under the Social Security Act are not intended by the lawmaking body to form part of the estate of the covered members. - Social Security Act is not a law of succession. Disposition Resolution of the Social Security Commission appealed is affirmed FRANCISCO DEL VAL v. ANDRES DEL VAL 29 PHIL 534 MORELAND; February 16, 1915 NATURE Appeal from a judgment of the Court of First Instance of the city of Manila dismissing the complaint with costs. FACTS - Plaintiffs and defendant are brothers and sisters; that they are the only heirs at law and next of kin of Gregorio Nacianceno del Val, who died in Manila on August 4, 1910, intestate - During the lifetime of the deceased he took out insurance on his life for the sum of P40,000 and made it payable to the defendant ANDRES DEL VAL as sole beneficiary. After his death the defendant collected the face of the policy. From said policy he paid the sum of P18,365.20 to redeem certain real estate which the decedent had sold to third persons with a right to repurchase. - The redemption of said premises was made by the attorney of defendant ANDRES in the name of the plaintiffs and the defendant as heirs of the deceased vendor. It further appears from the pleadings that the defendant, on the death of the deceased, took possession of most of his personal property, which he still has in his possession, and that he has also the balance on said insurance policy amounting to P21,634.80. - Plaintiffs contend that the amount of the insurance policy belonged to the estate of the deceased and not to the defendant personally; that, therefore, they are entitled to a partition not only of the real and personal property, but also of the P40,000 life insurance. The complaint prays a partition of all the property, both real and personal, left by the deceased; that the defendant account for P21,634.80, and that the sum be divided equally among the plaintiffs and defendant along with the other property of deceased. - The defendant denies the material allegations of the complaint and sets up as special defense and counterclaim that the redemption of the real estate sold by his father was made in the name of the plaintiffs and himself instead of in his name alone without his knowledge or consent. Andres contends that it was not his intention to use the proceeds of the insurance policy for the benefit of any person but himself, he alleging that he was and is the sole owner thereof and that it is his individual property. He, therefore, asks that he be declared the owner of the real estate redeemed by the payment of the P18,365.20, the owner of the remaining P21,634.80, the balance of the insurance policy, and that the plaintiffs account for the use and occupation of the premises so redeemed since the date of the redemption. - The trial court refused to give relief to either party and dismissed the action. In this appeal, it is claimed by the attorney for the plaintiffs that insurance provisions in the Code of Commerce are subordinated to the provisions of the Civil Code as found in article 1035. This article reads: "An heir by force of law surviving with others of the same character to a succession must bring into the hereditary estate the property or securities he may have received from the deceased during the life of the same, by way of dowry, gift, or for any good consideration, in order to compute it in fixing the legal portions and in the account of the division." - Counsel also claims that the proceeds of the insurance policy were a donation or gift made by the father during his lifetime to the defendant and that, as such, its ultimate destination is determined by those provisions of the Civil Code which relate to donations, especially article 819. This article provides that "gifts made to children which are not betterments shall be considered as part of their legal portion." ISSUES 1. WON the insurance belongs to the defendant and not to the decedents estate 2. WON the Civil code provisions on succession prevail over any other law with respect to the insurance HELD 1. YES - The SC agreed with the finding of the trial court that the proceeds of the life-insurance policy belong exclusively to the defendant as his individual and separate property, we agree. That the proceeds of an

Page
insurance policy belong exclusively to the beneficiary and not to the estate of the person whose life was insured, and that such proceeds are the separate and individual property of the beneficiary, and not of the heirs of the person whose life was insured, is the doctrine in America. We believe that the same doctrine obtains in these Islands by virtue of section 428 of the Code of Commerce, which reads: "The amounts which the underwriter must deliver to the person insured, in fulfillment of the contract, shall be the property of the latter, even against the claims of the legitimate heirs or creditors of any kind whatsoever of the person who effected the insurance in favor of the former." 2. NO - The contract of life insurance is a special contract and the destination of the proceeds thereof is determined by special laws which deal exclusively with that subject. The Civil Code has no provisions which relate directly and specifically to life-insurance contracts or to the destination of life insurance proceeds. That subject is regulated exclusively by the Code of Commerce which provides for the terms of the contract, the relations of the parties and the destination of the proceeds of the policy. - Assuming that the proceeds of the life-insurance policy being the exclusive property of the defendant and he having used a portion thereof in the repurchase of the real estate sold by the decedent prior to his death with right to repurchase, and such repurchase having been made and the conveyance taken in the names of all of the heirs instead of the defendant alone, plaintiffs claim that the property belongs to the heirs in common and not to the defendant alone. - The Court rejected this contention unless the fact appear or be shown that the defendant acted as he did with the intention that the other heirs should enjoy with him the ownership of the estate ---- in other words, that he proposed, in effect, to make a gift of the real estate to the other heirs. If it is established by the evidence that was his intention and that the real estate was delivered to the plaintiffs with that understanding, then it is probable that their contention is correct and that they are entitled to share equally with the defendant therein. If, however, it appears from the evidence in the case that the conveyances were taken in the name of the plaintiffs without his knowledge or consent, or that it was not his intention to make a gift to them of the real estate, then it belongs to him. If the facts are as stated, he

INSURANCE 53
has two remedies. The one is to compel the plaintiffs to reconvey to him and the other is to let the title stand with them and to recover from them the sum he paid on their behalf. - For the complete and proper determination of the questions at issue in this case, the Court was of the opinion that the cause should be returned to the trial court with instructions to permit the parties to frame such issues as will permit the settlement of all the questions involved and to introduce such evidence as may be necessary for the full determination of the issues framed. Upon such issues and evidence taken thereunder the court will decide the questions involved according to the evidence, subordinating his conclusions of law to the rules laid down in this opinion. REMANDED. GERCIO v. SUN LIFE ASSURANCE OF CANADA 48 PHIL 53 MALCOLM; September 28, 1925 NATURE Mandamus to compel Sun Life Assurance Co. of Canada to change the beneficiary in the policy issued by the defendant company on the life of the plaintiff Hilario Gercio FACTS - On January 29, 1910, the Sun Life Assurance Co. of Canada issued an insurance policy on the life of Hilario Gercio. The policy was what is known as a 20year endowment policy. By its terms, the insurance company agreed to insure the life of Hilario Gercio for the sum of P2,000, to be paid him on February 1, 1930, or if the insured should die before said date, then to his wife, Mrs. Andrea Zialcita, should she survive him; otherwise to the executors, administrators, or assigns of the insured. The policy did not include any provision reserving to the insured the right to change the beneficiary. - On the date the policy was issued, Andrea Zialcita was the lawful wife of Hilario Gercio. Towards the end of the year 1919, she was convicted of the crime of adultery. On September 4, 1920, a decree of divorce was issued in civil case no. 17955, which had the effect of completely dissolving their bonds of matrimony - On March 4, 1922, Hilario Gercio formally notified the Sun Life that he had revoked his donation in favor of Andrea Zialcita, and that he had designated in her stead his present wife, Adela Garcia de Gercio, as the beneficiary of the policy. Gercio requested the insurance company to eliminate Andrea Zialcita as beneficiary. This, the insurance company has refused and still refuses to do. ISSUES 1. (Preliminary) WON the provisions of the Code of Commerce and the Civil Code shall be in force in 1910, or the provisions of the Insurance Act now in force, or the general principles of law, guide the court in its decision 2. WON the insured, the husband, has the power to change the beneficiary, the former wife, and to name instead his actual wife, where the insured and the beneficiary have been divorced and where the policy of insurance does not expressly reserve to the insured the right to change the beneficiary HELD 1. Whether the case be considered in the light of the Code of Commerce, the Civil Code, or the Insurance Act, the deficiencies in the law will have to be supplemented by the general principles prevailing on the subject. To that end, we have gathered the rules which follow from the best considered American authorities. In adopting these rules, we do so with the purpose of having the Philippine Law of Insurance conform as nearly as possible to the modern Law of Insurance as found in the United States proper. - Courts first duty is to determine what law should be applied to the facts. The insurance policy was taken out in 1910, that the Insurance Act. No. 2427, became effective in 1914, and that the effort to change the beneficiary was made in 1922. - Code of Commerce- there can be found in it no provision either permitting or prohibiting the insured to change the beneficiary. - Civil Code- it would be most difficult, if indeed it is practicable, to test a life insurance policy by its provisions. In the case of Del Val vs. Del Val, it declined to consider the proceeds of the insurance policy as a donation or gift, saying "the contract of life insurance is a special contract and the destination of the proceeds thereof is determined by special laws which deal exclusively with that subject. The Civil Code has no provisions which relate directly and specifically to life-insurance contracts or to the destination of life-insurance proceeds. . . ." - Insurance Act- there is likewise no provision either permitting or prohibiting the insured to change the beneficiary.

Page
2. NO Ratio The wife has an insurable interest in the life of her husband. The beneficiary has an absolute vested interest in the policy from the date of its issuance and delivery. So when a policy of life insurance is taken out by the husband in which the wife is named as beneficiary, she has a subsisting interest in the policy. And this applies to a policy to which there are attached the incidents of a loan value, cash surrender value, an automatic extension by premiums paid, and to an endowment policy, as well as to an ordinary life insurance policy. If the husband wishes to retain to himself the control and ownership of the policy he may so provide in the policy. But if the policy contains no provision authorizing a change of beneficiary without the beneficiary's consent, the insured cannot make such change. Accordingly, it is held that a life insurance policy of a husband made payable to the wife as beneficiary, is the separate property of the beneficiary and beyond the control of the husband. - Unlike the statutes of a few jurisdictions, there is no provision in the Philippine Law permitting the beneficiary in a policy for the benefit of the wife of the husband to be changed after a divorce. It must follow, therefore, in the absence of a statute to the contrary, that if a policy is taken out upon a husband's life the wife is named as beneficiary therein, a subsequent divorce does not destroy her rights under the policy. Reasoning - Yore vs. Booth . . . It seems to be the settled doctrine, with but slight dissent in the courts of this country, that a person who procures a policy upon his own life, payable to a designated beneficiary, although he pays the premiums himself, and keeps the policy in his exclusive possession, has no power to change the beneficiary, unless the policy itself, or the charter of the insurance company, so provides. In policy, although he has parted with nothing, and is simply the object of another's bounty, has acquired a vested and irrevocable interest in the policy, which he may keep alive for his own benefit by paying the premiums or assessments if the person who effected the insurance fails or refuses to do so. - Connecticut Mutual Life Insurance Company vs Schaefer We do not hesitate to say, however, that a policy taken out in good faith and valid at its inception, is not avoided by the cessation of the insurable interest, unless such be the necessary effect of the

INSURANCE 54
provisions of the policy itself.. . . .In our judgment of life policy, originally valid, does not cease to be so by the cessation of the assured party's interest in the life insured. - Central National Bank of Washington City vs. Hume It is indeed the general rule that a policy, and the money to become due under it, belong, the moment it is issued, to the person or persons named in it as the beneficiary or beneficiaries, and that there is no power in the person procuring the insurance, by any act of his, by deed or by will, to transfer to any other person the interest of the person named. - In re Dreuil & Co. In so far as the law of Louisiana is concerned, it may also be considered settled that where a policy is of the semitontine variety, as in this case, the beneficiary has a vested right in the policy, of which she cannot be deprived without her consent - Wallace vs Mutual Benefit Life Insurance Co. As soon as the policy was issued Mrs. Wallace acquired a vested interest therein, of which she could not be deprived without her consent, except under the terms of the contract with the insurance company. No right to change the beneficiary was reserved. Her interest in the policy was her individual property, subject to be divested only by her death, the lapse of time, or by the failure of the insured to pay the premiums. She could keep the policy alive by paying the premiums, if the insured did not do so. It was contingent upon these events, but it was free from the control of her husband. He had no interest in her property in this policy, contingent or otherwise. Her interest was free from any claim on the part of the insured or his creditors. He could deprive her of her interest absolutely in but one way, by living more than twenty years. - Filley vs. Illinois Life Insurance Company The benefit accruing from a policy of life insurance upon the life of a married man, payable upon his death to his wife, naming her, is payable to the surviving beneficiary named, although she may have years thereafter secured a divorce from her husband, and he was thereafter again married to one who sustained the relation of wife to him at the time of his death. The rights of a beneficiary in an ordinary life insurance policy become vested upon the issuance of the policy, and can thereafter, during the life of the beneficiary, be defeated only as provided by the terms of the policy. - On the admitted facts and the authorities supporting the nearly universally accepted principles of insurance, we are irresistibly led to the conclusion that the question at issue must be answered in the negative Disposition The judgment appealed from will be reversed and the complaint ordered dismissed as to the appellant. SEPARATE OPINION JOHNSON [concur] - I agree with the majority of the court, that the judgment of the lower court should be revoked, but for a different reason. The purpose of the petition is to have declared the rights of certain persons in an insurance policy which is not yet due and payable. It may never become due and payable. The premiums may not be paid, thereby rendering the contract of insurance of non effect, and many other things may occur, before the policy becomes due, which would render it non effective. The plaintiff and the other parties who are claiming an interest in said policy should wait until there is something due them under the same. For the courts to declare now who are the persons entitled to receive the amounts due, if they ever become due and payable, is impossible, for the reason that nothing may ever become payable under the contract of insurance, and for many reasons such persons may never have a right to receive anything when the policy does become due and payable. In my judgment, the action is premature and should have been dismissed. PHIL. AMERICAN LIFE INSURANCE v. PINEDA 175 SCRA 416 PARAS; July 19, 1989 NATURE Petition for review on certiorari the orders of CFI Judge Pineda FACTS - In 1968, Private Respondent Rodolfo Dimayuga procured an ordinary life insurance policy from the petitioner company and designated his wife and children as irrevocable beneficiaries. On Feb. 22, 1980, Dimayuga filed with the CFI a petition to amend the designation of the beneficiaries in his life policy

Page
from irrevocable to revocable. Petitioner filed an Urgent Motion to reset hearing as well as its comment and/or Opposition to the respondents petition. - Respondent Judge denied petitioners Urgent Motion, thus allowing private respondent to adduce evidence, the consequence of which was the issuance of the questioned Order granting the petition. Petitioner then filed a MFR which was also denied hence this petition. ISSUE 1. WON the designation of the irrevocable beneficiaries could be changed or amended without the consent of all the irrevocable beneficiaries 2. WON the irrevocable beneficiaries herein, one of whom is already deceased while the others are all minors could validly give consent to the change or amendment in the designation of the irrevocable beneficiaries HELD 1. NO - Based on the provision of their contract and the law applicable, it is only with the consent of all the beneficiaries that any change or amendment in the policy concerning the irrevocable beneficiaries may be legally and validly effected. Both the law and the Policy do not provide for any other exception. Reasoning - Since the policy was procured in 1968, the applicable law in this case is the Insurance Act and under that law, the beneficiary designated in a life insurance contract cannot be changed without the consent of the beneficiary because he has a vested interest in the policy. - The Beneficiary Designation Indorsement in the policy in the name of Dimayuga states that the designation of the beneficiaries is irrevocable: no right or privilege under the Policy may be exercised, or agreement made with the Company to any change in or amendment to the Policy, without the consent of the said beneficiary/beneficiaries. - Contracts which are the private laws of the contracting parties should be fulfilled according to the literal sense of their stipulations, if their terms are clear and leave no room for doubt as to the intention of the contracting parties, for contracts are obligatory, no matter in what form they may be, whenever the essential requisites for their validity are present. - Finally, the fact that the contract of insurance does not contain a contingency when the change in the designation of beneficiaries could be validly effected

INSURANCE 55
means that it was never within the contemplation of the parties. 2. NO - The parent-insured cannot exercise rights and/or privileges pertaining to the insurance contract, for otherwise, the vested rights of the irrevocable beneficiaries would be rendered inconsequential. The alleged acquiescence of the 6 children beneficiaries cannot be considered an effective ratification to the change of the beneficiaries from irrevocable to revocable. They were minors at the time, and could not validly give consent. Neither could they act through their father-insured since their interests are quite divergent from one another. Disposition questioned Orders of respondent judge are nullified and set aside. SUN LIFE ASSURANCE v. INGERSOLL 41 PHIL 331 STREET; November 8, 1921 NATURE Action of interpleader FACTS - April 16, 1918, Sun Life Assurance Company of Canada (Sun Life), in consideration of the payment of a stipulated annual premium during the period of the policy, or until the premiums had been completely paid for twenty years, issued a policy of insurance on the life of Dy Poco for US$12,500, payable to the said assured or his assigns on the 21st day of February, 1938, and if he should die before that date then to his legal representatives. - June 23, 1919, the assured, Dy Poco, was adjudged an involuntary insolvent by the CFI Manila, and Frank B. Ingersoll was appointed assignee of his estate. - July 10, 1919, Dy Poco died, and on August 21, 1919, Tan Sit, was duly appointed as the administratrix of his intestate estate. - By the terms of the policy it was provided that after the payment of three full premiums, the assured could surrender the policy to the company for a "cash surrender value," indicated in an annexed table; but inasmuch as no more than two premiums had been paid upon the policy now in question up to the time of the death of the assured, this provision had not become effective; and it does not appear that the company would in accordance with its own usage or otherwise have made any concession to the assured in the event he had desired, before his death, to surrender the policy. It must therefore be accepted that this policy had no cash surrender value, at the time of the assured's death, either by contract or by convention practice of the company in such cases. - Both Ingersoll, as assignee, and Tan Sit, as administratix of Dy Poco's estate, asserted claims to the proceeds of the policy. The lower court found that Ingersoll had a better right and ordered Sun Life to pay the insurance proceeds to him. ISSUE WON Ingersoll, as assignee, has a right to the proceeds of the insurance HELD NO On the Philippine Insolvency Law (Act No. 1956) - The property and interests of the insolvent which become vested in the assignee of the insolvent are specified in section 32 of the Insolvency Law which reads as follows: "SEC. 32. As soon as an assignee is elected or appointed and qualified, the clerk of the court shall, by an instrument under his hand and seal of the court, assign and convey to the assignee all the real and personal property, estate, and effects of the debtor with all his deeds, books, and papers relating thereto, and such assignment shall relate back to the commencement of the proceedings in insolvency, and shall relate back to the acts upon which the adjudication was founded, and by operation of law shall vest the title to all such property, estate, and effects in the assignee, although the same is then attached on mesne process, as the property of the debtor. Such assignment shall operate to vest in the assignee all of the estate of the insolvent debtor not exempt by law from execution." - the Insolvency Law is in great part a copy of the Insolvency Act of California, enacted in 1895, though it contains a few provisions from the American Bankruptcy Law of 1898 - Under each of said laws the assignee acquires all the real and personal property, estate, and effects of the debtor, not exempt by law from execution, with all deeds, books and papers relating thereto; and while this language is broad, it nevertheless lacks the comprehensiveness of section 70 (a) of the American Bankruptcy Law of 1898 in at least two particulars; for under subsection 3 of section 70 (a) of the last mentioned law, the trustee in bankruptcy acquires the right to exercise any powers which the insolvent

Page
might have exercised for his own benefit, and under subsection 5 the trustee acquires any property of the insolvent which the latter could by any means have assigned to another. The Insolvency Law here in force, in common with the predecessor laws abovementioned, contains nothing similar to these provisions. On the applicability of the Insolvency Law - Sec 32 of the Insolvency Law among other things, declares that the assignment to be made by the clerk of the court "shall operate to vest in the assignee all of the estate of the insolvent debtor not exempt by law from execution." Moreover, by section 24, the court is required, upon making an order adjudicating any person insolvent, to stay any civil proceedings pending against him; and it is declared in section 60 that no creditor whose debt is provable under the Act shall be allowed, after the commencement of proceedings in insolvency, to prosecute to final judgment any action therefor against the debtor. In connection with the foregoing may be mentioned subsections 1 and 2 of section 36, as well as the opening words of section 33, to the effect that the assignee shall have the right and power to recover and to take into his possession, all of the estate, assets, and claims belonging to the insolvent, except such as are exempt by law from execution. - These provisions clearly evince an intention to vest in the assignee, for the benefit of all the creditors of the insolvent, such elements of property and property right as could be reached and subjected by process of law by any single creditor suing alone. And this is exactly as it should be: for it cannot be supposed that the Legislature would suppress the right of action of every individual creditor upon the adjudication of insolvency, and at the same time allow the insolvent debtor to retain anything subject to the payment of his debts in a normal state of solvency. - "leviable assets" and "assets in insolvency" are practically coextensive terms. Hence, in determining what elements of value constitute assets in insolvency, SC is at liberty to consider what elements of value are subject to be taken upon execution, and vice versa. On whether a policy of insurance having no cash surrender value, but payable to insured or his legal representative, is property that may be taken upon execution against him. - Philippine laws declare no exemption with respect to insurance policies; and this species of property is not enumerated, in section 48 of the Insolvency Law,

INSURANCE 56
among items from the ownership of which the assignee is excluded. Moreover, all life insurance policies are declared by law to be assignable, regardless of whether the assignee has an insurable interest in the life of the insured or not (Insurance Act No. 2427, sec. 166). - SC has held that insurance policies having a present cash surrender value are subject to be taken upon execution. (Misut Garcia vs. West Coast San Francisco Life Ins. Co.) - a policy devoid of a cash surrender value cannot be either "leviable assets" or "assets in insolvency." - the assignee in insolvency acquired no beneficial interest in the policy of insurance in question; that its proceeds are not liable for any of the debts provable against the insolvent in the pending proceedings, and that said proceeds should therefore be delivered to his administratrix. On applicable US case - In re McKinney: no beneficial interest in the policy had ever passed to the assignee over and beyond what constituted the surrender value, and that the legal title to the policy was vested in the assignee merely in order to make the surrender value-available to him. The assignee should surrender the policy upon the payment to him of said value, as he was in fact directed to do. The assignee in bankruptcy had no right to keep the estate unsettled for an indefinite period, for the mere purpose of speculating upon the chances of the bankrupt's death. As regards everything beyond the surrender value, the assignee in bankruptcy would, after the discharge of the bankrupt, have no insurable interest in the life of the bankrupt. - surrender value of a policy "arises from the fact that the fixed annual premiums is much in excess of the annual risk during the earlier years of the policy, an excess made necessary in order to balance the deficiency of the same premium to meet the annual risk during the latter years of the policy. This excess in the premium paid over the annual cost of insurance, with accumulations of interest, constitutes the surrender value. Though this excess of premiums paid is legally the sole property of the company, still in practical effect, though not in law, it is moneys of the assured deposited with the company in advance to make up the deficiency in later premiums to cover the annual cost of insurance, instead of being retained by the assured and paid by him to the company in the shape of greatly-increased premiums, when the risk is greatest. It is the 'net reserve' required by law to be kept by the company for the benefit of the assured, and to be maintained to the credit of the policy. So long as the policy remains in force the company has not practically any beneficial interest in it, except as its custodian, with the obligation to maintain it unimpaired and suitably invested for the benefit of the insured. This is the practical, though not the legal, relation of the company to this fund. "Upon the surrender of the policy before the death of the assured, the company, to be relieved from all responsibility for the increased risk, which is represented by this accumulating reserve, could well afford to surrender a considerable part of it to the assured, or his representative. A return of a part in some form or other is now Usually made." (In re McKinney) - the stipulation providing for a cash surrender value is a comparatively recent innovation in life insurance. Formerly the contracts provided as they still commonly do in the policies issued by fraternal organizations and benefit societies for the payment of a premium sufficient to keep the estimated risk covered; and in case of a lapse the policy-holder received nothing. Furthermore, the practice is common among insurance companies even now to concede nothing in the character of cash surrender value, until three full premiums have been paid, as in this case. - CONLUSION (from this case and other English and American cases cited following the same opinion): the assignee acquires no beneficial interest in insurance effected on the life of the insolvent, except to the extent that such insurance contains assets which can be realized upon as of the date when the petition of insolvency is filed. The explanation is to be found in the consideration that the destruction of a contract of life insurance is not only highly prejudicial to the insured and those dependent upon him, but is inimical to the interests of society. Insurance is a species of property that should be conserved and not dissipated. As is well known, life insurance is increasingly difficult to obtain with advancing years, and even when procurable after the age of fifty, the cost is then so great as to be practically prohibitive to many. Insolvency is a disaster likely to overtake men in mature life; and one who has gone through the process of bankruptcy usually finds himself in his declining years with the accumulated savings of years swept away and earning power diminished. The courts are therefore practically unanimous in refusing to permit the assignee in insolvency to wrest from the

Page
insolvent a policy of insurance which contains in it no present realizable assets. On the applicability of the Insolvency Law - Sec 32 of the Insolvency Law among other things, declares that the assignment to be made by the clerk of the court "shall operate to vest in the assignee all of the estate of the insolvent debtor not exempt by law from execution." Moreover, by section 24, the court is required, upon making an order adjudicating any person insolvent, to stay any civil proceedings pending against him; and it is declared in section 60 that no creditor whose debt is provable under the Act shall be allowed, after the commencement of proceedings in insolvency, to prosecute to final judgment any action therefor against the debtor. In connection with the foregoing may be mentioned subsections 1 and 2 of section 36, as well as the opening words of section 33, to the effect that the assignee shall have the right and power to recover and to take into his possession, all of the estate, assets, and claims belonging to the insolvent, except such as are exempt by law from execution. - These provisions clearly evince an intention to vest in the assignee, for the benefit of all the creditors of the insolvent, such elements of property and property right as could be reached and subjected by process of law by any single creditor suing alone. And this is exactly as it should be: for it cannot be supposed that the Legislature would suppress the right of action of every individual creditor upon the adjudication of insolvency, and at the same time allow the insolvent debtor to retain anything subject to the payment of his debts in a normal state of solvency. - "leviable assets" and "assets in insolvency" are practically coextensive terms. Hence, in determining what elements of value constitute assets in insolvency, SC is at liberty to consider what elements of value are subject to be taken upon execution, and vice versa. On whether a policy of insurance having no cash surrender value, but payable to the insured or his legal representative, is property that may be taken upon execution against him. - Philippine laws declare no exemption with respect to insurance policies; and this species of property is not enumerated, in section 48 of the Insolvency Law, among items from the ownership of which the assignee is excluded. Moreover, all life insurance policies are declared by law to be assignable, regardless of whether the assignee has an insurable

INSURANCE 57
interest in the life of the insured or not (Insurance Act No. 2427, sec. 166). - SC has held that insurance policies having a present cash surrender value are subject to be taken upon execution. (Misut Garcia vs. West Coast San Francisco Life Ins. Co., 41 Phil., 258.) - a policy devoid of a cash surrender value cannot be either "leviable assets" or "assets in insolvency." - the assignee in insolvency acquired no beneficial interest in the policy of insurance in question; that its proceeds are not liable for any of the debts provable against the insolvent in the pending proceedings, and that said proceeds should therefore be delivered to his administratrix. Disposition Judgment reversed. Sun Life is directed to pay the proceeds of the policy to Tan Sit. CHAPTER VI RESCISSION OF INSURANCE CONTRACTS: CONCEALMENT, MISREPRESENTATION, & BREACH OF WARRANTIES NG v. ASIAN CRUSADER LIFE ASSURANCE CORP 122 SCRA 461 ESCOLIN; May 30, 1983 FACTS - On May 12, 1962, Kwong Nam applied for a 20-year endowment insurance on his life for the sum of P20,000, with his wife, Ng Gan Zee, as beneficiary. - He died on Dec 1963 of cancer of the liver with metastasis. All premiums had been paid at the time of his death. - Ng presented a claim for payment of the face value of the policy. Appellant (Asian Crusader) denied the claim on the ground that the answers given by the insured to the questions appearing in his application for life insurance were untrue. -Appellant: the insured was guilty of misrepresentation when 1) he answered "No" to the question (in the application) of "Has any life insurance company ever refused your application for insurance or for reinstatement of a lapsed policy or offered you a policy different from that applied for?" when in fact, Insular Life denied his application for reinstatement of his lapsed life insurance policy 2) he gave the appellant's medical examiner false and misleading information as to his ailment and previous operation when he said he was operated on for a Tumor [mayoma] of the stomach associated with ulcer of stomach. Tumor taken out was hard and of a hen's egg size. Operation was two years ago in Chinese General Hospital by Dr. Yap. Claims he is completely recovered. Medical report show that insured was operated on for "peptic ulcer", involving the excision of a portion of the stomach, not tumor. ISSUE WON there was concealment (Was appellant, because of insured's aforesaid representation, misled or deceived into entering the contract or in accepting the risk at the rate of premium agreed upon?) HELD NO -"concealment exists where the assured had knowledge of a fact material to the risk, and honesty, good faith, and fair dealing requires that he should communicate it to the assurer, but he designedly and intentionally withholds the same." - It has also been held "that the concealment must, in the absence of inquiries, be not only material, but fraudulent, or the fact must have been intentionally withheld." Reasoning 1) The evidence shows that the Insular Life Assurance Co., Ltd. approved Kwong Nam's request for reinstatement and amendment of his lapsed insurance policy on April 24, 1962. It results, therefore, that when on May 12, 1962 Kwong Nam answered `No' to the question whether any life insurance company ever refused his application for reinstatement of a lapsed policy he did not misrepresent any fact. 2) Assuming that the aforesaid answer given by the insured is false, Sec. 278 of the Insurance Law nevertheless requires that fraudulent intent on the part of the insured be established to entitle the insurer to rescind the contract. And as correctly observed by the lower court, "misrepresentation as a defense of the insurer to avoid liability is an `affirmative defense. The duty to establish such a defense by satisfactory and convincing evidence rests upon the defendant. The evidence before the Court

Page
does not clearly and satisfactorily establish that defense." -Kwong Nam had informed the appellant's medical examiner that the tumor for which he was operated on was ''associated with ulcer of the stomach." In the absence of evidence that the insured had sufficient medical knowledge as to enable him to distinguish between "peptic ulcer" and "a tumor", his statement that said tumor was "associated with ulcer of the stomach" should be construed as an expression made in good faith of his belief as to the nature of his ailment and operation. Indeed, such statement must be presumed to have been made by him without knowledge of its incorrectness and without any deliberate intent on his part to mislead the appellant. 3) Waiver: While it may be conceded that, from the viewpoint of a medical expert, the information communicated was imperfect, the same was nevertheless sufficient to have induced appellant to make further inquiries about the ailment and operation of the insured. Section 32 of Insurance Law [Act No. 2427] provides: The right to information of material facts may be waived either by the terms of insurance or by neglect to make inquiries as to such facts where they are distinctly implied in other facts of which information is communicated. It has been held that where, "upon the face of the application, a question appears to be not answered at all or to be imperfectly answered, and the insurers issue a policy without any further inquiry, they waive the imperfection of the answer and render the omission to answer more fully immaterial. Disposition the judgment appealed from is hereby affirmed, with costs against appellant CANILANG v. CA (GREAT ASSURANCE CORP.) 223 SCRA 443 FELICIANO; June 17, 1993 PACIFIC LIFE

NATURE Petition for review on certiorari of the decision of the Court of Appeals FACTS - June 18, 1982 Jaime Canilang was diagnosed by Dr. Claudio to have sinus tachycardia. He was directed by the doctor to take a tranquilizer (Trazepam) and a beta-blocker drug (Aptin).

"Sec. 27. Such party to a contract of insurance must communicate to the other, in good faith, all facts within his knowledge which are material to the contract, and which the other has not the means of ascertaining, and as to which he makes no warranty."

INSURANCE 58
- August 3, 1982 Jaime consulted Dr. Claudio again and was diagnosed to have acute bronchitis. - August 4, 1982 Jaime applied for a nonmedical insurance policy with Great Pacific Life Assurance Company. He named his wife Thelma as his beneficiary. He was issue the policy with a face value of P19,700 effective August 9, 1982. - August 5, 1983 Jaime died of congestive heart failure, anemia and chronic anemia. Thelma filed her claim but the insurance company refused to grant it on the ground that Jaime had concealed information. - Thelma filed a complaint against Great Pacific to recover the insurance proceeds. She testified that she was not aware of her husbands ailments and that she thought he had died from a kidney disorder. - Great Pacific presented as witness Dr. Quismorio who testified that Jaimes insurance application was the basis of his medical declaration and she explained that an applicant was required to undergo medical examination only if the applicant had disclosed that he had previously been consulted with a doctor and had been hospitalized. - The Insurance Commissioner ordered Great Pacific to pay Thelma the insurance proceeds, including attorneys fees, holding that Jaimes illness was not that serious as to Great Pacifics decision to insure him and that there was no concealment on the part of Jaime with regard to his illness. Petitioners Claim: > Thelma argues that the non-disclosure of Jaime did not amount to fraud. > She also argues that the CA erred in not holding that the issue in the case agreed upon between the parties before the Insurance Commission is whether or not Jaime 'intentionally' made material concealment in stating his state of health; Respondents Comments: > The CA reversed the Insurance Commissioners decision, holding that the use of the word 'intentionally" by the Insurance Commissioner in defining and resolving the issue agreed upon by the parties at pre-trial before the Insurance Commissioner was not supported by the evidence and that the issue agreed upon by the parties had been whether Jaime made a material concealment as to the state of his health at the time of the filing of insurance application, justifying the denial of the claim. > It also found that the failure of Jaime to disclose previous medical consultation and treatment constituted material information which should have been communicated to Great Pacific to enable the latter to make proper inquiries. ISSUES 1. WON Jaime intentionally withheld information from Great Pacific 2. WON the information withheld would have been material to Great Pacifics decision to grant Jaime the insurance policy HELD 1. YES Ratio Section 27 of the Insurance Code of 1978 is properly read as referring to "any concealment without regard to whether such concealment is intentional or unintentional. The restoration in 1985 by B.P. Blg. 874 of the phrase "whether intentional or unintentional" merely underscored the fact that all throughout (from 1914 to 1985), the statute did not require proof that concealment must be "intentional" in order to authorize rescission by the injured party. Reasoning - Art. 27 of the 1978 Insurance Code reads that a concealment entitles the injured party to rescind a contract of insurance, which does not include the words whether intentional or unintentional from the previous statutes. The Insurance Commissioner relied on this deletion in arguing that the statute intended to limit the kinds of concealment which generate a right to rescind on the part of the injured party to "intentional concealments." - In the case at bar, the nature of the facts not conveyed to the insurer was such that the failure to communicate must have been intentional rather than merely inadvertent. > Jaime could not have been unaware that his heart beat would at times rise to high and alarming levels and that he had consulted a doctor twice two months before applying for non-medical insurance. > The last medical consultation took place just the day before the insurance application was filed. 2. YES Ratio Materiality relates rather to the "probable and reasonable influence of the facts" upon the party to whom the communication should have been made, in assessing the risk involved in making or omitting to make further inquiries and in accepting the application for insurance; that "probable and reasonable influence of the farts" concealed must, of course, be determined objectively, by the judge ultimately. Reasoning

Page
- The information which Jaime failed to disclose was material to the ability of Great Pacific to estimate the probable risk he presented as a subject of life insurance. - Had Canilang disclosed his visits to his doctor, the diagnosis made and the medicines prescribed by such doctor, in the insurance application, it may be reasonably assumed that Great Pacific would have made further inquiries and would have probably refused to issue a non-medical insurance policy or, at the very least, required a higher premium for the same coverage. - As held in the case of Saturnino vs. PhilippineAmerican Life Insurance, the waiver of medical examination in a non-medical insurance contract renders even more material the information inquired of the applicant concerning previous condition of health and diseases suffered, for such information necessarily constitutes an important factor which the insurer takes into consideration in deciding whether to issue the policy or not. Disposition the Petition for Review is DENIED for lack of merit and the Decision of the Court of Appeals dated 16 October 1989 in C.A.-G.R. SP No. 08696 is hereby AFFIRMED. YU PANG CHENG v. CA 105 PHIL 930 BAUTISTA ANGELO; May 29, 1959 FACTS - September 5, 1950: Yu Pang Eng submitted parts II and III of his application for insurance consisting of the medical declaration made by him to the medical examiner of defendant and the medical examiner's report - September 7: he submitted part I of his application which is the declaration made by him to an agent of defendant - September 8: defendant issued to the insured Policy No. 812858 - December 27, 1950: the insured entered St. Luke's Hospital for medical treatment but he died on February 27, 1951. - According to the death certificate, he died of "infiltrating medullary carcinoma, Grade 4, advanced cardiac and of lesser curvature, stomach metastases spleen." - Plaintiff, brother and beneficiary of the insured, demanded from defendant the payment of the

INSURANCE 59
proceeds of the insurance policy and when the demand was refused, he brought the present action. - The insured, in his application for insurance, particularly in his declarations to the examining physician, stated the following in answering the questions propounded to him: 14. Have you ever had any of the following diseases or symtoms? Each question must be read and answered "Yes" or "No.". "Gastritis, Ulcer of the Stomach or any disease of that organ? No. "Vertigo, Dizziness, Fainting-spells or Unconsciouness? No. "Cancer, Tumors or Ulcers of any kind? No. - 15. Have you ever consulted any physician riot included in any of the above answers? Give names and address or physicians list ailments or accidents and date. No." - It appears that the insured entered the Chinese General Hospital for medical treatment on January 29, 1950 having stayed there up to February 11, 1950. - An X-ray picture of his stomach was taken and the diagnosis made of him by his doctors showed that his illness was "peptic ulcer, bleeding." ISSUE WON the insured is guilty of concealment of some facts material to the risk insured against which has the effect of avoiding the policy as found by respondent court. HELD - It should be noted that the insured's confinement in the Chinese General Hospital took place from January 29, 1950 to February 11, 1950, whereas his application for insurance wherein he stated his answers to the questions propounded to him by the examining physician of defendant was submitted to defendant on September 5, 1950. - It is apparent that when the insured gave his answers regarding his previous ailment, particularly with regard to "Gastritis, Ulcer of the Stomach or any disease of that organ" and "Vertigo, Dizziness, Fainting-spells or Unconsciousness", he concealed the ailment of which he was treated in the Chinese General Hospital which precisely has direct connection with the subject of the questions propounded. - The negative answers given by the insured regarding his previous ailment, or his concealment of the fact that he was hospitalized and treated for sometime of peptic ulcer and had suffered from "dizziness, anemia, abdominal pains and tarry stools", deprived defendant of the opportunity to make the necessary inquiry as to the nature of his past illness so that it may form its estimate relative to the approval of his application. - Had defendant been given such opportunity, considering the previous illness of the insured as disclosed by the records of the Chinese General Hospital, defendant would probably had never consented to the issuance of the policy in question. In fact, according to the death certificate, the insured died of "infiltrating medullary carcinoma, Grade, 4, advanced cardiac and of lesser curvature, stomach metastases spleen", which may have a direct connection with his previous illness. - Our Insurance Law provides that "A neglect to communicate that which a party knows and ought to communicate, is called concealment" (Section 25, Act No. 2427). Whether intentional or unintentional, the concealment entitles the insurer to rescind the contract of insurance (Section 26). - Our law even requires the insured to communicate to the insurer all facts within his knowledge which are material to the contract and which the other party has not the means of ascertaining (Section 27), and the materiality is to be determined not by the event but solely by the probable and reasonable influence of the facts upon the party to whom the communication is due (Section 30). - Argente vs. West Coast Life Insurance Co.: "One ground for the rescission of a contract of insurance under the Insurance Act is 'a concealment', which in section 25 is defined 'A neglect to communicate that which a party knows and ought to communicate.' Appellant argues that the concealment was immaterial and insufficient to avoid the policy. We cannot agree. In an action on a life insurance policy where the evidence conclusively shows that the answers to questions concerning diseases were untrue, the truth or falsity of the answers become the determining factor. If the policy was procured by fraudulent representations, the contract of insurance apparently set forth therein was never legally existent. It can fairly be assumed that had the true facts been disclosed by the assured, the insurance would never have been granted." Disposition Decision affirmed.

Page

GREAT PACIFIC LIFE v. CA (supra p.34) PACIFIC BANKING CORP v. ASSURANCE CORPORATION) 168 SCRA 1 PARAS; November 28, 1988 NATURE Petition for review which set aside the in turn granted the civil case filed by Assurance. CA (ORIENTAL

on certiorari of the CA decision, decision of CFI Manila, which had complaint for a sum of money in Pacific Banking against Oriental

FACTS - October 21,1963: an open Fire Policy was issued to the Paramount Shirt Manufacturing Co. (insured), by which Oriental Assurance Corporation bound itself to indemnify the insured for any loss or damage, not exceeding P61,000.00, caused by fire to its property consisting of stocks, materials and supplies usual to a shirt factory, including furniture, fixtures, machinery and equipment while contained in the ground, second and third floors of the building situated at number 256 Jaboneros St., San Nicolas, Manila, for a period of one year commencing from that date to October 21, 1964. - Insured was at the time of the issuance of the policy and is up to this time, a debtor of Pacific Banking in the amount of not less P800,000.00 and the goods described in the policy were held in trust by the insured for the Pacific Banking under thrust receipts. - Said policy was duly endorsed to Pacific Banking as mortgagee/trustor of the properties insured, with the knowledge and consent of Oriental Assurance to the effect that "loss if any under this policy is payable to the Pacific Banking Corporation". - While the aforesaid policy was in full force and effect, a fire broke out on the subject premises destroying the goods contained in its ground and second floors. Counsel for the Pacific Banking sent a letter of demand to Oriental Assurance for indemnity due to the loss of property by fire. Oriental Assurance informed counsel that it was not yet ready to accede to the latter's demand as the former is awaiting the final report of the insurance adjuster, H.H. Bayne Adjustment Company. - Said insurance adjuster notified counsel for the Pacific Banking that the insured under the policy had not filed any claim with it, nor submitted proof of loss

INSURANCE 60
which is a clear violation of Policy Condition No.11, and for which reason, determination of the liability of Oriental Assurance could not be had. Pacific Banking's counsel replied asking the insurance adjuster to verify from the records of the Bureau of Customs the entries of merchandise taken into the customs bonded warehouse razed by fire as a reliable proof of loss. - For failure of the insurance company to pay the loss as demanded, Pacific Banking field before CFI an action for a sum of money against the Oriental Assurance, in the principal sum of P61,000.00 issued in favor of Paramount Shirt Manufacturing Co. Oriental Assurance defenses (a) lack of formal claim by insured over the loss and (b) premature filing of the suit as neither plaintiff nor insured had submitted any proof of loss on the basis of which defendant would determine its liability and the amount thereof, either to the Oriental Assurance or its adjuster H.H. Bayne Adjustment Co. Pacific Banking > presented evidence that insured has undeclared coinsurances with the following: P30,000.00 with Wellington Insurance; P25,000. 00 with Empire Surety and P250,000.00 with Asian Surety; undertaken by insured Paramount on the same property covered by its policy with Oriental Assurance whereas the only co-insurances declared in the subject policy are those of P30,000.00 with Malayan, P50,000.00 with South Sea, and P25.000.00 with Victory - NOTE: the defense of fraud and/or violation of nondeclaration of co-insurances was not pleaded in the answer, also not pleaded in the Motion to Dismiss. - CFI denied Oriental Assurance's motion on the ground that since the defense was raised for the first time, it must be deemed to have waived the requirement of proof of loss. Case was submitted for decision. But upon MR, Oriental Asurance was allowed to present additional evidence, "in order to prove that 'insured has committed a violation of condition No. 3 of the policy in relation to the other Insurance Clause.' " CFI eventually adjudged Oriental Assurance liable to the Pacific Banking under the said contract of insurance. - Court of Appeals reversed. Pacific Banking's MR denied. ISSUES 1. WON insured is guilty of fraud 2. WON mortgagee/assignee can still claim from the insurance HELD 1. YES - The crux of the controversy centers on two points: (a) unrevealed co-insurances which violated policy conditions No. 3; and (b) failure of the insured to file the required proof of loss prior to court action. - Policy Condition No. 3 explicitly provides: The Insured shall give notice to the Company of any insurance already effected, or which may subsequently be effected, covering any of the property hereby insured, and unless such notice be given and the particulars of such insurance or insurances be stated in or endorsed on this Policy by or on behalf of the Company before the occurrence of any loss or damage, all benefit under this policy shall be forfeited. - It is not disputed that the insured failed to reveal before the loss three other insurances. By reason of said unrevealed insurances, the insured had been guilty of a false declaration; a clear misrepresentation and a vital one because where the insured had been asked to reveal but did not, that was deception. Otherwise stated, had the insurer known that there were many co-insurances, it could have hesitated or plainly desisted from entering into such contract. Hence, the insured was guilty of clear fraud. - Pacific Banking's contention that the allegation of fraud is but a mere inference or suspicion is untenable. Concrete evidence of fraud or false declaration by the insured was furnished by the Pacific Banking itself when the facts alleged in the policy under clauses "Co-Insurances Declared" and "Other Insurance Clause" are materially different from the actual number of co-insurances taken over the subject property. Consequently, the whole foundation of the contract fails, the risk does not attach and the policy never becomes a contract between the parties. Representations of facts are the foundation of the contract and if the foundation does not exist, the superstructure does not arise. Falsehood in such representations is not shown to vary or add to the contract, or to terminate a contract which has once been made, but to show that no contract has ever existed (Tolentino). A void or inexistent contract is one which has no force and effect from the very beginning, as if it had never been entered into, and which cannot be validated either by time or by ratification. - As the insurance policy against fire expressly required that notice should be given by the insured of

Page
other insurance upon the same property, the total absence of such notice nullifies the policy. - Argument that notice of co-insurances may be made orally is preposterous and negates policy condition No. 20 which requires every notice and other communications to the insurer to be written or printed. 2. NO - Subject mortgage clause pecifically provides: Loss, if any, under this policy, shall be payable to the PACIFIC BANKING CORPORATION Manila mortgagee/trustor as its interest may appear, it being hereby understood and agreed that this insurance as to the interest of the mortgagee/trustor only herein, shall not be invalidated by any act or neglect except fraud or misrepresentation, or arson of the mortgagor or owner/trustee of the property insured; provided, that in case the mortgagor or owner/ trustee neglects or refuses to pay any premium, the mortgagee/ trustor shall, on demand pay the same. - The paragraph clearly states the exceptions to the general rule that insurance as to the interest of the mortgagee, cannot be invalidated; namely: fraud, or misrepresentation or arson. - Concealment of the aforecited co-insurances can easily be fraud, or in the very least, misrepresentation. It is but fair and just that where the insured who is primarily entitled to receive the proceeds of the policy has by its fraud and/or misrepresentation, forfeited said right, with more reason Pacific Banking which is merely claiming as indorsee of said insured, cannot be entitled to such proceeds. - The fact of fraud was tried by express or at least implied consent of the parties. Pacific Banking did not only object to the introduction of evidence but on the contrary, presented the very evidence that proved its existence. - Be that as it may, SC has ample authority to give beyond the pleadings where in the interest of justice and the promotion of public policy, there is a need to make its own finding to support its conclusion. Otherwise stated, the Court can consider a fact which surfaced only after trial proper. - Generally, the cause of action on the policy accrues when the loss occurs, but when the policy provides that no action shall be brought unless the claim is first presented extrajudicially in the manner provided in the policy, the cause of action will accrue from the time the insurer finally rejects the claim for payment.

INSURANCE 61
- In the case at bar, policy condition No. 11 specifically provides that the insured shall on the happening of any loss or damage give notice to the company and shall within fifteen (15) days after such loss or damage deliver to the Oriental Assurance (a) a claim in writing giving particular account as to the articles or goods destroyed and the amount of the loss or damage and (b) particulars of all other insurances, if any. Likewise, insured was required "at his own expense to produce, procure and give to the company all such further particulars, plans, specifications, books, vouchers, invoices, duplicates or copies thereof, documents, proofs and information with respect to the claim". - Evidence adduced shows that 24 days after the fire, Pacific Banking merely wrote letters to Oriental Assurance to serve as a notice of loss, thereafter, the former did not furnish the latter whatever pertinent documents were necessary to prove and estimate its loss. Instead, Pacific Banking shifted upon Oriental Assurance the burden of fishing out the necessary information to ascertain the particular account of the articles destroyed by fire as well as the amount of loss. - Oriental Assurance and its adjuster notified Pacific Banking that insured had not yet filed a written claim nor submitted the supporting documents in compliance with the requirements set forth in the policy. Despite the notice, the latter remained unheedful. Since the required claim by insured, together with the preliminary submittal of relevant documents had not been complied with, it follows that Oriental Assurance could not be deemed to have finally rejected Pacific Banking's claim and therefore the latter's cause of action had not yet arisen. Compliance with condition No. 11 is a requirement sine qua non to the right to maintain an action as prior thereto no violation of Pacific Banking's right can be attributable to Oriental Assurance. As before such final rejection, there was no real necessity for bringing suit. Pacific Banking should have endeavored to file the formal claim and procure all the documents, papers, inventory needed by Oriental Assurance or its adjuster to ascertain the amount of loss and after compliance await the final rejection of its claim. Indeed, the law does not encourage unnecessary litigation. - Pacific Banking prematurely filed the civil case and dismissal thereof was warranted under the circumstances. While it is a cardinal principle of insurance law that a policy or contract of insurance is to be construed liberally in favor of the insured and strictly as against the insurer company yet, contracts of insurance, like other contracts, are to be construed according to the sense and meaning of the terms which the parties themselves have used. If such terms are clear and unambiguous, they must be taken and understood in their plain, ordinary and popular sense. - Contracts of insurance are contracts of indemnity upon the terms and conditions specified in the policy. The parties have a right to impose such reasonable conditions at the time of the making of the contract as they may deem wise and necessary. The agreement has the force of law between the parties. The terms of the policy constitute the measure of the insurer's liability, and in order to recover, the insured must show himself within those terms. The compliance of the insured with the terms of the policy is a condition precedent to the light of recovery. - It appearing that insured has violated or failed to perform the conditions under No. 3 and 11 of the contract, and such violation or want of performance has not been waived by the insurer, the insured cannot recover, much less the herein Pacific Banking. Courts are not permitted to make contracts for the parties; the function and duty of the courts is simply to enforce and carry out the contracts actually made. Disposition Petition dismissed. CA affirmed. SUNLIFE ASSURANCE COMPANY v. CA (SPS. BACANI) 245 SCRA 268 QUIASON; June 22, 1995 NATURE A petition for review on certiorari. FACTS - April 15, 1986: Robert John B. Bacani procured a life insurance contract for himself from SUNLIFE (petitioner) valued at P100K. The designated beneficiary was his mother, Bernarda Bacani (respondent). - June 26, 1987: the insured died in a plane crash. Bernarda Bacani filed a claim with Sunlife, seeking the benefits of the insurance policy taken by her son. Petitioner conducted an investigation and its findings prompted it to reject the claim on the ground that the insured did not disclose facts material to the issuance of the policy. The insured gave false statements in the application when he answered in the negative to the

Page
question have you ever had or sought advice for urine, kidney, bladder disorder? - Sunlife discovered that two weeks prior to the issuance, insured was diagnosed with renal failure, was confined, and underwent tests. - November 17, 1988: Bacani and her husband filed for specific performance against Sunlife. RTC granted the plea on the ground that that the facts concealed by the insured were made in good faith and under the belief that they need not be disclosed, and that the disclosure was not material since the policy was nonmedical. - Sunlife appealed to the CA, but the latter denied the appeal on the ground that the cause of death was unrelated to the facts concealed by the insured. Petitioners Claim > The insured did not disclose facts relevant to the issuance of the policy, thus rescission of the contract may be invoked by the insurance company. Respondents Comments > The actual cause of death was not relevant to the concealed information, and the policy was entered into by the insured in good faith. ISSUE WON the concealment renders the insurance policy rescissible HELD YES Ratio The terms of the contract are clear. The insured is specifically required to disclose to the insurer matters relating to his health. Reasoning SEC. 26 (IC) A neglect to communicate that which a party knows and ought to communicate, is called a concealment. SEC. 31 (IC) Materiality is to be determined not by the event, but solely by the probable and reasonable influence of the facts upon the party to whom communication is due, in forming his estimate of the disadvantages of the proposed contract or in making his inquiries - The information which the insured failed to disclose was material and relevant to the approval and the issuance of the insurance policy. The matters concealed would have definitely affected petitioner's action on his application, either by approving it with

INSURANCE 62
the corresponding adjustment for a higher premium or rejecting the same. - Good faith is no defense in concealment. It appears that such concealment was deliberate on the part of the insured. - The waiver of a medical examination [in a nonmedical insurance contract] renders even more material the information required of the applicant concerning previous condition of health and diseases suffered, for such information necessarily constitutes an important factor which the insurer takes into consideration in deciding whether to issue the policy or not. - Anent the finding that the facts concealed had no bearing to the cause of death of the insured, it is well settled that the insured need not die of the disease he had failed to disclose to the insurer. It is sufficient that his non-disclosure misled the insurer in forming his estimates of the risks of the proposed insurance policy or in making inquiries Disposition Petition is granted and the decision of CA is reversed and set aside. EGUARAS v. GREAT EASTERN 33 PHIL. 263 TORRES.; January 24, 1916 NATURE Appeal filed through bill of exceptions from the judgment of the CFI FACTS - Francisca Eguaras filed a written complaint in court, alleging as a cause of action that her son-in-law Dominador Albay had applied in writing to the defendant insurance company to insure his life for the sum of P5,000, naming as the beneficiary in case of his death the plaintiff Francisca Eguaras; that after compliance with the requisites and the investigation carried on by the defendant company, it accepted the application for insurance and issued the policy; that, said policy being in force, the insured died, and despite the fact that the beneficiary submitted satisfactory proofs of his death and that the defendant company investigated the event, still it refused and continues to refuse to pay to the plaintiff the value of the policy. - Defendant set forth in special defense that the insurance policy issued in the name of Dominador Albay had been obtained through fraud and deceit known and consented to by the interested parties and is therefore completely illegal, void, and ineffective. - A criminal case for frustrated estafa was filed by defendant against Ponciano Remigio, Castor Garcia and Francisca Eguaras. They were acquitted, and claim that the judgment produces the effect of res judicata in the present suit. ISSUE WON the life insurance obtained by Dominador Albay was issued through fraud and deceit HELD YES Ratio In a contract where one of the contracting parties may have given his consent through error, violence, intimidation, or deceit, and in any of such cases the contract is void, even though, despite this nullity, no crime was committed. There may not have been estafa in the case at bar, but it was conclusively demonstrated by the trial that deceit entered into the insurance contract, fulfillment whereof is claimed, and therefore the conclusions reached by the court in the judgment it rendered in the criminal proceedings for estafa do not affect this suit, nor can they produce in the present suit the force of res adjudicata. Reasoning - It is proven that the signatures on the insurance applications reading "Dominado Albay" are false and forged; that the person who presented himself to Dr. Vidal to be examined was not the real Dominador Albay, but Castor Garcia who was positively identified by Dr. Vidal; that at the time of the application for insurance and the issuance of the policy which is the subject matter of this suit the real Dominador Albay was informed of all those machinations, wherefore it is plain that the insurance contract between the defendant and Dominador Albay is null and void because it is false, fraudulent and illegal. Disposition The judgment appealed from is reversed and the defendant absolved from the complaint without special finding as to the costs. QUA CHEE GAN v. LAW UNION AND ROCK 98 PHIL 85 REYES; December 17, 1955 FACTS - Qua Chee Gan insured 4 of his bodegas with Law Union & Rock Insurance Co in 1937. These bodegas

Page
were used for the storage of stocks of copra and of hemp, baled and loose. - Fire of undetermined origin that broke out in the early morning of July 21, 1940, and lasted almost one week, gutted and completely destroyed Bodegas Nos. 1, 2 and 4, with the merchandise stored therein. - Qua Chee Gan informed the insurance company of the fire. Fire adjusters of the company conducted an extensive investigation. Qua Chee Gan submitted the corresponding fire claims, totaling P398,562.81 (but reduced to the full amount of the insurance, P370,000), the Insurance Company resisted payment, claiming violation of warranties and conditions, filing of fraudulent claims, and that the fire had been deliberately caused by the insured or by other persons in connivance with him. - Qua Chee Gan, his brother and his employees were tried for arson, where counsel of the insurance company acted as a private prosecutor. They were acquitted. - This civil suit was then instituted to claim against the insurance company. The CFI ruled in favor of Qua Chee Gan and ordered Law Union Rock Co. to pay. ISSUES 1. WON there was a breach of the fire hydrant warranty 2. WON the insured violated the Hemp warranty 3. WON Qua Chee Gan is guilty of overvaluation 4. WON Qua Chee Gan caused the fire 5. WON there was an error in the amount of copra and hemp lost 6. WON the claims contained false and fraudulent statements HELD 1. NO - It is argued that he should have 11 fire hydrants in the compound, but he only had 2. We are in agreement with the trial Court that the appellant is barred by waiver (or rather estoppel) to claim violation of the so-called fire hydrants warranty, for the reason that knowing fully all that the number of hydrants demanded therein never existed from the very beginning, the appellant nevertheless issued the policies in question subject to such warranty, and received the corresponding premiums. 2. NO - The insurance company avers that the insured violated the hemp warranty when it admitted that it had 36 cans of gasoline in the building. It is well to

INSURANCE 63
note that gasoline is not specifically mentioned among the prohibited articles listed in the so-called "hemp warranty." The cause relied upon by the insurer speaks of "oils (animal and/or vegetable and/or mineral and/or their liquid products having a flash point below 300o Fahrenheit", and is decidedly ambiguous and uncertain; for in ordinary parlance, "Oils" mean "lubricants" and not gasoline or kerosene. And how many insured, it may well be wondered, are in a position to understand or determine "flash point below 003o Fahrenheit. Here, again, by reason of the exclusive control of the insurance company over the terms and phraseology of the contract, the ambiguity must be held strictly against the insurer and liberally in favor of the insured, especially to avoid a forfeiture - Another point that is in favor of the insured is that the gasoline kept in Bodega No. 2 was only incidental to his business, being no more than a customary 2 day's supply for the five or six motor vehicles used for transporting of the stored merchandise). "It is well settled that the keeping of inflammable oils on the premises though prohibited by the policy does not void it if such keeping is incidental to the business." (Bachrach vs. British American Ass. Co., 17 Phil. 555, 560) 3. NO - The charge that the insured failed or refused to submit to the examiners of the insurer the books, vouchers, etc. demanded by them was found unsubstantiated by the trial Court, and no reason has been shown to alter this finding. - In view of the discrepancy in the valuations between the insured and the adjuster Stewart for the insurer, the Court referred the controversy to a government auditor, Apolonio Ramos; but the latter reached a different result from the other two. Not only that, but Ramos reported two different valuations that could be reached according to the methods employed. Clearly then, the charge of fraudulent overvaluation cannot be seriously entertained. 4. NO - This defense is predicted on the assumption that the insured was in financial difficulties and set the fire to defraud the insurance company, presumably in order to pay off the Philippine National Bank, to which most of the insured hemp and copra was pledged. This defense is fatally undermined by the established fact that, notwithstanding the insurer's refusal to pay the value of the policies the extensive resources of the insured enabled him to pay off the National Bank in a short time; and if he was able to do so, no motive appears for attempt to defraud the insurer. While the acquittal of the insured in the arson case is not res judicata on the present civil action, the insurer's evidence, to judge from the decision in the criminal case, is practically identical in both cases and must lead to the same result, since the proof to establish the defense of connivance at the fire in order to defraud the insurer "cannot be materially less convincing than that required in order to convict the insured of the crime of arson. 5. NO - As to the defense that the burned bodegas could not possibly have contained the quantities of copra and hemp stated in the fire claims, the insurer's case rests almost exclusively on the estimates, inferences and conclusions of its adjuster investigator, Alexander D. Stewart, who examined the premises during and after the fire. His testimony, however, was based on inferences from the photographs and traces found after the fire, and must yield to the contradictory testimony of engineer Andres Bolinas, and specially of the then Chief of the Loan Department of the National Bank's Legaspi branch, Porfirio Barrios, and of Bank Appraiser Loreto Samson, who actually saw the contents of the bodegas shortly before the fire, while inspecting them for the mortgagee Bank 6. NO - Appellant insurance company also contends that the claims filed by the insured contained false and fraudulent statements that avoided the insurance policy. But the trial Court found that the discrepancies were a result of the insured's erroneous interpretation of the provisions of the insurance policies and claim forms, caused by his imperfect knowledge of English, and that the misstatements were innocently made and without intent to defraud. The trial courts ruling must be upheld. - For example, the occurrence of previous fires in the premises insured in 1939, altho omitted in the claims, Exhibits EE and FF, were nevertheless revealed by the insured in his claims Exhibits Q (filed simultaneously with them), KK and WW. Considering that all these claims were submitted to the smae agent, and that this same agent had paid the loss caused by the 1939 fire, we find no error in the trial Court's acceptance of the insured's explanation that the omission in Exhibits EE and FF was due to inadvertance, for the insured could hardly expect under such circumstances, that the 1939 would pass unnoticed by the insurance agents. Similarly, the 20 per cent overclaim on 70 per cent of the hemo stock, was explained by the insured

Page
as caused by his belief that he was entitled to include in the claim his expected profit on the 70 per cent of the hemp, because the same was already contracted for and sold to other parties before the fire occurred. Compared with other cases of over-valuation recorded in our judicial annals, the 20 per cent excess in the case of the insured is not by itself sufficient to establish fraudulent intent. Certainly, the insured's overclaim of 20 per cent in the case at bar, duly explained by him to the Court a quo, appears puny by comparison (compared to other cases cited by the court), and can not be regarded as "more than misstatement, more than inadvertence of mistake, more than a mere error in opinion, more than a slight exaggeration" that would entitle the insurer to avoid the policy. It is well to note that the overcharge of 20 per cent was claimed only on a part (70 per cent) of the hemp stock; had the insured acted with fraudulent intent, nothing prevented him from increasing the value of all of his copra, hemp and buildings in the same proportion. This also applies to the alleged fraudulent claim for burned empty sacks, that was likewise explained to our satisfaction and that of the trial Court. The rule is that to avoid a policy, the false swearing must be willful and with intent to defraud which was not the cause. Of course, the lack of fraudulent intent would not authorize the collection of the expected profit under the terms of the polices, and the trial Court correctly deducted the same from its award. Disposition Decision affirmed ARGENTE v. WEST COAST LIFE 51 PHIL 725 MALCOLM; March 19, 1928 FACTS - This is an action upon a joint life insurance policy for P15,000 issued by the West Coast Life Insurance Co., on May 15, 1925, in favor of Bernardo Argente, and his wife, Vicenta de Ocampo, the latter having died on November 18, 1925. Fraud in obtaining the policy was pleaded by way of special defense. On the issue thus suggested, the court adopted the theory of the defendant, and held the insurance policy null and void, with the result that the complaint was dismissed, with costs. -Bernardo Argente signed an application for joint insurance with his wife in the sum of P2,000. The wife, Vicenta de Ocampo, signed a like application for the same policy.

INSURANCE 64
- Bernardo Argente and his wife was examined by Dr. Cesareo Sta. Ana, a medical examiner for the West Coast Life Insurance Co. which did not show previous and existing health problems. - A temporary policy for P15,000 was issued to Bernardo Argente and his wife as of May 15, 1925. In view of the fact that more than thirty days had elapsed since the applicants were examined by the company's physician, each of them was required to file a certificate of health before the policy was delivered to them. - On November 18, 1925, Vicenta de Ocampo died of cerebral apoplexy. Thereafter Bernardo Argente presented a claim. Following investigation conducted by the Manager of the Manila office of the insurance company, it was apparently disclosed that the answers given by the insured in their medical examinations with regard to their health and previous illnesses and medical attendance were untrue. West Coast Life Insurance Co. refused to pay the claim of Bernardo Argente, and wrote him to the effect that the claim was rejected because the insurance was obtained through fraud and misrepresentation. - It is admitted that it appears in the Medical Examiner's Report that Bernardo Argente gave false responses. As well as with the Medical Examiner's Report that Vicenta de Ocampo. It is, however, not disputed that Vicenta de Ocampo was taken by a patrolman, at the request of her husband, Bernardo Argente, on May 19, 1924, to the Meisic police station, and from there was transferred to the San Lazaro Hospital. In San Lazaro Hospital, her case was diagnosed by the admitting physician as "alcoholism," but later Doctor Domingo made a diagnosis of probable "manic-depressive psychosis," and still, later in Mary Chiles Hospital, made a final diagnosis of "phycho-neurosis." - Bernardo Argente, while readily conceding most of the facts herein narrated, yet alleges that both he and his wife revealed to the company's physician, Doctor Sta. Ana, all the facts concerning their previous illnesses and medical attendance, but that Doctor Sta. Ana, presumably acting in collusion with the insurance agent, Jose Geronimo del Rosario, failed to record them in the medical reports. The evidence on these points consists of the testimony of the plaintiff and his subordinate clerk, Apolonio Espiritu, on the one hand, and of the testimony of Doctor Sta. Ana and Jose Geronimo del Rosario on the other. This was rejected by the Trial Court. Trial judge found with the insurance company with regard to the question of fact. SC agrees. There appears no motive whatever on the part of Doctor Sta. Ana to falsify the Medical Examiner's Reports and thereby not only jeopardize his career as a physician, but also gravely implicate himself criminally. ISSUE WON the contract of insurance may be rescinded

Page
"It therefore follows that the assurer in assuming a risk is entitled to know every material fact of which the assured has exclusive or peculiar knowledge, as well as all material facts which directly tend to increase the hazard or risk which are known by the assured, or which ought to be or are presumed to be known by him. And a concealment of such facts vitiates the policy. 'It does not seem to be necessary . . . that the . . . suppression of the truth should have been willful.' If it were but an inadvertent omission, yet if it were material to the risk and such as the plaintiff should have known to be so, it would render the policy void. But it is held that if untrue or false answers are given in response to inquiries and they relate to material facts the policy is avoided without regard to the knowledge or fraud of assured, although under the statute statements are representations which must be fraudulent to avoid the policy. So under certain codes the important inquiries are whether the concealment was willful and related to a matter material to the risk. xxx xxx xxx "If the assured has exclusive knowledge of material facts, he should fully and fairly disclose the same, whether he believes them material or not. But notwithstanding this general rule it will not infrequently happen, especially in life risks, that the assured may have a knowledge actual or presumed of material facts, and yet entertain an honest belief that they are not material. . . . The determination of the point whether there has or has not been a material concealment must rest largely in all cases upon the form of the questions propounded and the exact terms of the contract. Thus, where in addition to specifically named diseases the insured was asked whether he had had any sickness within ten years, to which he answered 'No,' and it was proven that within that period he had had a slight attack of pharyngitis, it was held a question properly for the jury whether such an inflammation of the throat was a 'sickness' within the intent of the inquiry, and the court remarked on the appeal decision that if it could be held as a matter of law that the policy was thereby avoided, then it was a mere device on the part of insurance companies to obtain money without rendering themselves liable under the policy. . . . " . . . The question should be left to the jury whether the assured truly represented the state of his health so as not to mislead or deceive the

HELD YES - Bernardo Argente and his wife applications were false with respect to their state of health during the period of five years preceding the date of such applications and that they knew the representations made by them in their applications were false. The question arises as to the state of the law in relation thereto. - One ground for the rescission of a contract of insurance under the Insurance Act is "a concealment," which in section 25 is defined as "A neglect to communicate that which a party knows and ought to communicate." In an action on a life insurance policy where the evidence conclusively shows that the answers to questions concerning diseases were untrue, the truth or falsity of the answers become the determining factor. If the policy was procured by fraudulent representations, the contract of insurance apparently set forth therein was never legally existent. It can fairly be assumed that had the true facts been disclosed by the assured, the insurance would never have been granted. - In Joyce, The Law of Insurance, second edition, volume 3, Chapter LV, is found the following: "The basis of the rule vitiating the contract in cases of concealment is that it misleads or deceives the insurer into accepting the risk, or accepting it at the rate of premium agreed upon; The insurer, relying upon the belief that the assured will disclose every material fact within his actual or presumed knowledge, is misled into a belief that the circumstance withheld does not exist, and he is thereby induced to estimate the risk upon a false basis that it does not exist. The principal question, therefore, must be, Was the assurer misled or deceived into entering a contract obligation or in fixing the premium of insurance by a withholding of material information or facts within the assured's knowledge or presumed knowledge?

INSURANCE 65
insurer; and if he did not deal in good faith with the insurer in that matter, then the inquiry should be made, Did he know the state of his health so as to be able to furnish a proper answer to such questions as are propounded? A Massachusetts case, if construed as it is frequently cited, would be opposed to the above conclusion; but, on the contrary, it sustains it, for the reason that symptoms of consumption had so far developed themselves within a few months prior to effecting the insurance as to induce a reasonable belief that the applicant had that fatal disease, and we should further construe this case as establishing the rule that such a matter cannot rest alone upon the assured's belief irrespective of what is a reasonable belief, but that it ought to be judged by the criterion whether the belief is one fairly warranted by the circumstances. A case in Indiana, however, holds that if the assured has some affection or ailment of one or more of the organs inquired about so well defined and marked as to materially derange for a time the functions of such organ, as in the case of Bright's disease, the policy will be avoided by a nondisclosure, irrespective of the fact whether the assured knew of such ailment or not. . . ." - Lastly, appellant contends that even if the insurance company had a right to rescind the contract, such right cannot now be enforced in view of the provisions of section 47 of the Insurance Act providing "Whenever a right to rescind a contract of insurance is given to the insurer by any provision of this chapter, such right must be exercised previous to the commencement of an action on the contract." This section was derived from section 2583 of the California Civil Code, but in contrast thereto, makes use of the imperative "must" instead of the permissive "may." Nevertheless, there are two answers to the problem as propounded. The first is that the California law as construed by the code examiners, at whose recommendation it was adopted, conceded that "A failure to exercise the right (of rescission), cannot, of course, prejudice any defense to the action which the concealment may furnish." (Codes of California Annotated; Tan Chay Heng vs. West Coast Life Insurance Company [1927], p. 80, ante.) The second answer is that the insurance company more than one month previous to the commencement of the present action wrote the plaintiff and informed him that the insurance contract was void because it had been procured through fraudulent representations, and offered to refund to the plaintiff the premium which the latter had paid upon the return of the policy for cancellation. As held in California as to a fire insurance policy, where any of the material representations are false, the insurer's tender of the premium and notice that the policy is canceled, before the commencement of suit thereon, operate to rescind the contract of insurance. (Rankin vs. Amazon Insurance Co. [1891], 89 Cal., 203.) Disposition Judgment affirmed, with the costs of this instance against the appellant. GREAT PACIFIC LIFE v. CA (NGO HING) 89 SCRA 543 DE CASTRO, J; April 30, 1979 NATURE Petition for certiorari FACTS - On March 14, 1957, private respondent Ngo Hing filed an application with the Great Pacific Life Assurance Co. (Pacific Life) for a 20 year endowment policy of P50k on the life of his 1 year old daughter, Helen. Ngo Hing supplied the essetntial data which petitioner Mondragon, branch manager of the Pacific Life in Cebu, wrote on the corresponding form in his own handwriting, later typing the data on an application form signed by Ngo Hing. The latter paid the P1077.75 annual premium but retained P1,317 as commission as he was also a duly authorized agent of Pacific Life. The binding deposit receipt was then issued to Ngo Hing; Mondragon handwrote his strong recommendation for the approval of the application on the back of the form. - On April 30, Mondragon received a letter from Pacific Life which stated that the 20 year endowment plan was not available for minors below 7, but that Pacific Life could consider the same under the Juvenile Triple Action Plan, advising that if the offer was acceptable, the Juvenile Non-Medical Declaration be sent to the company. -Mondragon allegedly failed to inform Ngo Hing of the non-acceptance of the insurance plan, instead writing Pacific Life again, recommending the approval of the endowment plan to children since customers had been asking for such coverage since 1954. -On May 28, 1957, Helen died of influenza. Ngo Hing sought the payment of the proceeds of the insurance, but having failed to do so, filed an action for recovery

Page
with the CFI of Cebu. The Court ordered Pacific Life to pay P50k with 6% interest, hence this petition. ISSUE WON the binding deposit receipt constituted temporary contract of the life insurance in question a

HELD NO - The binding deposit receipt is merely a provisional contract and only upon compliance with the ff conditions: (1) that the company be satisfied that the applicant was insurable on standard rates (2) that if the company does not accept the application and offers a different policy, the insurance contract shall not be binding until the applicant accepts the new policy (3) that if the applicant is not found to be insurable on standard rates and the application is disapproved, the insurance shall not be in force at any time and the premium be returned to the applicant. -This implies the receipt is merely an acknowledgement, on behalf of the company, that the Cebu branch of Pacific Life had received the premium and had accepted the application subject to processing by the insurance company, which will approve or reject it depending on whether the applicant is insurable on standard rates. As such, the receipt was never in forceit does not insure outright. No liability attaches until the principal approves the risk and a receipt is given by the agent; because private respondent failed to accept Pacific Lifes offer for the Juvenile Triple Action plan, there was no meeting of the minds and thus no contract. Also, being an authorized agent of Pacific Life, Ngo Hing must have known the company did not offer the insurance applied for and merely took a chance on Mondragons recommendation. Disposition the decision appealed from is set aside, absolving Pacific Life from their civil liabilities EDILLON v. MANILA BANKERS LIFE 117 SCRA 187 VASQUEZ; September 30, 1982 NATURE Appeal from a decision of the CFI FACTS - Sometime in April 1969, Carmen O, Lapuz applied with respondent insurance corporation for insurance coverage against accident and injuries. In the

INSURANCE 66
application form which was dated April 15, 1969, she gave the date of her birth as July 11, 1904. On the same date, she paid the sum of P20.00 representing the premium for which she was issued the corresponding receipt signed by an authorized agent of the respondent insurance corporation. Upon the filing of said application and the payment of the premium on the policy applied for, the respondent insurance corporation issued to Carmen O. Lapuz its Certificate of Insurance. The policy was to be effective for a period of 90 days. - On May 31, 1969 or during the effectivity of the Insurance, Carmen O. Lapuz died in a vehicular accident. - On June 7, 1969, petitioner Regina L. Edillon, a sister of the insured and who was the named beneficiary in the policy, filed her claim for the proceeds of the insurance, submitting all the necessary papers and other requisites with the private respondent. Her claim having been denied, Regina L. Edillon instituted this action in the Court of First Instance of Rizal. - In resisting the claim of the petitioner, the respondent insurance corporation relies on a provision contained in the Certificate of Insurance, excluding its liability to pay claims under the policy in behalf of "persons who are under the age of sixteen (16) years of age or over the age of sixty (60) years ..." It is pointed out that the insured being over sixty (60) years of age when she applied for the insurance coverage, the policy was null and void, and no risk on the part of the respondent insurance corporation had arisen therefrom. - RTC dismissed the complaint. ISSUE WON the acceptance by the private respondent insurance corporation of the premium and the issuance of the corresponding certificate of insurance should be deemed a waiver of the exclusionary condition of overage stated in the said certificate of insurance HELD YES - The age of the insured Carmen 0. Lapuz was not concealed to the insurance company. Her application for insurance coverage which was on a printed form furnished by private respondent and which contained very few items of information clearly indicated her age of the time of filing the same to be almost 65 years of age. Despite such information which could hardly be overlooked in the application form, considering its prominence thereon and its materiality to the coverage applied for, the respondent insurance corporation received her payment of premium and issued the corresponding certificate of insurance without question. The accident which resulted in the death of the insured, a risk covered by the policy, occurred on May 31, 1969 or FORTY-FIVE (45) DAYS after the insurance coverage was applied for. There was sufficient time for the private respondent to process the application and to notice that the applicant was over 60 years of age and thereby cancel the policy on that ground if it was minded to do so. If the private respondent failed to act, it is either because it was willing to waive such disqualification; or, through the negligence or incompetence of its employees for which it has only itself to blame, it simply overlooked such fact. Under the circumstances, the insurance corporation is already deemed in estoppel. Its inaction to revoke the policy despite a departure from the exclusionary condition contained in the said policy constituted a waiver of such condition. Disposition Judgment appealed from is REVERSED and SET ASIDE and respondent insurance corporation is ordered to pay to the petitioner the proceeds of Insurance HARDING v. COMMERCIAL UNION (supra p.36) TAN v. CA (PHILIPPINE INSURANCE COMPANY) 174 SCRA 403 GUTIERREZ; June 29, 1989 AMERICAN LIFE

Page
-respondent company denied petitioners' claim and rescinded the policy by reason of the alleged misrepresentation and concealment of material facts made by the deceased Tan Lee Siong in his application for insurance. The premiums paid on the policy were thereupon refunded - Petitioners filed on November 27, 1975, a complaint against the former with the Office of the Insurance Commissioner. Commissioner denied petition. CA affirmed Commissioners decision ISSUE WON according to Sec. 48 of the Insurance Code, insurance company is barred from rescinding contract HELD - Section 48. Whenever a right to rescind a contract of insurance is given to the insurer by any provision of this chapter, such right must be exercised previous to the commencement of an action on the contract. After a policy of life insurance made payable on the death of the insured shall have been in force during the lifetime of the insured for a period of two years from the date of its issue or of its last reinstatement, the insurer cannot prove that the policy is void ab initio or is rescindable by reason of the fraudulent concealment or misrepresentation of the insured or his agent. - According to the petitioners, the Insurance Law was amended and the second paragraph of Section 48 added to prevent the insurance company from exercising a right to rescind after the death of the insured - The so-called "incontestability clause" precludes the insurer from raising the defenses of false representations or concealment of material facts insofar as health and previous diseases are concerned if the insurance has been in force for at least two years during the insured's lifetime. The phrase "during the lifetime" found in Section 48 simply means that the policy is no longer considered in force after the insured has died. The key phrase in the second paragraph of Section 48 is "for a period of two years." - The policy was issued on November 6,1973 and the insured died on April 26,1975. The policy was thus in force for a period of only one year and five months. Considering that the insured died before the two-year period had lapsed, respondent company is not, therefore, barred from proving that the policy is void ab initio by reason of the insured's fraudulent concealment or misrepresentation.

NATURE Review on certiorari of the decision of the Court of Appeals affirming the decision of the Insurance Commissioner FACTS - On September 23,1973, Tan Lee Siong, father of herein petitioners, applied for life insurance in the amount of P 80,000.00 with respondent company. Said application was approved and was issued effective November 6, 1973 - On April 26,1975, Tan Lee Siong died of hepatoma (Exhibit B). Petitioners then filed with respondent company their claim for the proceeds of the life insurance policy

INSURANCE 67
- The petitioners contend that there could have been no concealment or misrepresentation by their late father because Tan Lee Siong did not have to buy insurance. He was only pressured by insistent salesmen to do so -The legislative answer to the arguments posed by the petitioners is the "incontestability clause" added by the second paragraph of Section 48. The insurer has two years from the date of issuance of the insurance contract or of its last reinstatement within which to contest the policy, whether or not, the insured still lives within such period. After two years, the defenses of concealment or misrepresentation, no matter how patent or well founded, no longer lie TAN CHAY HENG v. WEST INSURANCE 51 PHIL 80 JOHNS; November 21, 1927 COAST LIFE and in fact and as the plaintiff and his said coconspirators well knew, the said Tan Ceang was not single but was married and had several children; and was not a merchant but a mere employee of Tan Quina from whom he received only a meager salary, and that plaintiff was not a nephew of the said Tan Ceang. 2. Tan Ceang was seriously ill, suffering from pulmonary tuberculosis of about three years' duration, which illness was incurable and was well known to the plaintiff and his said coconspirators. 3. Locsin, in his capacity as medical examiner for the defendant, prepared and falsified the necessary medical certificate, in which it was made to appear, among other things, that Tan Ceang had never used morphine, cocaine or any other drug; that he was then in good health and had never consulted any physician; that he had never spit blood; and that there was no sign of either present or past disease of his lungs; whereas in truth and in fact, plaintiff and coconspirators well knew, Tan Ceang was addicted to morphine, cocaine, and opium and had been convicted and imprisoned therefor, and for about three year prior thereto had been suffering from pulmonary tuberculosis. 4. Plaintiff caused a confidential report to the defendant insurance company to be signed by one V. Sy Yock Kian, who was an employee of Go Chulian, in which it was falsely represented that Tan Ceang was worth about P40,000, had an annual income of from eight to ten thousand pesos net, had the appearance of good health, and never had tuberculosis. 5. After said application for insurance, medical certificate and confidential report had been prepared and falsified, plaintiff and coconspirators caused the same to be forwarded to the defendant at its office in Manila, the medical certificate thru the said Dr. V. S. Locsin as medical examiner, and said application for insurance and confidential report thru the said Francisco Sanchez in his capacity as one of the agents of the defendant insurance company in the Province of Occidental Negros; that the defendant, believing that the representations made in said document were true, and relying thereon, provisionally accepted the said application for insurance on the life of Tan Ceang in the sum of P10,000 and issued a temporary

Page
policy pending the final approval or disapproval of said application by defendant's home-office in San Francisco, California, where in case of approval a permanent policy was to be issued; that such permanent policy was never delivered to the plaintiff because defendant discovered the fraud before its delivery. 6. That the first agreed annual premium on the insurance in question of P936.50 not having been paid within 60 days after medical examination of the applicant as required by the regulations of the defendant insurance company, plaintiff and coconspirators caused Tan Ceang to sign a health certificate for reinstatement; that the said temporary policy was delivered by defendant to the insured on April 10, 1925, in the belief that said statements and representations were true and in reliance thereon. 7. 2 months after the supposed medical examination above referred to, and exactly 1 month after the date of the health certificate for reinstatement above set forth, Tan Ceang died in Valladolid, Occidental Negros, of pulmonary tuberculosis, the same illness from which suffering at the time it is supposed he was examined by Dr. V. S. Locsin, but that the plaintiff coconspirators, pursuant to their conspiracy, caused the said Dr. V. S. Locsin to state falsely in the certificate of death that the said Tan Ceang had died of cerebral hemorrhage. - Defendant also alleges that plaintiff was, like V. Sy Yock Kian, an employee of Go Chulian; that the latter was the ringleader of a gang of malefactors, who, during, and for some years previous to the dates above mentioned, were engaged in the illicit enterprise of procuring fraudulent life insurances from the present defendant, similar to the one in question, and which enterprise was capitalized by him by furnishing the funds with which to pay the premium on said fraudulent insurance; that the said Go Chulian was the one who furnished the money with which to pay the first and only annual premium on the insurance here in question, amounting to P936.50; that the said Go Chulian, on August 28, 1926, was convicted by the Court of First Instance of the City of Manila, in criminal case No. 31425 of that court, of the crime of falsification of private documents in connection with an fraudulent insurance, similar to the present, committed against this defendant in the month of September, 1924; that in the same case the said Francisco Sanchez was one of the coaccused of

FACTS - Plaintiff alleges that defendant accepted and approved a life insurance policy of for the sum of P10,000 in which the plaintiff was the sole beneficiary; that the policy was issued upon the payment by the said Tan Ceang of the first year's premium amounting to P936; that in and by its terms, the defendant agreed to pay the plaintiff as beneficiary the amount of the policy upon the receipt of the proofs of the death of the insured while the policy was in force; that without any premium being due or unpaid, Tan Ceang died on May 10, 1925; that in June, 1925, plaintiff submitted the proofs of the death of Tan Ceang with a claim for the payment of the policy which the defendant refused to pay, for which he prays for a corresponding judgment, with legal interest from the date of the policy, and costs. - Defendant alleges that the insurance policy on the life of Tan Ceang, upon which plaintiff's action is based, was obtained by the plaintiff in confabulation with one Go Chulian, of Bacolod, Negros Occidental; Francisco Sanchez of the same place; and Dr. V. S. Locsin, of La Carlota, Negros Occidental, thru fraud and deceit perpetrated against this defendant in the following manner, to wit: 1. Go, Sanchez and Locsin, caused Tan Caeng to sign an application for insurance with the defendant in the sum of P10,000, in which it was said that Tan Ceang was single and was a merchant, and that the plaintiff Tan Chai Heng, the beneficiary, was his nephew, whereas in truth

INSURANCE 68
the said Go Chulian but was discharged from the complaint, because he offered himself and was utilized as a state's witness; that there is another civil action now pending against Go Chulian and Sanchez in the Court of First Instance of Manila (civil case No. 28680), in which the present defendant is the plaintiff, for the recovery of the amounts of two insurance policies aggregating P19,000, fraudulently obtained by the said Go Chulian and Sanchez. - To this, plaintiff filed a demurrer which was granted. ISSUE WON defense is barred by Art. 47 HELD NO Ratio The word "rescind" has a well defined legal meaning, and as applied to contracts, it presupposes the existence of a contract to rescind. Reasoning - Plaintiff vigorously contends that section 47 of the Insurance Act should be applied, and that when so applied, defendant is barred and estopped to plead and set forth the matters alleged in its special defense. That section is as follows: Whenever a right to rescind a contract of insurance is given to the insurer by any provision of this chapter, such right must be exercised previous to the commencement of an action on the contract. - It will be noted that defendant does not seek to have the alleged insurance contract rescinded. It denies that it ever made any contract of insurance on the life of Tan Ceang or that any such a contract ever existed, and that is the question which it seeks to have litigated by its special defense. In the very nature of things, if the defendant never made or entered into the contract in question, there is no contract to rescind, and, hence, section 47 upon which the lower based its decision in sustaining the demurrer does not apply. As stated, an action to rescind a contract is founded upon and presupposes the existence of the contract which is sought to be rescinded. If all of the material matters set forth and alleged in the defendant's special plea are true, there was no valid contract of insurance, for the simple reason that the minds of the parties never met and never agreed upon the terms and conditions of the contract. We are clearly of the opinion that, if such matters are known to exist by a preponderance of the evidence, they would constitute a valid defense to plaintiff's cause of action. Upon the question as to whether or not they or are not true, we do not at this time have or express any opinion, but we are clear that section 47 does not apply to the allegations made in the answer, and that the trial court erred in sustaining the demurrer. Disposition The judgment of the lower court is reversed and the case is remanded for such other and further proceedings as are not inconsistent with this opinion, with costs against the plaintiff. PIONEER INSURANCE AND CORPORATION v. YAP 61 SCRA 426 FERNANDEZ; December 19, 1974 SURETY

Page
insurance. - August 29, 1962 : parties executed an endorsement on Policy 4219 stating: It is hereby declared and agreed that the coinsurance existing at present under this policy is as follows: P20,000.00 Northwest Ins., and not as originally stated. (emphasis supplied) Except as varied by this endorsement, all other terms and conditions remain unchanged. - September 26, 1962: Yap took out another fire insurance policy for P20,000 covering the same properties, from Federal Insurance Company. This policy was procured without notice to and the written consent of Pioneer, and was therefore not noted as a co-insurance in Policy 4219. - December 19, 1962: Fire burned Yaps store ISSUE WON petitioner should be absolved from liability on Fire insurance Policy No. 4219 on account of any violation by respondent Yap of the co-insurance clause therein HELD YES - The petitioner should be absolved. Reasoning - There was a violation by Yap of the co-insurance clause contained in Policy No. 4219 which resulted in the avoidance of the petitioners liability. - By the plain terms of the policy, other insurance without the consent of petitioner would ipso facto avoid the contract. It required no affirmative act of election on the part of the company to make operative the clause avoiding the contract, wherever the specified conditions should occur. Its obligations ceased, unless, being informed of the fact, it consented to the additional insurance. - The obvious purpose of the aforesaid requirement in the policy is to prevent over-insurance and thus avert the perpetration of fraud. The public, as well as the insurer, is interested in preventing the situation in which a fire would be profitable to the insured. According to Justice Story: "The insured has no right to complain, for he assents to comply with all the stipulation on his side, in order to entitle himself to the benefit of the contract, which, upon reason or principle, he has no right to ask the court to dispense with the performance of his own part of the agreement, and yet to bind the other party to

NATURE Appeal by certiorari from CA decision affirming a CFI decision which declared plaintiff Yap entitled to recover from defendant Pioneer Insurance and Surety Corp, the full amount of the damage inquired in Policy No. 4219 FACTS - Yap owned a store in a 2 storey building, where she sold shopping bags and footwear. Her son-in-law was in charge of the store - April 19, 1962- Yap took out Fire Insurance Policy No. 4216 from Pioneer with a face value of P25,000 covering her stocks, office furniture, fixtures, etc. - among the conditions set forth: The Insured shall give notice to the Company of any insurance or insurances already effected, or which may subsequently be effected, covering any of the property hereby insured, and unless such notice be given and the particulars of such insurance or insurances be stated in, or endorsed on this Policy by or on behalf of the Company before the occurrence of any loss or damage, all benefits under this Policy shall be forfeited. (emphasis supplied) It is understood that, except as may be stated on the face of this policy there is no other insurance on the property hereby covered and no other insurance is allowed except by the consent of the Company endorsed hereon. Any false declaration or breach or this condition will render this policy null and void. - At the time of insurance of Policy 4219(April 19, 1962), an insurance policy for P20,000 issued by the Great American Insurance Company covering the same properties was noted on said policy as co-

INSURANCE 69
obligations, which, but for those stipulation would not have been entered into." Disposition the appealed judgment of the Court of Appeals is reversed and set aside, and the petitioner absolved from all liability under the policy. NEW LIFE ENTERPRISES v. CA 207 SCRA 609 REGALADO; March 31, 1992 FACTS - Julian Sy and Jose Sy Bang are partners engaged in the business of selling construction materials under the business name New Life Enterprises. Julian Sy insured against fire the stocks in trade of New Life Enterprises with Western Guaranty Corporation, Reliance Surety and Insurance Co. Inc., and Equitable Insurance Corporation in the aggregate amount of P1,550,000.00. When the building where New Life Enterprises was located, along with the stocks in trade therein, were gutted by fire, petitioners filed an insurance claim against the three companies. The insurance companies all denied Julian Sys claim on the ground of breach of policy condition, (i.e., the other insurance clause which required New Life Enterprises to inform each of the insurance companies in case the former insures with another company the same property already insured by each of the insurance companies). - Because of the denial of their claims for payment by the 3 insurance companies, petitioners filed separate civil actions against the former before the Regional Trial Court of Lucena City, which cases were consolidated for trial. The trial court ruled in favor of petitioner. However, the Court of Appeals reversed the trial courts decision, found petitioner to have violated Clauses 3 and 27 of the separate insurance policies issued by the 3 companies, and exonerated the insurance companies from liability. ISSUE WON petitioners violated the Clause of the insurance policies Other Insurance by petitioners. It is further admitted by petitioners that Equitable's policy stated "nil" in the space thereon requiring indication of any co-insurance although there were 3 policies subsisting on the same stocks in trade at the time of the loss, namely, that of Western in the amount of P350,000.00 and two 2 policies of Reliance in the total amount of P1,000,000.00. - The coverage by other insurance or co-insurance effected or subsequently arranged by petitioners were neither stated nor endorsed in the policies of the 3 private respondents, warranting forfeiture of all benefits thereunder if we are to follow the express stipulation in Policy Condition No. 3. - The terms of the contract are clear and unambiguous. The insured is specifically required to disclose to the insurer any other insurance and its particulars which he may have effected on the same subject matter. The knowledge of such insurance by the insurer's agents, even assuming the acquisition thereof by the former, is not the "notice" that would stop the insurers from denying the claim. Besides, the so-called theory of imputed knowledge, that is, knowledge of the agent is knowledge of the principal, aside from being of dubious applicability here has likewise been roundly refuted by respondent court whose factual findings we find acceptable. The mere fact that Yap Kam Chuan was an agent for both Reliance and Equitable does not justify the allegation that the two are sister companies. Availment of the services of the same agents and adjusters by different companies is a common practice in the insurance business and such facts do not warrant the speculative conclusion of the trial court. - Considering the terms of the policy which required the insured to declare other insurances, the statement in question must be deemed to be a statement (warranty) binding on both insurer and insured, that there were no other insurance on the property. The annotation then, must be deemed to be a warranty that the property was not insured by any other policy. Violation thereof entitled the insurer to rescind. - The obvious purpose of the aforesaid requirement in the policy is to prevent over-insurance and thus avert the perpetration of fraud. The public, as well as the insurer, is interested in preventing the situation in which a fire would be profitable to the insured. The insured has no right to complain, for he assents to comply with all the stipulations on his side, in order to entitle himself to the benefit of the contract, which, upon reason or principle, he has no right to ask the

Page
court to dispense with the performance of his own part of the agreement, and yet to bind the other party to obligations, which, but for those stipulations, would not nave been entered into. - It is not disputed that the insured failed to reveal before the loss three other insurances. By reason of said unrevealed insurances, the insured had been guilty of a false declaration; a clear misrepresentation and a vital one because where the insured had been asked to reveal but did not, that was deception. Otherwise stated, had the insurer known that there were many co-insurances, it could have hesitated or plainly desisted from entering into such contract. Hence, the insured was guilty of clear fraud. - As the insurance policy against fire expressly required that notice should be given by the insured of other insurance upon the same property, the total absence of such notice nullifies the policy. - Additionally, insofar as the liability of respondent Reliance is concerned, it is not denied that the complaint for recovery was filed in court by petitioners only on January 31, 1984, or after more than one (1) year had elapsed from petitioners' receipt of the insurers' letter of denial on November 29, 1982. - The condition contained in an insurance policy that claims must be presented within one year after rejection is not merely a procedural requirement but an important matter essential to a prompt settlement of claims against insurance companies as it demands that insurance suits be brought by the insured while the evidence as to the origin and cause of destruction have not yet disappeared. QUA CHEE GAN v. LAW UNION (supra p.48)

HELD YES - Petitioners admit that the respective insurance policies issued by private respondents did not state or endorse thereon the other insurance coverage obtained or subsequently effected on the same stocks in trade for the loss of which compensation is claimed

YOUNG v. MIDLAND TEXTILE INSURANCE CO. 30 PHIL 617 JOHNSON; March 31, 1915 FACTS - K.S. Young had a candy and fruit store on the Escolta, Manila, and occupied a building at 321 Calle Claveria, as a residence and bodega. The Midland Textile Insurance Co. in consideration of the payment

INSURANCE 70
of a premium of P60, entered into a contract of insurance with Young by the terms of which the company, upon certain conditions, promised to pay Young the sum of P3,000 in case said residence and bodega and contents should be destroyed by fire. - One of the conditions of the contract is: "Warranty B It is hereby declared and agreed that during the pendency of this policy no hazardous goods be stored or kept for sale, and no hazardous trade or process be carried on, in the building to which this insurance applies, or in any building connected therewith." - Young placed in the residence and bodega three boxes filled with fireworks. Said residence and bodega and the contents thereof were partially destroyed by fire. - The fireworks had been given to Young by the former owner of the Luneta Candy Store. He intended to use them in the celebration of the Chinese New Year. However, the authorities of the city of Manila had prohibited the use of fireworks on said occasion, so Young then placed them in the bodega where they remained from the 4th or 5th of February, 1913 until after the fire of March 18, 1913. - Both parties agree that the fireworks come within the phrase "hazardous goods," mentioned in "Warranty B" of the policy; that the fireworks were found in a part of the building not destroyed by the fire and that they in no way contributed to the fire, or to the loss that resulted. - The lower court rendered a judgment in favor of Young for the sum of P2,708.78, and costs. ISSUE 1. WON the placing of the fireworks in the building insured, they being "hazardous goods," was a violation of the terms of the contract of insurance and especially of "Warranty B." HELD 1. YES. Reasoning It is admitted by both parties that the fireworks are hazardous goods. The defendant alleged that they were "stored." The plaintiff contends that under all the facts and circumstances of the case, they were not stored in said building, and that the placing of them in the building was not a violation of the terms of the contract. - Whether a particular article is "stored" or not must, in some degree, depend upon the intention of the parties. Nearly all of the cases cited by the lower court are cases where the article was being put to some reasonable and actual use, which might easily have been permitted by the terms of the policy, and within the intention of the parties, and excepted from the operation of the warranty, like the present. - (1) Where merchants have had or kept the "hazardous" articles in small quantities, and for actual daily use, for sale, such as gasoline, gunpowder, etc.; (2) Where such articles have been brought on the premises for actual use thereon, and in small quantities, such as oil, paints, etc; and (3) Where such articles or goods were used for lighting purposes, and in small quantities. - In the present case no claim is made that the "hazardous goods" were placed in the bodega for present or daily use. It is admitted that they were placed in the bodega "for future use," or for future consumption, or for safe keeping. It seems clear to us that the "hazardous goods" in question were "stored" in the bodega, as that word is generally defined. That being true, suppose the defendant had made an examination of the premises, even in the absence of a fire, and had found the "hazardous goods" there, would it not have been justified in declaring the policy null and of no effect by reason of a violation of its terms? If it might, then may it not repudiate its liability, even after the fire? If the "warranty" is a term of the contract, will not its violation cause a breach and justify noncompliance or repudiation? - Contracts of insurance are contracts of indemnity, upon the terms and conditions specified therein. Parties have a right to impose such reasonable conditions at the time of the making of the contract as they deem wise and necessary. The rate of premium is measured by the character of the risk assumed. The insurer, for a comparatively small consideration, undertakes to guarantee the insured against loss or damage, upon the terms and conditions agreed upon, and upon no other. When the insurer is called upon to pay, in case of loss, he may justly insist upon a fulfillment of the terms of the contract. If the insured, cannot bring himself within the terms and conditions of the contract, he is not entitled to recover for any loss suffered. The terms of the contract constitute the measure of the insurer's liability. If the contract has been terminated, by a violation of its terms on the part of the insured, there can be no recovery. Compliance with the terms of the contract is a condition precedent to the right of recovery. - Young argues that since the "storing" of the fireworks on the premises did not contribute in any way to the damage occasioned by the fire, he should

Page
be permitted to recover. That argument, however, is beside the question, if the "storing" was a violation of the terms of the contract. The violation of the terms of the contract, by virtue of the provisions of the policy itself, terminated, at the election of either party, the contractual relations. - Young paid a premium based upon the risk at the time the policy was issued. Certainly, the placing of the firecrackers in the building insured increased the risk. Young had not paid a premium based upon the increased risk, neither had the defendant issued a policy upon the theory of a different risk. He was enjoying, if his contention may be allowed, the benefits of an insurance policy upon one risk, whereas, as a matter of fact, it was issued upon an entirely different risk. The defendant had neither been paid nor had issued a policy to cover the increased risk. An increase of risk which is substantial and which is continued for a considerable period of time, is a direct and certain injury to the insurer, and changes the basis upon which the contract of insurance rests. Disposition Decision of the lower court is REVERSED. TAN v. CA (supra p.51) AREOLA v. CA (supra p.26) TAN CHAY v. WEST COAST (supra p.51) FILIPINAS LIFE ASSURANCE v. NAVA 17 SCRA 210 BAUTISTA ANGELO; May 20, 1966 NATURE Petition for review of a decision of the Court of Appeals FACTS - Before the war, Nava entered into a contract of insurance with Insular Life Assurance Co., Ltd. (face value of P5k), and 17 separate contracts of life insurance with Filipinas Life Assurance Co. (total face value of P90k). Each and everyone of the 18 policies issued by defendants to plaintiff contains a loan clause of the following tenor: Policy loans. After three full years' premiums have been paid upon this Policy, if no premium payment is in default, the Company, subject to its then existing rules, will advance on proper assignment and delivery of this Policy and on the sole security thereof a sum equal to, or at the

INSURANCE 71
option of the owner less than, the cash value specified in the Schedule of Policy Values, less any existing indebtedness on or secured by this Policy and any unpaid balance of the premium for the current policy-year; provided interest at six per centum per annum on the whole amount of the loan is paid in advance to the end of the current policyyear. At the end of the current policy-year interest at the same rate for one year in advance will be due and payable, and annually thereafter, and if not so paid will be added to the principal and bear the same rate of interest. Failure to repay any such loan or interest shall not avoid this Policy unless the total indebtedness shall equal or exceed the full amount of the loan value available hereunder. Any indebtedness on this Policy shall first be deducted from any money payable or in any settlement under this Policy. - Nava had so far paid to Insular a total of P2,574; and to Filipinas Life, a total of P32,072.60. - April 28, 1948: Nava applied to the companies for a P5k loan in line with the loan clause, but they refused to grant it because certain regulations issued by the Insurance Commissioner required the insurance companies to withhold the payments on premiums made during the Japanese occupation because the same shall be subject to future adjustments " as soon as debtor-creditor relationship is established" and because of such process of "withholding" plaintiff was not entitled to borrow any amount until such adjustment has been made. - Sept 30, 1948: Nava called the attention of the insurance companies to the SC decision (Haw Pia v. China Banking Corporation) establishing and recognizing the relationship of debtor and creditor with respect to payments in fiat currency made during the Japanese occupation on pre-war obligations. - Companies still refused saying that the SC decision was not applicable to transactions undertaken during Japanese occupation when they relate to life insurance policies. - Feb 4, 1949: Nava was again refused even if the total amount of the cash surrender values of the 18 policies reached the sum of P9,468.29. - Feb 10, 1949: Nava brought case to the CFI Manila praying for the rescission of the abovementioned 18 policies and for the refund to him of all the premiums so far paid by him to defendants in the amount of P31,633.80, plus 6% interest thereon as damages - Nov 28, 1951: companies passed a resolution which was approved by the Insurance Commissioner, giving full credit to all premium payments made by their policyholders in fiat currency during the Japanese occupation on account of pre-war policies for which reason they filed an amended answer offering to pay plaintiff the amount of P9,468.29 which represents the aggregate cash surrender values of all the policies in question as of February 10, 1949, but apparently this offer was refused. - CFI: (1) rescinded the insurance contracts; (2) ordered defendant Filipinas Life Assurance Co. to pay plaintiff the amount of P32,072.60; and (3) ordered defendant Insular Life Assurance Co., Ltd. to pay plaintiff the amount of P2,574.00 - CA affirmed. ISSUES 1. WON CA erred in ruling that as a consequence of the decision in the Haw Pia case petitioners violated the loan clause contained in the insurance policies thereby entitling respondent to their rescission 2. WON CA erred in ruling that by virtue of Article 1295 of the old Civil Code petitioners should refund to defendant all the premiums paid on his insurance policies as a consequence of their rescission 3. WON CA erred in not ruling that, even if respondent is entitled to the rescission of said insurance policies, he can only recover their cash surrender value at the time the complaint was filed HELD 1. NO. - Even assuming the validity of the Insurance Commissioners regulations, the fact however is that such requirement has already lost its legal effect and value when our Supreme Court rendered its decision in the Haw Pia case wherein it was declared, among others, that all payments made in fiat currency during the Japanese occupation in relation with any contractual obligation executed before the war were valid to all intents and purposes, and yet petitioners apparently did not give any importance to such decision for in their opinion it does not have any application to transactions which have any relation to payment of premiums on life insurance policies. - It cannot be denied that a life insurance policy involves a contractual obligation wherein the insured becomes duty bound to pay the premiums agreed upon, lest he runs the risk of having his insurance policy lapse if he fails to pay such premiums. - The fact that if the insured had paid in full the premiums corresponding to the first 3 years of the life

Page
of his policy he cannot be considered delinquent that would cause the lapse of his policy if the same contains an automatic premium payment clause cannot divest such policy of its contractual nature, for the result of such failure would only be for him to pay later the premium plus the corresponding interest depending upon the condition of the policy. But certainly it does not cease to be a contractual liability insofar as the payment of that premium is concerned for whether he likes it or not that premium has to be paid lest he allows the lapse of his policy. Consequently, the payment of premiums on the life insurance policies made by Nava before and during the war up to the time he applied for the loan in question with petitioners should be considered likewise as valid payments upon the theory that such insurance policies are in the nature of a contractual obligation within the meaning of the civil law. In effect, therefore, those payments were made by a debtor to a creditor within the meaning of the requirement of the regulations of the Insurance Commissioner and as such they can offer no excuse to petitioners for refusing to grant the loan as contemplated in the loan clause embodied in the policies in question. - It is clear from the foregoing that the petitioners violated the loan clause embodied in each of the 18 life insurance policies issued to respondent to rescind all said policies under Section 69 of the Insurance Act, which provides: "The violation of a material warranty, or other material provision of a policy, on the part of either party thereto, entitles the other to rescind." - "The general rule is that a breach of the agreement to make the loan does not entitle the insured to rescind the contract," is not controlling in this jurisdiction. Firstly, it was not shown that the insurance laws in the states where said ruling prevails contain a provision identical to Section 69 of our Insurance Law we quoted above, and secondly, the rule cited by Vance is not a rule uniformly followed by all states in the US, for on this matter there is a marked divergence of opinion. 2. NO - Considering that our Insurance Law does not contain an express provision as to what the court should do in cases of rescission of an insurance policy under Section 69, the provision that should apply is that embodied in Article 1225 of the old Civil Code, as postulated in Article 16 of the same Code, which provides that on matters which are not governed by special laws the provisions of said Code shall

INSURANCE 72
supplement its deficiency. And said Article 1295 provides: ART. 1295. Rescission makes necessary the return of the things which were the subject-matter of the contract, with their fruits, and of the price paid, with interest thereon. ...xxx - Said the petitioners: "Recovery of the full amount of the premium after the insurer has sustained for sometime the risk of the insurance and the insured has enjoyed the benefit of protection is obviously unjust and is so recognized by the better authorities." The ruling above quoted merely represents the minority rule in the US, the majority rule being that the insured can recover all premiums paid, in some cases with interest in case of wrongful cancellation, repudiation, termination or rescission of the contract of life insurance. - Contention that because respondent cannot restore to petitioners the "value of the benefit of protection" which he might have received under the 18 life insurance policies in question he is not entitled to rescind them under the provision of Article 1295 of the old Civil Code, is untenable because said article only contemplates a transaction whether material things are involved, and do not refer to intangible ones which cannot be the subject of restoration, for to interpret it otherwise would be to defeat the law itself with the result that rescission can never be had under Section 69 of our Insurance Law. - It cannot be denied that petitioners had in turn already derived material benefits from the use of premiums paid to them by respondent before, during and after the last war from which they must have realized huge profits, and in this light alone petitioners cannot claim prejudice or unfairness if they are ordered to refund the premiums paid by respondents. 3. NO. - Issue is corollary to preceding issue. No need to refute. Disposition Decision appealed from is AFFIRMED. Costs against petitioners CHAPTER VII. RISKS AND COVERAGES VDA. DE BATACLAN v. MEDINA 102 PHIL 181 MONTEMAYOR; October 22, 1957 FACTS - Juan Bataclan rode Bus 30 of Medina Transport, driven by Saylon, shortly after midnight. The bus was running very fast. One of the front tires burst. Bus fell into canal and turned turtle. 4 passengers couldnt get out, including Bataclan. Gasoline began to leak from the overturned bus. 10 men came to help. 1 carried a torch and when he approached the bus, fire started, killing the trapped passengers. - TC opined that proximate cause of Bataclans death was not the overturning of bus but the fire. At the time fire started, Bataclan, though injured, was still alive and damages were awarded, not for his death, but for physical injuries suffered. ISSUE/S WON the proximate cause is the overturning of the bus or the fire HELD - The proximate cause is the overturning of the bus. - Ordinarily, when a bus overturns and pins down passenger, merely causing him injuries. If through some event, unexpected and extraordinary, the bus is set on fire, and passenger is burned to death, one might contend that the proximate cause was the fire and not the overturning of the vehicle. - But here, the proximate cause of Bataclans death is the overturning of the bus, this for the reason that when the vehicle turned not only on its side but completely on its back, leaking of gasoline from the tank was not unnatural or unexpected. - The coming of the men with the torch was in response to the call for help, made only not by the passengers but even the driver and conductor, and because it was very dark, about 2:30 am, rescuers had to carry a light with them. Coming as they did from rural area where lanterns and flashlights were not available, they had to use a torch. What was more natural than that said rescuers should innocently approach the overturned vehicle to extend aid. - The coming of the men with the torch was to be expected, and was a natural sequence of the overturning of the bus, the trapping of some of its passengers and the call for outside help. - The burning of bus can also in part be attributed to negligence of carrier, through its driver and conductor. They, or at least the driver, should have known that in the position in which the overturned bus was, gasoline could and must have leaked from the gasoline tank and soaked the area in and around

Page
the bus. Gasoline can be smelt and detected even from a distance, and yet neither the driver nor the conductor would appear to have cautioned or taken steps to warn rescuers not to bring the lighted torch too near the bus. FINMAN GENERAL ASSURANCE CORPORATION v. CA (SURPOSA) 213 SCRA 493 NOCON; September 2, 1992 NATURE Certiorari FACTS - Oct. 22, 1986: Carlie Surposa was insured with Finman General Assurance Corporation under Finman General Teachers Protection Plan Master Policy No. 2005 and Individual Policy No. 08924 with his parents, spouses Julia and Carlos Surposa, and brothers Christopher, Charles, Chester and Clifton, all surnamed, Surposa, as beneficiaries. - While said insurance policy was in full force and effect, the insured, Carlie Surposa, died on October 18, 1988 as a result of a stab wound inflicted by one of the 3 unidentified men without provocation and warning on the part of the former as he and his cousin, Winston Surposa, were waiting for a ride on their way home after attending the celebration of the "Maskarra Annual Festival." - Thereafter, Julia Surposa and the other beneficiaries of said insurance policy filed a written notice of claim with the FINMAN Corp which denied said claim contending that murder and assault are not within the scope of the coverage of the insurance policy. - Feb. 24, 1989: Surposa filed a complaint with the Insurance Commission which subsequently ordered FINMAN to pay Surposa the proceeds of the policy with interest. - CA affirmed said decision. ISSUE WON CA committed GAD in applying the principle of "expresso unius exclusio alterius" in a personal accident insurance policy (since death resulting from murder and/or assault are impliedly excluded in said insurance policy considering that the cause of death of the insured was not accidental but rather a deliberate and intentional act of the assailant in killing the former as indicated by the location of the lone stab

INSURANCE 73
wound on the insured) [TF they cannot be made to indemnify the Surposa heirs] HELD NO - The record is barren of any circumstance showing how the stab wound was inflicted. While the act may not exempt the unknown perpetrator from criminal liability, the fact remains that the happening was a pure accident on the part of the victim. The insured died from an event that took place without his foresight or expectation, an event that proceeded from an unusual effect of a known cause and, therefore, not expected. Reasoning - De la Cruz vs. Capital Insurance & Surety Co., Inc (1966)~ The terms "accident" and "accidental" as used in insurance contracts have not acquired any technical meaning, and are construed by the courts in their ordinary and common acceptation. Thus, the terms have been taken to mean that which happen by chance or fortuitously, without intention and design, and which is unexpected, unusual, and unforeseen. An accident is an event that takes place without one's foresight or expectation an event that proceeds from an unknown cause, or is an unusual effect of a known cause and, therefore, not expected. Ratio The generally accepted rule is that, death or injury does not result from accident or accidental means within the terms of an accident-policy if it is the natural result of the insured's voluntary act, unaccompanied by anything unforeseen except the death or injury. There is no accident when a deliberate act is performed unless some additional, unexpected, independent, and unforeseen happening occurs which produces or brings about the result of injury or death. In other words, where the death or injury is not the natural or probable result of the insured's voluntary act, or if something unforeseen occurs in the doing of the act which produces the injury, the resulting death is within the protection of the policies insuring against death or injury from accident. - The personal accident insurance policy involved herein specifically enumerated only 10 circumstances wherein no liability attaches to FINMAN for any injury, disability or loss suffered by the insured as a result of any of the stimulated causes. -The principle of " expresso unius exclusio alterius" the mention of one thing implies the exclusion of another thing is therefore applicable in the instant case since murder and assault, not having been expressly included in the enumeration of the circumstances that would negate liability in said insurance policy: the failure of the FINMAN to include death resulting from murder or assault among the prohibited risks leads inevitably to the conclusion that it did not intend to limit or exempt itself from liability for such death. - A1377 NCC: The interpretation of obscure words or stipulations in a contract shall not favor the party who caused the obscurity. - NPC vs. CA [1986]~ It is well settled that contracts of insurance are to be construed liberally in favor of the insured and strictly against the insurer. Thus ambiguity in the words of an insurance contract should be interpreted in favor of its beneficiary. Disposition DENIED for lack of merit. CALANOC v. CA 98 PHIL 79 BAUTISTA; December 16, 1955 FACTS - Basilio was a watchman of the Manila Auto Supply located at the corner of Avenida Rizal and Zurbaran. He secured a life insurance policy from the Philippine American Life Insurance Company in the amount of P2,000 to which was attached a supplementary contract covering death by accident. On January 25, 1951, he died of a gunshot wound on the occasion of a robbery committed in the house of Atty. Ojeda at the corner of Oroquieta and Zurbaran streets. Calanoc, the widow, was paid the sum of P2,000, face value of the policy, but when she demanded the payment of the additional sum of P2,000 representing the value of the supplemental policy, the company refused alleging, as main defense, that the deceased died because he was murdered by a person who took part in the commission of the robbery and while making an arrest as an officer of the law which contingencies were expressly excluded in the contract and have the effect of exempting the company from liability. - It is contended in behalf of the company that Basilio was killed which "making an arrest as an officer of the law" or as a result of an "assault or murder" committed in the place and therefore his death was caused by one of the risks excluded by the supplementary contract which exempts the company from liability. This contention was upheld by the Court of Appeals. Hence, this petition.

Page

ISSUE WON the death of the victim comes within the purview of the exception clause of the supplementary policy and, hence, exempts the company from liability HELD NO - Basilio was a watchman of the Manila Auto Supply which was a block away from the house of Atty. Ojeda where something suspicious was happening which caused the latter to ask for help. While at first he declined the invitation of Atty. Ojeda to go with him to his residence to inquire into what was going on because he was not a regular policeman, he later agreed to come along when prompted by the traffic policeman, and upon approaching the gate of the residence he was shot and died. The circumstance that he was a mere watchman and had no duty to heed the call of Atty. Ojeda should not be taken as a capricious desire on his part to expose his life to danger considering the fact that the place he was in duty-bound to guard was only a block away. In volunteering to extend help under the situation, he might have thought, rightly or wrongly, that to know the truth was in the interest of his employer it being a matter that affects the security of the neighborhood. No doubt there was some risk coming to him in pursuing that errand, but that risk always existed it being inherent in the position he was holding. He cannot therefore be blamed solely for doing what he believed was in keeping with his duty as a watchman and as a citizen. And he cannot be considered as making an arrest as an officer of the law, as contended, simply because he went with the traffic policeman, for certainly he did not go there for that purpose nor was he asked to do so by the policeman. - Much less can it be pretended that Basilio died in the course of an assault or murder considering the very nature of these crimes. In the first place, there is no proof that the death of Basilio is the result of either crime for the record is barren of any circumstance showing how the fatal shot was fired. Perhaps this may be clarified in the criminal case now pending in court as regards the incident but before that is done anything that might be said on the point would be a mere conjecture. Nor can it be said that the killing was intentional for there is the possibility that the malefactor had fired the shot merely to scare away the people around for his own protection and not necessarily to kill or hit the victim. In any event, while

INSURANCE 74
the act may not exempt the triggerman from liability for the damage done, the fact remains that the happening was a pure accident on the part of the victim. The victim could have been either the policeman or Atty. Ojeda for it cannot be pretended that the malefactor aimed at the deceased precisely because he wanted to take his life. Disposition Decision set aside BIAGTAN v. THE INSULAR LIFE ASSURANCE COMPANY, LTD. 44 SCRA 58 MAKALINTAL; March 29, 1972 NATURE Appeal from decision of CFI Pangasinan. FACTS - Juan Biagtan was insured with Insular for P5k and a supplementary contract Accidental Death Benefit clause for another P5k if "the death of the Insured resulted directly from bodily injury effected solely through external and violent means sustained in an accident . . . and independently of all other causes." The clause, however, expressly provided that it would not apply where death resulted from an injury "intentionally inflicted by a third party." - One night, a band of robbers entered their house. Juan went out of his room and he was met with 9 knife stabs. He died. The robbers were convicted of robbery with homicide. - The family was claiming the additional P5k from Insular, under the Accidental Death Benefit clause. Insular refused on the ground that the death resulted from injuries intentionally inflicted by 3rd parties and was therefore not covered. Biagtans filed against Insular. CFI ruled in favor of Biagtans. ISSUE WON the injuries were intentionally inflicted HELD YES - Whether the robbers had the intent to kill or merely to scare the victim or to ward off any defense he might offer, it cannot be denied that the act itself of inflicting the injuries was intentional. The exception in the accidental benefit clause invoked by the appellant does not speak of the purpose whether homicidal or not of a third party in causing the injuries, but only of the fact that such injuries have been "intentionally" inflicted this obviously to distinguish them from injuries which, although received at the hands of a third party, are purely accidental. - Examples of unintentional: >> A gun which discharges while being cleaned and kills a bystander; >> a hunter who shoots at his prey and hits a person instead; >> an athlete in a competitive game involving physical effort who collides with an opponent and fatally injures him as a result. - In Calanoc vs. CA: Where a shot was fired and it turned out afterwards that the watchman was hit in the abdomen, the wound causing his death, the Court held that it could not be said that the killing was intentional for there was the possibility that the malefactor had fired the shot to scare the people around for his own protection and not necessarily to kill or hit the victim. A similar possibility is clearly ruled out by the facts in this case. For while a single shot fired from a distance, and by a person who was not even seen aiming at the victim, could indeed have been fired without intent to kill or injure, nine wounds inflicted with bladed weapons at close range cannot conceivably be considered as innocent insofar as such intent is concerned. - In Hucthcraft's Ex'r vs. Travelers' Ins. Co. (US case): where the insured was waylaid and assassinated for the purpose of robbery, the court rendered judgment for the insurance company and held that while the assassination of the insured was as to him an unforeseen event and therefore accidental, "the clause of the proviso that excludes the (insurer's) liability, in case death or injury is intentionally inflicted by any other person, applies to this case." Disposition CFI decision reversed. SEPARATE OPINION TEEHANKEE [dissent] - Calanoc v. CA is controlling in this case because the insurance company wasnt able to prove that the killing was intentional. (Burden of proof is with the insurance company) - Insurance, being contracts of adhesion, must be construed strictly against insurance company in cases of ambiguity. - The supplementary contract enumerated exceptions. The only exception which is not susceptible of

Page
classification is that provided in paragraph 5(e), the very exception herein involved, which would also except injuries "inflicted intentionally by a third party, either with or without provocation on the part of the insured, and whether or not the attack or the defense by the third party was caused by a violation of the law by the insured." - This ambiguous clause conflicts with all the other four exceptions in the same paragraph 5 particularly that immediately preceding it in item (d) which excepts injuries received where the insured has violated the law or provoked the injury, while this clause, construed as the insurance company now claims, would seemingly except also all other injuries, intentionally inflicted by a third party, regardless of any violation of law or provocation by the insured, and defeat the very purpose of the policy of giving the insured double indemnity in case of accidental death by "external and violent means" in the very language of the policy.' - It is obvious from the very classification of the exceptions and applying the rule of noscitus a sociis, that the double-indemnity policy covers the insured against accidental death, whether caused by fault, negligence or intent of a third party which is unforeseen and unexpected by the insured. All the associated words and concepts in the policy plainly exclude the accidental death from the coverage of the policy only where the injuries are self-inflicted or attended by some proscribed act of the insured or are incurred in some expressly excluded calamity such as riot, war or atomic explosion. - The untenability of insurer's claim that the insured's death fell within the exception is further heightened by the stipulated fact that two other insurance companies which likewise covered the insured for much larger sums under similar accidental death benefit clauses promptly paid the benefits thereof to plaintiffs beneficiaries. SUN INSURANCE v. CA (LIM) 211 SCRA 554 CRUZ; July 17, 1992 FACTS - The petitioner issued Personal Accident Policy to Felix Lim, Jr. with a face value of P200,000.00. Two months later, he was dead with a bullet wound in his head. As beneficiary, his wife Nerissa Lim sought payment on the policy but her claim was rejected. The

INSURANCE 75
petitioner agreed that there was no suicide. It argued, however, that there was no accident either. Pilar Nalagon, Lim's secretary, was the only eyewitness to his death. According to Nalagon, Lim was in a happy mood (but not drunk) and was playing with his handgun, from which he had previously removed the magazine. As she watched the television, he stood in front of her and pointed the gun at her. She pushed it aside and said it might be loaded. He assured her it was not and then pointed it to his temple. The next moment there was an explosion and Lim slumped to the floor. He was dead before he fell. - The term "accident" has been defined as follows: The words "accident" and "accidental" have never acquired any technical signification in law, and when used in an insurance contract are to be construed and considered according to the ordinary understanding and common usage and speech of people generally. In substance, the courts are practically agreed that the words "accident" and "accidental" mean that which happens by change or fortuitously, without intention or design, and which is unexpected, unusual, and unforeseen. The definition that has usually been adopted by the courts is that an accident is an event that takes place without one's foresight or expectation an event that proceeds from an unknown cause, or is an unusual effect of a known case, and therefore not expected. - An accident is an event which happens without any human agency or, if happening through human agency, an event which, under the circumstances, is unusual to and not expected by the person to whom it happens. It has also been defined as an injury which happens by reason of some violence or casualty to the insured without his design, consent, or voluntary cooperation. ISSUE WON what happened was an accident HELD YES - The petitioner, however, cites one of the four exceptions provided for in the insurance contract and contends that the private petitioner's claim is barred by such provision. It is there stated: Exceptions The company shall not be liable in respect of. 1. Bodily injury. xxx xxx xxx b. consequent upon. i) The insured persons attempting to commit suicide or wilfully exposing himself to needless peril except in an attempt to save human life. - To repeat, the parties agree that Lim did not commit suicide. Nevertheless, the petitioner contends that the insured willfully exposed himself to needless peril and thus removed himself from the coverage of the insurance policy. That posture is arguable. But what is not is that, as the secretary testified, Lim had removed the magazine from the gun and believed it was no longer dangerous. He expressed assured her that the gun was not loaded. It is submitted that Lim did not willfully expose himself to needless peril when he pointed the gun to his temple because the fact is that he thought it was not unsafe to do so. The act was precisely intended to assure Nalagon that the gun was indeed harmless. Disposition CA Affirmed DE LA CRUZ v. CAPITAL INSURANCE 17 SCRA 554 BARRERA; June 30, 1966 NATURE Appeal from the decision of the CFI of Pangasinan FACTS - Eduardo de la Cruz, employed in the Itogon-Suyoc Mines, Inc., was the holder of an accident insurance policy underwritten by the Capital Insurance & Surety Co., Inc., for the period beginning November 13, 1956 to November 12, 1957. - On January 1, 1957, the Itogon-Suyoc Mines, Inc. sponsored a boxing contest wherein the insured Eduardo de la Cruz participated. - In the course of his bout, Eduardo slipped and was hit by his opponent on the left part of the back of the head, causing Eduardo to fall, with his head hitting the rope of the ring. - He was brought to the Baguio General Hospital, but he died as a result of hemorrhage, intracranial, left. - Simon de la Cruz, the father and named beneficiary of the insured, filed a claim with the insurance company for payment of the indemnity, but it was denied. - He instituted the action in the CFI of Pangasinan for specific performance. - Defendant insurer set up the defense that the death of the insured, caused by his participation in a boxing

Page
contest, was not accidental and, therefore, not covered by insurance - The court rendered the decision in favor of the plaintiff, hence, the present appeal. ISSUE WON the death of the insured was not accidental and, therefore, not covered by insurance HELD NO - The terms "accident" and "accidental", as used in insurance contracts, have not acquired any technical meaning, and are construed by the courts in their ordinary and common acceptation. Thus, the terms have been taken to mean that which happen by chance or fortuitously, without intention and design, and which is unexpected, unusual, and unforeseen. An accident is an event that takes place without one's foresight or expectation, an event that proceeds from an unknown cause, or is an unusual effect of a known cause and, therefore, not expected. - The generally accepted rule is that, death or injury does not result from accident or accidental means within the terms of an accident-policy if it is the natural result of the insured's voluntary act, unaccompanied by anything unforeseen except the death or injury. There is no accident when a deliberate act is performed unless some additional, unexpected, independent, and unforeseen happening occurs which produces or brings about the result of injury or death. In other words, where the death or injury is not the natural or probable result of the insured's voluntary act, or if something unforeseen occurs in the doing of the act which produces the injury, the resulting death is within the protection of policies insuring against death or injury from accident. - In the present case, while the participation of the insured in the boxing contest is voluntary, the injury was sustained when he slid, giving occasion to the infliction by his opponent of the blow that threw him to the ropes of the ring. - The fact that boxing is attended with some risks of external injuries does not make any injuries received in the course of the game not accidental. In boxing as in other equally physically rigorous sports, such as basketball or baseball, death is not ordinarily anticipated to result. If, therefore, it ever does, the injury or death can only be accidental or produced by some unforeseen happening or event as what occurred in this case.

INSURANCE 76
- Furthermore, the policy involved herein specifically excluded from its coverage: (e) Death or disablement consequent upon the Insured engaging in football, hunting, pigsticking, steeplechasing, polo-playing, racing of any kind, mountaineering, or motorcycling. - Death or disablement resulting from engagement in boxing contests was not declared outside of the protection of the insurance contract. Failure of the defendant insurance company to include death resulting from a boxing match or other sports among the prohibitive risks leads inevitably to the conclusion that it did not intend to limit or exempt itself from liability for such death. Disposition The decision appealed from is affirmed FORTUNE INSURANCE v. CA (supra p.7) PHIL HOME ASSURANCE CORP v. CA (EASTERN SHIPPING) 257 SCRA 468 KAPUNAN; June 20, 1996 NATURE - Eastern Shipping Lines, Inc. loaded on board SS Eastern Explorer in Kobe, Japan, the following shipment for carriage to Manila and Cebu, freight prepaid and in good order and condition: (a) 2 boxes internal combustion engine parts, consigned to William Lines, Inc.; (b) 10 metric tons (334 bags) ammonium chloride, consigned to Orca's Company; (c) 200 bags Glue 300, consigned to Pan Oriental Match Company; and (d) garments, consigned to Ding Velayo. All consignations were made by virtue of a Bill of Lading. - While the vessel was off Okinawa, a small flame was detected on the acetylene cylinder located in the accommodation area near the engine room. As the crew was trying to extinguish the fire, the cylinder suddenly exploded, thus causing death and severe injuries to the crew and instantly setting fire to the whole vessel. - SS Eastern Explorer was then found to be a constructive total loss and its voyage was declared abandoned. - After the fire was extinguished, the cargoes which were saved were loaded to another vessel for delivery to their original ports of destination. ESLI charged the consignees several amounts corresponding to additional freight and salvage charges. - The charges were all paid by Philippine Home Assurance Corporation (PHAC) under protest for and in behalf of the consignees. PHAC, as subrogee of the consignees, then filed a complaint before the RTC of Manila, against ESLI to recover the sum paid under protest on the ground that the same were actually damages directly brought about by the fault, negligence, illegal act and/or breach of contract of ESLI. - ESLI contended that it exercised the diligence required by law in the handling, custody and carriage of the shipment; that the fire was caused by an unforeseen event; that the additional freight charges are due and demandable pursuant to the Bill of Lading; and that salvage charges are properly collectible under Act No. 2616, known as the Salvage Law. - RTC: dismissed PHAC's complaint and ruled in favor of ESLI. - The burning of the vessel was not the fault or negligence of defendant but a natural disaster or calamity. Salvage operations conducted by Fukuda Salvage Company was perfectly a legal operation and charges made on the goods recovered were legitimate charges. Section 19 of Act No. 2616, the Salvage Law is applicable. With respect to the additional freight charged by defendant from the consignees of the goods, the same are also validly demandable. - The burning of "EASTERN EXPLORER" while off Okinawa rendered it physically impossible for defendant to comply with its obligation of delivering the goods to their port of destination pursuant to the contract of carriage. Under Article 1266 of the Civil Code, the physical impossibility of the prestation extinguished defendant's obligation. - Note: The goods subject of the present controversy were neither lost nor damaged in transit by the fire that razed the carrier. In fact, these were all delivered to the consignees, even if the transshipment took longer than necessary. What is at issue therefore is not whether or not the carrier is liable for the loss, damage, or deterioration of the goods transported by them but who, among the carrier, consignee or insurer of the goods, is liable for the additional charges or expenses incurred by the owner of the ship in the salvage operations and in the
9
Section 1. When in case of shipwreck, the vessel or its cargo shall be beyond the control of the crew, or shall have been abandoned by them, and picked up and conveyed to a safe place by other persons, the latter shall be entitled to a reward for the salvage. Those who, not being included in the above paragraph, assist in saving a vessel or its cargo from shipwreck, shall be entitled to like reward.

Page
transshipment of the goods via a different carrier. In absolving respondent carrier of any liability, CA sustained the trial court's finding that the fire that gutted the ship was a natural disaster or calamity. ISSUE WON the burning of the SS Eastern Explorer rendering it a constructive total loss was a natural disaster or calamity HELD NO Ratio In our jurisprudence, fire may not be considered a natural disaster or calamity since it almost always arises from some act of man or by human means. It cannot be an act of God unless caused by lightning or a natural disaster or casualty not attributable to human agency. Reasoning - There was no showing, and none was alleged by the parties, that the fire was caused by a natural disaster. On the contrary, there is strong evidence indicating that the acetylene cylinder caught fire because of the fault and negligence of respondent ESLI, its captain and its crew: (1) The acetylene cylinder which was fully loaded should not have been stored near the engine room where the heat generated therefrom could cause the acetylene cylinder to explode by reason of spontaneous combustion. ESLI should have easily foreseen that the acetylene cylinder, containing highly inflammable material, was in a real danger of exploding. (2) ESLI should have known that by storing the acetylene cylinder in the accommodation area supposed to be reserved for passengers, it unnecessarily exposed its passengers to grave danger and injury. (3) The fact that the acetylene cylinder was checked, tested and examined and subsequently certified as having complied with the safety measures and standards by qualified experts before it was loaded in the vessel only shows to a great extent that negligence was present in the handling of the acetylene cylinder after it was loaded and while it was on board the ship. - From the foregoing premises, it indubitably follows that the cargo consignees cannot be made liable to respondent carrier for additional freight and salvage charges. Disposition Judgment appealed from is REVERSED

INSURANCE 77
and SET ASIDE. Respondent Eastern Shipping Lines, Inc. is ORDERED to return to petitioner Philippine Home Assurance Corporation the amount it paid under protest in behalf of the consignees herein. ISSUES 1. WON the premia remitted were in pursuance of the reinsurance treaty between Philamlife and Airco of January 1959, a contract antedating the Margin Law, and therefore, Philamlife exempted from paying margin fee 2. WON Margin Law impairs the obligation of contract 3. WON reinsurance contracts abroad would be made impractical by the imposition of the 25% margin fee HELD 1. NO - For an exemption to come into play, there must be a reinsurance policy or, as in the reinsurance treaty provided, a "reinsurance cession" which may be automatic or facultative. Ratio A reinsurance policy is thus a contract of indemnity one insurer makes with another to protect the first insurer from a risk it has already assumed. . . . In contradistinction, a reinsurance treaty is merely an agreement between two insurance companies whereby one agrees to cede and the other to accept reinsurance business pursuant to provisions specified in the treaty. The practice of issuing policies by insurance companies includes, among other things, the issuance of reinsurance policies on standard risks and also on substandard risks under special arrangements. The lumping of the different agreements under a contract has resulted in the term known to the insurance world as 'treaties.' Such a treaty is, in fact, an agreement between insurance companies to cover the different situations described. Reinsurance treaties and reinsurance policies are not synonymous. Treaties are contracts for insurance; reinsurance policies or cessions are contracts of insurance. Reasoning - Even if reinsurance treaty preceded the Margin Law by over nine years, nothing in the treaty obligates Philamlife to remit to Airco a fixed, certain, and obligatory sum by way of reinsurance premiums. The reinsurance treaty per se cannot give rise to a contractual obligation for the payment of foreign exchange. Philamlifes obligation to remit reinsurance premiums becomes fixed and definite upon the execution of the reinsurance cession. It is only after a reinsurance cession is made that payment of reinsurance premium may be exacted, as it is only after Philamlife seeks to remit that reinsurance premium that the obligation to pay the margin fee arises.

Page
2. NO Ratio. Existing laws form part of the contract "as the measure of the obligation to perform them by the one party and the right acquired by the other. If the obligation does not inhere and subsist in the contract itself, propio vigore, but in the law applicable to the contract.

PHILIPPINE AMERICAN LIFE INSURANCE COMPANY v. THE AUDITOR GENERAL 22 SCRA 135 SANCHEZ, JANUARY 18, 1968 NATURE PETITION FOR REVIEW of a ruling of the Auditor General. FACTS - Philamlife, a domestic life insurance corp., and American International Reinsurance Company (Airco), a corporation organized under the laws of the Republic of Panama, entered into a REINSURANCE TREATY wherein Philamlife agrees to reinsure with Airco on January 1950. Philamlife agreed to pay premiums for all reinsurances on an annual premium basis. - In July 16, 1959, the Margin Law was approved and became effective, which exempts certain obligations from payment of margin fees, particularly contractual obligations calling for payment of foreign exchange issued, approved and outstanding as of the date this Act takes place. - Central Bank of the Philippines collected P268,747.48 as foreign exchange margin on Philamlife remittances to Airco purportedly totalling $610,998.63 and made subsequent to July 16, 1959. Philamlife filed a claim for refund on the ground that the reinsurance premiums remitted were paid in pursuant to the January 1950 reinsurance treaty, and therefore exempted. - Monetary Board exempted Philamlife from payment of margin fee. However, Auditor of CB refused to pass in audit Philamlifes claim for refund. Philamlife sought reconsideration but was denied, saying reinsurance treaty NOT EXEMPTED.

Reasoning - . When petitioner entered into the reinsurance treaty of January 1, 1950 with Airco, it did so with the understanding that the municipal laws of the Philippines at the time said treaty was executed, became an unwritten condition thereof. Such municipal laws constitute part of the obligation of contract. -Rationale of Margin Law: to reduce the excessive demand on and prevent further decline of our international reserves; to provide the Central Bank with an additional instrument for effectively coping with the problem and achieving domestic and international stability of our currency; to reduce the excessive demand-for foreign exchange. - implementation of Margin Law in accordance with police power 3. NO Reasoning - First, there is no concrete evidence that such imposition of the 25% margin fee is unreasonable, Second, if really continuance of the existing reinsurance treaty becomes unbearable, that contract itself provides that petitioner may potestatively write finis thereto on ninety days' written notice. Petitioner is not forced to continue its reinsurance treaty indefinitely with Airco. Disposition For the reasons given, the petition for review is hereby denied, and the ruling of the Auditor General of October 24, 1961 denying refund is hereby affirmed. Costs against petitioner. So ordered. FIELDMEN'S INSURANCE CO SURETY & INSURANCE CO INC 34 SCRA 36 MAKALINTAL; July 31, 1970 INC v. ASIAN

FACTS - On various dates between April 11, 1960 and Jan. 9, 1961 the Asian Surety & Insurance Company, Inc.

INSURANCE 78
and the Fieldmen's Insurance Company, Inc. entered into 7 reinsurance agreements under which the former, as the ceding company undertook to cede to the latter, as the reinsuring company, a specified portion of the amount of insurance underwritten by ASIAN upon payment to FIELDMEN'S of a proportionate share of the gross rate of the premium applicable with respect to each cession after deducting a commission. Said agreements were to take effect from certain specific dates and were to be in force until cancelled by either party upon previous notice of at least 3 months by registered mail to the other party, the cancellation to take effect as of Dec. 31 of the year in which the notice was given. - On Sep. 19, 1961 FIELDMEN'S, by means of registered mail, served notice to ASIAN of the former's desire to be relieved from all participation in its various agreements with the latter effective Dec. 31, 1961. This communication, although admittedly received by ASIAN on Sep. 25, 1961, did not elicit any reply from ASIAN. - On Dec. 7, 1961 FIELDMEN'S sent another letter to ASIAN expressing regrets at alleged violations committed by the latter with respect to the various agreements between them and reiterated its position that it would consider itself "no longer at risk for any reinsurance and/or cession" given by ASIAN which might be in force on Dec. 31, 1961. Not having received any formal reply from ASIAN, FIELDMEN'S sent a new a letter on Feb. 17, 1962 reminding ASIAN of the cancellation of all the reinsurance treaties and cessions as of Dec. 31, 1961 and requested ASIAN to submit its final accounting of all cessions made to the former for the preceding months when the reinsurance agreements were in force. - Meanwhile one of the risks reinsured with FIELDMEN'S issued in favor of the GSIS became a liability when the insured property was burned on February 16, 1962. Since the policy was issued on July 1, 1961, it was supposed to expire on July 1, 1962. 2 The next day, Feb. 17, ASIAN immediately notified FIELDMEN'S of said fire loss. - FIELDMEN'S, relying on the sufficiency of its notice of termination dated September 19, 1961 and obviously bent on avoiding its liability under the reinsurance agreements with ASIAN, filed a petition for declaratory relief with the CFI of Manila to seek a declaration that all the reinsurance contracts entered into between them had terminated as of December 31, 1961 and to obtain an order directing ASIAN to render final accounting of the transactions between them with respect to said reinsurance treaties as of the cut-off date. - In its answer below ASIAN denied having received FIELDMEN'S letter dated Sep 19, 1961, and argued that even assuming it did, FIELDMEN'S could not have terminated the reinsurance treaties as of Dec 31, 1961 because the letter was merely an expression of FIELDMEN'S desire to cancel the treaties and not a formal notice of cancellation as contemplated in their reinsurance agreements. By way of special defense Asian contended that even if the Sep. 19 letter were considered sufficient notice of cancellation thereby rendering the reinsurance agreements terminated as of December 31, 1961 the liability of FIELDMEN'S with respect to policies or cessions issued under two of the said agreements prior to their cancellation continued to have full force and effect until the stated expiry dates of such policies or cessions. - On Dec. 4, 1962, the trial court declared 6 of the 7 reinsurance agreements in question cancelled as of Dec 31, 1961. At the same time, it upheld ASIAN'S position that all cessions of reinsurance made by it to FIELDMEN'S prior to the cancellation of the reinsurance treaties continued in full force and effect until expiry dates and ordered FIELDMEN'S to make an accounting of its business transactions with ASIAN within 30 days. - On appeal to the CA, the decision of the trial court was substantially affirmed, with the slight modification that the order for accounting was eliminated, without prejudice to the filing of a proper action between the parties for that purpose. ISSUE WON the cancellation as of Dec. 31, 1961 of the reinsurance treaties had the effect of terminating also the liability of FIELDMEN'S as reinsurer with respect to policies or cessions issued prior to the termination of the principal reinsurance contracts or treaties HELD NO to the 2 reinsurance contracts - Of the 6 reinsurance contracts, 2 contain provisions, which clearly and expressly recognize the continuing effectivity of policies ceded under them for reinsurance notwithstanding the cancellation of the contracts themselves. The said treaties provide "that in the event of termination of this Agreement . . ., the liability of the Fieldmen's under current cessions shall continue in full force and effect until their natural

Page
expiry . . .;" and the 4th paragraph of Article VI of the Personal Accident Reinsurance Treaty states: "4. On the termination of this Agreement from any cause whatever, the liability of the REINSURER (Fieldmen's) under any current cession including any amounts due to be ceded under the terms of this Agreement and which are not cancelled in the ordinary course of business shall continue in full force until their expiry unless the COMPANY (Asian) shall, prior to the thirty-first December next following such notice, elect to withdraw the existing cessions . . ." - Thus, insofar as the two reinsurance agreements are concerned, there is clearly no merit in FIELDMEN'S claim that their cancellation carried with it ipso facto the termination of all reinsurance cessions thereunder. Such cessions continued to be in force until their respective dates of expiration. Since it was under one of said agreements that the reinsurance cession corresponding to the GSIS policy had been made, FIELDMEN'S cannot avoid liability which arose by reason of the burning of the insured property. - With respect to the other 4 agreements, it would seem that the petition for declaratory relief is moot, and that no useful purpose would be served by defining the respective rights and obligations of the parties thereunder. The said agreements have been cancelled, and it does not appear that any claim by or liability in favor of the insured has actually arisen under any of the reinsurance cessions made prior to such cancellation. Future conflicts of the same nature as those which have motivated the present action can of course be obviated by using more precise and definite terminology in the reinsurance agreements which the parties may enter into henceforth. EQUITABLE INSURANCE v. RURAL INSURANCE 4 SCRA 343 BARRERA; January 31, 1962 FACTS - Plaintiff Equitable Insurance file a complaint with the CFI of Manila against defendant Rural Insurance alleging, as first cause of action, that they entered into a reciprocal facultative reinsurance agreement, wherein they agreed to cede to each other. Pursuant to said agreement, plaintiff reinsured for P2k with defendant the stock covered by fire insurance Policy No. 5880 issued by plaintiff which was later burned; the share of the loss of defendant as per insurance agreement was computed at P2,024 for which plaintiff

INSURANCE 79
sent to defendant a statement of account for payment by the latter. Despite repeated demands by plaintiff, defendant refused to pay. - On the second cause of action, plaintiff reinsured for P2k with defendant stock covered by fire insurance Policy No. 6062 which also burned. Again, defendant refused to pay its share of the loss of P1,334 hence said complaint. - Defendant filed a motion to dismiss on the ground that it states no cause of action, as pursuant to Art VIII of the Reinsurance Agreement between the parties, before a court action can be brought, the parties agreed to submit all disputes to a board of arbitrators. The Court denied the motion and required defendant to answer. - Defendant filed its answer, alleging that the nature of the agreement is self-liquidating between the parties, the reinsurer becoming the reinsured and vice versa; and that said agreement has not yet been abrogated so the liability of either to the other is not yet known. Defendant prayed that the complaint be dismissed and plaintiff filed a motion for judgment on the pleadings which the court denied. - Instead of going into a formal hearing, the parties submitted their case for decision stipulating the ff facts: defendant admits the allegations of the complaint and that plaintiff admits that the issues of the complaint were not submitted to a Board of Arbitrators as provided in par VIII of the complaint, but instead referred it to the Insurance Commissioner. The CFI rendered judgment in favor of plaintiff. Hence this appeal. ISSUES 1. WON Equitable had no cause of action as the matter was not referred to the decision of arbitrators 2. WON in a facultative obligation the right to choose an alternative remedy lies only with the debtor (here the defendant) under Art 1206 HELD 1. NO - The requirement of submission for decision to 2 arbitrators or an umpire the matter of losses by fire or the liability of the parties thereto under Art VIII of the agreement arises only if the same is disputed by one of the parties. In the instant case, there is no dispute between the parties; in the stipulation of facts defendant admitted that plaintiff had paid its liability and defendant likewise admitted that it ignored plaintiffs demands for reimbursement for defendants failure to pay its share as reinsurer. As held in Maligad v United Assurance Co., if in the course of the settlement of a loss, the action of the company or its agents amounts to refusal to pay, the company will be deemed to have waived the condition precedent with reference to arbitration and a suit upon the policy will lie. 2. NO - There is no connection between Art 1206 NCC and the agreement of this action. The term facultative is used in reinsurance contracts, and it is so used in this particular case, merely to define the right of the reinsurer to accept or not to accept participation in the risk insured. But once the share is accepted, as it was in the case at bar, the obligation is absolute and the liability assumed thereunder can be discharged by only one waythe payment of the share of the losses. Disposition judgment appealed from the TC is affirmed ARTEX DEVELOPMENT CO INC v. WELLINGTON INSURANCE CO INC 51 SCRA 352 TEEHANKEE; June 27, 1973 FACTS Wellington Insurance Co. Inc. insured for P24,346,509.00 the buildings, stocks and machinery of plaintiff Artex Development Co. Inc. against loss or damage by fire or lighting upon payment of the plaintiff of the corresponding premiums; that said properties were insured for an additional sum of P883,034.00; that defendant insured plaintiff against business interruption (use and occupancy) for P5,200,000.00; Wellington entered into a contract of reinsurance with Alexander and Alexander, Inc. of New York. USA. - The buildings, stocks and machineries of plaintiffs spinning department were burned. - Notice of the loss and damage was given the defendant; that as per report of the adjusters, the total property loss of the plaintiff was the sum of P10,106,554.40 and the total business interruption loss was P3,000,000.00; - That defendant has paid to the plaintiff the sum of P6,481,870.07 of the property loss suffered by plaintiff and P1,864,134.08 on its business interruption loss, leaving a balance of P3,624,683.43 and P1,748,460.00, respectively. - The counsel for Artex filed a Manifestation saying that in view of the Deeds of Discharge and Collateral

Page
Agreement, the only remaining liability subject of litigation shall be the proportion of the loss reinsured with or through Alexander and Alexander, Inc. of New York, USA, namely, P397,813.00. The document recited further that Artex acknowledges receipt of the sum of P3.6M paid by the insurer in full and final settlement of all or any claims of Artex against its insurer. It discharges its insurer from all actions, proceedings, claims, demands, costs and expenses in respect thereof. With regard the balance unpaid, Wellington contends that Artex should have been directed against the reinsurers to cover the liability and not against Wellington. ISSUE WON the insured (Artex) has a cause of action against the reinsurer HELD NO - Unless there is a specific grant in, or assignment of, the reinsurance contract in favor of the insured or a manifest intention of the contracting parties to the reinsurance contract to grant such benefit or favor to the insured, the insured, not being privy to the reinsurance contract, has no cause of action against the reinsurer. It is expressly provided in Section 91 the Insurance Act 1 that "(T)he original insured has no interest in a contract of insurance." PERLA COMPANIA DE SEGUROS v. CA(LIM) 208 SCRA 487 NOCON; May 7, 1992 NATURE Petition for certiorari by Perla Compania de Seguros and FOC Credit Corporation seeking to annul and set aside CA decision revering the RTC decision for replevin and damages. FACTS - Private respondents spouses Herminio and Evelyn Lim executed a promissory note in favor of Supercars, Inc. in the sum of P77,940.00, payable in monthly installments according to the schedule of payment indicated in said note, and secured by a chattel mortgage over a brand new red Ford Laser, which is registered under the name of private respondent Herminio Lim and insured with the petitioner Perla

INSURANCE 80
Compania de Seguros, Inc. (Perla for brevity) for comprehensive coverage. - On the same date, Supercars, Inc., with notice to private respondents spouses, assigned to petitioner FCP Credit Corporation (FCP for brevity) its rights, title and interest on said promissory note and chattel mortgage as shown by the Deed of Assignment. - At around 2:30pm Nov9, 1982, said vehicle was carnapped while parked at the back of Broadway Centrum. Evelyn Lim, who was driving said car before it was carnapped, immediately called up the AntiCarnapping Unit of the Philippine Constabulary to report said incident and thereafter, went to the nearest police substation to make a police report regarding said incident. - On Nov10, 1982, Evelyn Lim reported said incident to the LTO in compliance with the insurance requirement. She also filed a complaint with the Headquarters. Constabulary Highway Patrol Group. - On Nov11, 1982, private respondent filed a claim for loss with the petitioner Perla but said claim was denied on Nov18, 1982 on the ground that Evelyn Lim, who was using the vehicle before it was carnapped, was in possession of an expired driver's license at the time of the loss of said vehicle which is in violation of the authorized driver clause of the insurance policy, which states, to wit: "AUTHORIZED DRIVER: Any of the following: (a) The Insured (b) Any person driving on the Insured's order, or with his permission. Provided that the person driving is permitted, in accordance with the licensing or other laws or regulations, to drive the Scheduled Vehicle, or has been permitted and is not disqualified by order of a Court of Law or by reason of any enactment or regulation in that behalf." - On Nov17, 1982, private respondents requested from petitioner FCP for a suspension of payment on the monthly amortization agreed upon due to the loss of the vehicle and, since the carnapped vehicle was insured with petitioner Perla, said insurance company should be made to pay the remaining balance of the promissory note and the chattel mortgage contract. - Perla, however, denied private respondents' claim. Consequently, petitioner FCP demanded that private respondents pay the whole balance of the promissory note or to return the vehicle but the latter refused. - On July25, 1983, petitioner FCP filed a complaint against private respondents, who in turn filed an amended third party complaint against petitioner Perla on Dec8, 1983. After trial on the merits, TC ordered sps Lim to pay jointly and severally, plaintiff the sum of P55,055.93 plus interest thereon at the rate of 24% per annum from July 2, 1983 until fully paid; as well as the cost of suit. It also ordered the dismissal of the Third party complaint against ThirdParty Defendant. - Upon appeal, CA reversed said decision - After petitioners' separate MFRs were denied by CA, petitioners filed these separate petitions for review on certiorari. ISSUE 1. WON there was grave abuse of discretion on the part of the appellate court in holding that private respondents did not violate the insurance contract because the authorized driver clause is not applicable to the "Theft" clause of said Contract 2. WON the loss of the collateral exempted the debtor from his admitted obligations under the promissory note particularly the payment of interest, litigation expenses and attorney's fees HELD 1. NO - The comprehensive insurance policy issued by petitioner Perla undertook to indemnify the private respondents against loss or damages to the car (a) by accidental collision or overturning, or collision or overturning consequent upon mechanical breakdown or consequent upon wear and tear; (b) by fire, external explosion, self-ignition or lightning or burglary, housebreaking or theft; and (c) by malicious act. - Where a car is unlawfully and wrongfully taken without the owner's consent or knowledge, such taking constitutes theft, and, therefore, it is the "THEFT" clause, and not the "AUTHORIZED DRIVER" clause, that should apply.The risk against accident is distinct from the risk against theft. The 'authorized driver clause' in a typical insurance policy as in contemplation or anticipation of accident in the legal sense in which it should be understood, and not in contemplation or anticipation of an event such as theft. The distinction often seized upon by insurance companies in resisting claims from their assureds between death occurring as a result of accident and death occurring as a result of intent may apply to the case at bar. - If the insured vehicle had figured in an accident at the time she drove it with an expired license, then, appellee Perla Compania could properly resist appellants' claim for indemnification for the loss or

Page
destruction of the vehicle resulting from the accident. But in the present case, the loss of the insured vehicle did not result from an accident where intent was involved; the loss in the present case was caused by theft, the commission of which was attended by intent." - There is no causal connection between the possession of a valid driver's license and the loss of a vehicle. To rule otherwise would render car insurance practically a sham since an insurance company can easily escape liability by citing restrictions which are not applicable or germane to the claim, thereby reducing indemnity to a shadow. 2. The court agrees with FCP that Lim spouses are not relieved of their obligation to pay the former the installments due on the promissory note on account of the loss of the automobile. The chattel mortgage constituted over the automobile is merely an accessory contract to the promissory note. Being the principal contract, the promissory note is unaffected by whatever befalls the subject matter of the accessory contract. - The unpaid balance on the promissory note should be paid, and not just the installments due and payable before the automobile was carnapped, as erronously held by the CA - However, this does not mean that private respondents are bound to pay the interest, litigation expenses and attorney's fees stipulated in the promissory note. Because of the peculiar relationship between the three contracts in this case, i. e., the promissory note, the chattel mortgage contract and the insurance policy, the Court is compelled to construe all three contracts as intimately interrelated to each other, despite the fact that at first glance there is no relationship whatsoever between the parties thereto. - Under the promissory note, Lim spouses are obliged to pay Supercars, Inc. the amount stated therein in accordance with the schedule provided for. To secure said promissory note, private respondents constituted a chattel mortgage in favor of Supercars, Inc. over the automobile the former purchased from the latter. The chattel mortgage, in turn, required private respondents to insure the automobile and to make the proceeds thereof payable to Supercars, Inc. The promissory note and chattel mortgage were assigned by Supercars, Inc. to petitioner FCP, with the knowledge of private respondents. Private respondents were able to secure an insurance policy

INSURANCE 81
from petitioner Perla, and the same was made specifically payable to petitioner FCP. - From the abovementioned provision that upon the loss of the insured vehicle, the insurance company Perla undertakes to pay directly to the mortgagor or to their assignee, FCP, the outstanding balance of the mortgage at the time of said loss under the mortgage contract. If the claim on the insurance policy had been approved by petitioner Perla, it would have paid the proceeds thereof directly to petitioner FCP, and this would have had the effect of extinguishing private respondents' obligation to petitioner FCP. Therefore, private respondents were justified in asking petitioner FCP to demand the unpaid installments from petitioner Perla. - Because petitioner Perla had unreasonably denied their valid claim, private respondents should not be made to pay the interest, liquidated damages and attorney's fees as stipulated in the promissory note. As mentioned above, the contract of indemnity was procured to insure the return of the money loaned from petitioner FCP, and the unjustified refusal of petitioner Perla to recognize the valid claim of the private respondents should not in any way prejudice the latter. - Private respondents can not be said to have unduly enriched themselves at the expense of FCP since they will be required to pay the latter the unpaid balance of its obligation under the promissory note. - In view of the foregoing discussion, We hold that the Court of Appeals did not err in requiring petitioner Perla to indemnify private respondents for the loss of their insured vehicle. However, the latter should be ordered to pay petitioner FCP the amount of P55,055.93, representing the unpaid installments from December 30, 1982 up to July 1, 1983, as shown in the statement of account prepared by petitioner FCP, 18 plus legal interest from July 2, 1983 until fully paid. - As to the award of moral damages, exemplary damages and attorney's fees, private respondents are legally entitled to the same since Perla had acted in bad faith by unreasonably refusing to honor the insurance claim of the private respondents. Besides, awards for moral and exemplary damages, as well as attorney's fees are left to the sound discretion of the Court. Such discretion, if well exercised, will not be disturbed on appeal. Disposition the assailed decision of the CA is hereby MODIFIED to require private respondents to pay petitioner FCP the amount of P55,055.93, with legal interest from July 2, 1983 until fully paid. The decision appealed from is hereby affirmed as to all other respects. No pronouncement as to costs. SHAFER v. JUDGE 167 SCRA 386 PADILLA; November 14, 1988 NATURE Petition for review on certiorari FACTS - Sherman Shafer obtained a private car policy over his Ford Laser from Makati Insurance Company, Inc., for third party liability. During the effectivity of the policy, an information for reckless imprudence resulting in damage to property and serious physical injuries was filed against shafer. The information said that on or about the 17th day of May 1985, in the City of Olongapo. Shafer hit and bumped a Volkswagen car owned and driven by Felino llano y Legaspi, thereby causing damage in the total amount of P12,345.00 and as a result thereof one Jovencio Poblete, Sr. who was on board of the said Volkswagen car sustained physical injuries which injuries causing deformity on the face. The owner of the damaged Volkswagen car filed a separate civil action against petitioner for damages, while Jovencio Poblete, Sr., who was a passenger in the Volkswagen car, did not reserve his right to file a separate civil action for damages. Instead, in the course of the trial in the criminal case, Poblete, Sr. testified on his claim for damages for the serious physical injuries which he claimed to have sustained as a result of the accident. - The court issued an order dismissing the third party complaint on the ground that it was premature, based on the premise that unless the accused (herein petitioner) is found guilty and sentenced to pay the offended party (Poblete Sr.) indemnity or damages, the third party complaint is without cause of action. The court further stated that the better procedure is for the accused (petitioner) to wait for the outcome of the criminal aspect of the case to determine whether or not the accused, also the third party plaintiff, has a cause of action against the third party defendant for the enforcement of its third party liability (TPL) under the insurance contract. 6 Petitioner moved for reconsideration of said order, but the motion was denied; hence, this petition. ISSUE

Page
WON the court a quo erred in dismissing petitioner's third party complaint on the ground that petitioner had no cause of action yet against the insurance company HELD YES - There is no need on the part of the insured to wait for the decision of the trial court finding him guilty of reckless imprudence. The occurrence of the injury to the third party immediately gave rise to the liability of the insurer under its policy. Respondent insurance company's contention that the third party complaint involves extraneous matter which will only clutter, complicate and delay the criminal case is without merit. The civil aspect of the offense charged, i.e., serious physical injuries allegedly suffered by Jovencio Poblete, Sr., was impliedly instituted with the criminal case. Petitioner may thus raise all defenses available to him insofar as the criminal and civil aspects of the case are concerned. The claim of petitioner for payment of indemnity to the injured third party, under the insurance policy, for the alleged bodily injuries caused to said third party, arose from the offense charged in the criminal case, from which the injured (Jovencio Poblete, Sr.) has sought to recover civil damages. Hence, such claim of petitioner against the insurance company cannot be regarded as not related to the criminal action. - A third party complaint is a device allowed by the rules of procedure by which the defendant can bring into the original suit a party against whom he will have a claim for indemnity or remuneration as a result of a liability established against him in the original suit. 13 Third party complaints are allowed to minimize the number of lawsuits and avoid the necessity of bringing two (2) or more actions involving the same subject matter. They are predicated on the need for expediency and the avoidance of unnecessary lawsuits. If it appears probable that a second action will result if the plaintiff prevails, and that this result can be avoided by allowing the third party complaint to remain, then the motion to dismiss the third party complaint should be denied. - Compulsory Motor Vehicle Liability Insurance (third party liability, or TPL) is primarily intended to provide compensation for the death or bodily injuries suffered by innocent third parties or passengers as a result of a negligent operation and use of motor vehicles. The victims and/or their dependents are assured of

INSURANCE 82
immediate financial assistance, regardless of the financial capacity of motor vehicle owners. - The liability of the insurance company under the Compulsory Motor Vehicle Liability Insurance is for loss or damage. Where an insurance policy insures directly against liability, the insurer's liability accrues immediately upon the occurrence of the injury or event upon which the liability depends, and does not depend on the recovery of judgment by the injured party against the insured. - The injured for whom the contract of insurance is intended can sue directly the insurer. The general purpose of statutes enabling an injured person to proceed directly against the insurer is to protect injured persons against the insolvency of the insured who causes such injury, and to give such injured person a certain beneficial interest in the proceeds of the policy, and statutes are to be liberally construed so that their intended purpose may be accomplished. It has even been held that such a provision creates a contractual relation which inures to the benefit of any and every person who may be negligently injured by the named insured as if such injured person were specifically named in the policy. - In the event that the injured fails or refuses to include the insurer as party defendant in his claim for indemnity against the insured, the latter is not prevented by law to avail of the procedural rules intended to avoid multiplicity of suits. Not even a "no action" clause under the policy-which requires that a final judgment be first obtained against the insured and that only thereafter can the person insured recover on the policy can prevail over the Rules of Court provisions aimed at avoiding multiplicity of suits. Disposition instant petition is GRANTED. The questioned order dated 24 April 1987 is SET ASIDE and a new one entered admitting petitioner's third party complaint against the private respondent Makati Insurance Company, Inc. VDA DE MAGLANA v. CONSOLACION 212 SCRA 268 ROMERO; August 6, 1992 NATURE Petition for certiorari FACTS - Lope Maglana was an employee of the Bureau of Customs whose work station was at Lasa, here in Davao City. One day, when he was on his way to his work, he met an accident that resulted in his death. He died on the spot. - The PUJ jeep that bumped the deceased was driven by Pepito Into, operated and owned by defendant Destrajo. From the investigation conducted by the traffic investigator, the PUJ jeep was overtaking another passenger jeep that was going towards the city poblacion. While overtaking, the PUJ jeep of defendant Destrajo running abreast with the overtaken jeep, bumped the motorcycle driven by the deceased. The point of impact was on the lane of the motorcycle and the deceased was thrown from the road and met his untimely death. - Heirs of Lope Maglana, Sr. filed an action for damages and attorney's fees against operator Patricio Destrajo and the Afisco Insurance Corporation (AFISCO). An information for homicide thru reckless imprudence was also filed against Pepito Into. - During the pendency of the civil case, Into was sentenced to suffer an indeterminate penalty, with all the accessory penalties provided by law, and to indemnify the heirs of Lope Maglana, Sr. in the amount of twelve thousand pesos with subsidiary imprisonment in case of insolvency, plus five thousand pesos in the concept of moral and exemplary damages with costs. No appeal was interposed by accused who later applied for probation. - The lower court rendered a decision finding that Destrajo had not exercised sufficient diligence as the operator of the jeepney ordering him to pay plaintiffs the sum for loss of income; funeral and burial expenses of the deceased; moral damages, and attorney's fees and costs of suit. The defendant insurance company is ordered to reimburse defendant Destrajo whatever amounts the latter shall have paid only up to the extent of its insurance coverage. - Petitioners filed a motion for the reconsideration of the second paragraph of the decision contending that AFISCO should not merely be held secondarily liable because the Insurance Code provides that the insurer's liability is "direct and primary and/or jointly and severally with the operator of the vehicle, although only up to the extent of the insurance coverage." Hence, they argued that the P20,000.00 coverage of the insurance policy issued by AFISCO, should have been awarded in their favor. - AFISCO argued that since the Insurance Code does not expressly provide for a solidary obligation, the presumption is that the obligation is joint.

Page
- The lower court denied the motion for reconsideration ruling that since the insurance contract "is in the nature of suretyship, then the liability of the insurer is secondary only up to the extent of the insurance coverage." - Petitioners filed a second motion for reconsideration reiterating that the liability of the insurer is direct, primary and solidary with the jeepney operator because the petitioners became direct beneficiaries under the provision of the policy which, in effect, is a stipulation pour autrui. This motion was likewise denied for lack of merit. ISSUE WON AFISCO can be held directly liable HELD YES - As this Court ruled in Shafer vs. Judge, RTC of Olongapo City, Br. 75, "[w]here an insurance policy insures directly against liability, the insurer's liability accrues immediately upon the occurrence of the injury or even upon which the liability depends, and does not depend on the recovery of judgment by the injured party against the insured." - The underlying reason behind the third party liability (TPL) of the Compulsory Motor Vehicle Liability Insurance is "to protect injured persons against the insolvency of the insured who causes such injury, and to give such injured person a certain beneficial interest in the proceeds of the policy . . ." Since petitioners had received from AFISCO the sum of P5,000.00 under the no-fault clause, AFISCO's liability is now limited to P15,000.00. - However, we cannot agree that AFISCO is likewise solidarily liable with Destrajo. In Malayan Insurance Co., Inc. v. Court of Appeals, this Court had the opportunity to resolve the issue as to the nature of the liability of the insurer and the insured vis-a-vis the third party injured in an accident. We categorically ruled thus: While it is true that where the insurance contract provides for indemnity against liability to third persons, such third persons can directly sue the insurer, however, the direct liability of the insurer under indemnity contracts against third party liability does not mean that the insurer can be held solidarily liable with the insured and/or the other parties found at fault. The liability of the insurer is based on contract; that of the insured is based on tort. In the case at bar, petitioner as insurer of Sio Choy, is liable to respondent Vallejos (the injured third party), but it

INSURANCE 83
cannot, as incorrectly held by the trial court, be made "solidarily" liable with the two principal tortfeasors, namely respondents Sio Choy and San Leon Rice Mill, Inc. For if petitioner-insurer were solidarily liable with said, two (2) respondents by reason of the indemnity contract against third party liability under which an insurer can be directly sued by a third party this will result in a violation of the principles underlying solidary obligation and insurance contracts. - While in solidary obligations, the creditor may enforce the entire obligation against one of the solidary debtors, in an insurance contract, the insurer undertakes for a consideration to indemnify the insured against loss, damage or liability arising from an unknown or contingent event. - Similarly, petitioners herein cannot validly claim that AFISCO, whose liability under the insurance policy is also P20,000.00, can be held solidarily liable with Destrajo for the total amount of P53,901.70 in accordance with the decision of the lower court. Since under both the law and the insurance policy, AFISCO's liability is only up to P20,000.00, the second paragraph of the dispositive portion of the decision in question may have unwittingly sown confusion among the petitioners and their counsel. What should have been clearly stressed as to leave no room for doubt was the liability of AFISCO under the explicit terms of the insurance contract. Disposition present petition is hereby GRANTED. The award of P28,800.00 representing loss of income is INCREASED to P192,000.00 and the death indemnity of P12,000.00 to P50,000.00. - The two passengers instituted a suit for damages against La Mallorca who, while denying responsibility, instituted a third party complaint against Far Eastern Surety to recoup from the latter any award for damages that might be recovered by the passengers. - It would appear from the case that a sticker was placed in all the taxis of La Mallorca stating that passengers of the taxis were insured against accidents. This was done to entice the public into patronizing La Mallorca. - The trial court awarded to Misa and Pinto actual, moral and exemplary damages, and attorneys fees payable by La Mallorca and sentenced Far Eastern to pay La Mallorca P10,000. on its third party liability insurance. - On appeal, the CA, while holding that the collision was due to the fault of the driver of the gravel and sand truck, found the taxi company liable for damages to the passengers on the strength of its representation contained in the sticker above noted that the passengers were insured against accidents. In so ruling, the CA overruled the defense of the insurance company to the effect that it was responsible only if the insured, La Mallorca, was involved in accidents caused by, or arising out of, the use of the motor vehicle. A motion for reconsideration was filed in and dismissed by the CA. ISSUE WON Far Eastern Surety is liable to the insured on its insurance policy HELD NO - The award for damages made to the passengers was exclusively predicated on the representation made by La Mallorca that its passengers were insured against accidents and not because it was at fault in causing the accident. Reasoning - In this case, the findings of the CA and the trial court that the causative factor of the mishap was the negligence of the gravel and truck driver would have been sufficient to relieve the taxi company of any liability arising from the accident. However, in view of the sticker in all of its taxicabs, La Mallorca has insured its passengers against accidents, whether it was at fault or not. In other words, La Mallorca accepted the responsibility for damages or injuries to passengers even if it had no fault at all.

Page
- In the case of the insurance company, the SC ruled that it neither authorized nor consented to the representations made by the taxi company to its passengers. As such, the liability of the said insurance company based on its insurance contract is limited to the recovery by the insured of all sums, cost and expenses which the insured shall become legally liable. The insurance company therefore cannot be held liable for the award. - The taxi company is adjudged to be the sole party responsible for the award. Disposition The decision of the CA is modified by eliminating the award against Far Eastern. PEZA v. ALIKPALA 160 SCRA 31 NARVASA; April 15, 1988 NATURE Motion praying that Judge Alikpala be declared guilty of contempt of court for having decided the case on the merits despite the pendency in this Court of the certiorari action instituted by the plaintiffs FACTS - vehicular accident with 2 children running across the path of a Chevrolet "Carry-All", belonging to a partnership known as Diman & Company driven by its driver, Perfecto Amar, as it was passing a national highway at barrio Makiling Calamba, Laguna. They were killed. It was insured with the Empire Insurance Co., Inc. under a so-called 'comprehensive coverage" policy, loss by theft excluded. The policy was in force at the time of the accident. - Placida Peza, the managing partner of Diman & Co. filed a claim with Empire, for payment of compensation to the family of the 2 children who died as a result of the accident. Empire refused to pay on the ground that the driver had no authority to operate the vehicle, a fact which it expressly excepted from liability under the policy. What Peza did was to negotiate directly with the deceased children father for an out-of-court settlement. The father agreed to accept P6,200.00 in fun settlement of the liability of the vehicles owner and driver, and Peza paid him this sum. - Peza thereafter sued Empire to recover this sum of P6,200.00 as actual damages, as well as P20,000.00 as moral damages, P10,000.00 as exemplary damages, and P10,000.00 as attorney's fees. She

FAR EASTERN SURETY v. MISA 25 SCRA 663 REYES; October 26, 1968 NATURE Appeal by petition for review from a CA judgment FACTS - Socorro Dancel Vda.de Misa and Araceli Pinto hired a taxi cab operated by La Mallorca on September 3, 1957. The taxi they were riding in collided with a gravel and sand truck resulting to injuries to both Misa and Pinto.

INSURANCE 84
amended her complaint shortly thereafter to include Diman & Co. as alternative party plaintiff. - Empire's basic defense to the suit was anchored on the explicit requirement in the policy limiting the operation of the insured vehicle to the "authorized driver" therein defined, namely, (a) the insured, or (b) any person driving on the insured order or with his permission, provided that... that the person driving is permited in accordance with the licensing or other laws or regulations to drive the Motor vehicle or has been so permitted and is not disqualified by order of the Court of Law of by reason of any enactment or regulation in that behalf from driving such Motor Vehicle.- driver Perfecto Amar, only having a temporary operator's permit (TVR) [already expired] his drivers license having earlier been confiscated by an agent of the Land Transportation Commission for an alleged violation of Land Transportation and Traffic Rules, was not permitted by law and was in truth disqualified to operate any motor vehicle; Peza attempted to neutralize that fact by(1) the issuance of the TVR by the LTC officer to Amar; in proof of the proposition that there was no reason for confiscation of Amar's license (2) Amar's license had not expired, but had been renewed. - Judge Alikpala did not admit such evidence ISSUES 1. WON Judge Alikapala committed grave abuse of discretion in not admitting evidence 2. WON confiscation of license and expiration of TVR of the driver would serve as bar for Peza in recovering from Empire HELD 1. NO - Even positing error in the Judge's analysis of the evidence attempted to be introduced and his rejection thereof, it is clear that it was at most an error of judgment, not such an error as may be branded a grave abuse of discretion, i.e., such capricious and whimsical exercise of judgment as is equivalent to lack of jurisdiction, against which the writ of certiorari will lie. In any event, the established principle is "that ruling of the trial court on procedural questions and on admissibility of evidence during the course of the trial are interlocutory in nature and may not be the subject of separate appeal or review on certiorari, but are to be assigned as errors and reviewed in the appeal properly taken from the decision rendered by the trial court on the merits of the case. - In the meantime, Judge Alikpala rendered judgment on the merits, since the case was then already ripe for adjudication. The judgment ordered dismissal of the case for failure on the part of the plaintiff to prove their cause of action against Empire. Notice of the judgment was served on the parties in due course. 2. YES - It would seem fairly obvious that whether the LTC agent was correct or not in his opinion that driver Amar had violated some traffic regulation warranting confiscation of his license and issuance of a TVR in lieu thereof, this would not alter the undisputed fact that Amar's licence had indeed been confiscated and a TVR issued to him, and the TVR had already expired at the time that the vehicle being operated by him killed two children by accident. Neither would proof of the renewal of Amar's license change the fact that it had really been earlier confiscated by the LTC agent. Disposition petition is DISMISSED for lack of merit

Page
proving fault or negligence of any kind. Provided, That for purposes of this section (i) The indemnity in respect of any one person shall not exceed P5,000; (ii) The following proofs of loss, when submitted under oath, shall be sufficient evidence to substantiate the claim: (a) Police report of accident, and (b) Death certificate and evidence sufficient to establish, the proper payee, or (c) Medical report and evidence of medical or hospital disbursement in respect of which refund is claimed; (iii) Claim may be made against one motor vehicle only. In the case of an occupant of a vehicle, claim shall lie against the insurer of the vehicle in which the occupant is riding, mounting or dismounting from. In any other case, claim shall lie against the insurer of the directly offending vehicle. In all cases, the right of the party paying the claim to recover against the owner of the vehicle responsible for the accident shall be maintained. Perla denied its liability under the above provision and said that the insurer of the vehicle that the respondents were riding (Malayan Insurance in this case) should be liable. Its 2 MFRs denied, Perla filed this action ISSUE WON Perla is the insurer liable to indemnify under Sec. 378 HELD NO Ratio The law is very clear the claim shall lie against the insurer of the vehicle in which the occupant xxx is riding, and no other. The claimant is not free to choose from which insurer he will claim the no fault indemnity, as the law, by using the word shall, makes it mandatory that the claim be made against the insurer of the vehicle in which the occupant is riding, mounting or dismounting from. Reasoning - the rules on claims under the no fault indemnity provision, where proof of fault or negligence is not necessary for payment of any claim for death or injury to a passenger or to a 3rd party, are established: 1. A claim may be made against one motor vehicle only.

PERLA COMPANIA DE SEGUROS v. ANCHETA 164 SCRA 144 CORTES; August 8, 1988 NATURE Petition for certiorari and prohibition with prelim injunction to review orders of CFI Camarines Norte FACTS - Perla was the insurer of a Superlines bus which figured in a collision with a III Scout (its a kind of vehicle). Injured passengers of the latter (and respondents in this case) filed a complaint for damages against Superlines, the bus driver, and Perla (as insurer of the bus). CFI Judge Ancheta ordered that Perla should pay the respondents immediately the P5000 under the no fault clause as provided in Sec. 378. Sec. 378: Any claim for death or injury to any passenger or 3rd party pursuant to the provisions of this chapter shall be paid without the necessity of

INSURANCE 85
2. If the victim is an occupant of a vehicle, the claim shall lie against the insurer of the vehicle in which he is riding, mounting or dismounting from. 3. In any other case (i.e. if the victim is not an occupant of a vehicle), the claim shall lie against the insurer of the directly offending vehicle. 4. In all cases, the right of the party paying the claim to recover against the owner of the vehicle responsible for the accident shall be maintained. -That the vehicle ridden might not be the one that caused the accident is of no moment since the law itself provides that the party paying the claim under Sec. 378 may recover against the owner of the vehicle responsible for the accident. This is precisely the essence of no fault indemnity insurance which was introduced to and made part of our laws in order to provide victims of vehicular accidents or their heirs immediate compensation, although in a limited amount, pending final determination of who is responsible for the accident and liable for the victims' injuries or death. In turn, the no fault indemnity provision is part and parcel of the Code provisions on compulsory motor vehicle liability insurance and should be read together with the requirement for compulsory passenger and/or 3rd party liability insurance (Sec. 377) which was mandated in order to ensure ready compensation for victims of vehicular accidents. -Irrespective of whether or not fault or negligence lies with the driver of the Superlines bus, as respondents were not occupants of the bus, they cannot claim the no fault indemnity provided in Sec. 378 from Perla. The claim should be made against the insurer of the vehicle they were riding. Disposition Petition GRANTED. Orders of CFI ordering Perla to pay respondents immediately P5000 ANNULLED and SET ASIDE WESTERN GUARANTY CORPORATION (RODRIGUEZ & DE DIOS TRANS) 185 SCRA 652 FELICIANO; July 20, 1990 v. CA Transportation Co., Inc. Priscilla was thrown to the ground, hitting her forehead. She was treated at the Protacio Emergency Hospital and later on hospitalized at the San Juan De Dios Hospital. Her face was permanently disfigured, causing her serious anxiety and moral distress. Respondent bus company was insured with petitioner Western Guaranty Corporation ("Western") under its Master Policy which provided, among other things, for protection against third party liability, the relevant section reading as follows: Section 1. Liability to the Public ? Company will, subject to the Limits of Liability, pay all sums necessary to discharge liability of the insured in respect of ? (a) death of or bodily injury to or damage to property of any passenger as defined herein. (b) death of or bodily injury or damage to property of any THIRD PARTY as defined herein in any accident caused by or arising out of the use of the Schedule Vehicle, provided that the liability shall have first been determined. In no case, however, shall the Company's total payment under both Section I and Section 11 combined exceed the Limits of Liability set forth herein. With respect to death of or bodily injury to any third party or passenger, the company's payment per victim in any one accident shall not exceed the limits indicated in the Schedule of indemnities provided for in this policy excluding the cost of additional medicines, and such other burial and funeral expenses that might have been incurred. - Respondent Priscilla Rodriguez filed a complaint for damages before the Regional Trial Court of Makati against De Dios Transportation Co. and Walter A. Saga Respondent De Dios Transportation Co., in turn, filed a third-party complaint against its insurance carrier, petitioner Western. On 6 August 1985, the trial court rendered a decision in favor of respondent Priscilla E. Rodriguez, awarding moral damages, lossof earning and attorney's fees among others. - On appeal, the Court of Appeals affirmed in toto the decision of the trial court. - Petitioner contends that it cannot be held liable for loss of earnings, moral damages and attorney's fees because these items are not among those included in the Schedule of Indemnities set forth in the insurance policy. ISSUE WON petitioner can be held liable for loss of earnings, moral damages and attorney's fees

Page

NATURE Petition for review the decision of CA affirming in toto the damages awarded to private respondent by the trial court. FACTS - Respondent Priscilla E. Rodriguez was struck by a De Dios passenger bus owned by respondent De Dios

HELD YES - The Schedule of Indemnities does not purport to restrict the kinds of damages that may be awarded against Western once liability has arisen. It was merely meant to set limits to the amounts the movant would be liable for in cases of claims for death, bodily injuries of, professional services and hospital charges, for services rendered to traffic accident victims,' and not necessarily exclude claims against the insurance policy for other kinds of damages, such as those in question. - It will be seen that the above quoted Schedule of Indemnities establishes monetary limits which Western may invoke in case of occurrence of the particular kinds of physical injury there listed. - It must be stressed, however, that the Schedule of Indemnities does not purport to limit, or to enumerate exhaustively, the species of bodily injury occurrence of which generate liability for petitioner Western. A car accident may, for instance, result in injury to internal organs of a passenger or third party, without any accompanying amputation or loss of an external member (e.g., a foot or an arm or an eye). But such internal injuries are surely covered by Section I of the Master Policy, since they certainly constitute bodily injuries. - The Schedule of Indemnities does not purport to restrict the kinds of damages that may be awarded against Western once liability has arisen. Section 1, quoted above, does refer to certain "Limits of Liability" which in the case of the third party liability section of the Master Policy, is apparently P50,000.00 per person per accident. Within this over-all quantitative limit, all kinds of damages allowable by law" actual or compensatory damages"; "moral damages'; "nominal damages"; "temperate or moderate damages"; "liquidated damages"; and "exemplary damages" ? may be awarded by a competent court against the insurer once liability is shown to have arisen, and the essential requisites or conditions for grant of each species of damages are present. It appears to us self-evident that the Schedule of Indemnities was not intended to be an enumeration, much less a closed enumeration, of the specific kinds of damages which may be awarded under the Master Policy Western has issued. - The reading urged by Western of the Schedule of Indemnities comes too close to working fraud upon both the insured and the third party beneficiary of

INSURANCE 86
Section 1, quoted above. For Western's reading would drastically and without warning limit the otherwise unlimited and comprehensive scope of liability assumed by the insurer Western under Section 1: "all sums necessary to discharge liability of the insured in respect of [bodily injury to a third party]". This resultwhich is not essentially different from taking away with the left hand what had been given with the right hand we must avoid as obviously repugnant to public policy. If what Western now urges is what Western intended to achieve by its Schedule of Indemnities, it was incumbent upon Western to use language far more specific and precise than that used in fact by Western, so that the insured, and potential purchasers of its Master Policy, and the Office of the Insurance Commissioner, may be properly informed and act accordingly. - Moreover, an insurance contract is a contract of adhesion. The rule is well entrenched in our jurisprudence that the terms of such contract are to be construed strictly against the party which prepared the contract, which in this case happens to be petitioner Western. Disposition Petition denied. SUMMIT GUARANTY & INSURANCE COMPANY v. ARNALDO 158 SCRA 332 GANCAYCO; February 29, 1988 NATURE PETITION to review the order of the Insurance Commissioner. FACTS - On Nov. 26, 1976, a Ford Pick-up truck owned by Marcos Olasco was bumped by a cargo truck owned by Floralde. FGU Insurance Corporation (FG U) by reason of Motor Vehicle Insurance Policy No. IC-VF-07185 paid Olaso the sum of P2,817.50 as its share in the repair cost of the said Ford Pick-up. Having been subrogated to the rights and causes of action of Olaso in the said amount FGU formally demanded payment of said amount from Floralde and attempted to verify Floralde's insurance carrier but failed to do so. In 1978 FGU was able to ascertain the identity of Floralde's insurance carrier to be the Summit Guaranty and Insurance Company, Inc. (Summit) and thus requested the insurance commissioner for a conference with Summit and demanded from Summit through counsel on February 28, 1978 the payment of the damages sustained by the car of Olaso but to no avail. - Hence on May 22, 1978 FGU filed a case in the Insurance Commissioner's Office against Summit for recovery of said amount. - Summit filed a motion to dismiss on the ground of prescription under Section 384 of PD No. 612. Averring that the accident happened on November 26, 1976 while the complaint was filed on May 22, 1978 beyond the one-year period from the time of the accident provided for by the said provision. ISSUE WON the action must be dismissed on the ground of prescription under Section 384 of PD No. 612 HELD NO - The case do not fall within the meaning of proper cases' as contemplated in Section 384 of the Insurance Code. Reasoning - Section 384 of PD 612 (Insurance Code) Any person having any claim upon the policy issued pursuant to this chapter shall, without any unnecessary delay, present to the insurance company concerned a written notice of claim setting forth the amount of his loss, and/or the nature, extent and duration of the injuries sustained as certified by a duly licensed physician. Notice of claim must be filed within six months from date of the accident, otherwise, the claim shall be deemed waived. Action or suit for recovery of damage due to loss or injury must be brought, in proper cases, with the Commissioner or the Courts within one year from date of accident, otherwise, the claimant's right of action shall prescribe. - It is very clear that the one-year period is only required In proper cases. Had the lawmakers intended it to be the way Petitioner Company assumes it to be, then the phrase 'in proper cases' would not have been inserted. - in Aisporna. vs. Court of Appeals: 'Legislative intent must be ascertained from a consideration of the statute as a whole. The particular words, clauses and phrases should not be studied as detached and isolated expressions, but the whole and every part of the statute must be

Page
considered in fixing the meaning of any of its parts and in order to produce a harmonious whole. A statute must be so construed as to harmonize and give effect to all its provisions whenever possible.' - Petitioner company is trying to use Section 384 of the Insurance Code as a cloak to hide itself from its liabilities. The facts of these cases evidently reflect the deliberate efforts of petitioner company to prevent the filing of a formal action against it. Bearing in mind that if it succeeds in doing so until one year lapses from the date of the accident it could set up the defense of prescription, petitioner company made private respondents believe that their claims would be settled in order that the latter will not find it necessary to immediately bring suit. In violation of its duties to adopt and implement reasonable standards for the prompt investigation of claims and to effectuate prompt, fair and equitable settlement of claims, and with manifest bad faith, petitioner company devised means and ways of stalling the settlement proceedings. In G.R. No. L-50997, no steps were taken to process the claim and no rejection of said claim was ever made even if private respondent had already complied with all the requirements. - In G.R. No. L-48758-petitioner company even provided legal assistance to one of the private respondents in the criminal case filed against him leading Private respondents to believe that it was ready to pay. In the same case, petitioner company admits that it took no final action or adjudication of the claim. Worse still, in G.R. No. L-48679, assurances of payment were constantly given and petitioner company even said that a check was ready for release. This Court has made the observation that some insurance companies have been inventing excuses to avoid their just obligations and it is only the State that can give the protection which the insuring public needs from possible abuses of the insurers. In view of the foregoing, - It is not denied that an extrajudicial demand for payment was made by respondent FGU on petitioner but petitioner failed to respond to the same. Nevertheless the complaint was filed even before a denial of the claim was made by petitioner. For all legal purposes, the one-year prescriptive period provided for in Section 384 of the Insurance Code has not begun to run.The cause of action arises only and starts to run upon the denial of the claim by the insurance company.The court takes note of the dilatory tactics employed by petitioner in this as in the

INSURANCE 87
several cases aforecited to avoid payment of its liabilities. VILLACORTA v. THE INSURANCE COMMISSION 100 SCRA 467 TEEHANKEE; October 30, 1980 FACTS - JEWEL VILLACORTA was the owner of a Colt Lancer, Model 1976, insured with respondent company for P35,000.00 - Own Damage; P30,000.00 - Theft; and P30,000.00 - Third Party Liability, effective May 16, 1977 to May 16, 1978. - On May 9, 1978, the vehicle was brought to the Sunday Machine Works, Inc., for general check-up and repairs. On May 11, 1978, while it was in the custody of the Sunday Machine Works, the car was allegedly taken by six (6) persons and driven out to Montalban, Rizal. While travelling along Mabini St., Sitio Palyasan, Barrio Burgos, going North at Montalban, Rizal, the car figured in an accident, hitting and bumping a gravel and sand truck parked at the right side of the road going south. As a consequence, the gravel and sand truck veered to the right side of the pavement going south and the car veered to the right side of the pavement going north. The driver, Benito Mabasa, and one of the passengers died and the other four sustained physical injuries. The car, as well, suffered extensive damage. Complainant, thereafter, filed a claim for total loss with the respondent company but claim was denied. Hence, complainant was compelled to institute the present action." - The comprehensive motor car insurance policy for P35,000.00 issued by respondent Empire Insurance Company admittedly undertook to indemnify the petitioner-insured against loss or damage to the car (a) by accidental collision or overturning, or collision or overturning consequent upon mechanical breakdown or consequent upon wear and tear; (b) by fire, external explosion, self-ignition or lightning or burglary, housebreaking or theft; and (c) by malicious act. - Respondent insurance commission, however, dismissed petitioner's complaint for recovery of the total loss of the vehicle against private respondent, sustaining respondent insurer's contention that the accident did not fall within the provisions of the policy either for the Own Damage or Theft coverage, invoking the policy provision on "Authorized Driver" clause, which clause limits the use of the insured vehicle to two (2) persons only, namely: the insured himself or any person on his (insured's) permission. Apparently, the Insurance commission sees the unauthorized taking of the vehicle for a joyride as a violation of the 'Authorized Driver' clause of the policy." - Respondent commission likewise upheld private respondent's assertion that the car was not stolen and therefore not covered by the Theft clause, ruling that "(T)he element of 'taking' in Article 308 of the Revised Penal Code means that the act of depriving another of the possession and dominion of a movable thing is coupled . . . with the intention, at the time of the 'taking', of withholding it with the character of permanency ISSUE WON the Insurance commissions findings are in accord with law HELD NO - First, respondent commission's ruling that the person who drove the vehicle in the person of Benito Mabasa, who, according to its own finding, was one of the residents of the Sunday Machine Works, Inc. to whom the car had been entrusted for general checkup and repairs was not an "authorized driver" of petitioner-complainant is too restrictive and contrary to the established principle that insurance contracts, being contracts of adhesion where the only participation of the other party is the signing of his signature or his "adhesion" thereto, "obviously call for greater strictness and vigilance on the part of courts of justice with a view of protecting the weaker party from abuse and imposition, and prevent their becoming traps for the unwary." - The main purpose of the "authorized driver" clause, as may be seen from its text, supra, is that a person other than the insured owner, who drives the car on the insured's order, such as his regular driver, or with his permission, such as a friend or member of the family or the employees of a car service or repair shop must be duly licensed drivers and have no disqualification to drive a motor vehicle. A car owner who entrusts his car to an established car service and repair shop necessarily entrusts his car key to the shop owner and employees who are presumed to have the insured's permission to drive the car for legitimate purposes of checking or road-testing the car. The mere happenstance that the employee(s) of

Page
the shop owner diverts the use of the car to his own illicit or unauthorized purpose in violation of the trust reposed in the shop by the insured car owner does not mean that the "authorized driver" clause has been violated such as to bar recovery, provided that such employee is duly qualified to drive under a valid driver's license. - Secondly, and independently of the foregoing (since when a car is unlawfully taken, it is the theft clause, not the "authorized driver" clause, that applies), where a car is admittedly as in this case unlawfully and wrongfully taken by some people, be they employees of the car shop or not to whom it had been entrusted, and taken on a long trip to Montalban without the owner's consent or knowledge, such taking constitutes or partakes of the nature of theft as defined in Article 308 of the Revised Penal Code. - The Court rejects respondent commission's premise that there must be an intent on the part of the taker of the car "permanently to deprive the insured of his car" and that since the taking here was for a "joy ride" and "merely temporary in nature," a "temporary taking is held not a taking insured against." - The insurer must therefore indemnify the petitioner owner for the total loss of the insured car in the sum of P35,000.00 under the theft clause of the policy, subject to the filing of such claim for reimbursement or payment as it may have as subrogee against the Sunday Machine Works, Inc.

CHAPTER VIII. MARINE INSURANCE MAGSAYSAY INC v. AGAN 96 PHIL 504 REYES; January 31, 1955 FACTS - The S S "San Antonio", vessel owned and operated by plaintiff, left Manila on October 6, 1949, bound for Basco, Batanes, vis Aparri, Cagayan, with general cargo belonging to different shippers, among them the defendant. The vessel reached Aparri, but while still in the port, it ran aground at the mouth of the Cagayan river, and, attempts to refloat it under its own power having failed, plaintiff had it refloated by

INSURANCE 88
the Luzon Stevedoring Co. at an agreed compensation. Once afloat the vessel returned to Manila to refuel and then proceeded to Basco, the port of destination. There the cargoes were delivered to their respective owners or consignees, who, with the exception of defendant, made a deposit or signed a bond to answer for their contribution to the average. - On the theory that the expenses incurred in floating the vessel constitute general average to which both ship and cargo should contribute, plaintiff brought the present action in the CFI of Manila to make defendant pay his contribution, which, as determined by the average adjuster, amounts to P841.40. - Defendant denies liability to his amount, alleging, among other things, that the stranding of the vessel was due to the fault, negligence and lack of skill of its master, that the expenses incurred in putting it afloat did not constitute general average, and that the liquidation of the average was not made in accordance with law. - The lower court found for plaintiff ISSUE WON the expenses incurred in floating a vessel so stranded should be considered general average and shared by the cargo owners HELD NO Ratio The law on averages is contained in the Code of Commerce. Under that law, averages are classified into simple or particular and general or gross. Generally speaking, simple or particular averages include all expenses and damages caused to the vessel or cargo which have not inured to the common benefit (Art. 809), and are, therefore, to be borne only by the owner of the property gave rise to same (Art. 810); while general or gross averages include "all the damages and expenses which are deliberately caused in order to save the vessel, its cargo, or both at the same time, from a real and known risk" (Art. 811). Being for the common benefit, gross averages are to be borne by the owners of the articles saved (Art. 812). Reasoning - the stranding of plaintiff's vessel was due to the sudden shifting of the sandbars at the mouth of the river which the port pilot did not anticipate. The standing may, therefore, be regarded as accidental. - Tolentino, in his commentaries on the Code of Commerce, gives the following requisites for general average: First, there must be a common danger. This means, that both the ship and the cargo, after has been loaded, are subject to the same danger, whether during the voyage, or in the port of loading or unloading; that the danger arises from the accidents of the sea, dispositions of the authority, or faults of men, provided that the circumstances producing the peril should be ascertained and imminent or may rationally be said to be certain and imminent. This last requirement exclude measures undertaken against a distant peril. Second, that for the common safety part of the vessel or of the cargo or both is sacrificed deliberately. Third, that from the expenses or damages caused follows the successful saving of the vessel and cargo. Fourth, that the expenses or damages should have been incurred or inflicted after taking proper legal steps and authority. - With respect to the first requisite, the evidence does not disclose that the expenses sought to be recovered from defendant were incurred to save vessel and cargo from a common danger...it is the safety of the property, and not of the voyage, which constitutes the true foundation of the general average. - As to the second requisite, we need only repeat that the expenses in question were not incurred for the common safety of vessel and cargo, since they, or at least the cargo, were not in imminent peril. - With respect to the third requisite, the salvage operation, it is true, was a success. But as the sacrifice was for the benefit of the vessel to enable it to proceed to destination and not for the purpose of saving the cargo, the cargo owners are not in law bound to contribute to the expenses. - The final requisite has not been proved, for it does not appear that the expenses here in question were incurred after following the procedure laid down in article 813. Disposition Wherefore, the decision appealed from is reversed. JARQUE v. SMITH, BELL & CO. 56 PHIL 758 OSTRAND; November11, 1932 NATURE Appeal from judgment of the lower court

Page
FACTS - Plaintiffs motorboat, Pandan was insured on a marine insurance policy with National Union Fire Insurance Company (NUFIC) for P45K. According to the provisions of a rider attached to the policy, the insurance was against the absolute total loss of the vessel only. On Oct. 31, 1928, the ship ran into very heavy sea and it became necessary to jettison a portion of the cargo. As a result of the jettison, the NUFIC was assessed P2,610.86 as its contribution to the general average. - The insurance company, insisting that its obligation did not extend beyond the insurance of the absolute total loss of the vessel only, and to pay proportionate salvage of the declared value, refused to contribute to the settlement of the gen. ave. The present action was thereupon instituted, and after trial the court below rendered judgment in favor of the plaintiff and ordered the defendant to pay the plaintiff P2,610.86 as its part of the indemnity for the gen. ave. brought about by the jettison of cargo. The insurance company then appealed to the SC. - The insurance contract is printed in the English common form of marine policies. One of the clauses of the document originally read as follows: Touching the Adventures and Perils which the said NUFIC is content to bear, and to take upon them in this Voyage; they are of the Seas, Men-of-War, Fire, Pirates, Thieves, Jettison, Letters of Mart and Countermart, Surprisals, and Takings at Sea. Arrests, Restraints and Detainments, of all Kings, Princes and People of what Nation, Condition or Quality soever; Barratry of the Master and Marines, and of all other Perils, Losses and Misfortunes, that have or shall come to the Hurt, Detriment, or Damage of the said Vessel or any part thereof; and in case of any Loss or Misfortunes, it shall be lawful for the Assured, his or their Factors, Servants, or assigns, to sue, labour and travel for, in and about the Defence. Safeguard, and recovery of the said Vessel or any part thereof, without Prejudice to this Insurance; to the Charges whereof the said Company, will contribute, according to the rate and quantity of the sum herein assured... - Attached to the policy over and above the said clause is a rider containing typewritten provisions, among which appears in capitalized type the following clause: AGAINST THE ABSOLUTE TOTAL LOSS OF THE VESSEL ONLY, AND TO PAY PROPORTIONATE SALVAGE CHARGES OF THE DECLARED VALUE.

INSURANCE 89
ISSUES 1. WON the lower court erred in disregarding the typewritten clause endorsed upon the policy, expressly limiting insurer's liability thereunder of the total loss of the wooden vessel Pandan and to proportionate salvage charges 2. WON lower court erred in concluding that defendant and appellant, NUFIC is liable to contribute to the general average resulting from the jettison of a part of said vessel's cargo HELD 1. NO Ratio In case repugnance exists between written and printed portions of a policy, the written portion prevails. Reasoning - Section 291 of the Code of Civil Procedure provides that when an instrument consists partly of written words and partly of a printed form and the two are inconsistent, the former controls the latter. 2. NO Ratio The liability for contribution in general average is not based on the express terms of the policy, but rests upon the theory that from the relation of the parties and for their benefit, a quasi contract is implied by law. Reasoning - In the absence of positive legislation to the contrary, the liability of the defendant insurance company on its policy would, perhaps, be limited to absolute loss of the vessel only, and to pay proportionate salvage of the declared value. But the policy was executed in this jurisdiction and warranted to trade within the waters of the Philippine Archipelago only. Here, Art. 859 of the Code of Commerce is still in force: ART. 859. The underwriters of the vessel, of the freight, and of the cargo shall be obliged to pay for the indemnity of the gross average in so far as is required of each one of these objects respectively. - The article is mandatory in its terms, and the insurers (whether for the vessel or for the freight or for the cargo) are bound to contribute to the indemnity of the general average. The provision simply places the insurer on the same footing as other persons who have an interest in the vessel, or the cargo therein, at the time of the occurrence of the general average and who are compelled to contribute (Art. 812, Code of Commerce). - In the present case it is not disputed that the ship was in grave peril and that the jettison of part of the cargo was necessary. If the cargo was in peril to the extent of call for general average, the ship must also have been in great danger, possibly sufficient to cause its absolute loss. The jettison was therefore as much to the benefit of the underwriter as to the owner of the cargo. The latter was compelled to contribute to the indemnity; why should not the insurer be required to do likewise? If no jettison had taken place and if the ship by reason thereof had foundered, the underwriter's loss would have been many times as large as the contribution now demanded. Disposition Appealed judgment is affirmed GO TIACO v. UNION INSURANCE 40 PHIL 40 STREET; September 1, 1919 FACTS - Union Insurance Society of Canton, Ltd., issued a marine insurance policy upon a cargo of rice belonging to the Go Tiaoco Brothers, which was transported in the early days of May, 1915, on the steamship Hondagua from the port of Saigon to Cebu. - On discharging the rice from one of the compartments in the after hold, upon arrival at Cebu, it was discovered that 1473 sacks had been damaged by sea water. The loss was P3,875.25. - The trial court found that the inflow of the sea water during the voyage was due to a defect in one of the drain pipes of the ship and concluded that the loss was not covered by the policy of insurance. The trial court made the ff findings: The drain pipe which served as a discharge from the water closet passed down through the compartment where the rice in question was stowed and thence out to sea through the wall of the compartment, which was a part of the wall of the ship. The joint or elbow where the pipe changed its direction was of cast iron; and in course of time it had become corroded and abraded until a longitudinal opening had appeared in the pipe about one inch in length. This hole had been in existence before the voyage was begun, and an attempt had been made to repair it by filling with cement and bolting over it a strip of iron. The effect of loading the boat was to submerge the vent, or

Page
orifice, of the pipe until it was about 18 inches or 2 feet below the level of the sea. As a consequence the sea water rose in the pipe. Navigation under these conditions resulted in the washing out of the cement-filling from the action of the sea water, thus permitting the continued flow of the salt water into the compartment of rice. - The court found in effect that the opening above described had resulted in course of time from ordinary wear and tear and not from the straining of the ship in rough weather on that voyage. The court also found that the repairs that had been made on the pipe were slovenly and defective and that, by reason of the condition of this pipe, the ship was not properly equipped to receive the rice at the time the voyage was begun. For this reason the court held that the ship was unseaworthy. - The policy purports to insure the cargo from the following among other risks: "Perils . . . of the seas, men, of war, fire, enemies, pirates, rovers, thieves, .jettisons, . . . barratry of the master and mariners, and of all other perils, losses, and misfortunes that have or shall come to the hurt, detriment, or damage of the said goods and merchandise or any part thereof." ISSUE WON Union Insurance is liable for the loss of the Go Tiaco Brothers HELD NO - the words "all other perils, losses, and misfortunes" are to be interpreted as covering risks which are of like kind (ejusdem generis) with the particular risks which are enumerated in the preceding part of the same clause of the contract. ''According to the ordinary rules of construction, these words must be interpreted with reference to the words which immediately precede them. They were no doubt inserted in order to prevent disputes founded on nice distinctions. X x x For example, if the expression 'perils of the seas' is given its widest sense the general words have little or no effect as applied to that case. If on the other hand that expression is to receive a limited construction, as apparently it did in Cullen vs. Butler (5 M. & S., 461), and loss by perils of the seas is to be confined to loss ex marine tempestatis discrimine, the general words become

INSURANCE 90
most important. X x x" (Thames and Mersey Marine Insurance Co. vs. Hamilton, Fraser & Co.) - a loss which, in the ordinary course of events, results from the natural and inevitable action of the sea, from the ordinary wear and tear of the ship, or from the negligent failure of the ship's owner to provide the vessel with proper equipment to convey the cargo under ordinary conditions, is not a peril of the sea. Such a loss is rather due to what has been aptly called the "peril of the ship." The insurer undertakes to insure against perils of the sea and similar perils, not against perils of the ship. There must, in order to make the insurer liable, be "some casualty, something which could not be foreseen as one of the necessary incidents of the adventure. The purpose of the policy is to secure an indemnity against accidents which may happen, not against events which must happen." (Wilson, Sons & Co. vs. Owners of Cargo per the Xantho) - In the present case the entrance of the sea water into the ship's hold through the defective pipe already described was not due to any accident which happened during the voyage, but to the failure of the ship's owner properly to repair a defect of the existence of which he was apprised. The loss was therefore more analogous to that which directly results from simple unseaworthiness than to that which results from perils of the sea. - there is no room to doubt the liability of the shipowner for such a loss as occurred in this case. By parity of reasoning the insurer is not liable; for, generally speaking, the shipowner excepts the perils of the sea from his engagement under the bill of lading, while this is the very peril against which the insurer intends to give protection. As applied to the present case it results that the owners of the damaged rice must look to the shipowner for redress and not to the insurer. The same conclusion must be reached if the question be discussed with reference to the seaworthiness of the ship. It is universally accepted that in every contract of insurance upon anything which is the subject of marine insurance, a warranty is implied that the ship shall be seaworthy at the time of the inception of the voyage. This rule is accepted in our own Insurance Law (Act No. 2427, sec. 106). It is also well settled that a ship which is seaworthy for the purpose of insurance upon the ship may yet be unseaworthy for the purpose of insurance upon the cargo (Act No. 2427, sec. 106). Disposition Decision of trial court is affirmed CATHAY INSURANCE CO. v. CA (REMINGTON INDUSTRIAL SALES CORP.) 151 SCRA 710 PARAS; June 30 1987 FACTS - Remington Industrial Sales Corp insured its shipment of seamless steel pipes. It incurred losses and damages (I gather the steel pipes rusted during the voyage from Japan to the Phils. on board vessel SS "Eastern Mariner) and filed complaint against Cathay Insurance Co seeking collection of the sum of P868,339.15 - TC decided for Remington. Cathay filed MR, which was denied. CA affirmed. - CA said (among other things): 1. Coverage of private respondent's loss under the insurance policy issued by petitioner is unmistakable; 2. Alleged contractual limitations contained in insurance policies are regarded with extreme caution by courts and are to be strictly construed against the insurer; obscure phrases and exceptions should not be allowed to defeat the very purpose for which the policy was procured; 3. Rust is not an inherent vice of the seamless steel pipes without interference of external factors - Cathay contend (among other things): 1. private respondent has admitted that the questioned shipment is not covered by a "square provision of the contract," but private respondent claims implied coverage from the phrase "perils of the sea" mentioned in the opening sentence of the policy; 2. The insistence of private respondent that rusting is a peril of the sea is erroneous; 3. Rusting is not a risk insured against, since a risk to be insured against should be a casualty or some casualty, something which could not be foreseen as one of the necessary incidents of adventure; 4. A fact capable of unquestionable demonstration or of public knowledge needs no evidence. This fact of unquestionable demonstration or of public knowledge is that heavy rusting of steel or iron pipes cannot occur within a period of a seven (7) day voyage. Besides, petitioner had introduced the clear cargo receipts or tally sheets indicating that there was no damage on the steel pipes during the voyage. ISSUE WON rusting is a peril of the sea

Page
HELD YES - There is no question that the rusting of steel pipes in the course of a voyage is a "peril of the sea" in view of the toll on the cargo of wind, water, and salt conditions. At any rate if the insurer cannot be held accountable therefor, We would fail to observe a cardinal rule in the interpretation of contracts, namely, that any ambiguity therein should be construed against the maker/issuer/drafter thereof, namely, the insurer. Besides the precise purpose of insuring cargo during a voyage would be rendered fruitless. Disposition WHEREFORE, this petition is hereby DENIED, and the assailed decision of the Court of Appeals is hereby AFFIRMED. ROQUE v. IAC (PIONEER INSURANCE SURETY CORP.) 139 SCRA 596 GUTIERREZ; November 11, 1985 AND

NATURE Petition for certiorari to review the decision of the IAC FACTS - February 19, 1972 Common carrier Manila Bay Lighterage Corp. entered into a contract with Roque Timber Enterprises and Chiong. The contract stated that Manila Bay would carry 422.18 cu. meters of logs on its vessel Mable 10 from Malampaya Sound, Palawan to Manila North Harbor. Roque insured the logs with Pioneer Insurance for P100,000. - February 29, 1972 811 logs were loaded in Malampaya but en route to Manila, Mable 10 sank. - March 8,1972 Roque and Chiong wrote a letter to Manila Bay, demanding payment of P150,000.00 for the loss of the shipment plus P100,000.00 as unrealized profits but the latter ignored the demand. - A letter was also sent to Pioneer, claiming the full amount of P100,000.00 under the insurance policy but Pioneer refused to pay on the ground that its liability depended upon the "Total Loss by Total Loss of Vessel only". - After hearing, the trial court favored Roque. Pioneer and Manila Bay were ordered to pay Roque P100,000. Pioneer appealed the decision. - January 30, 1984 Pioneer was absolved from liability after finding that there was a breach of implied warranty of seaworthiness on the part of the petitioners and that the loss of the insured cargo was

INSURANCE 91
caused by the "perils of the ship" and not by the "perils of the sea". It ruled that the loss is not covered by the marine insurance policy. - It was alleged that Mable 10 was not seaworthy and that it developed a leak - The IAC found that one of the hatches was left open, causing water to enter the barge and because the barge was not provided with the necessary cover or tarpaulin, the splash of sea waves brought more water inside the barge. - Petitioners contend that the implied warranty of seaworthiness provided for in the Insurance Code refers only to the responsibility of the shipowner who must see to it that his ship is reasonably fit to make in safety the contemplated voyage. - The petitioners state that a mere shipper of cargo, having no control over the ship, has nothing to do with its seaworthiness. They argue that a cargo owner has no control over the structure of the ship, its cables, anchors, fuel and provisions, the manner of loading his cargo and the cargo of other shippers, and the hiring of a sufficient number of competent officers and seamen. ISSUE WON the loss should have been covered by the marine insurance policy HELD NO Ratio It is universally accepted that in every contract of insurance upon anything which is the subject of marine insurance, a warranty is implied that the ship shall be seaworthy at the time of the inception of the voyage. In marine insurance, the risks insured against are classified as 'perils of the sea, which includes such losses that are of extraordinary nature, or arise from some overwhelming power, which cannot be guarded against by the ordinary exertion of human skill and prudence. Reasoning - Based on Sec. 113 and Sec. 99 of the Insurance Code, the term "cargo" can be the subject of marine insurance and that once it is so made, the implied warranty of seaworthiness immediately attaches to whoever is insuring the cargo whether he be the shipowner or not. - The fact that the un-seaworthiness of the ship was unknown to the insured is immaterial in ordinary marine insurance and may not be used by him as a defense in order to recover on the marine insurance policy. - Since the law provides for an implied warranty of seaworthiness in every contract of ordinary marine insurance, it becomes the obligation of a cargo owner to look for a reliable common carrier which keeps its vessels in seaworthy condition. The shipper of cargo my have no control over the vessel but he has full control in the choice of the common carrier that will transport his goods. - In marine cases, the risks insured against are 'perils of the sea. The term extends only to losses caused by sea damage, or by the violence of the elements, and does not embrace all losses happening at sea. - It is quite unmistakable that the loss of the cargo was due to the perils of the ship rather than the perils of the sea. - Loss which, in the ordinary course of events, results from the natural and inevitable action of the sea, from the ordinary wear and tear of the ship, or from the negligent failure of the ship's owner to provide the vessel with proper equipment to convey the cargo under ordinary conditions, is not a peril of the sea but is called peril of the ship. Disposition Decision appealed from is affirmed. LA RAZON v. UNION INSURANCE SOCIETY OF CANTON, LTD. 40 PHIL 40 STREET; September 1, 1919 FACTS - This is an action on a policy of marine insurance issued by the Union Insurance Society of Canton, Ltd., upon a cargo of rice belonging to the plaintiffs, Go Tiaoco Brothers, which was transported on the steamship Hondagua from the port of Saigon to Cebu. - On discharging the rice from one of the compartments in the after hold, upon arrival at Cebu, it was discovered that 1,473 sacks had been damaged by sea water. - The loss so resulting to the owners of rice, after proper deduction had been made for the portion saved, was P3,875.25. - The trial court found that the inflow of the sea water during the voyage was due to a defect in one of the drain pipes of the ship and concluded that the loss was not covered by the policy of insurance. Judgment was accordingly entered in favor of the defendant and the plaintiffs appealed. - The court found in effect that the opening above described had resulted in course of time from ordinary wear and tear and not from the straining of the ship

Page
in rough weather on that voyage. The court also found that the repairs that had been made on the pipe were slovenly and defective and that, by reason of the condition of this pipe, the ship was not properly equipped to receive the rice at the time the voyage was begun. For this reason the court held that the ship was unseaworthy. ISSUE WON the insurer is liable HELD - The question whether the insurer is liable on this policy for the loss caused in the manner above stated presents two phases which are in a manner involved with each other. One has reference to the meaning of the expression "perils of the seas and all other perils, losses, and misfortunes," as used in the policy; the other has reference to the implied warranty, on the part of the insured, as to the seaworthiness of the ship. - The meaning of the expression "perils * * * of the seas * * * and all other perils, losses, and misfortunes," used in describing the risks covered by policies of marine insurance, has been the subject of frequent discussion; and certain propositions relative thereto are now so generally accepted as to be considered definitely settled. - The words "all other perils, losses, and misfortunes" are to be interpreted as covering risks which are of like kind (ejusdem generis) with the particular risks which are enumerated in the preceding part of the same clause of the contract. - A loss which, in the ordinary course of events, results from the natural and inevitable action of the sea, from the ordinary wear and tear of the ship, or from the negligent failure of the ship's owner to provide the vessel with proper equipment to convey the cargo under ordinary conditions, is not a peril of the sea. Such a loss is rather due to what has been aptly called the "peril of the ship." The insurer undertakes to insure against perils of the sea and similar perils, not against perils of the ship. - As was said by Lord Herschell in Wilson, Sons & Co. vs. Owners of Cargo per the Xantho, there must, in order to make the insurer liable, be "some casualty, something which could not be foreseen as one of the necessary incidents of the adventure. The purpose of the policy is to secure an indemnity against accidents which may happen, not against events which must happen."

INSURANCE 92
- In the present case the entrance of the sea water into the ship's hold through the defective pipe already described was not due to any accident which happened during the voyage, but to the failure of the ship's owner properly to repair a defect of the existence of which, he was apprised. The loss was therefore more analogous to that which directly results from simple unseaworthiness than to that which results from perils of the sea. - It is universally accepted that in every contract of insurance upon anything which is the subject of marine insurance, a warranty is implied that the ship shall be seaworthy at the time of the inception of the voyage. This rule is accepted in our own Insurance Law (Act No. 2427, see. 106). - It is also well settled that a ship which is seaworthy for the purpose of insurance upon the ship may yet be unseaworthy for the purpose of insurance upon the cargo (Act No. 2427, see. 106). Disposition Jjudgment affirmed. MALAYAN INSURANCE v. CA (supra p.10) FILIPINO MERCHANTS INS. CO. v. CA (supra p.19) COASTWISE LIGHTERAGE CORP v. CA (PHILIPPINE GENERAL INSURANCE COMPANY) 245 SCRA 796 FRANCISCO; July 12, 1995 NATURE Petition for review of CA Decision affirming decision of RTC Manila holding that Coastwise is liable to pay PhilGen Insurance the amount of P700thou plus legal interest thereon, another sum of P100thou as attorney's fees and the cost of the suit. FACTS - Pag-asa Sales, Inc. entered into a contract to transport molasses from the province of Negros to Manila with Coastwise, using the latter's dumb barges. The barges were towed in tandem by the tugboat MT Marica, also owned by Coastwise. Upon reaching Manila Bay, while approaching Pier 18, one of the barges struck an unknown sunken object. The forward buoyancy compartment was damaged, and water gushed in through a hole "two inches wide and twenty-two inches long." - As a consequence, the molasses at the cargo tanks were contaminated and rendered unfit for the use it was intended. This prompted consignee Pag-asa Sales to reject the shipment of molasses as a total loss. Thereafter, Pag-asa Sales filed a formal claim with the insurer of its lost cargo (PhilGen) and against the carrier (Coastwise). Coastwise denied the claim and it was PhilGen which paid Pag-asa Sales the amount of P700k representing the value of the damaged cargo of molasses. - PhilGen then filed an action against Coastwise before the RTC Manila, seeking to recover the P700k which it paid to Pag-asa Sales for the latter's lost cargo. PhilGen now claims to be subrogated to all the contractual rights and claims which the consignee may have against the carrier, which is presumed to have violated the contract of carriage. - RTC awarded the amount prayed for by PhilGen. CA affirmed. Hence, this petition. ISSUES 1. WON Coastwise Lighterage was transformed into a private carrier, by virtue of the contract of affreightment which it entered into with the consignee, Pag-asa Sales, Inc. (Corollarily, if it were in fact transformed into a private carrier, did it exercise the ordinary diligence to which a private carrier is in turn bound?) 2. WON the insurer was subrogated into the rights of the consignee against the carrier, upon payment by the insurer of the value of the consignee's goods lost while on board one of the carrier's vessels HELD 1. NO - The distinction between the two kinds of charter parties (i.e. bareboat or demise and contract of affreightment) is more clearly set out in the case of Puromines, Inc. vs. Court of Appeals, wherein SC ruled: Under the demise or bareboat charter of the vessel, the charterer will generally be regarded as the owner for the voyage or service stipulated. The charterer mans the vessel with his own people and becomes the owner pro hac vice, subject to liability to others for damages caused by negligence. To create a demise, the owner of a vessel must completely and exclusively relinquish possession, command and navigation thereof to the charterer, anything short of such a complete transfer is a contract of affreightment (time or voyage charter party) or not a charter party at all.

Page
On the other hand a contract of affreightment is one in which the owner of the vessel leases part or all of its space to haul goods for others. It is a contract for special service to be rendered by the owner of the vessel and under such contract the general owner retains the possession, command and navigation of the ship, the charterer or freighter merely having use of the space in the vessel in return for his payment of the charter hire... An owner who retains possession of the ship though the hold is the property of the charterer, remains liable as carrier and must answer for any breach of duty as to the care, loading and unloading of the cargo. - Although a charter party may transform a common carrier into a private one, the same however is not true in a contract of affreightment on account of the aforementioned distinctions between the two. - SC agrees with Coastwise's admission that the contract it entered into with the consignee was one of affreightment. Pag-asa Sales, Inc. only leased three of petitioner's vessels, in order to carry cargo from one point to another, but the possession, command and navigation of the vessels remained with Coastwise. As such, Coastwise, by the contract of affreightment, was not converted into a private carrier, but remained a common carrier and was still liable as such. - Therefore, the mere proof of delivery of goods in good order to a carrier and the subsequent arrival of the same goods at the place of destination in bad order makes for a prima facie case against the carrier. The presumption of negligence that attaches to common carriers, once the goods it transports are lost, destroyed or deteriorated, applies to Coastwise. This presumption, which is overcome only by proof of the exercise of extraordinary diligence, remained unrebutted in this case. - The damage to the barge which carried the cargo of molasses was caused by its hitting an unknown sunken object as it was heading for Pier 18. The object turned out to be a submerged derelict vessel. The evidence on record appeared that far from having rendered service with the greatest skill and utmost foresight, and being free from fault, the carrier was culpably remiss in the observance of its duties. - Jesus R. Constantino, the patron of the vessel "Coastwise 9" admitted that he was not licensed. The Code of Commerce, which subsidiarily governs common carriers (which are primarily governed by the

INSURANCE 93
provisions of the Civil Code) provides: Art. 609. Captains, masters, or patrons of vessels must be Filipinos, have legal capacity to contract in accordance with this code, and prove the skill capacity and qualifications necessary to command and direct the vessel, as established by marine and navigation laws, ordinances or regulations, and must not be disqualified according to the same for the discharge of the duties of the position. - Clearly, Coastwise Lighterage's embarking on a voyage with an unlicensed patron violates this rule. It cannot safely claim to have exercised extraordinary diligence, by placing a person whose navigational skills are questionable, at the helm of the vessel which eventually met the fateful accident. It may also logically, follow that a person without license to navigate, lacks not just the skill to do so, but also the utmost familiarity with the usual and safe routes taken by seasoned and legally authorized ones. Had the patron been licensed, he could be presumed to have both the skill and the knowledge that would have prevented the vessel's hitting the sunken derelict ship that lay on their way to Pier 18. - As a common carrier, Coastwise is liable for breach of the contract of carriage, having failed to overcome the presumption of negligence with the loss and destruction of goods it transported, by proof of its exercise of extraordinary diligence. 2. YES - Coastwise is liable for breach of the contract of carriage it entered into with Pag-asa Sales, Inc. However, for the damage sustained by the loss of the cargo which petitioner-carrier was transporting, it was not the carrier which paid the value thereof to Pagasa Sales, Inc. but the latter's insurer, herein private respondent PhilGen. - Article 2207 of the Civil Code: If the plaintiffs property has been insured, and he has received indemnity from the insurance company for the injury or loss arising out of the wrong or breach of contract complained of, the insurance company shall be subrogated to the rights of the insured against the wrongdoer or the person who violated the contract. - This legal provision is founded on the well-settled principle of subrogation. If the insured property is destroyed or damaged through the fault or negligence of a party other than the assured, then the insurer, upon payment to the assured will be subrogated to the rights of the assured to recover from the wrongdoer to the extent that the insurer has been obligated to pay. Payment by the insurer to the assured operated as an equitable assignment to the former of all remedies which the latter may have against the third party whose negligence or wrongful act caused the loss. The right of subrogation is not dependent upon, nor does it grow out of, any privity of contract or upon written assignment of claim. It accrues simply upon payment of the insurance claim by the insurer. - Undoubtedly, upon payment by respondent insurer PhilGen of the amount of P700,000.00 to Pag-asa Sales, Inc., the consignee of the cargo of molasses totally damaged while being transported by petitioner Coastwise Lighterage, the former was subrogated into all the rights which Pag-asa Sales, Inc. may have had against the carrier, herein petitioner Coastwise Lighterage. Disposition Petition denied. CA affrimed. THE PHILIPPINE AMERICAN GENERAL INSURANCE COMPANY INC v. CA (FELMAN SHIPPING LINES) 273 SCRA 226 BELLOSILLO; June 11, 1997 FACTS - Coca-Cola Bottlers Philippines, Inc., loaded on board MV Asilda, a vessel owned and operated by Felman 7,500 cases of 1-liter Coca-Cola softdrink bottles to be transported from Zamboanga City to Cebu for consignee Coca-Cola Bottlers Philippines, Inc., Cebu. The shipment was insured with petitioner Philippine American General under Marine Open Policy. - The vessel sank in the waters of Zamboanga del Norte bringing down her entire cargo with her including the subject 7,500 cases of 1-liter Coca-Cola softdrink bottles. - The consignee filed a claim with respondent FELMAN for recovery of damages it sustained as a result of the loss of its softdrink bottles that sank with MV Asilda. Respondent denied the claim thus prompting the consignee to file an insurance claim with PHILAMGEN which paid its claim of P755,250.00. - Claiming its right of subrogation PHILAMGEN sought recourse against respondent FELMAN which disclaimed any liability for the loss. Consequently, PHILAMGEN sued the shipowner for sum of money and damages. - PHILAMGEN alleged that the sinking and total loss of MV Asilda and its cargo were due to the vessels unseaworthiness as she was put to sea in an unstable

Page
condition. It further alleged that the vessel was improperly manned and that its officers were grossly negligent in failing to take appropriate measures to proceed to a nearby port or beach after the vessel started to list. - FELMAN filed a motion to dismiss based on the affirmative defense that no right of subrogation in favor of PHILAMGEN was transmitted by the shipper, and that, in any event, FELMAN had abandoned all its rights, interests and ownership over MV Asilda together with her freight and appurtenances for the purpose of limiting and extinguishing its liability under Art. 587 of the Code of Commerce. - Trial court dismissed the complaint of PHILAMGEN. On appeal the Court of Appeals set aside the dismissal and remanded the case to the lower court for trial on the merits. FELMAN filed a petition for certiorari with this Court but it was subsequently denied on 13 February 1989. - Trial court rendered judgment in favor of FELMAN. It ruled that MV Asilda was seaworthy when it left the port of Zamboanga as confirmed by certificates issued by the Philippine Coast Guard and the shipowners surveyor attesting to its seaworthiness. Thus the loss of the vessel and its entire shipment could only be attributed to either a fortuitous event, in which case, no liability should attach unless there was a stipulation to the contrary, or to the negligence of the captain and his crew, in which case, Art. 587 of the Code of Commerce should apply. - CA ruled that MV Asilda was unseaworthy for being top- heavy as 2,500 cases of Coca-Cola softdrink bottles were improperly stowed on deck. Nonetheless, the appellate court denied the claim of PHILAMGEN on the ground that the assureds implied warranty of seaworthiness was not complied with. Perfunctorily, PHILAMGEN was not properly subrogated to the rights and interests of the shipper. Furthermore, respondent court held that the filing of notice of abandonment had absolved the shipowner/agent from liability under the limited liability rule. ISSUES 1. WON MV Asilda was seaworthy when it left the port of Zamboanga 2. WON the limited liability under Art. 587 of the Code of Commerce should apply 3. WON PHILAMGEN was properly subrogated to the rights and legal actions which the shipper had against FELMAN, the shipowner

INSURANCE 94
HELD 1. YES - MV Asilda was unseaworthy when it left the port of Zamboanga. We subscribe to the findings of the Elite Adjusters, Inc., and the Court of Appeals that the proximate cause of the sinking of MV Asilda was its being top-heavy. Contrary to the ship captains allegations, evidence shows that approximately 2,500 cases of softdrink bottles were stowed on deck. Several days after MV Asilda sank, an estimated 2,500 empty Coca-Cola plastic cases were recovered near the vicinity of the sinking. Considering that the ships hatches were properly secured, the empty Coca-Cola cases recovered could have come only from the vessels deck cargo. It is settled that carrying a deck cargo raises the presumption of unseaworthiness unless it can be shown that the deck cargo will not interfere with the proper management of the ship. However, in this case it was established that MV Asilda was not designed to carry substantial amount of cargo on deck. The inordinate loading of cargo deck resulted in the decrease of the vessels metacentric height thus making it unstable. The strong winds and waves encountered by the vessel are but the ordinary vicissitudes of a sea voyage and as such merely contributed to its already unstable and unseaworthy condition. 2. NO - The ship agent is liable for the negligent acts of the captain in the care of goods loaded on the vessel. This liability however can be limited through abandonment of the vessel, its equipment and freightage as provided in Art. 587. Nonetheless, there are exceptional circumstances wherein the ship agent could still be held answerable despite the abandonment, as where the loss or injury was due to the fault of the shipowner and the captain. The international rule is to the effect that the right of abandonment of vessels, as a legal limitation of a shipowners liability, does not apply to cases where the injury or average was occasioned by the shipowners own fault. 3. YES - The doctrine of subrogation has its roots in equity. It is designed to promote and to accomplish justice and is the mode which equity adopts to compel the ultimate payment of a debt by one who in justice, equity and good conscience ought to pay. Therefore, the payment made by PHILAMGEN to Coca-Cola Bottlers Philippines, Inc., gave the former the right to bring an action as subrogee against FELMAN. Having failed to rebut the presumption of fault, the liability of FELMAN for the loss of the 7,500 cases of 1-liter Coca-Cola softdrink bottles is inevitable. - Sec. 113 of the Insurance Code provides that (i)n every marine insurance upon a ship or freight, or freightage, or upon anything which is the subject of marine insurance, a warranty is implied that the ship is seaworthy. Under Sec. 114, a ship is seaworthy when reasonably fit to perform the service, and to encounter the ordinary perils of the voyage, contemplated by the parties to the policy. Thus it becomes the obligation of the cargo owner to look for a reliable common carrier which keeps its vessels in seaworthy condition. He may have no control over the vessel but he has full control in the selection of the common carrier that will transport his goods. He also has full discretion in the choice of assurer that will underwrite a particular venture. - In policies where the law will generally imply a warranty of seaworthiness, it can only be excluded by terms in writing in the policy in the clearest language. And where the policy stipulates that the seaworthiness of the vessel as between the assured and the assurer is admitted, the question of seaworthiness cannot be raised by the assurer without showing concealment or misrepresentation by the assured. - PHILAMGENs action against FELMAN is squarely sanctioned by Art. 2207 of the Civil Code which provides: Art. 2207. If the plaintiffs property has been insured, and he has received indemnity from the insurance company for the injury or loss arising out of the wrong or breach of contract complained of, the insurance company shall be subrogated to the rights of the insured against the wrongdoer or the person who has violated the contract. If the amount paid by the insurance company does not fully cover the injury or loss, the aggrieved party shall be entitled to recover the deficiency from the person causing the loss or injury. Disposition Petition is GRANTED. Respondent FELMAN SHIPPING LINES is ordered to pay petitioner PHILIPPINE AMERICAN GENERAL INSURANCE CO., INC., Seven Hundred Fifty-five Thousand Two Hundred and Fifty Pesos (P755,250.00) plus legal interest thereon counted from 29 November 1983, the date of judicial demand, pursuant to Arts. 2212 and 2213 of the Civil Code.

Page

PHILIPPINE MFTG. CO. v. UNION INSURANCE SOCIETY OF CANTON 42 PHIL 378 JOHNS; November 22, 1921 FACTS - The plaintiffs steel tank lighter was insured by defendant company for absolute total loss. As a result of a typhoon, the lighter sunk in Manila Bay. The plaintiff demanded payment from the defendant insurance company but the latter refused. The company asked the plaintiff to salvage the ship, which it was able to do so. - With the plaintiff able to raise the lighter, reconstruct it and placed it in commission, the defendant insurance company claims that it was only liable for a total absolute loss and that there was no total destruction of the lighter. - The trial court decided in favor of the defendant, saying that the policy only covered an actual total loss, not a constructive total loss. ISSUES 1. WON there was an absolute total loss that can be covered by the policy 2. WON the Marine Law of Great Britain applies HELD 1. YES - At the time that the lighter was at the bottom of the bay, it was of no value to the owner, thus there was an actual total loss. - The ship was sunk in July 1, 1918. After several futile attempts, it was finally raised on Sept. 20, 1918. It is faitr to assume that in its then condition much further time would be required to make the necessary repairs and install the new machinery before it could again be placed in commission. During that time the owner would be deprived of the use of its vessel or the interest on its investment. When those questions are considered the testimony is conclusive that the cost of salvage, repair and reconstruction was more than the original cost of the ship at the time the policy was issued. As found by the trial court, t is difficult to see how there could have been a more complete loss of the vessel than that which actually occurred. Upon the facts shown here, any other construction would nullify the statute and as applied to the conditions existing in the Manila

INSURANCE 95
Bay, this kind of policy would be worthless, and there would not be any consideration for the premium. 2. NO - The defendant argues that the policy contains the provision that it shall be of as force and effect as the surest writing or policy of insurance made in London. However, for such law to apply to our courts the existence of such law must be proven. It cannot apply when such proof is lacking. Nevertheless, in the English practice, a ship is a total loss when she has sustained such extensive damages that it would not be reasonably practical to repair her. Disposition Decision reversed delivered to petitioner, 403 were in bad order. The surveys showed that the bad order bags suffered spillage and loss later valued at P33,117.63. Petitioner filed a claim for said loss dated February 16, 1977 against respondent insurance company in the amount of P33,117.63 as the insured value of the loss. - Respondent insurance company rejected the claim alleging that assuming that spillage took place while the goods were in transit, petitioner and his agent failed to avert or minimize the loss by failing to recover spillage from the sea van, thus violating the terms of the insurance policy sued upon; and that assuming that the spillage did not occur while the cargo was in transit, the said 400 bags were loaded in bad order, and that in any case, the van did not carry any evidence of spillage. - Petitioner filed a complaint in the RTC against the insurance company seeking payment of the sum of P33,117.63 as damages plus attorney's fees and expenses of litigation. Insurance company denied all the material allegations of the complaint and raised several special defenses as well as a compulsory counterclaim. Insurance company filed a third-party complaint against respondents Ben Lines and broker. - RTC dismissed the complaint, the counterclaim and the third-party complaint with costs against the petitioner. Appealed in CA but denied. MFR was denied as well. ISSUE WON insurance company should be held liable even if the technical meaning in marine insurance of an insurance against all risk" is applied HELD YES - In Gloren Inc. vs. Filipinas Cia. de Seguros, 12 it was held that an all risk insurance policy insures against all causes of conceivable loss or damage, except as otherwise excluded in the policy or due to fraud or intentional misconduct on the part of the insured. It covers all losses during the voyage whether arising from a marine peril or not, including pilferage losses during the war. - In the present case, the "all risks" clause of the policy sued upon reads as follows: "5. This insurance is against all risks of loss or damage to the subject matter insured but shall in no case be deemed to extend to cover loss, damage, or expense proximately caused by delay

Page
or inherent vice or nature of the subject matter insured. Claims recoverable hereunder shall be payable irrespective of percentage." - The terms of the policy are so clear and require no interpretation. The insurance policy covers all loss or damage to the cargo except those caused by delay or inherent vice or nature of the cargo insured. It is the duty of the respondent insurance company to establish that said loss or damage falls within the exceptions provided for by law, otherwise it is liable therefor. - An "all risks" provision of a marine policy creates a special type of insurance which extends coverage to risks not usually contemplated and avoids putting upon the insured the burden of establishing that the loss was due to peril falling within the policy's coverage. The insurer can avoid coverage upon demonstrating that a specific provision expressly excludes the loss from coverage. - In this case, the damage caused to the cargo has not been attributed to any of the exceptions provided for nor is there any pretension to this effect. Thus, the liability of respondent insurance company is clear. Disposition the decision appealed from is hereby REVERSED AND SET ASIDE and another judgment is hereby rendered ordering the respondent Filipinas Merchants Insurance Company, Inc. to pay the sum of P33,117.63 as damages to petitioner with legal interest from the filing of the complaint, plus attorney's fees and expenses of litigation in the amount of P10,000.00 as well as the costs of the suit. FILIPINO MERCHANTS INS. CO. v. CA (supra p.19) ABOITIZ SHIPPING v. PHILAMGEN INSURANCE 179 SCRA 357 GANCAYCO; October 5, 1989 NATURE Petition for review on certiorari FACTS Marinduque Mining Industrial Corporation (Marinduque) shipped on board SS Arthur Maersk from Boston, U.S.A. a shipment of 1 skid carton parts for valves. The shipment was ordered from Jamesbury, Singapore PTE, LTD., which issued the cargo's packing list and Invoice number showing the contents of the carton. The Philippine Consulate in Singapore issued invoice for the shipment showing

CHOA TIEK SENG v. CA (FILIPINO MERCHANTS INSURANCE) 183 SCRA 223 GANCAYO; March 15, 1990 NATURE Appeal from a decision of the Court of Appeals FACTS - Petitioner imported some lactose crystals from Holland. - The importation involved fifteen (15) metric tons packed in 600 6-ply paper bags with polythelene inner bags, each bag at 25 kilos net. The goods were loaded at the port at Rotterdam in sea vans on board the vessel "MS Benalder' as the mother vessel, and thereafter aboard the feeder vessel "Wesser Broker V25" of respondent Ben Lines Container, Ltd. (Ben Lines for short). The goods were insured by the respondent Filipino Merchants' Insurance Co., Inc. (insurance company for short) for the sum of P98,882.35, the equivalent of US$8,765.00 plus 50% mark-up or US $13,147.50, against all risks under the terms of the insurance cargo policy. Upon arrival at the port of Manila, the cargo was discharged into the custody of the arrastre operator respondent E. Razon, Inc. (broker for short), prior to the delivery to petitioner through his broker. Of the 600 bags

INSURANCE 96
the contents and its total price of $39,419.60 and the freight and other charges of $2,791.73. When the cargo arrived in Manila, it was received and deposited in the office of Aboitiz Shipping Corporation (Aboitiz) for transhipment to Nonoc Island. - In July 1980, Marinduque, as consignee of the cargo, made a report that said cargo was pilfered on July 3, 1980 due to heavy rain at the Aboitiz terminal and that of the total value of the cargo of $42,209.33, only $7,412.00 worth remains of the cargo with the recommendation that the claim be made against Aboitiz. - The services of the Manila Adjusters and Surveyors Co. (Manila Adjusters) were engaged by the PhilAmerican General Insurance Co., Inc. (Phil Am) which came out with the report that the cargo in question, when inspected, showed that it was pilfered. A confirmatory report was submitted by the Manila Adjusters. - On August 11, 1980 Marinduque then filed a claim against Aboitiz in the amount of P246,430.80 representing the value of the pilfered cargo. On the same day Marinduque filed a claim for the same amount against the Phil-Am on the latter's policy. Phil-Am paid Marinduque the sum of P246,430.80 as insurer of the cargo. - Phil-Am then filed a complaint in RTC Manila against Aboitiz for recovery of same amount alleging that it has been subrogated to the rights of Marinduque. Complaint dismissed and MFR denied. CA reversed. MFR thereof was denied. Hence, this petition. ISSUE WON petitioner Aboitiz was properly held liable to the private respondent Phil-Am by the appellate court HELD YES - The questioned shipment is covered by a continuing open insurance coverage (which took effect after Sept. 1, 1975, as contained in Marine Open Policy No. 100184) from the time it was loaded aboard the SS Arthur Maersk in Boston, U.S.A. to the time it was delivered to the possession of petitioner at its offices at Pier 4 in Manila until it was pilfered when the great majority of the cargo was lost on July 3, 1980. Hence, petitioner Aboitiz was properly held liable to Phil-Am. Reasoning [a] Records of the case show that Phil-Am executed a continuous and open insurance coverage covering goods of Marinduque imported into and exported from the Philippines which took effect after Sept. 1, 1975, as contained in Marine Open Policy No. 100184. A similar insurance coverage was also executed by petitioner in favor of Marinduque for all its goods shipped or moved within the territorial limits of the Philippines also effective after Sept. 1, 1975 and contained in Marine Open Policy No. 100185. [b] TC in dismissing the complaint apparently relied on Marine Risk Note No. 017545 issued by private respondent Phil-Am only on July 28, 1980 after the shipment in question was already pilfered. Obviously TC mistook said Marine Risk Note as an insurance policy when it is NOT. It is only an acknowledgment or declaration of the private respondent confirming the specific shipment covered by its Marine Open Policy, the evaluation of the cargo and the chargeable premium. [c] The contention of the Aboitiz that it could not be liable for the pilferage of the cargo as it was stolen even before it was loaded on its vessel is untenable. Aboitiz received cargo when it arrived in Manila at its offices, and it was while in its possession and before loading it in its vessel that the cargo was pilfered. Its liability is clear. Disposition Petition DISMISSED. ORIENTAL ASSURANCE v. CA (PANAMA SAW MILL) 200 SCRA 459 MELENCIO-HERRERA; August 9, 1991 NATURE Petition for review on certiorari FACTS - Sometime in January 1986, private respondent Panama Sawmill Co., Inc. (Panama) bought, in Palawan, 1,208 pieces of apitong logs, with a total volume of 2,000 cubic meters. It hired Transpacific Towage, Inc., to transport the logs by sea to Manila and insured it against loss for P1-M with petitioner Oriental Assurance Corporation (Oriental Assurance). - While the logs were being transported, rough seas and strong winds caused damage to one of the two barges resulting in the loss of 497 pieces of logs out of the 598 pieces loaded thereon. - Panama demanded payment for the loss but Oriental Assurance refuse on the ground that its contracted liability was for "TOTAL LOSS ONLY."

Page
- Unable to convince Oriental Assurance to pay its claim, Panama filed a Complaint for Damages against Oriental Assurance before the Regional Trial Court. - RTC ordered Oriental Assurance to pay Panama with the view that the insurance contract should be liberally construed in order to avoid a denial of substantial justice; and that the logs loaded in the two barges should be treated separately such that the loss sustained by the shipment in one of them may be considered as "constructive total loss" and correspondingly compensable. CA affirmed in toto. ISSUE WON Oriental Assurance can be held liable under its marine insurance policy based on the theory of a divisible contract of insurance and, consequently, a constructive total loss HELD NO - The terms of the contract constitute the measure of the insurer liability and compliance therewith is a condition precedent to the insured's right to recovery from the insurer. Whether a contract is entire or severable is a question of intention to be determined by the language employed by the parties. The policy in question shows that the subject matter insured was the entire shipment of 2,000 cubic meters of apitong logs. The fact that the logs were loaded on two different barges did not make the contract several and divisible as to the items insured. The logs on the two barges were not separately valued or separately insured. Only one premium was paid for the entire shipment, making for only one cause or consideration. The insurance contract must, therefore, be considered indivisible. - More importantly, the insurer's liability was for "total loss only." A total loss may be either actual or constructive (Sec. 129, Insurance Code). An actual total loss is caused by: (a) A total destruction of the thing insured; (b) The irretrievable loss of the thing by sinking, or by being broken up; (c) Any damage to the thing which renders it valueless to the owner for the purpose for which he held it; or (d) Any other event which effectively deprives the owner of the possession, at the port of destination, of the thing insured. (Section 130, Insurance Code).

INSURANCE 97
- A constructive total loss is one which gives to a person insured a right to abandon, under Section 139 of the Insurance Code. This provision reads: SECTION 139. A person insured by a contract of marine insurance may abandon the thing insured, or any particular portion thereof separately valued by the policy, or otherwise separately insured, and recover for a total loss thereof, when the cause of the loss is a peril injured against, (a) If more than three-fourths thereof in value is actually lost, or would have to be expended to recover it from the peril; (b) If it is injured to such an extent as to reduce its value more than three-fourths; xxx xxx xxx - The requirements for the application of Section 139 of the Insurance Code, quoted above, have not been met. The logs involved, although placed in two barges, were not separately valued by the policy, nor separately insured. Resultantly, the logs lost in the damaged barge in relation to the total number of logs loaded on the same barge cannot be made the basis for determining constructive total loss. The logs having been insured as one inseparable unit, the correct basis for determining the existence of constructive total loss is the totality of the shipment of logs. Of the entirety of 1,208, pieces of logs, only 497 pieces thereof were lost or 41.45% of the entire shipment. Since the cost of those 497 pieces does not exceed 75% of the value of all 1,208 pieces of logs, the shipment cannot be said to have sustained a constructive total loss under Section 139(a) of the Insurance Code. Disposition judgment under review is SET ASIDE PAN MALAYAN INSURANCE v. CA (THE FOOD AND AGRICULTURAL ORGANIZATION OF THE UNITED NATIONS) 201 SCRA 382 REGALADO; September 5, 1991 FACTS - The Food and Agricultural Organization of the United Nations (hereinafter referred to as FAO), ntended and made arrangements to send to Kampuchea 1,500 metric petitions of IR-36 certified rice seeds to be distributed to the people for seedling purposes - LUZTEVECO was to ship the cargo amounting to US$83,325.92 in respect of one lot of 1,500 metric petitions winch is the subject of the present action. The cargo was loaded on board LUZTEVECO Barge No. LC-3000 and consisted of 34,122 bags of IR-36 certified rice seeds purchased by FAO from the Bureau of Plant Industry for P4,602,270.00 - FAO secured insurance coverage in the amount of P5,250,000.00 from petitioner, Pan Malayan Insurance Corporation - On June 16, 1980, FAO gave instructions to LUZTEVECO to leave for Vaung Tau, Vietnam to deliver the cargo which, by its nature, could not withstand delay because of the inherent risks of termination and/or spoilage. On the same date, the insurance premiums on the shipment was paid by FAO petitioner - On June 26, 1980, FAO was advised of the sinking of the barge in the China Sea, hence it informed petitioner thereof and, later, formally filed its claim under the marine insurance policy. On July 29, 1980, FAO was informed by LUSTEVECO of the recovery of the lost shipment, for which reason FAO formally filed its claim with LUZTEVECO for compensation of damage to its cargo - LUZTEVECO failed and refused to pay. Pan Malayan likewise failed to pay for the losses and damages sustained by FAO by reason of its inability to recover the value of the shipment from LUZTEVECO - Pan Malayan claims that part of the cargo was recovered and thus the claim by FAO was unwarranted. This is evidenced by two surveys upon the cargo wherein it was found that only around 78% was lost. - FAO filed a civil case against both LUZTEVECO and Pan Malayan. Trial court found in favor of FAO and ordered both to pay jointly and severally the full amount of the claim. This was affirmed by CA ISSUE 1. WON respondent court committed a reversible error in holding that the trial court is correct in holding that there is a total loss of the shipment HELD 1. NO - The law classifies loss into either total or partial. Total loss may be actual or absolute, or it may otherwise be constructive or technical. Petitioner submits that respondent court erred in ruling that there was total loss of the shipment despite the fact that only 27,922 bags of rice seeds out of 34,122 bags were rendered valueless to FAO and the shipment sustained only a loss of 78%. - FAO, however, claims that, for all intents and purposes, it

Page
has practically lost its total or entire shipment in this case, inclusive of expenses, premium fees, and so forth, despite the alleged recovery by defendant LUZTEVECO. As found by the court below and reproduced with approval by respondent court, FAO "has never been compensated for this total loss or damage, a fact which is not denied nor controverted - If there were some cargoes saved, by LUZTEVECO, private respondent abandoned it and the same was sold or used for the benefit of LUZTEVECO or Pan Malayan Corporation. Under Sections 129 and 130 of the New Insurance Code, a total loss may either be actual or constructive. In case of total loss in Marine Insurance, the assured is entitled to recover from the underwriter the whole amount of his subscription - SEC. 130. An actual total loss is caused by: (c) Any damage to the thing which renders it valueless to the owner for the purpose for which he held it; or (d) Any other event which effectively deprives the owner of the possession, at the port of destination of the thing insured. -as said and proven, the seeds were of fragile nature. And the wetting of said seeds affected the state of seeds. Thus rendering them useless for FAO. Although there were bags which were recovered, these were stained and not in the same condition it was brought in. in addition to this, FAO did not receive any compensation for said recovered bags as the same were distributed by LUZVETECO without authorization of FAO - the complete physical destruction of the subject matter is not essential to constitute an actual total loss. Such a loss may exist where the form and specie of the thing is destroyed, although the materials of which it consisted still exist (Great Western Ins. Co. vs. Fogarty, N.Y., 19 Wall 640, 22 L. Ed. 216), as where the cargo by the process of decomposition or other chemical agency no longer remains the same kind of thing as before (Williams vs. Cole, 16 Me. 207). - It is thus clear that FAO suffered actual total loss under Section 130 of the Insurance Code, specifically under paragraphs (c) and (d) thereof, recompense for which it has been denied up to the present -Section 135 of the Insurance Code explicitly provides that "(u)pon an actual total loss, a person insured is entitled to payment without notice of abandonment." This is a statutory adoption of a long standing doctrine in maritime insurance law that in case of actual total loss, the right of the insured to claim the

INSURANCE 98
whole insurance is absolute, without need of a notice of abandonment PHILIPPINE AMERICAN LIFE INSURANCE COMPANY v. CA (ELIZA PULIDO) 344 SCRA 260 GONZAGA-REYES; November 15, 2000 NATURE This petition for review on certiorari seeks to reverse the Decision of the Special Second Division of the Court of Appeals FACTS - On January 9, 1989, petitioner received from one Florence Pulido an application for life insurance, dated December 16, 1988, in the amount of P100,000.00 which designated her sister, herein private respondent, as its principal beneficiary. Because the insurance applied for was non-medical, petitioner did not require a medical examination and issued a policy on the sole basis of the application on February 11, 1989. On April 1992, petitioner received private respondents claim, which declared that the insured, Florence Pulido, died of acute pneumonia on September 10, 1991. - Petitioner withheld payment on the ground that the policy claimed under was void from the start for having been procured in fraud. It is petitioners contention that even before they received private respondents claim for death benefits, their investigation concerning the subject policy yielded the information that the insured, Florence Pulido, died in 1988, before the application for insurance on her life was made. While this was communicated to private respondent in a letter dated April 29, 1992, private respondent had already filed her claim earlier that month. In another letter dated July 27, 1992, however, petitioner confirmed to private respondent receipt of the claim papers and assured her that her case was being given preferential attention and prompt action. - Following the filing by private respondent of her claim, petitioner caused another investigation respecting the subject policy. Pursuant to the findings of this second investigation, petitioner stood by its initial decision to treat the policy as void and not to honor the claim. On November 9, 1992, private respondent enlisted the services of counsel in reiterating her claim for death benefits. Petitioner still refused to make payment and thus, this action. - Petitioner: the results of its investigations having indicated that the insured was already dead at the time the policy was applied for. It also counterclaimed for attorneys fees. The first report, prepared by one Dr. Benedicto Briones, was dated April 1, 1992, and had attached to it a questionnaire, responded to by one Ramon Piganto, who represented to be the brother-in-law of the insured and the barangay chairman of Cardiz, Bagulin, La Union. To the question Where does [Florence Pulido] reside now?, Piganto had replied that Florence Pulido used to live in Cardiz, but was dead since 1988. Pigantos statement was signed by him, and witnessed by his wife, Nenita Piganto. This report was petitioners basis for treating the disputed policy as void since April 1992, even before receipt of private respondents claim. ISSUE WON there was fraud (whether the insured, Florence Pulido, was in fact dead before the application for insurance on her life was made) HELD NO - This the lower courts had effected ruled on, upon a preponderance of the evidence duly received from both parties. We see no reversible error in the finding of both respondent court and the trial court in favor of the correctness of the entries in Certificate of Death, duly registered with the Local Civil Registrar of Bagulin, La Union, which declared that Florence Pulido died of acute pneumonia on September 10, 1991. Dr. Irineo Gutierrez, the Municipal Health Officer of Bagulin, La Union whose signature appeared in the death certificate, testified in addition that he ministered to the ailing Florence Pulido for two days immediately prior to her death. This fact is likewise noted in the death certificate. - Death certificates, and notes by a municipal health officer prepared in the regular performance of his duties, are prima facie evidence of facts therein stated. A duly-registered death certificate is considered a public document and the entries found therein are presumed correct, unless the party who contests its accuracy can produce positive evidence establishing otherwise. Petitioners contention that the death certificate is suspect because Dr. Gutierrez was not present when Florence Pulido died, and knew of Florences death only through Ramon Piganto, does not merit a conclusion of fraud. No motive was

Page
imputed to Dr. Gutierrez for seeking to perpetuate a falsity in public records. Petitioner was likewise unable to make out any clear motive as to why Ramon Piganto would purposely lie. Mere allegations of fraud could not substitute for the full and convincing evidence that is required to prove it. A failure to do so would leave intact the presumption of good faith and regularity in the performance of public duties, which was the basis of both respondent court and the trial court in finding the date of Florence Pulidos death to be as plaintiff-private respondent maintained. - We cannot likewise give credence to petitioners submission that the inconsistencies in the testimonies of the witnesses for plaintiff-private respondent are in themselves evidence of fraud. Such alleged inconsistencies are matters of credibility which had been ably passed upon by the lower court. - The absence of fraud, as a factual finding of the lower court adopted by the Court of Appeals, entirely consistent with the evidence on record, will not be reversed and, hence, is final and conclusive upon this Court. Disposition The instant petition is DENIED CHAPTER IX. SUBROGATION CLAIMS SETTLEMENT &

LONDRES v. NATIONAL LIFE INSURANCE 94 PHIL 627 BAUTISTA ANGELO; March 29, 1954 NATURE Appeal from a decision of the Court of First Instance of Manila ordering defendant to pay to plaintiff the sum of P3,000, Philippine currency, plus legal interest thereon from the time of the filing of the complaint until its full payment. FACTS - On April 14, 1943, the National Life Insurance Company of the Philippines issued a policy on the life of Jose C. Londres whereby it undertook to pay its beneficiary upon his death the sum of P3,000. All the premiums due under the policy were actually paid on their dates of maturity and the policy was in force when the insured died on February 7, 1945. Salvacion V. Londres, as beneficiary, demanded from the company the payment of the proceeds of the policy, and her demand having been refused, she instituted the present action against the company in the Court

INSURANCE 99
of First Instance of Manila. - Defendant in its answer denied, for lack of sufficient proof, the allegation that the insured died on February 7, 1945, and set up the following special defenses: (a) that plaintiff's claim is covered by the Moratorium Law; (b) that the policy having been issued during the Japanese occupation, it is presumed that its face value should be paid in Japanese currency, there being no provision in the policy from which can be inferred that the parties contemplated payment in any other currency; (c) that the money paid by the insured as premiums, together with the money received from other policy-holders, was all deposited by the defendant in the Philippine National Bank and said deposit was declared without value by Executive Order No. 49 of the President of the Philippines; and (d) that the policy having been issued under abnormal circumstances, it should be considered in the light of equity which does not permit anyone to enrich himself at the expense of another. Defendant, however, as a proof of good faith, offered to pay the value of the policy in accordance with the Ballantyne scale of values, or the sum of P2,400, Philippine currency. - It appears that the deceased took up the policy under consideration on April 15, 1943 for the sum of P3,000. All the premiums due under the policy were actually paid on their dates of maturity and the policy was in force when the insured died on February 7, 1945. On said date, the battle of the liberation of the City of Manila was still raging. While the northern part may have been liberated, not so the southern part, as shown from the very affidavits submitted by appellee wherein it was stated that on the aforesaid date, the insured, Jose Londres, and his two sons were taken by the Japanese soldiers from their house at Singalong Street and were massacred by their captors. It may therefore be said that the policy became due when the City of Manila was still under the yoke of the enemy and became payable only after liberation which took place on March 10, 1945 when President Osmena issued Proclamation No. 6 following the restoration of the civil government by General Douglas Mac Arthur. And we say that the policy became payable only after liberation even if it matured sometime before, because before that eventuality the insurance company, appellant herein, was not yet in a position to pay the value of the policy for the simple reason that it had not yet reopened. ISSUE WON the amount of P3,000 which appellant bound itself to pay to the insured under the policy upon his death should be paid in accordance with the present currency or should be adjusted under the Ballantyne scale of values HELD YES, present currency. Reasoning - In the case of Rutter vs. Esteban, 93 Phil., 68, the Moratorium Law was declared invalid and unconstitutional. - During those days of liberation, while the people were rejoicing because of the happy event, the banks, the insurance companies, and for that matter other commercial and business firms, were still feeling the adverse effects of the sudden fall of values and were uncertain and apprehensive as to the manner the readjustment would be made by the new Government. It is for this reason that the beneficiary, after realizing the truth about the death of her husband, and after gathering evidence to substantiate his death, had difficulty in effecting the collection of her claim from the insurance company because at that time it had not yet reopened for business purposes. Although the record does not disclose the exact date on which the insurance company reopened for this purpose, this Court can take judicial notice that it only did so after liberation. At that time the legal tender was already the present currency. - As final plea, appellant invokes equity in its favor in view of the nullification of the deposits made by it with the Philippine National Bank of all fiat money received from its policyholders, which money was declared without value by Executive Order No. 49 of the President of the Philippines. Appellant claims that, considering the unexpected circumstances that developed, the indemnity to be paid by it should be suffered by it under Article 307 of the Code of Commerce which provides: "When the deposits are of cash, with a specification of the coins constituting them, . . . the increase or reduction which their value may suffer shall be for the account of the depositor." Appellant, by entering into an insurance contract, cannot claim, if it suffers loss, that the beneficiary cannot enrich herself at its expense. This is a risk attendant to any wagering contract. One who gambles and loses cannot be heard to complain of his loss. To appellant, we can only repeat the following admonition:

Page
"The parties herein gambled and speculated on the date of the termination of the war and the liberation of the Philippines by the Americans. This can be gleaned from the stipulation about redemption, particularly that portion to the effect that redemption could be effected not before the expiration of one year from June 24, 1944. This kind of agreement is permitted by law. We find nothing immoral or unlawful in it." (Gomez vs. Tabia) Disposition Wherefore, the decision appealed from is affirmed, with costs against appellant.

VDA. DE FERNANDEZ v. NATIONAL INSURANCE CO OF THE PHILS 105 PHIL 59 ENDENCIA; January 27, 1959

LIFE

NATURE Appeal from CFI decision applying the Ballantyne scale of values upon the proceeds of life insurance taken and maturing during the Japanese occupation but claimed after liberation FACTS - National Life Insurance Company (NLIC) insured J. Fernandezs life for P10,000 upon his payment of P444 from July 15, 1944 to July 14, 1945 - The insured died on November 2, 1944, while the policy was in force - After more than 7 years, in 1952, Atty de la Torre, representing the benficiaries of the policy, informed the company that Fernandez had died in 1944, and claimed the proceeds of the policy. The company said that the status of the policies issued during the Japanese occupation was still pending consideration before the courts. NLIC said that because the policy matured upon the insureds death in November, 1944, they should compute the value of their claim under the Ballantyne scale of values (which would amount only to P500) - beneficiaries commenced suit, and the lower court sustained the stand of the company, dismissed the complaint. - beneficiaries maintain that the obligation of the company to pay accrued not upon the death of Fernandez, but only upon the receipt and approval by

INSURANCE 100
the company, on proof of death of the insured, which was in 1954. The policy reads: National Life Insurance Company of the Philippine hereby agrees to pay at its Home Office, Manila, Ten Thousand Pesos to Juan D. Fernandez (hereinafter called the insured) on the 15th day of July, 1964, if the Insured is living and this Policy is in force, or upon receipt and approved at its Office of due proofs of the title of the claimant and of the prior death of the Insured while this Policy is in force to Teresa Duat Vda. De Fernandez, Maria T. and Manuela Fernandez, mother and sisters respectively of the Insured (Hereinafter called the Beneficiary) subject to the right of the Insured to change the beneficiary as stated on the second page of this Policy. - The above stipulation is apparently based on Sec. 91-A of the Insurance Law which provides as follows: The proceeds of a life insurance policy shall be paid immediately upon maturity of the policy, unless such proceeds are made payable in installments or a as an annuity, in which case the installments or annuities shall be paid as they become due: Provided, however, That in case of a policy maturing by the death of the insured, the proceeds thereof shall be paid within sixty days after presentation of the claim and filing of the proof of the death of the insured. Refused to pay the claim within the time prescribed herein will entitle the beneficiary to collect interest on the proceeds of the policy for the duration of the delay at the rate of six per centum per annum, unless such failure or refusal to pay is based on the ground that the claim is fraudulent . . . . - Based on the foregoing provision of law and the aforequoted stipulation as well as on the allegation that the filing of proof of death by the beneficiaries is a condition precedent of the demandability of the obligation of the insurer to pay the proceeds, appellants claim that they should be paid P10,000 in Philippine currency and not under the Ballantyne scale of values. ISSUE WON the policy matured upon the death of the insured HELD YES Ratio In life insurance, the policy matures either upon the expiration of the term set forth therein, or upon his death occuring at any time prior to the expiration of such stipulated term, in which case, the proceeds are payable to his beneficiaries within sixty days after their filing of proof of death. Reasoning - The sixty day period fixed by law within which to pay the proceeds after presentation of proof of death Is merely procedural in nature, evidently to determine the exact amount to be paid and the interest thereon to which the beneficiaries may be entitled to collect in case of unwarranted refusal of the company to pay, and also to enable the insurer to verify or check on the fact of death which it may even validly waive. It is the happening of the suspensive condition of death that renders a life policy matured, and not ht efiling of proof of death which, as above stated, is merely procedural. The insured having died during the Japanese occupation, the proceeds of his policy should be adjusted accordingly, for The rule is already settled that where a debtor could have paid his obligation at any time during the Japanese occupation, payment after liberation must be adjusted in accordance with the Ballantyne schedule (De Asis vs. Agdamag, among other cases). (Collaboration is defined as the acts of working together in a joint project. Disposition Judgment affirmed TIO KHE CHIO v. CA (EASTERN ASSURANCE & SURETY) 202 SCRA 119 FERNAN; September 30, 1991 FACTS - Petitioner Tio Khe Chio imported 1,000 bags of fishmeal valued at $36,000.30 from Agro Impex, S.A. Dallas, Texas, U.S.A. The goods were insured with respondent EASCO and shipped on board the M/V Peskov, a vessel owned by Far Eastern Shipping Company. When the goods reached Manila, they were found to have been damaged by sea water which rendered the fishmeal useless. Petitioner filed a claim with EASCO and Far Eastern Shipping. Both refused to pay. Whereupon, petitioner sued them before the then Court of First Instance of Cebu for damages. EASCO, as the insurer, filed a counterclaim against the petitioner for the recovery of the unpaid insurance premiums. - The trial court rendered judgment in favor of petitioner. The judgment became final as to EASCO but the shipping company appealed to the Court of

Page
Appeals and was absolved from liability by the said court. - The trial court, upon motion by petitioner, issued a writ of execution against EASCO. The sheriff enforcing the writ reportedly fixed the legal rate of interest at 12%. Respondent EASCO moved to quash the writ alleging that the legal interest to be computed should be 6% per cent per annum in accordance with Article 2209 of the Civil Code. The trial court denied EASCO's motion. On appeal, the Court of Appeals reversed the trial courts denial of EASCOs motion and ruled that the applicable interest is 6% per annum. Hence, this petition. ISSUE WON the applicable rate of interest is 12% per annum HELD NO - Sections 243 and 244 of the Insurance Code apply only when there is an unjustified refusal or withholding of payment on the insureds claim. In this case, EASCO's refusal to settle the claim to Tio Khe Chio was based on some ground which, while not sufficient to free it from liability under its policy, nevertheless is sufficient to negate any assertion that in refusing to pay, it acted unjustifiably. Simply put, the said provisions of the Insurance Code are not pertinent to the instant case. They apply only when the court finds an unreasonable delay or refusal in the payment of the claims. - Circular No. 416 of the Central Bank, which raised the legal rate of interest from 6% to 12% per annum refers only to loans or forbearances of money, goods or credits and court judgments thereon but not to court judgments for damages arising from injury to persons and loss of property which does not involve a loan. Clearly, the applicable law is Article 2209 of the Civil Code. - And in the light of the fact that the contending parties did not allege the rate of interest stipulated in the insurance contract, the legal interest was properly pegged by the Appellate Court at 6% per cent. CATHAY INSURANCE v. CA(LUGAY) 174 SCRA 11 GRINO-AQUINO; June 5, 1989 FACTS - Petitioners are 6 insurance companies that issued fire insurance policies for the total sum of P4,000,000

INSURANCE 101
to the Cebu Filipina Press owned by Emilia Chan Lugay. The fire policies described the insured property as "stocks of Printing materials, papers and general merchandise usual to the Assured's trade" stored in a one-storey building of strong materials housing the Cebu Filipina Press located at UNNO Pres. Quirino cor. Don V. Sotto Sts., Mabolo, Cebu City. The co-insurers were indicated in each of the policies. All, except one policy (Paramount's), were renewals of earlier policies issued for the same property. - On December 18, 1981, the Cebu Filipina Press was razed by electrical fire together with all the stocks and merchandise stored in the premises. On January 15, 1982, Lugay submitted sworn Statements of Loss and Formal Claims to the insurers, through their adjusters. She claimed a total loss of P4,595,000. - After nearly 10 months of waiting, she sued to collect on December 15, 1982. The insurance companies denied liability, alleging violation of certain conditions of the policy, misdeclaration, and even arson which was not seriously pressed for, come the pre-trial, the petitioners offered to pay 50% of her claim, but she insisted on full recovery. - Trial court rendered judgment in her favor ordering the insurers to pay her a total of P4,000,000 as indemnity, P48,000 representing expenses of the plaintiff, a separate amount of 20% of the P4,000,000 representing fees of counsel, interests at the rate of twice the ceiling being prescribed by the Monetary Board starting from the time when the case was filed, and finally, with costs. CA affirmed. ISSUES 1. WON the insured's cause of action had already accrued before she filed her complaint 2. WON sufficient proofs of loss had been presented by the insured 3. WON the private respondents claim for loss was inflated 4. WON lower court erred in awarding damages to the private respondent in the form of interest equivalent to double the interest ceiling set by the Monetary Board 5. WON attorney's fees awarded were exorbitant HELD 1. YES - As the fire which destroyed the Cebu Filipina Press occurred on December 19, 1981 and the proofs of loss were submitted from January 15, 1982 through June 21, 1982 in compliance with the adjusters' numerous requests for various documents, payment should have been made within 90 days thereafter (Sec 243), or on or before September 21, 1982. Hence, when the assured filed her complaint on December 15, 1982, her cause of action had already accrued. 2. YES - There is no merit in the petitioners' contention that the proofs of loss were insufficient because Lugay failed to comply with the adjuster's request for the submission of her bank statements. Condition No. 13 of the policy does not require the insured to produce her bank statements. Therefore, the insured was not obligated to produce them and the insurers had no right to ask for them. Condition No. 13 was prepared by the insurers themselves, hence, it should be taken most strongly against them. 3. NO - Both the trial court and the CA noted that the proofs were ample and more than enough for defendant insurers to do a just assessment supporting the 1981 fire claim for an amount exceeding four million pesos. 4. NO - The award of double interest on the claim is lawful and justified under Sections 243 and 244 of the Insurance Code which provide: Sec. 243 Refusal or failure to pay the loss or damage within the time prescribed herein will entitle the assured to collect interest on the proceeds of the policy for the duration of the delay at the rate of twice the ceiling prescribed by the Monetary Board. Sec. 244 In case of any litigation for the enforcement of any policy or contract of insurance, it shall be the duty of the Commissioner or the Court, as the case may be, to make a finding as to whether the payment of the claim of the insured has been unreasonably denied or withheld; and in the affirmative case, the insurance company shall be adjudged to pay damages which shall consist of attorney's fees and other expenses incurred by the insured person by reason of such unreasonable denial or withholding of payment plus interest of twice the ceiling prescribed by the Monetary Board of the amount of claim due the insured. - The petitioners' contention that the charging of double interest was improper because no unreasonable delay in the processing of the fire claim was proven is refuted by the trial court's explicit finding that "there was a delay that was not reasonable in processing the claim and doing payments". Under Section 244, a prima facie evidence

Page
of unreasonable delay in payment of the claim is created by the failure of the insurer to pay the claim within the time fixed in both Sec. 242 and 243 of the IC. - In view of the not insubstantial value of the private respondent's claims and the considerable time and effort expended by them and their counsel in prosecuting these claims for the past 8 years, attorney's fees were properly awarded to the private respondents. 5. YES - An award equivalent to 10% of the proceeds of the policies would be more reasonable than the 20% awarded by the trial court and the CA. Disposition Decision of the CA AFFIRMED with MODIFICATION. NODA v. CRUZ 151 SCRA 227 FERNAN; June 22, 1987 NATURE Petition to review Commissioner decision of the Insurance

FACTS - In 1977, Noda obtained from Zenith Insurance Corporation 2 fire insurance policies: [1] No. F-03724 with a face value of P30k covering the goods and stocks in trade in his business establishment at the market site in Mangagoy, Bislig, Surigao del Sur and [2] No. F-03734 with a face value in the aggregate amount of P100k and consisting of Item 1 for P40k on household furniture, fixtures, fittings and other personal effects, and Item 2 for P60k on stocks in trade usual to petitioner's retail business situated in a two-storey building at 039 Barreda St., Mangagoy, Bislig, Surigao del Sur. - While both policies were in force, fire destroyed petitioner's insured properties at the market site on September 5, 1977 and at Barreda St. on November 9, 1977. - When petitioner failed to obtain indemnity on his claims from Zenith, he filed a complaint with the Insurance Commission praying that Zenith be ordered to pay him P130kj representing the value of the 2 policies insured by respondent with interest at 12% per annum, plus damages, attorney's fees and other expenses of litigation. ...

INSURANCE 102
- Zenith interposed that petitioner had no cause of action; that Policy No. F-03724 was not in full force and effect at the time of the fire because the premium on the policy was not paid; that Zenith's liability under Policy No. F-03734, if any, was limited to P15,472.50 in view of the co-insurance; and that petitioner failed to substantiate his claim as to the value of the goods reputedly destroyed by fire. - While the case was pending, Zenith settled petitioners fire loss claim under Item 1 of Policy No. 03734 in the amount of P15,472.50. - Insurance Commissioner allowed petitioner to recover under said policy and ordered Zenith to pay him the amount of P20k with legal interest from the date the complaint was filed, including P1k as attorney's fees but excluding the actual, moral and exemplary damages prayed for. As for petitioner's claim under Policy No. F-03734, she held that in view of the payment of P15,472.50 to petitioner, Zenith had fully discharged its liability under said policy which covered furniture, fixtures, fittings and other personal belongings of petitioner. - In allowing recovery under Policy No. F-03734, Commissioner placed much weight on the final report prepared by Dela Merced Adjustment Corporation, an independent fire, marine and casualty adjuster contracted by Zenith to investigate the claims of its various policyholders. Said report concluded that "the sound value of P26,666.67 represented the whole loss and damage" incurred by petitioner, but with the application of the three-fourths loss clause, Zenith's liability was reduced to P20k. ISSUES 1. WON Insurance Commissioner erred in denying petitioner's demand for P60k under Item 2 of Policy No. F-03734 2. WON Insurance Commissioner erred in not awarding in favor of petitioner exemplary damages for Zenith's unjustified and wanton refusal to pay petitioner's claim under the said two insurance contracts HELD 1. YES - To prove the existence of the stocks in trade covered by Policy No. F-03734, petitioner offered his testimony and that of his wife as well as documentary exhibits. The foregoing evidence for petitioner preponderantly showed the presence of some P590k worth of goods in his retail store during the fire of November 9, 1977. - While the insurer, and the Insurance Commissioner for that matter, have the right to reject proofs of loss if they are unsatisfactory, they may not set up for themselves an arbitrary standard of satisfaction. Substantial compliance with the requirements will always be deemed sufficient. - Zenith introduced in evidence the final report on Policy No. F-03734 submitted by its own adjuster, Dela Merced Adjustment Corporation. Respondent Commissioner however ignored such report, reasoning that with regard to Item 2 of Policy No. F03734 the claim for loss of the stocks in trade was not successfully proven in view of petitioner's failure to present evidence; that the adjuster's report deserved scant consideration since the allegations therein were not substantiated, and that said report did not even make a recommendation for payment. - A scrutiny of the abovementioned adjuster's report reveals that together with the formal demand for full indemnity, petitioner submitted his income tax return for 1978, purchase invoices, certification from his suppliers as to his purchases, and other supporting papers. The report even took into account the appraisals of the other adjusters and concluded that the total loss sustained by petitioner in his household effects and stocks in trade reached P379,302.12. But after apportioning said amount among petitioner's six different insurers [the co-insurance being known to Zenith], the liability of Zenith was placed at P60,592.10. It therefore recommended that Zenith pay the petitioner the amount of P60, 592.10. - Said document was offered as evidence by Zenith itself and could very well be considered as an admission of its liability up to the amount recommended. Being in the nature of an admission against interest, it is the best evidence which affords the greatest certainty of the facts in dispute. Respondent Commissioner should not have perfunctorily dismissed that particular evidence as a worthless piece of paper. 2. NO - There is no showing that Zenith, in contesting payment, had acted in a wanton, oppressive or malevolent manner to warrant the imposition of corrective damages. Disposition Zenith Insurance Corporation ordered to pay petitioner Norman R. Noda the sum of P60,592.10 with legal interest from the filing of the complaint until full payment, but deducting therefrom

Page
the amount of P15,472.50 which it had earlier paid to petitioner. DELSAN TRANSPORT, INC. v. CA (AMERICAN HOME ASSURANCE) 369 SCRA 24 DE LEON, JR; November 15, 2001 NATURE A petition for review on certiorari of the decision of CA. FACTS - Caltex entered into a contract of affreightment with the petitioner, Delsan Transport Lines, Inc. (petitioner), for a period of one year whereby the said common carrier agreed to transport Caltexs industrial fuel oil from the Batangas-Bataan Refinery to different parts of the country. Delsan took on board its vessel, MT Maysun, 2,277.314 kiloliters of industrial fuel oil of Caltex to be delivered to the Caltex Oil Terminal in Zamboanga City. The shipment was insured by American Home Assurance Corporation (respondent). - August 14, 1986: MT Maysun set sail from Batangas for Zamboanga City. The vessel sank in the early morning of August 16, 1986 near Panay Gulf in the Visayas taking with it the entire cargo of fuel oil. - Respondent paid Caltex P5,096,635.57 representing the insured value of the lost cargo. Exercising its right of subrogation under Article 2207 of the New Civil Code, the private respondent demanded of the petitioner the same amount it paid to Caltex. Delsan refused to pay, forcing American home to file a case for collection in the RTC. - RTC found that the vessel, MT Maysun, was seaworthy to undertake the voyage, and that the incident was caused by an unexpected inclement weather condition or force majeure, thus exempting the common carrier from liability for the loss of its cargo. - CA reversed RTC decision on the basis of evidence from PAG-ASA that there were no 20 ft. waves in the area. CA ruled that the petitioner is liable on its obligation as common carrier to respondent insurance company as subrogee of Caltex. Petitioners Claim > In every marine insurance upon a ship or freight, or freightage, or upon any thing which is the subject of marine insurance there is an implied warranty by the

INSURANCE 103
shipper that the ship is seaworthy.10 When private respondent paid Caltex the value of its lost cargo, the act of the private respondent is equivalent to a tacit recognition that the ill-fated vessel was seaworthy. Respondents Comment > American Home Assurance is entitled to payment by its right of subrogation. ISSUES 1. WON payment made by American Home to Caltex for the insured value of the lost cargo amounted to an admission that the vessel was seaworthy, thus precluding any action for recovery against the petitioner 2. WON MT Maysun was seaworthy at the time of the voyage (outline topic) 3. WON non-presentation of the marine insurance policy bars the complaint for recovery of sum of money for lack of cause of action HELD 1. NO Ratio The fact of payment grants American Home the subrogatory right which enables it to exercise legal remedies that would otherwise be available to Caltex as owner of the lost cargo against the petitioner common carrier. Reasoning Art. 2207. (Civil Code) If the plaintiffs property has been insured, and he has received indemnity from the insurance company for the injury or loss arising out of the wrong or breach of contract complained of, the insurance company shall be subrogated to the rights of the insured against the wrongdoer or the person who has violated the contract. If the amount paid by the insurance company does not fully cover the injury or loss, the aggrieved party shall be entitled to recover the deficiency from the person causing the loss or injury. - The right of subrogation is designed to promote and to accomplish justice and is the mode which equity adopts to compel the ultimate payment of a debt by one who in justice and good conscience ought to pay. It is not dependent upon, nor does it grow out of, any privity of contract or upon written assignment of claim. It accrues simply upon payment by the insurance company of the insurance claim.
10

Page
2. NO Ratio Seaworthiness relates to a vessels actual condition. Neither the granting of classification or the issuance of certificates establishes seaworthiness. Reasoning - Common carriers are bound to observe extraordinary diligence in the vigilance over the goods and for the safety of passengers transported by them, according to all the circumstances of each case. There is no liability if the loss, destruction or deterioration is by force majeure. - The tale of strong winds and big waves by the said officers of the petitioner however, was effectively rebutted and belied by the weather report from PAGASA. MT Maysun sank with its entire cargo for the reason that it was not seaworthy. There was no squall or bad weather or extremely poor sea condition in the vicinity when the said vessel sank. - Petitioner may not escape liability by presenting in evidence certificates that tend to show that at the time of dry-docking and inspection by the Philippine Coast Guard MT Maysun, was fit for voyage. These pieces of evidence do not necessarily take into account the actual condition of the vessel at the time of the commencement of the voyage. At the time of dry-docking and inspection, the ship may have appeared fit. The certificates issued, however, do not negate the presumption of unseaworthiness triggered by an unexplained sinking. - Authorities are clear that diligence in securing certificates of seaworthiness does not satisfy the vessel owners obligation. Also securing the approval of the shipper of the cargo, or his surveyor, of the condition of the vessel or her stowage does not establish due diligence if the vessel was in fact unseaworthy, for the cargo owner has no obligation in relation to seaworthiness. 3. NO Ratio The presentation in evidence of the marine insurance policy is not indispensable in this case before the insurer may recover from the common carrier the insured value of the lost cargo in the exercise of its subrogatory right. The subrogation receipt, by itself, is sufficient to establish not only the relationship of respondent as insurer and Caltex, as the assured shipper of the lost cargo of industrial fuel oil, but also the amount paid to settle the insurance claim. The right of subrogation accrues simply upon payment by the insurance company of the insurance claim. Disposition Petition is denied, and the decision of the CA is affirmed.

FINMAN GENERAL ASSURANCE INOCENCIO 179 SCRA 480 FELICIANO; November 15, 1989

CORP

v.

FACTS - Pan Pacific is a recruitment and employment agency. It posted surety bond issued by Finman General Assurance and was granted license to operate by POEA. - Inocencio, Palero, Cardones, Hernandez filed with POEA complaints against Pan Pacific for violation of Labor Code and for refund of placement fees. POEA Administrator motu propio impleaded Finman as surety for Pan Pacific. - Pan Pacific moved out and no notice of transfer was furnished to POEA as required. POEA considered that constructive service of complaints had been effected. - Finman denied liability and said that POEA had no jurisdiction over surety bonds; jurisdiction is vested in Insurance Commission - Finman had not violated Labor Code - Complainants have no cause of action against Finman - Amounts claimed were paid as deposits and not as placement fees. - POEA Administrator issued Order that respondents should pay. Finman appealed to Secretary of Labor. Secretary upheld the POEA order. ISSUE WON Finman can be held liable for complainants claims against Pan Pacific HELD YES - Under Insurance Code, liability of surety in a surety bond is joint and several with the principal obligor. - Conditions of a bond specified and required in the provisions of a statute providing for submission of the bond, are incorporated into all bonds tendered under that statute even though not set out in printers ink. - POEA held and Secretary of Labor affirmed that Pan Pacific had violated Labor Code, and at least one of the conditions for the grant and continued use of the

Sec. 113 Insurance Code

INSURANCE 104
recruitment license. POEA and Secretary of Labor can require Pan Pacific to refund the placement fees and to impose the fine. - If Pan Pacific is liable, and if Finman is solidarily liable with Pan Pacific, then Finman is liable both to private respondents and to POEA. Cash and surety bonds are required from recruitment companies as means of ensuring prompt and effective recourse against such companies when held liable. Public policy will be effectively negated if POEA and the DoLE were held powerless to compel a surety company to make good on its solidary undertaking. EAGLE STAR INSURANCE CO LTD v. CHIA YU 96 PHIL 696 REYES; March 31, 1955 NATURE Certiorari FACTS - Atkin, Kroll & Co., loaded on the S. S. Roeph Silverlight owned and operated by Leigh Hoegh & Co., A/S, of San Francisco California, 14 bales of assorted underwear valued at P8,085.23 consigned to Chia Yu in the City of Manila. - The shipment was insured against all risks by Eagle Star Ins. Co. of San Francisco, California, under a policy issued to the shipper and by the latter assigned to the consignee. - The vessel arrived in Manila but of the 14 bales (a.k.a. freights =p) consigned to Chia Yu only 10 were delivered to him as the remaining 3 could not be found.3 of those delivered were also found damaged to the extent of 50 per cent. -Chia Yu claimed indemnity for the missing and damaged bales. But the claim was declined, first, by the carrier and afterward by the insurer, whereupon Chia Yu brought the present action against both, including their respective agents in the Philippines. - An action was filed at the CFI after more than 2 years after delivery of the damaged bales and the date when the missing bales should have been delivered, the action was resisted by the Atkins and Eagle Star principally on the ground of prescription. -TC favored Chia Yu and CA affirmed. *** CARRIERs defense of prescription is made to rest on the following stipulation of the bill of lading: In any event the carrier and the ship shall be discharged from all liability in respect of loss or damage unless suit is brought within one year after the delivery of the goods or the date when the goods should have been delivered. (This stipulation is but a repetition of a provision in the CA 65 which says that bills of lading covering shipments from the US to the Phils should be brought w/in one year after the delivery of the goods or the date when the goods should have been delivered to hold the carrier liable.) *** INSURERs claim of prescription is founded upon the terms of the policy and not upon the bill of lading. (But in our jurisdiction, as per A1144, prescription is 10 years after action accrues.) No suit action on this Policy, for the recovery of any claim, shall be sustainable in any Court of law or equity unless the insured shall have fully complied with all the terms and conditions of this Policy nor unless commenced with twelve (12) months next after the happening of the loss . . . ISSUE WON ATKIN s action has prescribed HELD NO - Being contrary to the law of the forum, the stipulation in the policy cannot be given effect as it would reduce the period allowed the insured for bringing his action to less than one year (because the prescription period begins from the happening of the loss and that before any suit could be sustained the insured shall have to comply with the terms and conditions of the policy first TF lessening the period to less than a year. ) - Insular Government vs. Frank(13 Phil. 236)~ "matters respecting a remedy, such as the bringing of suit, admissibility of evidence, and statute of limitations, depend upon the law of the place where the suit is brought" TF any policy clause repugnant to this amendment to the Insurance Act cannot be given effect in an action in our courts. SEC. 61-A. (Insurance Code) ~ Any condition, stipulation or agreement in any policy of insurance, limiting the time for commencing an action thereunder to a period of less than one year from the time when the cause of action accrues, is void. - The prescription clause could be harmonized with section 61-A of the Insurance Act by taking it to mean that the time given the insured for bringing his suit is twelve months after the cause of action accrues.

Page
- If so, when did the cause of action accrue? Chia Yus action did not accrue until his claim was finally rejected by the insurance company. This is because, before such final rejection, there was no real necessity for bringing suit. - As the policy provides that the insured should file his claim, first, with the carrier and then with the insurer, he had a right to wait for his claim to be finally decided before going to court. - Furthermore, there is nothing in the record to show that the claim was rejected in the year 1947, either by the insurance company in London or its settling agents in the Philippines. - For the purpose of this action, Chia Yu's claim was considered to have been finally rejected by the insurer on April 22, 1948. Having been filed within twelve months form that date, the action cannot be deemed to have prescribed even on the supposition that the period given the insured for bringing suit under the prescriptive clause of the policy is twelve months after the accrual of the cause of action. - Contractual limitations contained in insurance policies are regarded with extreme jealousy by courts and will be strictly construed against the insurer and should not be permitted to prevent a recovery when their just and honest application would not produce that result. (46 C. J. S. 273.) Disposition Judgment appealed from is REVERSED with respect to the carrier and its agents but AFFIRMED with respect to the insurance company and its agents.

ACCFA v. ALPHA INSURANCE 24 SCRA 151 REYES; July 29, 1968 FACTS - In order to guarantee the Asingan Farmers' Cooperative Marketing Association, Inc. (FACOMA) against loss on account of "personal dishonesty, amounting to larceny or estafa of its SecretaryTreasurer, Ladines, the appellee, Alpha Insurance & Surety Company had issued, on 14 February 1958, its bond, No. P-FID-15-58, for the sum of P5,000 with said Ladines as principal and the appellee as solidary surety. On the same date, the Asingan FACOMA assigned its rights to the appellant, Agricultural Credit

INSURANCE 105
Cooperative and Financing Administration (ACCFA for short), with approval of the principal and the surety. - During the effectivity of the bond, Ladines converted and misappropriated, to his personal benefit, some P11,513.22 of the FACOMA funds, of which P6,307.33 belonged to the ACCFA. Upon discovery of the loss, ACCFA immediately notified in writing the survey company on 10 October 1958, and presented the proof of loss within the period fixed in the bond; but despite repeated demands the surety company refused and failed to pay. Whereupon, ACCFA filed suit against appellee on 30 May 1960. - Defendant Alpha Insurance & Surety Co., Inc., (now appellee) moved to dismiss the complaint for failure to state a cause of action, giving as reason that (1) the same was filed more than one year after plaintiff made claim for loss, contrary to the eighth condition of the bond, providing as follows: EIGHT LIMITATION OF ACTION: No action, suit or proceeding shall be had or maintained upon this Bond unless the same be commenced within one year from the time of making claim for the loss upon which such action, suit or proceeding, is based, in accordance with the fourth section hereof. (2) the complaint failed to show that plaintiff had filed civil or criminal action against Ladines, as required by conditions 4 and 11 of the bond; and (3) that Ladines was a necessary and indispensable party but had not been joined as such. - At first, the Court of First Instance denied dismissal; but, upon reconsideration, the court reversed its original stand, and dismissed the complaint on the ground that the action was filed beyond the contractual limitation period. Hence, this appeal. ISSUE WON the provision of a fidelity bond that no action shall be had or maintained thereon unless commenced within one year from the making of a claim for the loss upon which the action is based, is valid, in view of Section 61-A of the Insurance Act invalidating stipulations limiting the time for commencing an action thereon to less than one year from the time the cause of action accrues HELD NO - A fidelity bond is, in effect, in the nature of a contract of insurance against loss from misconduct, and is governed by the same principles of interpretation. Consequently, the condition of the bond in question, limiting the period for bringing action thereon, is subject to the provisions of Section 61-A of the Insurance Act (No. 2427), as amended by Act 4101 of the pre-Commonwealth Philippine Legislature, prescribing that: SEC. 61-A: A condition, stipulation or agreement in any policy of insurance, limiting the time for commencing an action thereunder to a period of less than one year from the time when the cause of action accrues is void. - Since a "cause of action" requires, as essential elements, not only a legal right of the plaintiff and a correlative obligation of the defendant but also "an act or omission of the defendant in violation of said legal right," the cause of action does not accrue until the party obligated refuses, expressly or impliedly, to comply with its duty (in this case, to pay the amount of the bond). The year for instituting action in court must be reckoned, therefore, from the time of appellee's refusal to comply with its bond; it can not be counted from the creditor's filing of the claim of loss, for that does not import that the surety company will refuse to pay. In so far, therefore, as condition eight of the bond requires action to be filed within one year from the filing of the claim for loss, such stipulation contradicts the public policy expressed in Section 61-A of the Philippine Insurance Act. - Condition eight of the bond, therefore, is null and void, and the appellant is not bound to comply with its provisions. The discouraging of unnecessary litigation must be deemed a rule of public policy, considering the unrelieved congestion in the courts. As a consequence of the foregoing, action may be brought within the statutory period of limitation for written contracts (New Civil Code, Article 1144). ANG v. FULTON FIRE INSURANCE CO. 2 SCRA 945 LABRADOR; July 31, 1961 NATURE Appeal from judgment of the CFI ordering the defendant Fulton Fire Insurance Co. to pay the plaintiffs the sum of P10,000.00, with interest, and an additional sum of P2,000.00 as attorney's fees, and costs. FACTS - The stocks of general merchandise in the store of the Ang spouses are insured with Fulton. While the

Page
insurance was in force, fire destroyed the goods. The Angs filed their first claim immediately after the fire. - Their claim was denied on April 6, 1956. They received notice on April 19, 1956. - The Angs brought an action against the agent on May 11, 1956. The court denied the suit and the mfr on Sept. 3 and 12, 1957. - The Angs filed against Fulton on May 26, 1958. - There was a clause in the policy: 13.If the claim be in any respect fraudulent, or if any false declaration is made or used in support thereof, or if any fraudulent means or devices are used by the Insured or any one acting on his behalf to obtain any benefit under this Policy, or, if the loss or damage be occasioned by the wilful act or with the connivance of the Insured, or, if the claim be made and rejected and an action or suit be not commenced within twelve months after such rejection or (in case of arbitration taking place in pursuance of the 18th condition of this Policy) within twelve months after the arbitrator or arbitrators or umpire shall have made their award all benefit under this Policy shall be forfeited." ISSUE WON the suit against the agent tolled the prescription period, such that the filing against Fulton was only 9 months after the claim was rejected HELD NO - The bringing of the action against the Paramount Surety & Insurance Company, the agent of the defendant company, cannot have any legal effect except that of notifying the agent of the claim. Beyond such notification, the filing of the action can serve no other purpose. There is no law giving any effect to such action upon the principal. Besides, there is no condition in the policy that the action must be filed against the agent, and the Court can not by interpretation extend the clear scope of the agreement beyond what is agreed upon by the parties. - Their contract is the law between the parties, and their agreement that an action on a claim denied by the insurer must be brought within one year from the denial, governs, not the rules on the prescription of actions. Disposition The judgment appealed from is hereby set aside and the case dismissed, with costs against plaintiffs-appellees.

INSURANCE 106
TRAVELLERS INSURANCE & SURETY CORP. v. CA (MENDOZA) 272 SCRA 536 HERMOSISIMA, JR; May 22, 1997 NATURE The petition herein seeks the review and reversal of the decision of respondent Court of Appeals affirming in toto the judgment of the Regional Trial Court in an action for damages filed by private respondent Vicente Mendoza, Jr. as heir of his mother who was killed in a vehicular accident. FACTS -an old lady was hit by a taxicab. The taxicab was later identified and a case was filed against the driver and owner. Later, an amendment was filed to include the insurance company. RTC and CA ordered that the owner, driver as well as the insurance company be held solidarily liable. ISSUE WON RTC and CA erred HELD YES - Where the contract provides for indemnity against liability to third persons, then third persons to whom the insured is liable can sue the insurer. Where the contract is for indemnity against actual loss or payment, then third persons cannot proceed against the insurer, the contract being solely to reimburse the insured for liability actually discharged by him thru payment to third persons, said third persons' recourse being thus limited to the insured alone. But in the case at bar, there was no contract shown. What then was the basis of the RTC and the CA to say that the insurance contract was a third-party liability insurance policy? Consequently, the trial court was confused as it did not distinguish between the private respondent's cause of action against the owner and the driver of the Lady Love taxicab and his cause of action against petitioner. The former is based on torts and quasi-delicts while the latter is based on contract. - Even assuming arguendo that there was such a contract, private respondent's cause of action can not prevail because he failed to file the written claim mandated by the Insurance Code (before it was amended-action must be brought within six months from date of the accident (this is whats applicable here) ; after amendment"action or suit for recovery of damage due to loss or injury must be brought in proper cases, with the Commissioner or the Courts within one year from denial of the claim, otherwise the claimant's right of action shall prescribe" ). He is deemed, under this legal provision, to have waived his rights as against petitioner-insurer. Disposition petition granted SUN INSURANCE v. CA (supra p.57) COASTWISE v. CA (supra p.70) CEBU SHIPYARD v. WILLIAM LINES (supra p.3) MANILA MAHOGANY MANUFACTURING CORP v. CA (ZENITH INSURANCE CORP) 154 SCRA 652 PADILLA; October 12, 1987 NATURE Petition to review CA decision ordering Manila Mahogany Manufacturing Corporation to pay Zenith Insurance Corporation P5,000 with 6% annual interest, attorney's fees, and costs of suit FACTS - From 6 March 1970 to 6 March 1971, MLA MAHOGANY insured its Mercedes Benz 4-door sedan with ZENITH. - On 4 May 1970, the insured vehicle was bumped and damaged by a truck owned by San Miguel Corporation. For the damage caused, ZENITH paid MLA MAHOGANY P5,000 in amicable settlement. MLA MAHOGANY's general manager executed a Release of Claim, subrogating respondent company to all its right to action against San Miguel Corporation. - On 11 Dec 1972, ZENITH wrote Insurance Adjusters, Inc. to demand reimbursement from San Miguel. Insurance Adjusters, Inc. refused reimbursement, alleging that San Miguel had already paid petitioner P4,500, as evidenced by a cash voucher and a Release of Claim executed by the General Manager of petitioner. ZENITH thus demanded from petitioner reimbursement of the sum of P4,500 paid by San Miguel. - City Court ordered petitioner to pay respondent P4,500. - CFI affirmed the City Court's decision in toto.

Page
- CA affirned CFI, with the modification that petitioner was to pay the total amount of P5,000 it had earlier received from ZENITH. Petitioners Claims > It is not bound to pay P4,500, and much more, P5,000 to ZENITH as the subrogation in the Release of Claim it executed in favor of respondent was conditioned on recovery of the total amount of damages petitioner had sustained. Since total damages were valued by petitioner at P9,486.43 and only P5,000 was received by petitioner, MLA MAHOGANY argues that it was entitled to go after San Miguel to claim the additional P4,500. > It cites Art. 220711 and Art. 130412 of the Civil Code, and claims a preferred right to retain the amount coming from San Miguel, despite the subrogation in favor of ZENITH. Respondents Arguments > There was no qualification to its right of subrogation under the Release of Claim executed by petitioner, the contents having expressed all intents and purposes of the parties. ISSUE WON the insurer may recover the sum of P5,000 HELD YES Ratio Since the insurer can be subrogated to only such rights as the insured may have, should the insured, after receiving payment from the insurer, release the wrongdoer who caused the loss, the insurer loses his rights against the latter. But in such a case, the insurer will be entitled to recover from the insured whatever it has paid to the latter, unless the release was made with the consent of the insurer. Reasoning - Although petitioners right to file a deficiency claim against San Miguel is with legal basis, without prejudice to the insurer's right of subrogation, nevertheless, when Manila Mahogany executed
11

Article 2207: If the plaintiff's property has been insured, and he has received indemnity from the insurance company for the injury or loss arising out of the wrong or breach of contract complained of the insurance company shall be subrogated to the rights of the insured against the wrongdoer or the person who has violated the contract. If the amount paid by the insurance company does not fully cover the injury or loss the aggrieved party shall be entitled to recover the deficiency from the person causing the loss or injury. 12 Article 1305: A creditor, to whom partial payment has been made, may exercise his right for the remainder, and he shall be preferred to the person who has been subrogated in his place in virtue of the partial payment of the same credit.

INSURANCE 107
another release claim discharging San Miguel from "all actions, claims, demands and rights of action that now exist or hereafter arising out of or as a consequence of the accident" after the insurer had paid the proceeds of the policy - the compromise agreement of P5,000 being based on the insurance policy - the insurer is entitled to recover from the insured the amount of insurance money paid. Since petitioner by its own acts released San Miguel, thereby defeating private respondents right of subrogation, the right of action of petitioner against the insurer was also nullified. - As held in Phil. Air Lines v. Heald Lumber Co., under Art. 2207, the real party in interest with regard to the portion of the indemnity paid is the insurer and not the insured. SUBROGATION: The right of subrogation can only exist after the insurer has paid the insured, otherwise the insured will be deprived of his right to full indemnity. If the insurance proceeds are not sufficient to cover the damages suffered by the insured, then he may sue the party responsible for the damage for the remainder. To the extent of the amount he has already received from the insurer enjoys the right of subrogation. Disposition Petition DENIED. Judgment appealed from is AFFIRMED with costs against petitioner. PIONEER INSURANCE v. CA (BORDER MACHINERY & HEAVY EQUIPMENT INC) 175 SCRA 668 GUTIERREZ, JR.; July 28, 1989 NATURE Petitions for review on certiorari of a decision of the CA FACTS - In 1965, Jacob S. Lim was engaged in the airline business as owner-operator of Southern Air Lines (SAL), a single proprietorship. -On May 17, 1965, at Tokyo, Japan, Japan Domestic Airlines (JDA) and Lim entered into and executed a sales contract for the sale and purchase of two (2) DC-3A Type aircrafts and one (1) set of necessary spare parts for the total agreed price of US $109,000.00 to be paid in installments. - On May 22, 1965, Pioneer Insurance and Surety Corporation, as surety, executed and issued its Surety Bond No. 6639 in favor of JDA, in behalf of its principal, Lim, for the balance price of the aircrafts and spare parts. -Border Machinery and Heavy Equipment Company, Inc. (Bormaheco), Francisco and Modesto Cervantes (Cervanteses) and Constancio Maglana contributed some funds used in the purchase of the above aircrafts and spare parts. They executed two (2) separate indemnity agreements in favor of Pioneer, one signed by Maglana and the other jointly signed by Lim for SAL, Bormaheco and the Cervanteses. - On June 10, 1965, Lim doing business under the name and style of SAL executed in favor of Pioneer as deed of chattel mortgage as security for the latter's suretyship in favor of the former. It was stipulated therein that Lim transfer and convey to the surety the two aircrafts. - Lim defaulted on his subsequent installment payments prompting JDA to request payments from the surety. - Pioneer paid a total sum of P298,626.12. - Pioneer then filed a petition for the extrajudicial foreclosure of the said chattel mortgage before the Sheriff of Davao City. - The Cervanteses and Maglana, however, filed a third party claim alleging that they are co-owners of the aircrafts, - On July 19, 1966, Pioneer filed an action for judicial foreclosure with an application for a writ of preliminary attachment against Lim and respondents, the Cervanteses, Bormaheco and Maglana. **Maglana, Bormaheco and the Cervanteses filed cross-claims against Lim alleging that they were not privies to the contracts signed by Lim and, by way of counterclaim, sought for damages for being exposed to litigation and for recovery of the sums of money they advanced to Lim for the purchase of the aircrafts in question. (this constitutes the second petition but will no longer be discussed because it is not relevant to the topic) - After trial on the merits, a decision was rendered holding Lim liable to pay Pioneer but dismissed Pioneer's complaint against all other defendants. - CA modified the trial court's decision in that the plaintiffs complaint against all the defendants (including Lim) was dismissed. ISSUE WON the petition of Pioneer Insurance and Surety Corporation against all defendants was rightly dismissed

Page

HELD YES - Both the TC and CA made the finding that Pioneer reinsured its risk of liability under the surety bond it had executed in favor of JDA, collected the proceeds of such reinsurance in the sum of P295,000, and paid with the said amount the bulk of its alleged liability to JDA under the said surety bond. The total amount paid by Pioneer to JDA is P299,666.29. Since Pioneer has collected P295,000.00 from the reinsurers, the uninsured portion of what it paid to JDA is the difference between the two amounts, or P3,666.28. This is the amount for which Pioneer may sue defendants, assuming that the indemnity agreement is still valid and effective. But since the amount realized from the sale of the mortgaged chattels are P35,000.00 for one of the airplanes and P2,050.00 for a spare engine, or a total of P37,050.00, Pioneer is still overpaid by P33,383.72. Therefore, Pioneer has no more claim against defendants. - The payment to the petitioner made by the reinsurers was not disputed. Considering this admitted payment, the only question was the effect of payment made by the reinsurers to the petitioner - In general a reinsurer, on payment of a loss acquires the same rights by subrogation as are acquired in similar cases where the original insurer pays a loss (Universal Ins. Co. v. Old Time Molasses Co.). - The rules of practice in actions on original insurance policies are in general applicable to actions or contracts of reinsurance (Delaware, Ins. Co. v. Pennsylvania Fire Ins. Co.). - Hence the applicable law is Article 2207 of the new Civil Code, to wit: Art. 2207. If the plaintiffs property has been insured, and he has received indemnity from the insurance company for the injury or loss arising out of the wrong or breach of contract complained of, the insurance company shall be subrogated to the rights of the insured against the wrongdoer or the person who has violated the contract. If the amount paid by the insurance company does not fully cover the injury or loss, the aggrieved party shall be entitled to recover the deficiency from the person causing the loss or injury - If a property is insured and the owner receives the indemnity from the insurer, it is provided in said article that the insurer is deemed subrogated to the rights of the insured against the wrongdoer and if the amount paid by the insurer does not fully cover the

INSURANCE 108
loss, then the aggrieved party is the one entitled to recover the deficiency. Evidently, under this legal provision, the real party in interest with regard to the portion of the indemnity paid is the insurer and not the insured (. Air Lines, Inc. v. Heald Lumber Co., and Manila Mahogany Manufacturing Corporation v. Court of Appeals) - It is clear from the records that Pioneer sued in its own name and not as an attorney-in-fact of the reinsurer. Accordingly, the appellate court did not commit a reversible error in dismissing the petitioner's complaint as against the respondents for the reason that the petitioner was not the real party in interest in the complaint and, therefore, has no cause of action against the respondents. Disposition Petitions dismissed. Questioned decision of CA affirmed. PAN MALAYAN INSURANCE CORPORATION v. CA (FABIE, HER UNKNOWN DRIVER) 184 SCRA 54 CORTES, April 3, 1990 NATURE PETITION to review the decision of the Court of Appeals FACTS - Pan Malayan Insurance Company (Panmalay) insured the Mitsubishi Colt Lancer car registered in the name of Canlubang Automotive Resources Corporation (Canlubang) under its motor vehicle insurance policy. Among the provisions of the policy was a own-damage clause whereby Panmalay agrees to indemnify Canlubang in cases of damage caused by accidental collision or overturning, or collision or overturning consequent upon mechanical breakdown or consequent upon wear and tear. - On 1985, the insured car was sideswept and damaged by a car owned by Erlinda Fabie, driven by an unknown driver who fled the scene. Panmalay, in accordance with the policy, defrayed the cost of repair of the insured car and was subrogated to the rights of Canlubang against the driver and owner of the pickup. Panmalay then filed a complaint for damages with RTC Makati against Erlinda Fabie and her driver on the grounds of subrogation, with the latter failing and refusing to pay their claim. Fabie filed a Motion for Bill of Particulars. - RTC: dismissed complaint for lack of cause of action (payment by PANMALAY of CANLUBANG's claim under the "own damage" clause of the insurance policy was an admission by the insurer that the damage was caused by the assured and/or its representatives) Panmalay appealed - CA: dismissed appeal, affirmed RTC (applying the ejusdem generis rule held that Section III-1 of the policy, which was the basis for settlement of CANLUBANG's claim, did not cover damage arising from collision or overturning due to the negligence of third parties as one of the insurable risk) ISSUE WON the insurer PANMALAY may institute an action to recover the amount it had paid its assured in settlement of an insurance claim against private respondents as the parties allegedly responsible for the damage caused to the insured vehicle, in accordance with A2207, NCC HELD YES Ratio Article 2207 of the Civil Code is founded on the well-settled principle of subrogation. If the insured property is destroyed or damaged through the fault or negligence of a party other than the assured, then the insurer, upon payment to the assured, will be subrogated to the rights of the assured to recover from the wrongdoer to the extent that the insurer has been obligated to pay. Payment by the insurer to the assured operates as an equitable assignment to the former of all remedies which the latter may have against the third party whose negligence or wrongful act caused the loss. The right of subrogation is not dependent upon, nor does it grow out of, any privity of contract or upon written assignment of claim. It accrues simply upon payment of the insurance claim by the insurer Exceptions (1) if the assured by his own act releases the wrongdoer or third party liable for the loss or damage, from liability, the insurer's right of subrogation is defeated; (2) where the insurer pays the assured the value of the lost goods without notifying the carrier who has in good faith settled the assured's claim for loss, the settlement is binding on both the assured and the insurer, and the latter cannot bring an action against the carrier on his right of subrogation;

Page
(3) where the insurer pays the assured for a loss which is not a risk covered by the policy, thereby effecting "voluntary payment", the former has no right of subrogation against the third party liable for the loss Reasoning - Both TC and CA are incorrect. ON TC: Own damage (not found in the insurance policy) simply meant that Panmalay had assumed to reimburse the cost for repairing the damage to the insured vehicle. Its different from Third Party Liability coverage (liabilities arising from the death of or bodily injuries suffered by 3rd parties) and from Property Damage coverage (liabilities from damage caused by insured vehicle to properties of 3rd parties) ON CA: the terms of a contract are to be construed according to the sense and meaning of the terms which the parties thereto have used. In the case of property insurance policies, the evident intention of the contracting parties, i.e., the insurer and the assured, determine the import of the various terms and provisions embodied in the policy. It is only when the terms of the policy are ambiguous, equivocal or uncertain, such that the parties themselves disagree about the meaning of particular provisions, that the courts will intervene. In such an event, the policy will be construed by the courts liberally in favor of the assured and strictly against the insurer - Both Panmalay and Canlubang had the same interpretation regarding the coverage of insured risk regarding accidental collision or overturning to include damages caused by 3rd party to Canlubang so it was improper for CA to ascribe meaning contrary to the clear intention and understanding of the parties. - Court on several occasions defined accident or accidental as taking place without ones foresight or expectation, an event that proceeds from an unknown cause, or is an unusual effect of a known cause and, therefore, not expected [Dela Cruz v. Capital Insurance & Surety Co.] The concept "accident" is not necessarily synonymous with the concept of "no fault". It may be utilized simply to distinguish intentional or malicious acts from negligent or careless acts of man. - damage/loss to insured vehicle due to negligence of 3rd parties not listed as exceptions to coverage in the insurance policy - Interpretation given by Panmalay is more in keeping with rationale behind rules on interpretation of insurance contracts in favor of assured or beneficiary: indemnity or payment

INSURANCE 109
- EVEN if voluntarily indemnified Canlubang, as interpreted by TC: the insurer who may have no rights of subrogation due to "voluntary" payment may never. theless recover from the third party responsible for the damage to the insured property under Article 1236 of the Civil Code. [Sveriges Angfartygs Assurans Forening v. Qua Chee Gan] Disposition the present petition is GRANTED. Petitioner's complaint for damages against private respondents is hereby REINSTATED. Let the case be remanded to the lower court for trial on the merits. FIREMAN'S FUND INSURANCE COMPANY JAMILA & COMPANY, INC. 70 SCRA 323 AQUINO; April 1976 v. of the complaint on the ground of lack of cause of action. Its basis for its contention were: (1) that the complaint did not allege that Firestone, pursuant to the contractual stipulation quoted in the complaint, had investigated the loss and that Jamila was represented in the investigation and (2) that Jamila did not consent to the subrogation of Fireman's Fund to Firestone's right to get reimbursement from Jamila and its surety. The lower court in its order of dismissal had sustained the second ground. - Jamila in its MR invoked the first ground which had never been passed upon by the lower court. But the lower court in its order granting Jamila's motion for reconsideration, completely ignored that first ground. It reverted to the second ground which was relied upon in its order previous order. The lower court reiterated its order, stating that Fireman's Fund had no cause of action against Jamila because Jamila did not consent to the subrogation. The court did not mention Firestone, the co-plaintiff of Fireman's Fund. - Firestone and Fireman's Fund filed an MR on the ground that Fireman's Fund was suing on the basis of legal subrogation whereas the lower court erroneously predicated its dismissal order on the theory that there was no conventional subrogation because the debtor's consent was lacking. The plaintiffs cited article 2207 of the Civil Code which provides that "if the plaintiff's property has been insured, and he has received indemnity from the insurance company for the injury or loss arising out of the wrong or breach of contract complained of, the insurance company shall be subrogated to the rights of the insured against the wrongdoer or the person who has violated the contract". - The lower court denied plaintiff's motion. They filed a second MR, calling the lower court's attention to the fact that the issue of subrogation was of no moment because Firestone, the subrogor, is a party-plaintiff and could sue directly Jamila in its own right. Without resolving that contention, the lower court denied plaintiffs' second MR. ISSUE WON the complaint of Firestone and Fireman's Fund states a cause of action against Jamila HELD YES - Fireman's Fund's action against Jamila is squarely sanctioned by article 2207. As the insurer, Fireman's Fund is entitled to go after the person or entity that

Page
violated its contractual commitment to answer for the loss insured against. The trial court erred in applying to this case the rules on novation. The plaintiffs in alleging in their complaint that Fireman's Fund "became a party in interest in this case by virtue of a subrogation right given in its favor by" Firestone, were not relying on the novation by change of creditors as contemplated in articles 1291 and 1300 to 1303 of the Civil Code but rather on article 2207. - Article 2207 is a restatement of a settled principle of American jurisprudence. Subrogation has been referred to as the doctrine of substitution. It is an arm of equity that may guide or even force one to pay a debt for which an obligation was incurred but which was in whole or in part paid by another. - Subrogation is founded on principles of justice and equity, and its operation is governed by principles of equity. It rests on the principle that substantial justice should be attained regardless of form, that is, its basis is the doing of complete, essential, and perfect justice between all the parties without regard to form. Subrogation is a normal incident of indemnity insurance. Upon payment of the loss, the insurer is entitled to be subrogated pro tanto to any right of action which the insured may have against the third person whose negligence or wrongful act caused the loss. The right of subrogation is of the highest equity. The loss in the first instance is that of the insured but after reimbursement or compensation, it becomes the loss of the insurer. Although many policies including policies in the standard form, now provide for subrogation, and thus determine the rights of the insurer in this respect, the equitable right of subrogation as the legal effect of payment inures to the insurer without any formal assignment or any express stipulation to that effect in the policy. Stated otherwise, when the insurance company pays for the loss, such payment operates as an equitable assignment to the insurer of the property and all remedies which the insured may have for the recovery thereof. That right is not dependent upon, nor does it grow out of, any privity of contract, or upon written assignment of claim, and payment to the insured makes the insurer an assignee in equity. On the other hand, Firestone is really a nominal party in this case. It had already been indemnified for the loss which it had sustained. Obviously, it joined as a party-plaintiff in order to help Fireman's Fund to recover the amount of the loss from Jamila and First Quezon City. Firestone had tacitly assigned to

FACTS - Jamila & Co., Inc. or the Veterans Philippine Scouts Security Agency contracted to supply security guards to Firestone. Jamila assumed responsibility for the acts of its security guards. The First Quezon City Insurance Co., Inc. executed a bond in the sum of P20,000 to guarantee Jamila's obligations under that contract. - On May 18, 1963 properties of Firestone valued at P11,925 were lost allegedly due to the acts of its employees who connived with Jamila's security guard. Fireman's Fund, as insurer, paid to Firestone the amount of the loss. Fireman's Fund was subrogated to Firestone's right to get reimbursement from Jamila. Jamila and its surety, First Quezon City failed to pay the amount of the loss in spite of repeated demands. Upon defendant's motions, the lower court dismissed the complaint as to Jamila on the ground that there was no allegation that it had consented to the subrogation and, therefore, Fireman's Fund had no cause of action against it. It also dismissed the complaint as to First Quezon City on the ground of res judicata. It appears that the same action was previously filed in a civil case which was dismissed because of the failure of the same plaintiffs and their counsel to appear at the pre-trial. - Upon an MR, the lower court set aside its order of dismissal and sustained plaintiff's contention that there was no res judicata as to First Quezon City because the civil case was dismissed without prejudice. However, due to inadvertence, the lower court did not state in its order why it set aside its prior order dismissing the complaint with respect to Jamila. Jamilla had originally moved for the dismissal

INSURANCE 110
Fireman's Fund its cause of action against Jamila for breach of contract. Sufficient ultimate facts are alleged in the complaint to sustain that cause of action. TABACALERA v. NORTH FRONT SHIPPING 272 SCRA 527 BELLOSILLO; May 16, 1997 FACTS - 20,234 sacks of corn grains valued at P3.5M were shipped on board North Front 777, defendants vessel. The cargo was consigned to Republic Flour Mills Corp. under Bill of Lading No. 001 and insured with Tabacalera, Prudential Guarantee & Assurance, and New Zealand Insurance. - Republic Flour was advised of the vessels arrival in Manila, but did not immediately commence the unloading operations. Unloading was sometimes stopped due to varying weather and sometimes for no apparent reason. Unloading was only completed 20 days after the arrival of the barge; by then, the cargo was short 26.333 metric tons and the rest was already moldy and deteriorating. - Analyses showed that the deterioration was caused by moisture content from salt water, which could be arrested by drying. However, Republic Flour rejected the entire cargo and demanded that defendant North Front Shipping pay the damages suffered by it. The demands were unheeded and the insurance companies were obliged to pay Republic Flour P2,189,433 - By virtue of the insurance companies payment, they were subrogated to the rights of Republic Flour. Petitioners filed a complaint against North Front Shipping, claiming the loss was exclusively attributable to the latters fault and negligence. Having surveyed the vessel, it was found that the barge had cracks in its bodega. The hatches on the crates of grain were not sealed and the tarpaulins used in covering them were not new, contrary to North Front Shippings claims. North Front Shipping reiterated that the barge was inspected prior to loading and found seaworthy and were issued a permit to sail by the Coast Guard. They further averred that the grains were farm wet and not properly dried before loading. - The court dismissed the complaint, ruling that the contract entered into was a charter-party agreement; as such, only ordinary diligence in the care of the goods was required of North Front Shipping. ISSUE WON defendant is required to observe extraordinary diligence in its vigilance over the goods it transports HELD YES - As a corporation engaged in the business of transporting cargo offering its services indiscriminately to the public, it is without a doubt a common carrier. As such, it has the burden of proving that it observed extraordinary diligence to avoid responsibility for the lost cargo. The clean bill of lading it issued disprove the master of the vessels claim that the grains were farm wet when loaded. If they were wet, the master of the vessel should have known that the grains would eventually deteriorate when sealed in hot compartments in hatches of a ship and should have undertaken precautionary measures to avoid this. The arrival of the goods at the place of destination in bad order makes a prima facie case against the common carrier, which must prove its non-liability. - While petitioners presented evidence of the vessels bad shape and a laboratory analysis revealing that the grains were contaminated with salt water, defendants failed to rebut said arguments or even endeavor to establish that the loss, destruction or deterioration was due to a fortuitous event; an act/omission of the owner of the goods; the character of the goods or defects in their packing; or an order or act of a competent public authority. - However, Republic Flour is also found to be guilty of contributory negligence for not immediately staring the unloading operations and for providing no explanation for the delay. As such, it should share at least 40% of the loss. Disposition The decision of the CA is REVERSED and SET ASIDE PHILIPPINE AMERICAN LIFE INSURANCE COMPANY v. CA (ELIZA PULIDO) 344 SCRA 360 GONZAGA-REYES; November 15, 2000 NATURE This petition for review on certiorari seeks to reverse the Decision of the Special Second Division of the Court of Appeals FACTS

Page
- On January 9, 1989, petitioner received from one Florence Pulido an application for life insurance, dated December 16, 1988, in the amount of P100,000.00 which designated her sister, herein private respondent, as its principal beneficiary. Because the insurance applied for was non-medical, petitioner did not require a medical examination and issued a policy on the sole basis of the application on February 11, 1989. On April 1992, petitioner received private respondents claim, which declared that the insured, Florence Pulido, died of acute pneumonia on September 10, 1991. - Petitioner withheld payment on the ground that the policy claimed under was void from the start for having been procured in fraud. It is petitioners contention that even before they received private respondents claim for death benefits, their investigation concerning the subject policy yielded the information that the insured, Florence Pulido, died in 1988, before the application for insurance on her life was made. While this was communicated to private respondent in a letter, private respondent had already filed her claim earlier that month. In another letter, however, petitioner confirmed to private respondent receipt of the claim papers and assured her that her case was being given preferential attention and prompt action. - Petitioner caused another investigation respecting the subject policy. Pursuant to the findings of this second investigation, petitioner stood by its initial decision to treat the policy as void and not to honor the claim. On November 9, 1992, private respondent enlisted the services of counsel in reiterating her claim for death benefitsPetitioner still refused to make payment and thus, this action. ISSUE WON lower court erred in holding that there was no fraud HELD - The records bear out that since the onset of this case, the main issue has always been whether there was fraud in the obtainment of the disputed policy, or put differently, whether the insured, Florence Pulido, was in fact dead before the application for insurance on her life was made. This the lower courts had effected ruled on, upon a preponderance of the evidence duly received from both parties. We see no reversible error in the finding of both respondent court and the trial court in favor of the correctness of

INSURANCE 111
the entries in Certificate of Death, duly registered with the Local Civil Registrar of Bagulin, La Union, which declared that Florence Pulido died of acute pneumonia on September 10, 1991. Dr. Irineo Gutierrez, the Municipal Health Officer of Bagulin, La Union whose signature appeared in the death certificate, testified in addition that he ministered to the ailing Florence Pulido for two days immediately prior to her death. This fact is likewise noted in the death certificate. - Death certificates, and notes by a municipal health officer prepared in the regular performance of his duties, are prima facie evidence of facts therein stated. A duly-registered death certificate is considered a public document and the entries found therein are presumed correct, unless the party who contests its accuracy can produce positive evidence establishing otherwise. Petitioners contention that the death certificate is suspect because Dr. Gutierrez was not present when Florence Pulido died, and knew of Florences death only through Ramon Piganto, does not merit a conclusion of fraud. No motive was imputed to Dr. Gutierrez for seeking to perpetuate a falsity in public records. Petitioner was likewise unable to make out any clear motive as to why Ramon Piganto would purposely lie. Mere allegations of fraud could not substitute for the full and convincing evidence that is required to prove it. A failure to do so would leave intact the presumption of good faith and regularity in the performance of public duties, which was the basis of both respondent court and the trial court in finding the date of Florence Pulidos death to be as plaintiff-private respondent maintained. - We cannot likewise give credence to petitioners submission that the inconsistencies in the testimonies of the witnesses for plaintiff-private respondent are in themselves evidence of fraud. Such alleged inconsistencies are matters of credibility which had been ably passed upon by the lower court. Disposition the instant petition is DENIED ST.PAUL FIRE & MARINE INSURANCE CO v. MACONDRAY & CO INC 70 SCRA 122 ANTONIO; March 25, 1976 FACTS - Winthrop Products, Inc. shipped aboard the SS "Tai Ping", owned and operated by Wilhelm Wilhelmsen, 218 cartons and drums of drugs and medicine, with the freight prepaid, which were consigned to Winthrop-Stearns, Inc. Barber Steamship Lines, Inc., agent of Wilhelm Wilhelmsen issued Bill of Lading No. 34, in the name of Winthrop Products, Inc. as shipper, with arrival notice in-Manila to consignee WinthropStearns, Inc. The shipment was insured by the shipper against loss and/or damage with the St. Paul Fire & Marine Insurance Company. - The SS "Tai Ping" arrived at the Port of Manila and discharged its aforesaid shipment into the custody of Manila Port Service, the arrastre contractor for the Port of Manila. The said shipment was discharged complete and in good order with the exception of one (1) drum and several cartons which were in bad order condition. Because consignee failed to receive the whole shipment and as several cartons of medicine were received in bad order condition, the consignee filed the corresponding claim in the amount of P1,109.67 representing the C.I.F. value of the damaged drum and cartons of medicine with the carrier and the Manila Port Service. However, both refused to pay such claim. Consequently, the consignee filed its claim with the insurer, St. Paul Fire & Marine Insurance Co., the insurance company, on the basis of such claim, paid to the consignee the insured value of the lost and damagcd goods, including other expenses in connection therewith, in the total amount of $1,134.46 U.S. currency. - As subrogee of the rights of' the shipper and/or consignee, the insurer, St. Paul Fire & Marine Insurance Co., instituted an action against the defendants for the recovery of said amount of $1,134.46, plus costs. - The defendants resisted the action. However, for the purpose only of avoiding litigation without admitting liability to the consignee, the defendants offered to settle the latters claim in full by paying the C.I.F. value of the damaged cargo, but this offer was declined by the plaintiff. - The LC rendered judgment ordering the defendants to pay the plaintiff the sum of P300.00. The plaintiff filed a MFR contending that it should recover the amount of $1,134.46 or its equivalent in pesos at the rate of P3.90, instead of P2.00, but this was denied. Hence, this appeal. ISSUES 1. WON in case of loss or damage, the liability of the carrier to the consignee is limited to the C.I.F. value of the goods which were lost or damaged 2. WON the insurer who has paid the claim in dollars to the consignee should be reimbursed in its peso

Page
equivalent on the date of discharge of the cargo or on the date of the decision HELD 1. YES Ratio The purpose of the bill of lading is to provide for the rights and liabilities of the parties in reference to the contract to carry. The stipulation in the bill of lading limiting the common carrier's liability to the value of the goods appearing in the bill, unless the shipper or owner declares a greater value, is valid and binding. This limitation of the carrier's liability is sanctioned by the freedom of the contracting parties to establish such stipulations, clauses, terms, or conditions as they may deem convenient, provided they are not contrary to law, morals, good customs and public policy. A stipulation fixing or limiting the sum that may be recovered from the carrier on the loss or deterioration of the goods is valid, provided it is (a) reasonable and just under the circumstances, and (b) has been fairly and freely agreed upon. In the case at bar, the liabilities of the defendants-appellees with respect to the lost or damaged shipments are expressly limited to the C.I.F. value of the goods as per contract of sea carriage embodied in the bill of lading. - The plaintiff-appellant, as insurer, after paying the claim of the insured for damages under the insurance, is subrogated merely to the rights of the assured. As subrogee, it can recover only the amount that is recoverable by the latter. Since the right of the assured, in case of loss or damage to the goods, is limited or restricted by the provisions in the bill of lading, a suit by the insurer as subrogee necessarily is subject to like limitations and restrictions. 2. On the date of the discharge of the cargo. The peso equivalent was based by the consignee on the exchange rate of P2.015 to $1.00 which was the rate existing at that time. PHILAM v. AUDITOR (supra p.59) FIELDMENS v. ASIAN SURETY (supra p.60) EQUITABLE v. RURAL INSURANCE (supra p.60) COQUIA v. FIELDMEN'S INSURANCE CO. INC. 26 SCRA 178 CONCEPCION; November 29, 1968 NATURE

INSURANCE 112
Appeal from the decision of the CFI certified by CA FACTS - December 1, 1961, appellant Fieldmen's Insurance Company, Inc. issued, in favor of the Manila Yellow Taxicab Co., Inc. a common carrier accident insurance policy, covering the period from December 1, 1961 to December ,1962. It was stipulated in said policy that: "The Company will, subject to the Limits of Liability and under the Terms of this Policy, indemnify the Insured in the event of accident caused by or arising out of the use of Motor Vehicle against all sums which the Insured will become legally liable to pay in respect of: Death or bodily injury to any fare-paying passenger including the Driver, Conductor and/or Inspector who is riding in the Motor Vehicle insured at the time of accident or injury." - While the policy was in force, or on February 10, 1962, a taxicab of the Insured, driven by Carlito Coquia, met a vehicular accident to which he died. The Insured filed therefor a claim for P5,000.00 to which the Company replied with an offer to pay P2,000.00, by way of compromise. The Insured rejected it and made a counter-offer for P4,000.00, but the Company did not accept it. - On September 18, 1962, the Insured and Carlito's parents filed a complaint against the Company to collect the proceeds of the policy. In its answer, the Company admitted the existence thereof, but pleaded lack of cause of action on the part of the plaintiffs. - TC rendered a decision sentencing the Company to pay to the plaintiffs the sum of P4,000.00 and the costs. Hence, this appeal by the Company, which contends that plaintiffs have no cause of action because: 1) the Coquias have no contractual relation with the Company; and 2) the Insured has not complied with the provisions of the policy concerning arbitration. ISSUES 1. WON there was contractual relations between the Coquias and the Company 2. WON the insured has not complied with the provisions of the policy concerning arbitration HELD 1. Although, in general, only parties to a contract may bring an action based thereon, this rule is subject to exceptions, one of which is found in the Art 1311 CC, reading: "If a contract should contain some stipulation in favor of a third person, he may demand its fulfillment provided he communicated his acceptance of the obligor before its revocation. A mere incidental benefit or interest of a person is not sufficient. The contracting parties must have clearly and deliberately conferred a favor upon a third person." - Does the policy in question belong to such class of contracts pour autrui? In this connection, said policy provides, inter alia: "Section I Liability to Passengers. 1. The Company will, subject to the Limits of Liability and under the Terms of this Policy, indemnify the Insured in the event of accident caused by or arising out of the use of Motor Vehicle against all sums which the Insured will become legally liable to pay in respect of: Death or bodily injury to any fare-paying passenger including the Driver. . . who is riding in the Motor Vehicle insured at the time of accident or injury. "Section II. Liability to the Public "3. In terms of and subject to the limitations of and for the purposes of this Section, the Company will indemnify any authorized Driver who is driving the Motor Vehicle . . . " "Conditions "7. In the event of death of any person entitled to indemnify under this Policy, the Company will, in respect of the liability incurred by such person, indemnify his personal representatives in terms of and subject to the limitations of this Policy, provided, that such representatives shall, as though they were the Insured, observe, fulfill and be subject to the Terms of this Policy insofar as they can apply. "8. The Company may, at its option, make indemnity payable directly to the claimants or heirs of claimants, with or without securing the consent of or prior notification to the Insured, it being the true intention of this Policy to protect, to the extent herein specified and subject always to the Terms of this Policy, the liabilities of the Insured towards the passengers of the Motor Vehicle and the Public." - Thus, the policy under consideration is typical of contracts pour autrui, this character being made more manifest by the fact that the deceased driver paid fifty percent (50%) of the corresponding premiums, which were deducted from his weekly commissions. Under these conditions, it is clear that the Coquias who, admittedly, are the sole heirs of the deceased have a direct cause of action against the Company, and, since they could have maintained this action by themselves, without the assistance of the Insured, it

Page
goes without saying that they could and did properly join the latter in filing the complaint herein. 2. Based upon Section 17 of the policy: "If any difference or dispute shall arise with respect to the amount of the Company's liability under this Policy, the same shall be referred to the decision of a single arbitrator to be agreed upon by both parties or failing such agreement of a single arbitrator, to the decision of two arbitrators, one to be appointed in writing by each of the parties within one calendar month after having been required in writing so to do by either of the parties and in case of disagreement between the arbitrators, to the decision of an umpire who shall have been appointed in writing by the arbitrators before entering on the reference and the costs of and incidental to the reference shall be dealt with in the Award. And it is hereby expressly stipulated and declared that it shall be a condition precedent to any right of action or suit upon this Policy that the award by such arbitrator, arbitrators or umpire of the amount of the Company's liability hereunder if disputed shall be first obtained." - The record shows that none of the parties to the contract invoked this section, or made any reference to arbitration, during the negotiations preceding the institution of the present case. In fact, counsel for both parties stipulated, in the trial court, that none of them had, at any time during said negotiations, even suggested the settlement of the issue between them by arbitration, as provided in said section. Their aforementioned acts or omissions had the effect of a waiver of their respective right to demand an arbitration. Disposition The decision appealed from should be as it is hereby affirmed in toto, with costs against the herein defendant-appellant, Fieldmen's Insurance Co., Inc. COUNTRY BANKERS INSURANCE CORP v. LIANGA BAY DE LEON; January 25, 2002 NATURE Petition for review on certiorari FACTS - Lianga Bay is a duly registered cooperative judicially declared insolvent and is here represented by, Cornelio Jamero. Country Bankers Insurance and Lianga Bay entered into a contract of fire insurance.

INSURANCE 113
Country Bankers insured the respondents stocks-intrade against fire loss, damage or liability during the period starting from June 20, 1989 at 4:00 p.m. to June 20, 1990 at 4:00 p.m., for the sum of P200,000.00. - On July 1, 1989, at or about 12:40 a.m., the respondents building at Barangay Diatagon, Lianga, Surigao del Sur was gutted by fire, resulting in the total loss of the respondents stocks-in-trade, pieces of furnitures and fixtures, equipments and records. - Due to the loss, the respondent filed an insurance claim with the petitioner under its Fire Insurance Policy, submitting: (a) the Spot Report of Pfc. Arturo V. Juarbal, INP Investigator, dated July 1, 1989; (b) the Sworn Statement of Jose Lomocso; and (c) the Sworn Statement of Ernesto Urbiztondo. - The petitioner, however, denied the insurance claim on the ground that, based on the submitted documents, the building was set on fire by 2 NPA rebels who wanted to obtain canned goods, rice and medicines as provisions for their comrades in the forest, and that such loss was an excepted risk under paragraph No. 6 of the policy conditions of Fire Insurance Policy No. F-1397, which provides: This insurance does not cover any loss or damage occasioned by or through or in consequence, directly or indirectly, of any of the following occurrences, namely: (d) Mutiny, riot, military or popular uprising, insurrection, rebellion, revolution, military or usurped power. Any loss or damage happening during the existence of abnormal conditions (whether physical or otherwise) which are occasioned by or through or in consequence, directly or indirectly, of any of said occurrences shall be deemed to be loss or damage which is not covered by this insurance, except to the extent that the Insured shall prove that such loss or damage happened independently of the existence of such abnormal conditions. - Finding the denial of its claim unacceptable, Lianga Bay then instituted in the trial court the complaint for recovery of "loss, damage or liability" against Country Bankers. - RTC ruled in favor of the cooperative. CA affirmed. ISSUE WON the cause of the loss was an excepted risk under the terms of the fire insurance policy HELD - Where a risk is excepted by the terms of a policy which insures against other perils or hazards, loss from such a risk constitutes a defense which the insurer may urge, since it has not assumed that risk, and from this it follows that an insurer seeking to defeat a claim because of an exception or limitation in the policy has the burden of proving that the loss comes within the purview of the exception or limitation set up. If a proof is made of a loss apparently within a contract of insurance, the burden is upon the insurer to prove that the loss arose from a cause of loss which is excepted or for which it is not liable, or from a cause which limits its liability. Stated else wise, since Country bank here is defending on the ground of non-coverage and relying upon an exemption or exception clause in the fire insurance policy it has the burden of proving the facts upon which such excepted risk is based, by a preponderance of evidence. But petitioner failed to do so. - The petitioner relies on the Sworn Statements of Jose Lomocso and Ernesto Urbiztondo as well as on the Spot Report of Pfc. Arturo V. Juarbal. A witness can testify only to those facts which he knows of his personal knowledge, which means those facts which are derived from his perception. Consequently, a witness may not testify as to what he merely learned from others either because he was told or read or heard the same. Such testimony is considered hearsay and may not be received as proof of the truth of what he has learned. Disposition the appealed Decision is MODIFIED. The rate of interest on the adjudged principal amount of Two Hundred Thousand Pesos (P200,000.00) shall be six percent (6%) per annum computed from the date of filing of the Complaint in the trial court. The awards in the amounts of Fifty Thousand Pesos (P50,000.00) as actual damages, Fifty Thousand Pesos (P50,000.00) as exemplary damages, Five Thousand Pesos (P5,000.00) as litigation expenses, and Ten Thousand Pesos (P10,000.00) as attorney?s fees are hereby DELETED. DBP POOL OF ACCREDITED RADIO MINDANAO NETWORK 480 SCRA 314 MARTINEZ; January 27, 2006 NATURE Petition for certiorari INSURANCE v.

Page
FACTS - In the evening of July 27, 1988, the radio station of Radio Mindanao Network located at the SSS Building in Bacolod City was burned down causing damage in the amount of over one million pesos. Respondent sought to recover under two insurance policies but the claims were denied on the basis that the case of the loss was an excepted risk under condition no. 6 (c) and (d), to wit: 6. This insurance does not cover any loss or damage occasioned by or through or in consequence, directly or indirectly, of any of the following consequences, namely: (c) War, invasion, act of foreign enemies, hostilities, or warlike operations (whether war be declared or not), civic war. (d) Mutiny, riot, military or popular uprising, insurrection, rebellion, revolution, military or usurped power. - The insurers maintained that based on witnesses and evidence gathered at the site, the fire was caused by the members of the Communist Party of the Philippines/New Peoples Army. Hence the refusal to honor their obligations. - The trial court and the CA found in favor of the respondent. In its findings, both courts mentioned the fact that there was no credible evidence presented that the CCP/NPA did in fact cause the fire that gutted the radio station in Bacolod. ISSUE WON the insurance companies are liable to pay Radio Mindanao Network under the insurance policies HELD YES - The Court will not disturb the factual findings of the appellant and trial courts absent compelling reason. Under this mode of review, the jurisdiction of the court is limited to reviewing only errors of law. - Particularly in cases of insurance disputes with regard to excepted risks, it is the insurance companies which have the burden to prove that the loss comes within the purview of the exception or limitation set up. It is sufficient for the insured to prove the fact of damage or loss. Once the insured makes out a prima facie case in its favor, the duty or burden of evidence shifts to the insurer to controvert said prima facie case. Disposition Petition dismissed. Decision of the CA is affirmed.

INSURANCE 114
LEA MER INDUSTRIES v. MALAYAN INSURANCE 471 SCRA 698 PANGANIBAN; September 30, 2005 NATURE Petition for Review FACTS - Ilian Silica Mining entered into a contract of carriage with Lea Mer Industries for the shipment of 900 metric tons of silica sand valued at P565,000. Consigned to Vulcan Industrial and Mining Corporation, the cargo was to be transported from Palawan to Manila. The silica sand was placed on board Judy VII, a barge leased by Lea Mer, the vessel sank, resulting in the loss of the cargo. ISSUE WON Lea Mer is liable for the loss of the cargo HELD YES - Common carriers are bound to observe extraordinary diligence in their vigilance over the goods and the safety of the passengers they transport, as required by the nature of their business and for reasons of public policy. Extraordinary diligence requires rendering service with the greatest skill and foresight to avoid damage and destruction to the goods entrusted for carriage and delivery. - Common carriers are presumed to have been at fault or to have acted negligently for loss or damage to the goods that they have transported. This presumption can be rebutted only by proof that they observed extraordinary diligence, or that the loss or damage was occasioned by any of the following causes: (1) Flood, storm, earthquake, lightning, or other natural disaster or calamity; (2) Act of the public enemy in war, whether international or civil; (3) Act or omission of the shipper or owner of the goods; (4) The character of the goods or defects in the packing or in the containers; (5) Order or act of competent public authority. - To excuse the common carrier fully of any liability, the fortuitous event must have been the proximate and only cause of the loss. It should have exercised due diligence to prevent or minimize the loss before, during and after the occurrence of the event. - Petitioner bore the burden of proving that it had exercised extraordinary diligence to avoid the loss, or that the loss had been occasioned by a fortuitous event -- an exempting circumstance. - The evidence presented by petitioner in support of its defense of fortuitous event was sorely insufficient. It was not enough for the common carrier to show that there was an unforeseen or unexpected occurrence. Disposition Petition is DENIED and the assailed Decision and Resolution are AFFIRMED. Costs against petitioner. LOADSTAR SHIPPING CO INC v. PIONEER ASIA INSURANCE CORP GR No. 157481 QUISUMBING; January 24, 2006 NATURE Review on certiorari (1) the Decision dated October 15, 2002 and (2) the Resolution dated February 27, 2003 of CA FACTS - June 6, 1984 - Petitioner Loadstar Shipping Co., Inc. (LOADSTAR), registered owner and operator of the vessel M/V Weasel, entered into a voyage-charter with Northern Mindanao Transport Company, Inc. for the carriage of 65,000 bags of cement from Iligan City to Manila. The shipper was Iligan Cement Corporation, while the consignee in Manila was Market Developers, Inc. (MARKET) - June 24, 1984 - 67,500 bags of cement were loaded on board M/V Weasel and stowed in the cargo holds for delivery to the consignee. The shipment was covered by petitioners Bill of Lading dated June 23, 1984. - Prior to the voyage, the consignee insured the shipment of cement with respondent Pioneer Asia Insurance Corporation (PIONEER) for P1,400,000, for which it issued Marine Open Policy No. MOP-006 dated September 17, 1980, covering all shipments made on or after September 30, 1980 - June 25, 1984 - Captain Montera of M/V Weasel ordered the vessel to be forced aground which rendered the entire shipment of cement as good as

Page
gone due to exposure to sea water. LOASTAR thus failed to deliver the goods to MARKET in Manila. MARKET demanded from LOADSTAR full reimbursement of the cost of the lost shipment. LOADSTAR refused to reimburse the MARKET despite repeated demands. - March 11, 1985 PIONEER paid the MARKET P1,400,000 plus an additional amount of P500,000, the value of the lost shipment of cement. In return, the MARKET executed a Loss and Subrogation Receipt in favor of PIONEER concerning the latters subrogation rights against LOADSTAR. - October 15, 1986 PIONEER filed a complaint against LOADSTAR with the RTC Manila alleging that: (1) the M/V Weasel was not seaworthy at the commencement of the voyage; (2) the weather and sea conditions then prevailing were usual and expected for that time of the year and as such, was an ordinary peril of the voyage for which the M/V Weasel should have been normally able to cope with; and (3) LOADSTAR was negligent in the selection and supervision of its agents and employees then manning the M/V Weasel. - LOADSTAR alleged that no fault nor negligence could be attributed to it because it exercised due diligence to make the ship seaworthy, as well as properly manned and equipped and failure to deliver was due to force majeure. - February 15, 1993 - RTC decided in favor of PIONEER and that LOADSTAR , as a common carrier, bears the burden of proving that it exercised extraordinary diligence in its vigilance over the goods it transported. The trial court explained that in case of loss or destruction of the goods, a statutory presumption arises that the common carrier was negligent unless it could prove that it had observed extraordinary diligence. LOADSTARS defense of force majeure was found bereft of factual basis as a PAG-ASA report that at the time of the incident, tropical storm Asiang had moved away from the Philippines was presented. - October 15, 2002 CA affirmed RTC Decision with modification ISSUES 1. WON LOADSTAR is a common carrier under Article 1732 CC 2. Assuming it is a common carrier, WON proximate cause of the loss of cargo was not a fortuitous event but was allegedly due to the failure of petitioner to exercise extraordinary diligence

INSURANCE 115
HELD 1. YES - A1732 CC defines a common carrier as follows: Common carriers are persons, corporations, firms or associations engaged in the business of carrying or transporting passengers or goods or both, by land, water, or air, for compensation, offering their services to the public. - LOADSTAR is a corporation engaged in the business of transporting cargo by water and for compensation, offering its services indiscriminately to the public. Thus, without doubt, it is a common carrier. Even if it entered into a voyage-charter agreement with Northern Mindanao Transport Company, Inc, it did not in any way convert the common carrier into a private carrier. > Planters Products, Inc. v. CA - public carrier shall remain as such, notwithstanding the charter of the whole or portion of a vessel by one or more persons, provided the charter is limited to the ship only, as in the case of a time-charter or voyagecharter. It is only when the charter includes both the vessel and its crew, as in a bareboat or demise that a common carrier becomes private, at least insofar as the particular voyage covering the charter-party is concerned. 2. YES - As a common carrier, LOADSTAR is required to observe extraordinary diligence in the vigilance over the goods it transports. When the goods placed in its care are lost, LOADSTAR is presumed to have been at fault or to have acted negligently. LOADSTAR has the burden of proving that it observed extraordinary diligence in order to avoid responsibility for the lost cargo. - Compania Maritima V CA - It requires common carriers to render service with the greatest skill and foresight and to use all reasonable means to ascertain the nature and characteristics of goods tendered for shipment, and to exercise due care in the handling and stowage, including such methods as their nature requires. - A1734 CC enumerates the instances when a carrier might be exempt from liability for the loss of the goods. (1) Flood, storm, earthquake, lightning, or other natural disaster or calamity; (2) Act of the public enemy in war, whether international or civil; (3) Act or omission of the shipper or owner of the goods; (4) The character of the goods or defects in the packing or in the containers; and (5) Order or act of competent public authority - LOADSTAR claims that the loss of the goods was due to a fortuitous event under paragraph 1. Yet, its claim is not substantiated. It is supported by evidence that the loss of the entire shipment of cement was due to the gross negligence of LOADSTAR - Records show that in the evening of June 24, 1984, the sea and weather conditions in the vicinity of Negros Occidental were calm. The records reveal that LOADSTAR took a shortcut route, instead of the usual route, which exposed the voyage to unexpected hazard. LOADSTAR has only itself to blame for its misjudgment. Disposition petition is DENIED

Page

You might also like